Vous êtes sur la page 1sur 311

Mad about

Modern Physics
Mad about
Modern Physics
Braintwisters, Paradoxes, and Curiosities

Franklin Potter
and
Christopher Jargodzki

John Wiley & Sons, Inc.


This book is printed on acid-free paper.

Copyright 2005 by Franklin Potter and Christopher Jargodzki. All rights reserved

Illustrations on pages 2, 4, 9, 26, 31, 134, and 161 copyright 2005 by Tina Cash-Walsh.

Published by John Wiley & Sons, Inc., Hoboken, New Jersey


Published simultaneously in Canada

Design and production by Navta Associates, Inc.

No part of this publication may be reproduced, stored in a retrieval system, or transmitted in any
form or by any means, electronic, mechanical, photocopying, recording, scanning, or otherwise,
except as permitted under Section 107 or 108 of the 1976 United States Copyright Act, without
either the prior written permission of the Publisher, or authorization through payment of the appro-
priate per-copy fee to the Copyright Clearance Center, 222 Rosewood Drive, Danvers, MA 01923,
(978) 750-8400, fax (978) 646-8600, or on the web at www.copyright.com. Requests to the Pub-
lisher for permission should be addressed to the Permissions Department, John Wiley & Sons, Inc.,
111 River Street, Hoboken, NJ 07030, (201) 748-6011, fax (201) 748-6008.

Limit of Liability/Disclaimer of Warranty: While the publisher and the author have used their best
efforts in preparing this book, they make no representations or warranties with respect to the accu-
racy or completeness of the contents of this book and specically disclaim any implied warranties
of merchantability or tness for a particular purpose. No warranty may be created or extended by
sales representatives or written sales materials. The advice and strategies contained herein may not
be suitable for your situation. You should consult with a professional where appropriate. Neither
the publisher nor the author shall be liable for any loss of prot or any other commercial damages,
including but not limited to special, incidental, consequential, or other damages.

For general information about our other products and services, please contact our Customer Care
Department within the United States at (800) 762-2974, outside the United States at (317) 572-
3993 or fax (317) 572-4002.

Wiley also publishes its books in a variety of electronic formats. Some content that appears in print
may not be available in electronic books. For more information about Wiley products, visit our web
site at www.wiley.com.

Library of Congress Cataloging-in-Publication Data:

Potter, Frank, date


Mad about modern physics : braintwisters, paradoxes and curiosities / Franklin Potter and
Christopher Jargodzki.
p. cm.
Includes index.
ISBN 0-471-44855-9
1. Physics--Popular works. I. Jargodzki, Christopher II. Title
QC24.5.P68 2004
530dc22
2004014941

Printed in the United States of America

10 9 8 7 6 5 4 3 2 1
To my late parents, who nourished my formative years and
have now crossed that portal to another world.

F. P.

To my late grandmotherZoa Lesinska,


who instilled in me the idea that the visible world
owes its being to the invisible one.

C. J.
Contents

Preface . . . . . . . . . . . . . . . . . . . . . . . . . . ix

Acknowledgments. . . . . . . . . . . . . . . . . . xii

To the Reader . . . . . . . . . . . . . . . . . . . . . xiii

Chapter 1 The Heat Is On . . . . . . . . . . . . . . . . . . . . 1

Chapter 2 Does Anybody Really Know What


Time It Is? . . . . . . . . . . . . . . . . . . . . . . . . 11

Chapter 3 Crazy Circles . . . . . . . . . . . . . . . . . . . . . 19

Chapter 4 Fly Me to the Moon. . . . . . . . . . . . . . . . . 29

Chapter 5 Go Ask Alice . . . . . . . . . . . . . . . . . . . . . . 39

Chapter 6 Start Me Up . . . . . . . . . . . . . . . . . . . . . . 49

Chapter 7 A Whole New World. . . . . . . . . . . . . . . . . 63

Chapter 8 Chances Are . . . . . . . . . . . . . . . . . . . . . . 75

Chapter 9 Can This Be Real? . . . . . . . . . . . . . . . . . 91

Chapter 10 Over My Head. . . . . . . . . . . . . . . . . . . . . 105

Chapter 1 1 Crystal Blue Persuasion . . . . . . . . . . . . . 117

vii
Answers
The Heat Is On . . . . . . . . . . . . . . . . . . . . . . 125
Does Anybody Really Know What
Time It Is? . . . . . . . . . . . . . . . . . . . . . . . 139
Crazy Circles . . . . . . . . . . . . . . . . . . . . . . . 151
Fly Me to the Moon . . . . . . . . . . . . . . . . . . 164
Go Ask Alice . . . . . . . . . . . . . . . . . . . . . . . 181
Start Me Up . . . . . . . . . . . . . . . . . . . . . . . . 192
A Whole New World . . . . . . . . . . . . . . . . . . 206
Chances Are. . . . . . . . . . . . . . . . . . . . . . . . 224
Can This Be Real? . . . . . . . . . . . . . . . . . . . 24 1
Over My Head . . . . . . . . . . . . . . . . . . . . . . 257
Crystal Blue Persuasion . . . . . . . . . . . . . . 27 7

Index . . . . . . . . . . . . . . . . . . . . . . . . . . 287

viii Contents
Preface
T
his book of almost 250 puzzles begins where our rst book, Mad
About Physics: Braintwisters, Paradoxes, and Curiosities (2001)
endedwith the physics of the late nineteenth and early twenti-
eth centuries. The Michelson-Morley experiment of 1887, the
challenges posed by atomic spectra and blackbody radiation, the
unexpected discoveries of X-rays in 1895, radioactivity in 1896, and
the electron in 1897 all loosened the protective belt of ad hoc hypothe-
ses around the mechanistic physics the nineteenth century had so
laboriously built. Anomalies and paradoxes abounded, ultimately
necessitating a radical rethinking of the very foundations of physics
and culminating in the theory of relativity and quantum mechanics.
Numerous applications of these new and strange concepts followed
very quickly as atomic and nuclear physics led to semiconductor
devices on the small scale and nuclear energy on the large scale. There-
fore we have developed a whole new set of challenges to tickle the
minds of our scientically literate readers, from science students to
engineers to professionals in the sciences.
The challenges begin with the classical problem of getting a cooked
egg into a bottle through a narrow bottleneck and back out again and
progress gradually to the famous aging-twin paradox of the theory of
special relativity and eventually reach problems dealing with the large-
scale universe. In between, we explore the nature of time and of space
as well as how the world of lms and television tends to sacrice
physics for the sake of entertainment. We also consider some of the
more startling questions in relativity. For example, we ask whether a
person can go on a space journey out to a star 7,000 light-years distant
and return while aging only 40 years! And we certainly want to
emphasize the practical applications of microphysics through an exam-
ination of some properties of exotic uids, unusual motors running on
air or on random motion, as well as thermal, electrical, and photonic
properties of materials in a challenging journey into the atomic world.

ix
Particularly important microworld challenges include: What happened
to Schrdingers cat? Can a cup of coffee be the ultimate quantum
computer? Why is a Bose-Einstein condensate a new state of matter?
Why is quantum mechanical coherent scattering so important in devel-
oping new detectors for neutrinos and gravitational waves? When we
reach the nucleus, there are challenges about the accuracy of carbon-14
dating, the reason for neutron decay, and the amount of human
radioactivity. Then our journey reverses as we reach for the stars to con-
sider Olbers paradox about why the night sky is dark instead of burst-
ing with light, how gravitational lensing by galaxies works, and what
the total energy in the universe might be. This book nishes with a pot-
pourri of challenges from all categories that ranges from using bicycle
tracks in the mud to determine the direction of travel, to analyzing
water-spouting alligators, and ending with a space-crawling mechanical
invention that seems to defy the laws of physics.
The puzzles range in difculty from simple questions (e.g., Will
an old mechanical watch run faster or slower when taken to the
mountains?) to subtle problems requiring more analysis (e.g., Is the
Bragg scattering of X-rays from an ideal crystal a coherent scattering
process?) Solutions and more than 300 references are provided, and
they constitute about two-thirds of the book.
As these examples demonstrate, most of the puzzles contain an ele-
ment of surprise. Indeed, one nds that commonsense conjecture and
proper physical reasoning often clash throughout this volume. Ein-
stein characterized common sense as the collection of prejudices
acquired by age eighteen, and we agree: at least in science, common
sense is to be rened and often transcended rather than venerated.
Many of the challenges were devised to undermine physical precon-
ceptions by employing paradoxes (from the Greek para and doxos,
meaning beyond belief) to create cognitive dissonance. Far from
being simply amusing, paradoxes are uniquely effective in addressing
specific deficiencies in understanding. Usually the contradiction
between gut instinct and physical reasoning for some people will be so
painful that they will go to great lengths to escape it even if it means
having to learn some physics in the process.
Philosopher Ludwig Wittgenstein considered paradoxes to be an
embodiment of disquietude, and as we have learned, these disqui-
etudes often foreshadow revolutionary developments in our thinking

x Preface
about the natural world. The counterintuitive upheavals resulting
from relativity theory and quantum mechanics in the twentieth cen-
tury only enhanced the reputation of the paradox as an agent for
change in our understanding of physical reality.
Such disquietudes, rather than unexplained experimental facts,
writes Gerald Holton in Thematic Origins of Scientic Thought, were
what led Einstein to rethink the foundations of physics in his three
papers of 1905. Each begins with the statement of formal asymmetries
of a predominantly aesthetic nature, then proposes a general postu-
late, not derivable directly from experience, that removes the asym-
metries. For example, in the paper on the quantum theory of light,
formal asymmetry existed between the discontinuous nature of parti-
cles and the continuous functions used to describe electromagnetic
radiation. As Holton notes, The discussion of the photoelectric
effect, for which this paper is mostly remembered, occurs toward the
end, in a little over two pages out of the total sixteen. Consistent
with this approach is Einsteins statement in Physics and Reality
(1936), We now realize . . . how much in error are those theorists
who believe that theory comes inductively from experience, and later
in The Evolution of Physics (1938), coauthored with the Polish physi-
cist Leopold Infeld, Physical concepts are free creations of the human
mind, and are not, however it may seem, uniquely determined by the
external world.
As another sore point, the term quantum mechanics is really a
misnomer: quantum systems cannot be regarded as made up of sepa-
rate building blocks. In the helium atom, for instance, we do not have
electron A and electron B but simply a two-electron pattern in which
all separate identity is lost. This indivisible unity of the quantum world
is paralleled by another kind of unitybetween subject and object. Is
light a wave or a particle? The answer seems to depend on the experi-
mental setup. In the double-slit experiment, the observations of light
yield characteristics of the box and its slits as much as of light itself.
Is reality then observer-dependent? And would this justify Einsteins
insistence on the power of pure thought in the construction of physi-
cal reality? Modern physics seems particularly adept at generating such
disquietudes. If thats the case, then perhaps the word Mad in the title
of our book should not be construed as a mere metaphor!

Preface xi
Acknowledgments

W
e all stand on the shoulders of giants as we develop our
minds to become individuals living today on our planet Earth.
And we owe so much to so many people that we cannot
acknowledge all of them.
Franklin Potter would like to express appreciation to his wife,
Patricia, and their two sons, David and Steven, for their love and
inspiration through many wonderful years of family adventures. He
also treasures the numerous inspiring physics discussions over the
decades with many friends and colleagues: Howard G. Preston,
Gregory Endo, Fletcher Goldin, David M. Scott, John Priest, Lowell
Wood, Julius S. Miller, George E. Miller, Leigh H. Palmer, Charles W.
Peck, Myron Bander, Joseph Weber, Richard Feynman, Willard Libby,
Edward Teller, and Kamal Das Gupta.
Christopher Jargodzki would like to express appreciation to
Myron Bander of the University of California at Irvine; Stephen Reu-
croft of Northeastern University in Boston; and James H. Taylor of
Central Missouri State University in Warrensburg. His interactions
with close to twenty thousand students (and counting!) in his classes
at UC Irvine, Northeastern University, and CMSU have been, over the
years, never-ending sources of stimulation, as well as occasional exas-
peration. In fact, the present volume got its start in 1975 when one of
us (C. J.), still a graduate student at UC Irvine, put together a proposal
for a book of paradoxes in modern physics, partly to allay his own
exasperation with the koanlike conundrums that abound in modern
physics. Alas, the project had to wait several decades for the author to
mature and join forces with Franklin Potter in our joint inquiry into
the nature of physical reality. The authors hope that physical reality is
duly impressed with their efforts.
Both authors sincerely thank Kate C. Bradford, senior editor at
John Wiley & Sons, Inc., who continues to support our paradoxical
adventures into the world of physics.

xii
To the Reader

T
hese puzzles are meant to be fun. How many puzzles you solve is
not as important as how many you enjoy thinking about. Some
of them are even challenging to research physicists, and some
were generated by research articles that have appeared only recently in
physics journals, so these topics may not have been part of physics just
10 years ago! It would be a rare reader who could provide detailed
solutions to all the puzzles. Indeed, sometimes you may need to think
a bit to even understand the answer. If we included all the steps, this
book would double its present size. We offer no apologies, but we do
try to provide all the key steps to make each answer complete on its
own. If you nd the puzzles perplexing and intriguing, we have suc-
ceeded in our mission.
Mad about Modern Physics can be read with prot by anyone who
has had some exposure to a year of introductory physics and is eager
to learn more about its applications and its more recent discoveries.
Most puzzles are nonmathematical in character and require only a
qualitative application of fundamental physics principles. Many
physics concepts are dened directly or indirectly in the questions or
in the answers, so they can be found with the aid of the index. How-
ever, even someone who knows the subject will quickly realize that the
application of physics to the real world can be quite challenging, and
in this sense this is not an elementary book.
More than three hundred follow-up references provide further
resources for interested readers. These referencesto journal research
papers, books, and magazine articlesare included with only some of
the puzzles, typically those that are either controversial or that involve
relatively new concepts. There was no space to include a more com-
plete list of references. Consequently we had to make choices, and we
apologize to the authors whose work may have been left out or inad-
vertently overlooked.

xiii
Any errors are solely those of the authors, and we would appre-
ciate your communications via e-mail to Franklin Potter (see
www.sciencegems.com) with regard to the puzzles and their
answers.

xiv To the Reader


1 The Heat
Is On
S CIENCE IN THE HOME CONTRIBUTES IMMENSELY

to our everyday repertoire of activities, although


most of us are unaware of exactly how science does so.
Physics, in particular, is all around us and plays a crucial role
in determining what we can and cannot do. One enjoyable
activity for many people is cooking, which is an application
of physics and chemistry to satisfy our gastronomical tastes.
Or are physics and chemistry just other modes of cooking?
Well let you decide. Most of the challenges in this chapter
involve physics from a high-school-level course. But be care-
ful. Quick responses may be correct occasionally, but you
should not rely on your intuition very much, for Nature,
particularly in the kitchen, is nonintuitive for the most part.
Anyone who has tried to make a souf can attest to how
limited a recipe can be!

1
We can detect five 1. Egg into a Bottle
basic tastesfour are Egg
very familiar: sweet, Perhaps the most intriguing
sour, bitter, and salty. physics-in-the-kitchen demon-
The fifth, while familiar stration for all ages is get-
in East Asia, is less well ting a hard-boiled egg with
known in Western the shell removed into a
cuisineit is called bottle that has an opening
Umami and is the taste diameter smaller than the
of monosodium gluta- minimum diameter of the
mate, MSG. MSG is egg. One solution is to very
used widely in Eastern
carefully drop some bits of
cooking and that is
burning paper into
probably why it is rec-
the upright bottle and
ognized as a separate
taste sensation more then place the egg at
the opening. Soon, if Burning
readily by those familiar paper
with that cuisine. How- the sequence is done
ever, many common with the correct tim-
western foods contain ing, the egg will have the urge to go inside. What is the
large amounts of MSG, correct timing, and why does the egg have this urge?
notably tomatoes and
parmesan cheese.
P ETER B ARHAM ,
T HE S CIENCE OF C OOKING 2. Egg out of a Bottle
Perhaps the most challenging physics-in-the-kitchen
demonstration for all ages is getting a hard-boiled egg
I was raised in Alabama with the shell removed out of a bottle that has an open-
and Florida a Southern ing diameter smaller than the minimum diameter of
Baptist, a lad given the egg. Of course, one could cut up the egg with a
simple answers to pro- knife inserted into the bottle and then pour out the
found questions. At the pieces. However, we want the egg out whole and
same time I came to undamaged.
love science, which Long ago, physics professor Julius Sumner Miller,
seeks profound answers (who was Professor Wonderful on the early Mickey
to simple questions. Mouse Club shows) was on the Tonight Show with
E DWARD O. W ILSON host Johnny Carson and showed rst how to get the
egg into the bottle and then, taking no more than three

2 Mad about Modern Physics


seconds, had the same egg back in his hand. What is the Why is it that tea made
procedure? (Hint: the same physics principles that put with microwave-heated
the egg into the bottle can get the egg out.) water doesnt taste as
good as tea made with
teakettle water? The
3. Sugar main reason is that
microwaves heat only
Add two cups of sugar to one cup of water in a the outer inch or so of
saucepan and stir while heating slightly. All the sugar the water all around the
will dissolve. About how much total sugar will dissolve cup, because thats as
in one cup of water? What is the physics? far as they can pene-
trate. The water in the
middle of the cup gets
4. Kneading Bread hot more slowly, through
contact with the outer
Bread made with yeast is usually kneadedthat is, portions. When the
drawn out and pressed together to create a distribution outer portions of the
of the ingredients. Then the bread dough is set aside to water have reached boil-
rise. Why is some bread then kneaded a second time ing temperature and
and sometimes even a third time before baking? start to bubble, you can
be tricked into thinking
that all the water in the
5. Measuring Out Butter cup is that hot. But the
average temperature
Suppose you have a solid chunk of butter and a meas- may be much lower, and
uring cup in the kitchen. You desire to accurately your tea will be short-
measure one-half cup of butter chunks without melting changed of good flavor.
them. What is a quick, easy way to do so? Often one ROBERT L. WOLKE , W HAT
encounters the statement in cookbooks that Archi- E INSTEIN TOLD H IS C OOK :
K ITCHEN S CIENCE E XPLAINED
medes principle is being used. What is this principle,
and why is the statement erroneous?

If there were one drop


6. Milk and Cream of water less in the
universe,
You are given two identical bottles, one with milk and
the whole world would
the other with cream, both lled to the top. Quick now,
thirst.
which is heavier? And is light cream lighter than heavy U GO B ETTI ,
cream? I TALIAN P LAYWRIGHT

The Heat Is On 3
CALORIC REQUIREMENT 7. Straw and Potato
BASED ON BODY WEIGHT
The basal calorie A paper or plastic drinking straw can be pushed
requirement of the through an uncooked potato. Explain the physics. If
average adult is ten you plan to try this demonstration, be sure that you
times the ideal weight take appropriate safety precautionskeep your hands
in pounds (e.g., 1,270 and body out of harms way.
for 127 lbs.), plus 30
percent for light activity
(i.e., 1,650 kcal), 50 Push down
percent for moderate Straw
activity (1,905 kcal),
and 100 percent for
Potato
heavy activity (2,540
kcal). Expressed slightly
differently, the basal
energy requirement is
about 1 kilocalorie per
hour for every kilogram
(2.2 lbs.) of ideal body
weight. Of course, any
8. Blueberry Muffins
estimate of calorie Marion loves to bake warm, fresh blueberry mufns,
requirements based on with the blueberries almost uniformly distributed
such formulas is just throughout the mufn. She knows that if one simply
thatan estimate . prepares the batter and mixes in the blueberries, they
Individual requirements may be uniformly distributed before entering the oven,
vary widely with age,
but upon baking they will gravitate to lodge in the
health, body size,
lower part of the mufn. How does she prevent this
and environmental
natural downward drift?
temperature.

9. Can of Soup
Some people buy canned soup and store the cans in the
When men reach their
cupboard. Some people even turn these soup cans
sixties and retire, they
go to pieces. Women go upside down for storage. If we open a can of soup that
right on cooking. was stored in the upright position by removing the top,
G AIL S HEEHY, A MERICAN quite often all the concentrated ingredients are on the
J OURNALIST bottom and must be scooped out with a spoon. Even

4 Mad about Modern Physics


then, not all the concentrate is removed. Suppose, The boiling temperature
instead, we turn the same can upside down and open of water decreases
the bottom. Upon turning the can over, the soup simply about 1.9F for every
rushes out into the pot. Why so? 1,000 feet above sea
level. So in Denver, the
mile-high city, water will
10. Salt and Sugar boil at 202Fthat is,
at 94.4C. Tempera-
Salts have been used for thousands of years to preserve
tures above 165F are
meats, and sugar has been used to preserve fruits and
generally thought to be
berries. How do they work?
high enough to kill most
germs, so there is no
11. Defrosting Tray danger on this account
until you get to about
In catalogs and cookware stores one can buy a mira- 25,000 feet.
cle defrosting tray advertised as made of an
advanced, space-age super-conductive alloy that
takes heat right out of the air. How does this defrost-
ing tray work?
On the average we get
about 9 (food) calories
12. Ice Cream Delight (kcal) of energy from
Most of us have made ice cream or seen ice cream being each gram of fat and
made. Milk, eggs, sugar, and flavorings are slowly 4 calories from each
gram of protein or
chilled. Terri likes to make ice cream in a simpler and
carbohydrate. To lose a
more efcient way. Practicing proper safety precautions,
pound (454 g) of fat,
she pours liquid nitrogen directly into the ingredients in
we have to cut the food
a metal bowl. About equal volumes of liquid nitrogen intake by 3,500 calo-
and the mixture are used for ice cream or sorbet, and ries. The discrepancy in
she stirs while adding the coolant until the ice cream is numbers is due to the
nicely stiff. Why does this method produce absolutely fact that body fat is
marvelous ice cream, and what is the physics here? only about 85 percent
actual fat, the rest
coming from connective
13. Cooking a Roast tissue, blood vessels,
For many types of meatbeef, pork, lamb, etc.one and other things.
can buy a roast from the butcher with or without the
bone inside. Suppose we have two beef roasts of the same

The Heat Is On 5
Light bounces off mir- weight of 4.4 pounds (2 kg) and cook them in identical
rors; microwaves bounce ovens at the same temperature. One roast has the bone in
off metal. If what you and the other does not. Which roast cooks faster? Why?
put in the microwave
oven reflects too many
microwaves back 14. Cooking Chinese Style
instead of absorbing Estimates of Chinese meals include more than 3,000
them, the magnetron
varieties, possibly more meal types than the total num-
tube that generates the
ber of meals by all other cultures combined. Many of
microwaves can be
the Chinese dishes use meats cut into small cubes or
damaged. There must
always be something in other small volumes. Certainly, these small volumes are
the oven to absorb much easier to eat with chopsticks. Are there any sig-
microwaves. Thats why nicant scientic reasons for cutting up the meats into
you should never run it small volumes?
empty.
Metals in microwave
ovens can behave
15. Baked Beans
unpredictably. Micro- If you buy dry beans in bulk, they must be soaked in
waves set up electrical water overnight in a covered container before they are
currents in metals, and ready to be baked. To bake them without soaking
if the metal object is too would require an enormous amount of cooking time.
thin it may not be able An alternative preparation is to parboil them in a
to support the current
cooking potthat is, simmer them. Simmer means to
and will turn red hot and
be on the verge of boiling.
melt. And if it has sharp
How does one know that the beans have simmered
points, it may even act
like a lightning rod and enough? The test involves good physics. Take up a few
concentrate so much beans in a spoon and, after making sure that no liquid
microwave energy at the is in the spoon, blow a stream of air gently with pursed
points that it will send lips against the beans. If the bean skin cracks, the beans
off lightning-like are ready for baking. Why must the lips be pursed, and
sparks. why do the bean skins then crack open?
ROBERT L. WOLKE , W HAT
E INSTEIN TOLD H IS C OOK :
K ITCHEN S CIENCE E XPLAINED 16. Ice Water
Normally, to cool a pitcher of water quickly, one adds
ice. The ice oats at the top. Suppose one could add the
same amount of ice so it could be held in the water at

6 Mad about Modern Physics


the bottom of the pitcher. Which technique would lead A standard 12-ounce
to faster cooling of the water? aluminum can, whose
wall surfaces are
thinner than two pages
17. Peeling Vegetables from this book (about
A friend of ours peels ripe tomatoes by impaling the 0.00762 cm), with-
tomato on a fork, then holding it over a gas ame and stands more than 90
pounds of pressure per
rotating gently. If you try this procedure, use appropri-
square inchthree
ate safety procedures to protect your eyes and body.
times the pressure in an
Peeling fresh beets is also a messy chore. Their col-
automobile tire.
ored liquid stains everything, including your ngers. W ILLIAM H OSFORD AND
Another friend of ours peels fresh beets by rst boiling J OHN D UNCAN , T HE
them, then immediately holding them under cold water A LUMINUM B EVERAGE C AN ,
S CIENTIFIC A MERIC AN ,
with a fork. What is the physics in both of these meth- S EPTEMBER 1994
ods used for preparing vegetables for peeling?

18. Igniting a Sugar Cube


Sugar burns in air. But igniting
a sugar cube is much more dif-
ficult than expected. Put a
sugar cube on the end of a Decaffeinated coffee
toothpick and bring a lighted still contains caffeine!
match flame under a remote A regular cup of
coffee has 80 to 135
corner. The sugar melts
milligrams of caffeine.
instead of burning, and the
For a coffee to be con-
brown, gooey stuff is caramel.
sidered decaffeinated,
However, we wish to burn at least 97 percent of
the sugar, not melt it! We want to see it on re with a the coffees caffeine
ame of its own. Why is this process so difcult to must be removed. Test-
achieve? How can we succeed in lighting the sugar cube ing shows that decafs
with the burning match? typically have 2 to 6
milligrams of caffeine
per cup.
19. Water Boiling
An open pot of water is boiling on the kitchen stove.
Sprinkle some room-temperature table salt (which

The Heat Is On 7
An object at room tem- contains mostly NaCl and some KCl) into the clear
perature (20C) emits boiling water, and the boiling ceases. Isnt it amazing
radiation with a peak at how the water ceases its boiling as the salt warms up!
the wavelength 9.89 Can you explain the physics? What is the surprise here?
micrometers, roughly
.01 mm, in the infrared
region of the electro-
magnetic spectrum. 20. Put the Kettle On
Bring water to a boil in a teakettle with a spout. Let it
cook! Now watch the mouth of the spout carefully.
What do you see? Can you see the water vapor come
out?

21. The Watched Pot


You have probably heard the expression A watched
pot never boils. Is this statement correct physics? That
For an isolated water
is, when would this statement be good physics? (Hint:
molecule the H-O-H
angle is 104.5. In ice One should interpret the phrase never boils here to
each water molecule mean that the cooking takes a longer time.)
forms hydrogen bonds
to four nearest neigh-
bors in a tetrahedral
arrangement. The tetra- 22. Ice in a Microwave
hedral bond geometry The microwave oven emits microwaves that are
explains the openness absorbed by water molecules in food. Microwaves
and relatively low den- make the polar water molecules rotate or oscillate, and
sity of ice (i.e., why
their friction within the material converts some of
water expands upon
this kinetic energy into thermal energy to raise the tem-
freezing). In ice the
perature of the food.
H-O-H angles are
nearly the same as the Suppose you made an ice block that had liquid
perfect tetrahedral water trapped in a large cavity inside and then you
angle of 109.5. placed the block into a microwave oven. Could the
trapped water be brought to a boil while the ice
remained ice?

8 Mad about Modern Physics


23. The Glycemic Index Night cooling by evapo-
ration of water and heat
The glycemic index is an important number for anyone radiation had been per-
concerned with the conversion of food to blood sugar fected by the peoples
(sucrose), for the gylcemic index gives the measured of Egypt and India, and
rate of this conversion process. The higher the glycemic several ancient cultures
index value, the faster the conversion rate to sucrose. had partially investi-
There are types of sugar molecules other than sucrose. gated the ability of
Glucose, for example, is normally the standard reference salts to lower the freez-
for the conversion rate to sucrose, with a value of 100. ing temperature of
Some sample values of the glycemic index for foods water. Both the ancient
Greeks and Romans
are: brown rice, 59; white rice, 88; table sugar,
had figured out that
65; grapefruit, 25; spaghetti, 25 to 45; potato, boiled,
previously boiled water
55; potato, baked, 85; and dates, 103. Brown rice has
will cool more rapidly
more outer layer intact than white rice, so its lower than unboiled water, but
value is evident. But why would a baked potato have a they did not know why;
much higher glycemic index than a boiled potato? And boiling rids the water
how could the value for dates, or any food, be higher of carbon dioxide and
than 100? other gases that other-
wise retard the lowering
of water temperature.
24. Electric Pickle TOM S HACHTMAN , A BSOLUTE
Z ERO AND THE C ONQUEST
Some specialty and novelty stores sell an electrical
OF C OLD
appliance that cooks hot dogs between two metal
electrodes. A protective cover with a safety interlock

Interestingly, microwave
ovens are not very good
at melting ice. The
water molecules in ice
are bound pretty tightly
together into a crystal
lattice, so they cant flip
back and forth under
the influence of micro-
waves oscillation.

The Heat Is On 9
Although it flies in the closes over the device before electrical energy in the
face of common sense, form of a standard AC current can be applied. Suppose
people with more insula- that instead of a hot dog one places a pickle between the
tionfatwhose body electrodes. When the room lights are dimmed, the
core is better protected pickle glows impressively, predominantly at one end.
from the cold, may feel What is the physics, and what might the glow look like?
cold more quickly than
thinner people with less
protection. The reason 25. Space-Age Cooking
is that insulation keeps
Microwave ovens were probably the rst new method
heat in the core, away
from the skin, which for making heat for cooking in more than a million
gets cold. When the years. In addition, two newer methods have become
skin gets cold, you feel available for the kitchen. Magnetic induction cooktops
cold. Paradoxically, have been available for about fteen years in Europe
women may feel colder and Japan and are now becoming known in the United
than men because States. And for the modern chef, cooking with light in
women are better a light oven has been done since the mid-1990s and
insulated. may become a fad in the immediate future. How do
J AMES G ORMAN , B EYOND both of these cooking sources work?
B RR : T HE E LUSIVE S CIENCE
OF C OLD, T HE N EW YORK
T IMES , F EBRUARY 10, 2004

The main compartment


of a refrigerator should
always be below 40F
(4.4C). Above that
temperature, bacteria
can multiply fast
enough to be dangerous.

10 Mad about Modern Physics


2 Does Anybody
Really Know
What Time It Is?
is?
W HAT IS TIME ? S T. A UGUSTINE FAMOUSLY

wrote. If no one asks me, I know. But if I


wanted to explain it to one who asks me, I plainly do not
know. Time itself is a strange quantity to some people. To
many of us, time never seems to be going at the right rate
sometimes too fast, sometimes too slow. In some parts of the
world, promptness and being on time are important aspects
of the local culture. In other regions, time is almost irrele-
vant. In this chapter, we have created a mixture of familiar
challenges and many new ones in preparation for later chap-
ters in which time shares its role with space as a major ingre-
dient of motion, chapters that look at concepts such as the
space-time of the special theory of relativity and the world of
astrophysics.

11
How did the day get to be 26. January Summer
divided into 24 hours?
The night appears to have Contrary to the popular belief that Earth is closest to the
been divided first, by the Sun on about June 23 or possibly December 22 each
ancient Egyptians. year, the date of perihelion actually falls between January
According to Prof. Owen 2 and January 5! In the Northern Hemisphere, we expe-
Gingerich, a historian of rience winter on this January date because the North
science at Harvard Uni- Polar axis is tilted away from the Sun. The Southern
versity, they divided the Hemisphere enjoys a warm summer at this time. Will the
heavens into intervals of Northern Hemisphere ever enjoy summer in January?
10 degrees of arc, making
it possible to squeeze 12
hours, each of 10 27. Proximity of Winter
degrees, into the shortest Solstice and Perihelion
night. When the day also
Earth reaches perihelionthe point in its orbit when
became divided, the
hours of night and day its closest to the Sunbetween January 2 and 5,
were of unequal length, depending on the year. Thats about two weeks after
and the system of so- the December solstice, December 21 or 22. Thus win-
called unequal hours, ter begins in the Northern Hemisphere at about the
12 each for night and time that the Earth is nearest the Sun. Is there a reason
day, lasted well into the why the times of solstice and perihelion are so close, or
Middle Ages, coexisting is this a coincidence?
with another reckoning
of the equal hours.
28. Earths Speed
Q & A, S CIENCE T IMES ,
T HE N EW YORK T IMES , The time interval required for Earth to travel from the
D ECEMBER 13, 1983
autumnal equinox to the vernal equinox (approximately
179 days) is less than the time interval from the vernal to
You must remember this,
the autumnal equinox (roughly 186 days). Why?
A kiss is still a kiss,
A sigh is just a sigh,
The fundamental things 29. The Equinox Displaced
apply,
At the time of the spring equinox (usually March 20) or
As time goes by.
the fall equinox (September 22 or 23), night and day
H ERMAN H UPFELD,
A S T IME G OES B Y are supposed to be of equal duration. But according to
the almanacs of sunrise and sunset times, on the dates
of the equinoxes, daytime is longer by 8 to 10 minutes.
How come?

12 Mad about Modern Physics


30. The Dark Days of December The minute first
appeared as a division of
At latitude 40 degrees north, earliest sunset occurs on the hour about A.D. 1320
about December 8 and latest sunrise on about January in Paris editions of the
5. The shortest day of the year, the winter solstice, is so-called Alfonsine
December 21 or 22. Why are all these dates not the Mean Motion Tables,
same? sponsored by King
Alfonso the Wise of
Spain. But the idea of
31. Days of the Year the minute was implicit
all the time in a method
The length of the year (i.e., the interval of time between of reckoning used by
two successive passages of Earth through the same early astronomers. They
point in its orbit) is about 365.2422 days. How many employed a system of
entire rotations on its own axis does Earth execute dur- sexagesimal fractions,
ing that time? first devised by the
Babylonians, based on
successive powers of
32. Leap Years 60. Any unit could be
divided into 60 parts;
Every four years, in years divisible by four, is a leap
these were called in
year, when an extra day is added to the month of Feb- Latin partes minutae
ruary, except years divisible by 100. For example, primae, or first very
1700, 1800, and 1900 were not leap years, yet 2000 small parts, yielding the
was a leap year. Why? word minute. A minute
in turn was eventually
divided into 60 partes
33. Full Moons minutae secundae,
hence the word second.
Is the interval of time between one full Moon and the
Q & A, S CIENCE T IMES ,
next equal to 28 days? T HE N EW YORK T IMES ,
D ECEMBER 13, 1983

34. Moon Time


Cheryl is sitting at a desk in an ofce and the clock
When it comes to
shows 12:20 and the Moon is seen through the window procrastinating, I do it
as a thin crescent with the open side pointing down- right away!
ward to the right. What do you make of this scene? A NONYMOUS
Where could the Sun be?

Does Anybody Really Know What Time It Is? 13


In Wicca, February 2
(Groundhog Day) is one
of the four greater
sabbats that divide the
year at the midpoints
between the solstices
and equinoxes.
12:20
Sundials tell Sun time
while clocks tell mean
time. The true Sun
leads or lags the mean
Sun, crossing the
meridian from 16 min- 35. Lunar Calendar
utes, 25 seconds ear-
lier than the mean Sun Although there have been numerous calendars over the
(in early November) to millennia of civilizations, they fall into two basic types,
14 minutes, 20 seconds solar and lunar calendars. Today, while practically every-
later (in February). Only one uses the solar calendar with 365.2422 days per trop-
on or about April 16, ical year, rice farmers in many parts of the world
June 14, September 2, continue to use the lunar calendar based on a 29.53-day
and December 25 are lunar month. Can you gure out a scientic reason why?
the true and mean Suns
together as they cross
the meridian. 36. The Sandglass
For a sandglass timer one could simply have a straight
glass or plastic tube with equally spaced markings and
The angle between the
then the whole tube would be inverted to start the time
Equator and the ecliptic
measurement. Why do ruled sandglasses have a tapered
(i.e., the plane of Earths
orbit), also known as the hourglass shape instead?
tilt of the globe, was
23 26' 32" in 2002. 37. Old Watch
Through the ages, this
value varies between Lenni has an old mechanical watch in pristine condi-
21 and 28. At present tion that has an internal balance wheel that operates
it goes down by 0.47" perfectly. She takes a drive into the mountains. Will the
per year. watch run fast or slow?

14 Mad about Modern Physics


38. Reading a Digital Timer More people are born on
October 5 in the United
Many digital timers show the elapsed time to one- States than on any
hundredth of a second. What is the minimum uncer- other day. Not so sur-
tainty in the value? What value should be reported? prising, as conception
would have fallen on
New Years Eve.
39. Eternal Clocks?
There are laser and atomic clocks in special laboratory
environments that are accurate to one second in 300
If 23 students are in a
million years! Yet their lifetimes are typically less than classroom and you pick
30 years. Some wristwatches run longer! There are two at random, the
mechanical clocks in development that could last about probability that their
10,000 years! But they would need periodic winding. birthdays (month and
Why do the laser and atomic clocks have such short day) match is about
lifetimes? How might one build a mechanical clock that 1/365. The probability
would survive so long? that at least two of the
23 have the same birth
date, however, is a trifle
40. Room Light better than 12. The
Suppose there is a photodetector with a ash lamp at reason is that now there
the exact center of a 3 m 3 m 3 m dark, barren are 1 + 2 + 3 + . . . + 22
= 253 possible match-
room with reective walls. The ash lamp ashes for
ing pairs.
one nanosecond. For simplicity, assume that the light is
M ARTIN G ARDNER , M ATHE -
emitted isotropically in all directions when the lamp MATICAL G AMES , S CIENTIFIC
ashes. If the photodetector simply sums the light from A MERIC AN (O CTOBER 1972)
all directions, what is its recorded intensity versus time?
If the photodetector is an array capable of discerning
different angular directions, what is the intensity versus There are 365 days in
time for several different directions? Suppose the lamp the year. Note the
ashes for one microsecond. What now? following:
365 = 102 + 112 + 122
41. Right to Left Driving Switch = 132 + 142
Coincidence? Preestab-
Suppose you live in a country in which the driving is
lished harmony? You be
on the right and there is to be a change to driving the judge!
on the left. If highways with on-ramps and off-ramps,

Does Anybody Really Know What Time It Is? 15


Studying mid- and so on are built for driving on the right, will they
twentieth-century work equally well for driving on the left? Of course,
scientists, psychologist we must assume the same patterns of driving speeds
Bernice T. Eiduson as before.
found a disproportionate
number who had been
confined to their beds 42. Light Clock
for large amounts of
time by childhood ill- Some museums and labora-
tories have a light clock with Mirror
nesses. During these
travails, they searched two parallel mirrors and a
for resources within pulse of light bouncing back
themselves and became and forth repeatedly, retrac-
comfortable being by ing the same path over and
themselves; most over, keeping very accurate
turned to reading, and time as each complete tran-
through reading they sit of the light pulse is
developed a bent for Mirror
detected and counted. The
intellectual work. Not mirror separation is usually
very good at sports,
about a meter or less, so a
unfit by illness to com-
very large number of reections occur during each sec-
pete in childhood games,
ond of time. Suppose this light clock is moved sideways
they remained emotion-
ally fragile throughout parallel to the mirrors at a constant velocity, and
life, deriving satisfaction assume that the light will continue to reect off both
mostly from intense mirrors during this sideward movement. Will the clock
involvement in science. continue to keep accurate time?
TOM S HACHTMAN , A BSOLUTE
Z ERO AND THE C ONQUEST
OF C OLD
43. Time Reversal
Frame 1 Frame 4

16 Mad about Modern Physics


A movie is made showing successive frames for an In a Proustian moment
object accelerating downward. If the sequence is run an unexpected smell or
backward, the object accelerates (a) upward or (b) taste or perhaps a song
downward. Explain. from your past can
unleash in you a raging
torrent of realistic and
44. Molecular Clock graphic memory. The
phrase recalls a scene
Different species of organisms have enormous regions in Marcel Prousts
of DNA that are the same or very similar. Humans and Remembrance of
chimpanzees, for example, share about 98 percent of Things Past when a
their DNA. We share much less of our DNA with madeleine cake (a
rodents and amphibians and insects. small, rich cookie-like
In a general way, the percentage of shared DNA pastry) enables the
might be a means to establish a molecular clockthat narrator to experience
is, the more DNA that is shared, the more recent was the past completely as
the separation of the family tree. And, if by accident, a simultaneous part of
the changes in the DNA happened to proceed at a com- his present existence:
And suddenly the mem-
mon rate, then one could set up a timeline also.
ory revealed itself: The
However, the genetic changes do not occur with
taste was that of the
any regularity. Why not? little piece of madeleine
which on Sunday morn-
ings at Combray
45. SAD (because on those
Most animals experience dramatic seasonal cycles: they mornings I did not go
migrate, hibernate, mate, and molt at specic times of out before mass), when
the year. These cycles appear to be hardwired; they I went to say good
occur even when the temperature is held constant and morning to her in her
bedroom, my aunt
the light and dark periods are varied. But humans are
Leonie used to give me,
among the least seasonally sensitive creatures, having
dipping it first in her
only a vestige of seasonal effects known as seasonal own cup of tea or
affective disorder (SAD), an extremely mild version of tisane.
the cyclical responses animals experience. Only about 5
percent of adults overtly sense the seasonal changes and
suffer from SAD during the winter days of longer dark-
ness. Amazingly, light therapylooking into a light that
mimics sunlightor merely sleeping until dawn helps

Does Anybody Really Know What Time It Is? 17


Now he has departed the people with SAD in northern latitudes. Would these
from this strange world therapies be effective on people living at the Equator?
a little ahead of me.
That signifies nothing.
For us believing physi- 46. Two Metronomes
cists, the distinction
Suppose the timekeeping abilities of two identical
between past, present,
and future is only a metronomes are compared over several hours. They
stubbornly persistent will drift faster or slower at different rates. When both
illusion. metronomes are placed on a skateboard that moves
A LBERT E INSTEIN ON LIFE - freely horizontally, their drifts change gradually as they
LONG FRIEND M ICHELE B ESSO, tend to synchronize. Each metronome has been sub-
IN A LETTER OF CONDOLENCE
TO THE B ESSO FAMILY, M ARCH
jected to the driving force of the other, the result being
21, 1955, LESS THAN A MONTH the phenomenon called phase-locking or mode-
BEFORE HIS OWN DEATH . A LICE
locking. Suppose now that each metronome on the
C ALAPRICE , T HE E XPANDED
QUOTABLE E INSTEIN skateboard begins with different initial conditions, but
one of the two metronomes is driven by perturbations
that uctuate randomly in time. Can the metronomes
become synchronized?
HOW TO FIND NORTH
USING A WATCH
In the Northern Hemi- 47. Time Symmetry
sphere, hold the watch The fundamental equations of physicsat least those
horizontal and point the that derive from symmetries in natureall exhibit time
hour hand at the Sun. symmetry because they are second-order differential
Bisect the angle
equations. Newtons second law and Maxwells equa-
between the hour hand
tions are immediate examples. However, one can con-
and the 12 oclock mark
sider time running forward or backward. Even
to get the north-south
line. If your watch is set general-relativity equations formulated in tensor math-
on daylight saving time, ematics exhibit time symmetry. Assuming that all these
use the midway point equations are correct, must nature at its most funda-
between the hour hand mental level obey time symmetry? (Note: Entropy rela-
and one oclock. The tions are not derived from a fundamental symmetry
farther you are from and therefore are excluded.)
the Equator, the more
accurate this method
will be.

18 Mad about Modern Physics


3 Crazy
Circles
S PACE IS RELATED TO POSITION , DISTANCE , AND

size and has its own paradoxes and inuences. We


live in a space of three dimensions, but our ability to visual-
ize three-dimensional relationships among objects is not as
easy as judging distance. Our brain activity relies on neural
connections in a 3-D biomass that would probably become
moronic if limited to two dimensions. However, robots usu-
ally operate in our 3-D space by following computer pro-
grams that maneuver in multidimensional configuration
spaces that often far exceed three dimensions. Recent theo-
retical research in quantum physics hints that the natural
world may be as large as 11-dimensional, with seven dimen-
sions curled up too small for our senses, leaving the four
dimensions of space-time. In this chapter we have created a
mixture of familiar challenges and many new ones regarding
space in preparation for a later chapter on the space-time of
the special theory of relativity.

19
48. Spider and Fly
A Pythagorean triplet is On a plane the shortest distance between two points
a set of three numbers is a straight line. Suppose a spider sits on a cube and
that describes the sides wants to catch a y sitting on the opposite face. How
of a right triangle. would you determine the path of shortest distance for
Pythagoras invented his the spider to crawl on the surface to catch the y?
theorem around 550
B.C., but the Babyloni-
ans had catalogued per- 49. Moon Distance
haps hundreds of In measuring the length of a 1-meter table with a
triplets by 2000 B.C., meterstick to within 0.1 millimeter, the uncertainty in
long before Pythagoras.
the measurement is one part in ten thousand. Meter-
One of the triplets the
sticks, however, are usually inconvenient for measuring
Babylonians found is
the distance to the Moon. Instead, a laser light pulse
the enormous
3,367:3,456:4,825. can be reected from a stationary corner reector on
D ICK T ERESI , L OST the Moon similar to the reectors on bicycles, and the
D ISCOVERIES : T HE A NCIENT R OOTS total duration of the pulse from Earth to Moon and
OF M ODERN S CIENCE FROM THE
back to Earth again is timed. What do you estimate for
B ABYLONIANS TO THE M AYA
the uncertainty in the measurement for the Moons dis-
tance? Which determination would you expect to have
the greater distance uncertainty, the table length or the
In the hallowed groves distance to the Moon?
of the academe they
whisper the tale of a
physicist who spent
50. Ideal Billiards Table
the first half of his life
trying to become
famous, at which he
failed; then spent the
second half of his life
trying to convince him-
self it wasnt important
to be famous, at which
he also failed.

20 Mad about Modern Physics


Suppose you have an ideal rectangular billiards table on In the Conics of Apollo-
which a ball collides with any wall (called the cushion) nius, the words ellipse
so that the angles of incidence and reection are equal. (defect), parabola
Let there be pockets at the corners only. Describe how (equality), and hyper-
to shoot a given ball into a specic corner pocket with bola (excess) were
either zero, one, two, or three banks of the ball. applied to the three
curves now known by
these names because of
51. Wallpaper Geometry the relationships y 2 <
px , y 2 = px , and y 2 >
Some of the old video games used an interesting but
px , respectively, where
simple visual technique to extend the playing eld. A
p is the parameter
character running off the right side of the screen then (latus rectum) of the
entered the left side while the background scenery curve which is so placed
remained xed. That is, the right side edge is matched upon a coordinate sys-
to the left side edge, and the top and bottom are tem that a vertex is at
matched also. One could even have a rectangular array the origin, and the axis
of video screens, each right edge matched to a left of the curve lies along
edge, etc., each screen showing the same image. Faster the axis of abscissas.
systems later came along, and the scenery moved Hence one can see that
instead, and these 2-D views were eventually replaced Apollonius applied the
by 3-D views. name ellipse to indicate
Consider now a 3-D regular array of cubes touch- not a defective circle but
a defective parabola.
ing face to face and top to bottom, the 3-D space ana-
C ARL B. B OYER , L ETTERS ,
log to the old style 2-D video game. Let opposite cube S CIENTIFIC A MERIC AN
faces be matched and imagine that these face surfaces (F EBRUARY 1960)
are invisible. You are standing in one cube inside this
space and look to your right. Behold! You see yourself!
I put tape on the mirrors
in my house so I dont
accidentally walk
through into another
dimension.
S TEVEN W RIGHT, C OMEDIAN
You are in
this cube.

Crazy Circles 21
The Chinese mathe- What exactly do you see? What do you see when look-
matician Liu Hui calcu- ing upward?
lated a value for
(3.1416) in A.D. 200
that remained the most 52. Space-Filling Geometry
accurate estimation for
a thousand years.
D ICK T ERESI , L OST
D ISCOVERIES : T HE A NCIENT
R OOTS OF M ODERN S CIENCE
FROM THE B ABYLONIANS
TO THE M AYA

FOUR-DIMENSIONAL
GEOMETRY IN THE BIBLE?
St. Pauls Letter to the
Cubes can be placed next to each other in three direc-
Ephesians contains the
tions to ll all of 3-D space. Regular octahedrons can
following passage: that
you, being rooted and ll 3-D space also. Spheres of the same radius cannot.
grounded in love, may Can regular tetrahedrons ll all of 3-D space and leave
be able to comprehend no gaps? Can regular dodecahedrons and regular icosa-
with all the saints what hedrons?
is the width and length
and depth and height
(Ephesians 3: 1718). 53. Archimedes Gravestone
M ARTIN G ARDNER , M ATHE - Archimedes gravestone is said to have a sphere inside
MATICAL G AMES , S CIENTIFIC
A MERIC AN (S EPTEMBER 1975)
a cylinder etched into the stone as well as the symbol .
How are the two 3-D objects related if they have the
same radius? And why are they on his gravestone?
A SILLY SYLLOGISM
Nothing is better than
eternal life;
A salami sandwich is
better than nothing;
Therefore, a salami
sandwich is better
than eternal life!

22 Mad about Modern Physics


54. Brain Connections THE LATE APPEARANCE IN
ENGLISH OF THE WORD
The human brain has more than 100 billion neurons, SCIENTIST
with each neuron receiving input signals from 10 to In 1840 William Whewell
1,000 other neurons. Schematic representations of noted that there was no
these connections in the brain always show an incredi- simple and natural way
ble web of lines representing the neurons, either as a to refer to a cultivator
2-D or a 3-D image. Suppose you created a scaled- of science in general.
down computer model of this human brain using only He was, he concluded,
1 million neurons in a 3-D space. On average, how inclined to call him a
many input connections would each neuron have? scientist. Before
What is the surprise here? Whewell scientists
tended to refer to each
other as philosophers,
55. Configuration Space or more fully, as natural
philosophers. For this
Suppose we have a robotic arm that mimics the move-
reason Newtons treatise
ments of a persons arm. The arm exists in the familiar
on mathematical physics
3-D physical space. Consider a simplication of the was given the title The
robotic arm that assumes just three connected parts: Mathematical Principles
upper arm, forearm, and hand, all in the shape of of Natural Philosophy
straight rods that are connected. The body of the robot, (1687).
including the shoulder, remains xed in position. We
wish to have the robotic arm touch a particular point-
like object in the room. How many numbers are
required in a computer program to describe the arm
position? The Indian mathematician
Srinivasa Ramanujan
(18871920) discovered
56. Farmer Chasing a Goose an approximation to
that is remarkable for its
Farmers know that to catch a stray goose one does not
precision and concise-
run after the goose in an open eld. A better strategy is
ness: (2143/22)1/4 =
to corner the goose. However, suppose the farmer and
3.14159265258 . . .
the goose are in an open eld and they both run with (to be compared with =
the same speed, V, to provide us with some semblance 3.14159265358 . . .).
of fair play. Furthermore, restrict the farmer to chasing
the goose along the instantaneous line of sight to the
goose. When will the farmer catch the goose?

Crazy Circles 23
Stiglers law of 57. A Spooky Refrigerator
eponymy, formulated by
statistician Stephen Christina notices that food is disappearing from her
Stigler, states that no refrigerator and yet her surveillance camera shows that
scientific discovery is no one is opening the door. Suppose our 3-D spatial
named after its original world were really a 4-D spatial world, but we did not
discoverer. Journalist know anything about the existence of the fourth spatial
Jim Holt points out that dimension. There is still the single time dimension.
Stiglers law itself is Could a 4-D being remove food from her 3-D refriger-
self-confirming, given ator without opening the refrigerator door?
that Stigler admits that
it was discovered by
someone else: Robert 58. Fractional Dimensions?
Merton, a sociologist of
A point has zero dimensions. A line has one dimension.
science. The most noto-
A plane has two dimensions. Space has three dimensions.
rious example of
Stiglers law is probably Can something have 1.585 . . . spatial dimensions?
the Pythagorean theo-
rem, widely known by 59. Platonic Solids
the ancient Egyptians,
Babylonians, and Indi-
ans long before
Pythagoras.
A DAPTED FROM J IM H OLT,
M ISTAKEN I DENTITY T HEORY,
L INGUA F RANC A (M ARCH 2000)

DIVINE MADNESS
The word theory comes
from the Greek word
theoria, meaning
ecstatic contemplation
of the truth, as exem-
plified in Platos belief
that the greatest truths There are five 3-D regular polyhedrons called the
are those that come to Platonic solids: the regular tetrahedron (4 faces), the
us through divine regular hexahedron (cube), the regular octahedron (8
madness. faces), the regular dodecahedron (12 faces), and the

24 Mad about Modern Physics


regular icosahedron (20 faces). All these solids have a The size of the Moon
twofold rotational symmetry axis through the center of compared to the Earth
each edgethat is, a rotation about this axis by 180 is 3:11 (with accuracy of
degrees leaves the object looking the same as the initial 99.9 percent). This
view. But the regular tetrahedron does not have inver- EarthMoon proportion
sion symmetry. is also precisely invoked
by our two planetary
If we intersect two identical regular tetrahedrons so
neighbors, Venus and
their centers coincide, can the composite object have a
Mars. The closest : far-
twofold rotational symmetry axis? Can it have inver-
thest distance ratio that
sion symmetry? each experiences of the
other is, incredibly, 3:11
60. Intersecting Spheres (with accuracy of 99.9
percent). Quite by
chance, 3:11 is 27.3
percent, and the Moon
orbits the Earth every
27.3 days, also the
average rotation period
of a sunspot.
J OHN M ARTINEAU ,
A L ITTLE B OOK OF C OINCIDENCE

If in 2-D we intersect two circles (called one-spheres by


mathematicians), the intersection is either a point, two The Roman numeral
representing five,
points, or a circle. In 3-D the intersection of two
symbolized by the letter
spheres (each called a two-sphere) will be either a
V, derives from the
point, a circle, or a sphere. What can the intersection of
shape of the space
two three-spheres be? And three three-spheres? between the open
thumb and index finger.
61. Arm Contortions The Roman numeral for
ten, the letter X, is
Normally, the rotation of an object about a xed axis actually two Vs.
by 360 degrees brings the object back to its initial ori-
entation. However, Barbara has the agility to do the
following double rotation. She places a small object or
book in her right hand, holding the object horizontal

Crazy Circles 25
The biblical approxima- and noting its orientation in the room. While imagining
tion of is given in a vertical axis from oor to ceiling, the book is moved
1 Kings 7:23 and is inward rst and then under the upper arm, keeping the
repeated in 2 Chroni- book horizontal and rotating the object completely
cles 4:2. Both verses around this vertical axis back to its initial position. Her
speak of a circular sea arm is now twisted. Can she untwist by rotating her
of cast bronze with a
arm a second time in the same direction?
diameter of 10 cubits
and a circumference of
30. The Greeks used a
more accurate value of
22/7 (error 0.04 per-
cent), and the Egyptians
used a ratio of two
squares 256/81 (error
0.6 percent).

The other day, I was


walking my dog around
my building . . . on the
ledge. Some people are
afraid of heights. Not
me, I am afraid of
widths.
S TEVEN W RIGHT, C OMEDIAN

R. G. Duggleby, a bio-
chemist at the Univer-
sity of Ottawa, found
that the sum of to the
fourth power (97.40909
. . .) and to the fifth
power (306.01968 . . .)
is e (i.e., 2.7182818 . . .)
to the sixth power
(403.42879 . . .),
correct to four decimal
places!

26 Mad about Modern Physics


62. The Rotating Cup The ancient composers
of Vedic literature in
Place a cup with a handle on a shelf at eye height. Now India had to develop a
walk in a straight line at a nearly constant speed past method of evaluating
the cup, all the while rotating your head to observe the square roots. The tech-
orientation of the cup. Notice what you see. The cup nique apparently
appears to rotate in the direction opposite your walk- evolved from a need to
ing direction, at rst very slowly, then quickly, then double the size of a
slowly again. Now consider yourself to be stationary square altar. One needs
and imagine the cup itself moving past in a straight line a square whose sides
with constant speed. You could try to demonstrate this are the square root of
2. In the Sulbasutras, a
with the cup in your hand. What do you see now?
collection that dictates
the shapes and areas of
63. Space and Time Together altars and the location
of the sacred fires, the
To explain Einsteins 1905 special theory of relativity square root of 2 is
and Minkowskis 1908 unication that combines three stated as 1.414215 . . . ,
space dimensions and one time dimension into a four- an amazingly accurate
dimensional space-time continuum, most introductory value! The Sulbasutras
physics textbooks use a four-dimensional coordinate were written between
system, with three real coordinates for space and one 800 and 500 B.C.,
imaginary coordinate for the time coordinate. Why not making them at least as
four real coordinates? Why not have three imaginary old as the earliest
space coordinates and one real-time coodinate? Greek mathematics.
The Greeks, however,
had no positional nota-
64. Space > 3-D? tion system. Hence
their approximations of
Can you provide arguments for why space has three
the square roots were
dimensions? Hint: Are planetary orbits stable in a space
rather crude.
of n dimensions, where n > 3? Is the hydrogen atom
G EORGE G HEVERGHESE
stable when n > 3? J OSEPH , T HE C REST OF THE
P EACOCK : N ON -E UROPEAN
R OOTS OF M ATHEMATICS

Crazy Circles 27
4 Fly Me to
the Moon
W E LIVE IN A WORLD OBEYING THE RULES OF

nature. But this natural world described by physics


and the other sciences can be superseded and replaced by the
imagination of the human mind. The articial worlds cre-
ated in the many forms of literature and in audio and visual
renderings today cast a powerful inuence on the minds of
everyone in the modern world. In fact, more people prefer to
live in these articial fantasy worlds than in the real world
than are willing to admit. In these challenges we focus on
some of the fuzzy science prevalent in movies and televi-
sion shows. In certain ways, the awareness of the correct sci-
ence can enhance your enjoyment of the entertainment
product, just like knowing how a bee communicates to the
other bees in its hive enhances the beauty of the bee itself.

29
The anagram of 65. Gunfight
MOON STARERS is
ASTRONOMERS. Some TV programs and lms have high drama scenes
based on a victim being shot by the pursuer and being
blown backward a meter or two by the projectile
impact. Is this dramatic response Hollywood hype, or
Jonathan Swifts is there good physics here?
Gullivers Travels,
published in 1726, 66. Body Cushion
describes Gullivers
many adventures, A fall from a height of several stories onto pavement or
including his Voyage to even onto a lawn will produce serious injuries or even
Laputa. Gulliver learns death. Yet we have seen the movie hero going over the
that the scientists there edge of a roof holding another human body in position
discovered two moons just beneath to cushion the fall on impact. Certainly,
of Mars, which revolve collision with this second body is better than direct col-
around the planet at lision with the ground. What do you think about the
distances from its cen- advantages here?
ter equal to 3 and 5
Martian diameters.
When the moons of 67. Cartoon Free Fall
Mars were discovered So many of us in our youth learned the laws of nature
by Asaph Hall in 1877, it from cartoons. Some of us are still learning from
turned out that Swift cartoons! The cartoon character steps forward off
not only had the number a cliff and remains there in
of the moons right, but
suspension until realizing the
he also placed them
situation, then the acceleration
close to the actual
downward begins. As you
distances: 1.4 and 3.5
diameters of Mars. recall the scene, what violations
The two moons, named of physics can you discern?
Phobos and Deimos, are
tiny. Phobos, which 68. Silhouette
measures 27 by 19 kilo-
of Passage
meters, is shaped rather
like a potato. Deimos, When a cartoon character
too, is oddly shaped, smashes through a solid wall or
and measures 15 by 11 other object, we see the perfo-
kilometers. ration as the crisp outline of the

30 Mad about Modern Physics


character. What would a condensed matter physicist Jules Vernes From the
say about this cookie cutter type of material response? Earth to the Moon was
published in 1865.
Breaking with literary
69. Artificial Gravity tradition, which called
for recounting such a
voyage only as an imagi-
nary undertaking, Verne
based his account on an
extrapolation of contem-
porary scientific princi-
ples. The resulting
prophetic qualities of
this novel are uncanny.
For instance, Verne
We all know that a body will tend to oat around in chose a launch site not
a space station orbiting Earth or in a spaceship cruising far from Cape Canaveral
in Florida; he also gave
at a constant velocity with respect to the stars. Some
his readers the initial
lms depict a dumbbell-shaped space station rotating
velocity required for
about an axis through its middle perpendicular to the escaping the earths
long axis in order to provide articial gravity. What gravitation. In the sequel,
interesting behavior patterns might be experienced by Around the Moon, Verne
an astronaut who walks across the axis from one end correctly described the
to the other? effects of weightless-
ness, and he even
pictured the space-
70. Small crafts fiery reentry and
Wings splashdown in the
Pacific Ocean
Space heroes who
amazingly, at a site just
visit other planets three miles from where
have encountered alien Apollo 11 landed on its
beings who suspend them- return from the Moon in
selves in the air with two 1969.
small beating wings, each A RTHUR E VANS AND RON
about 40 centimeters long, M ILLER , J ULES V ERNE ,
M ISUNDERSTOOD V ISIONARY,
attached to their backs. These S CIENTIFIC A MERIC AN
characters are less than a (A PRIL 1997)

Fly Me to the Moon 31


The first public suggestion meter tall but probably have a mass of at least 20 kilo-
that Abraham Lincoln be grams. Could these wings sufce?
the Republican candidate
for president is believed to
be a letter written Novem- 71. Shrunken People
ber 6, 1858, and published Suppose someone is shrunken by some gimmick in the
in the Cincinnati Gazette.
movies. Lets say that you suffer this consequence and
The writer, Israel Green
are now 100 times smaller in all dimensions. Actually,
(a druggist in Findlay,
Ohio), proposed a ticket there is a lot of space between the atoms and molecules
of Lincoln for president, of our bodies, but lets ignore any increase in repulsive
and John Kennedy for forces, etc., and assume that this shrinkage can be
vice president. The pro- done. What does physics tell you will be a major prob-
posed Kennedy was John lem as you walk?
Pendleton Kennedy of
Maryland, a prominent
author and politician who 72. Spaceship Designs
had been Millard Fillmores The simple but effective spaceships of Buck Rogers and
Secretary of the Navy.
Flash Gordon have been superseded by flashy new
M ARTIN G ARDNER ,
T HE M AGIC N UMBERS OF
designs with interesting shapes, sizes, and abilities. The
D R . M ATRIX advent of the space age in the 1950s brought about a
heightened awareness of the practical physics charac-
According to NASA, there terizing a successful rocket or spaceship. Yet today,
is only one proven case of more than 50 years later, the ingenuity of the movie
a human who was hit by an industry continues to defy the laws of physics. We see
object from outer space, the latest nuclear-powered spaceships operating in
Lannie Williams of Tulsa, space coming in for a landing on Earth (or other com-
Oklahoma. In 1997, when parable planet) at a spaceport and then taking off for
Ms. Williams was power space a little while later in the same ship from the same
walking, she felt a tap on
spaceport. Why cant we do this feat with present-day
the back of her shoulder.
space vehicles?
As something rolled off her
shoulder, she heard it hit
the sidewalk with a metallic 73. Warp Speed
thud. When she looked
back, she saw a mangled Spaceships are known for their ability to turn on their
lump of metal, which later warp drives to accelerate to speeds beyond the speed of
turned out to be a piece light. Can present-day physics conceptually explain this
of a U.S. rocket. capability?

32 Mad about Modern Physics


74. North Pole Ice Melt H. G. Wellss 1914 novel
The World Set Free, a
Environmental disasters have always been popular speculative history of
with lmmakers. In recent years, the trend has been the future, contains the
toward disasters on a global scale because the public following sentence:
has become more aware of global environmental Nothing could have
problems. If there is a global warming trend, there been more obvious to
could be much ice melting at the poles of Earth. Some the people of the early
lms have portrayed seacoasts being inundated by the twentieth century than
rising water level. What would you predict for the sea the rapidity with which
level change if the ice only at the North Pole melted war was becoming
impossible. And as cer-
completely?
tainly they did not see
it. They did not see it
until the atomic bombs
75. Lightning and Thunder burst in their fumbling
hands.
We see the ash of distant lightning and hear its thun-
der roll simultaneously in the movie. But we all know
that the lightning ash arrives before the thunder in the
real world, there being about ve seconds of sound
delay for each mile of distance to the lightning. Suppose How would you suspend
you were in charge of a battle scene in a war movie. 500,000 pounds of
When editing the scenes of the explosions on the bat- water in the air with no
tleeld, how would you ensure the correct experience visible means of sup-
for the theater patron? port? (Answer: build a
cloud!)
B OB M ILLER , A RTIST

76. Explosions in Outer Space


Explosions in outer space on the big screen are magnif-
icent to behold. Brilliant colors of stuff shooting On November 10, 1907,
outward in all directions, decreasing their density as the magazine section of
the inverse square of the distance. The sound of the the New York Times
explosion rocks the spaceship with a thundering roar included the headline
just as the light ash is rst seen. Finally, bits of debris Martians Probably
scream past. What do you think about this space Superior to Us.
physics?

Fly Me to the Moon 33


The American socialist 77. Space Wars
writer Edward Bellamy
(18501898), in his One space battlecruiser after another shoots powerful
best-selling 1888 novel laser beams that destroy the enemys space battlecruiser.
Looking Backward: We see the powerful red laser beams strike the oppo-
20001887, antici- nent, and we hear the explosion as the object blows
pated, by seven years, apart. What wonderful physics can be learned here?
the motif of time travel
H. G. Wells was to use
in his Time Machine 78. Security Lasers
(1895). Bellamys vision
of the future included, Quite often the drama in a crime movie or an adventure
among other things, movie is enhanced by having crisscrossed visible laser
shopping malls and beams around the item to be protected from theft. The
credit cards (his own thief must avoid intersecting these beams during the
expression). escapade to steal the item; otherwise a security alarm
will notify the appropriate authorities and the thief will
be caught. If you were the movies director, how would
you make this scene to ensure good physics?

PERVERSITY OF
INANIMATE OBJECTS? 79. Bullet Fireworks
As many a scientist Bullets bounce everywhere. The bad guys shoot a
concluded with con- lengthy burst of submachinegun re as the hero runs
sternation, there is a through an industrial plant. The bullets impacting on
reason why there is steel railings, for example, give off bright ashes of
a word demon in
light. This scene is a dramatic event for almost anyone
demonstrations.
watching the hero in a time of great peril. What can
you say about the physics here?

80. Internet Gaming


Were the sun and moon
to doubt For years people have been playing live time games
Theyd immediately go over the Internet. If the game is checkers or poker, for
out. example, each player must take his or her turn in
W ILLIAM B LAKE proper order, so short delays are not a problem. Even

34 Mad about Modern Physics


when the game is a world-domination board game with PERIODIC CLEANUP
multiple players, each player can submit moves at any OPERATION
time before the deadline. But many video games require Every time the solar
simultaneous play by several players, so delays can cycle peaks, it causes
mean life or death for a players combatant in a shoot- Earths atmosphere to
em-up type of action game. One can hear comments expand and pull in low-
by some action game players that they tried to make orbiting debris, which
their move but the Internet was too slow. What is the burn up on reentry.
possible truth here?
Jules Verne died in
1905. A memorial
81. Cartoon Stretching sculpture placed over
Objects in cartoons are stretched and squeezed into his grave depicts Verne
amazing distortions and then released. Some of the rising from his tomb,
characters suffer the same fate. When the body material one arm reaching
of a cartoon character is being pulled, we often see the toward the stars. Some
two decades later an
part closer to the applied force stretch rst and then the
American periodical
rest follow with a small time delay. For example, a car-
called Amazing Stories
toon dog may pull on a characters leg, which we see
the first magazine
being stretched while the torso remains normal, until exclusively to feature
nally the torso stretches, the arms stretch, and the tales of science and
character releases his or her handgrip from the door- adventureused a rep-
way. Using some physics concepts, what can you say resentation of Vernes
about the speed of sound in a cartoon characters body? tomb as a logo. To
describe these narra-
tives, the publisher,
82. Infrared Images Hugo Gernsback, coined
the term scientifiction,
In crime dramas and in adventure lms the result of an which was later changed
infrared vision device is often reconstructed and shown to science fiction.
in a sharp greenish or black-and-white format. We see A RTHUR B. E VANS AND RON
the infrared faces of people as if they were originally M ILLER , J ULES V ERNE ,
M ISUNDERSTOOD V ISIONARY,
color images seen normally with ones eyes, but now
S CIENTIFIC A MERIC AN
these color images have been converted to black and (A PRIL 1997)
white. Is there any physics violation here in depicting
the infrared images via this process?

Fly Me to the Moon 35


One of the big uncer- 83. Light Sabers
tainties in calculating
the path of an asteroid Enemies dueling with light sabers have graced the silver
involves how much sun- screen for several decades now. Isnt this type of
light it absorbs and then weapon the most ridiculous thing youve ever seen?
reradiates as thermal
energy. Such radiation
can, over the centuries, 84. Force Fields
gently push the asteroid In battle scenes of many science ction movies we see
into a different orbit, the baddies roll up with their giant laser guns to shoot
much as a tiny rocket
the good guys, who are protected by a visibly transpar-
would. So, if scientists
ent force eld. Why do the laser beams suffer deection
in future years should
at the force eld?
conclude that a collision
looks ever more likely,
then they can probably
85. Cold Silence of Space
find ways to alter the
asteroids radiation pat- In the cold silence of space begins many a description
tern by dusting its sur- of space between planets. Can this statement survive a
face with soot or physics analysis?
powdered chalk or drap-
ing it with reflective
Mylar. Such tinkering 86. Nuclear Submarine
could be enough to
Several movies have involved an out-of-control nuclear
nudge the asteroid
reactor aboard a nuclear submarine. We are told that
safely away.
the containment vessel is about to fail and that the best
E DITORIAL D ESK ,
E NCOUNTER W ITH AN action is to move the sub several hundred meters
A STEROID, N EW YORK T IMES underwater. When the explosion occurs down there,
(A PRIL 8, 2002)
what might happen?

If you are a scientist, 87. Plutonium vs. Uranium


youre more likely to be
killed in a film than a Suppose you nd a nuclear bomb and decide to trans-
member of any other port the device to a safe hiding place. Would there
profession, including a be any difference with regard to your safety as to
Mafia hit man. whether the device is made of uranium-235 or
C ARL S AGAN plutonium-239?

36 Mad about Modern Physics


88. Nuclear Detonation In a sense, 1:Sun = Moon,
and 1:Moon = Sun!
The threat of the detonation of a hydrogen nuclear 1:365.242 = 0.0027379,
warhead by striking one with another object such as a which in days is 3 min-
missile or the shrapnel from a nearby explosion lurks in utes and 56 seconds, the
a scriptwriters creative mind for many war and adven- difference between side-
ture lms. Suppose the nuclear warhead is aboard an real and solar days, while
ICBM and is struck by an interceptor missile or the 1:27.322 = 0.0366,
warhead itself is penetrated by fast-moving BBs. What which in days is 52 min-
will happen? utes, the difference
between lunar and solar
days.
89. Fabric of Space-time ROBIN H EATH , S UN ,
M OON , & E ARTH
Conjectures about the fabric of space-time and
tears in the space-time continuum abound in science
ction movies. An entertaining 2001 lm involved a On the average there is
protagonist who derived an equation for the time and one catalogued satellite
place of a temporary tear in the fabric of space-time. that falls back to Earth
Several characters jumped off the Brooklyn Bridge uncontrolled every single
through the temporary space-time tear, acting as a day and has been since
portal to another dimension to the year 1876, and then the early 1960s. Most of
returned through the next temporary tear by jumping them vaporize high in the
off the bridge again days later. In addition, lm charac- atmosphere.
ters have used the phrase speed of gravity in an
ambiguous way. What can you say about the physics
The first sentence in
here?
H. G. Wellss 1908 novel
The War in the Air reads:
Lower Manhattan was
soon a furnace of crimson
flames, from which there
was no escape.

Fly Me to the Moon 37


5 Go Ask
Alice
P ERHAPS NO OTHER ASPECT OF TWENTIETH -

century physics has captured the imagination of the


general public more than the concepts of the special theory
of relativity (STR). Absolute time and absolute space are
forever cast aside in favor of a union of space and time into
one important entity called space-time. This four-dimen-
sional world of space-time has spawned an enormous num-
ber of conjectures about the behavior of nature. Among
these conjectures are time travel, two people aging at differ-
ent rates when one remains on Earth and the other travels on
a space journey, the ability to see the back side of an
approaching cube, and the conversion of mass into energy.
As you know, the STR is based on the idea that two
observers in different inertial reference frames must each
experience physics described by the same basic laws. Even
though these two inertial reference frames are moving with

39
a constant velocity with respect to each other, the speed
of light in a vacuum is the same for both observers. The
important quantities in STR are the invariants. For
many people, the most useful invariant is the space-
time interval , dened by 2 = c2 t 2 x2 y2 z 2.
For others, the four-momentum invariant E2 p2c2 =
m2c4 is the most useful because E0 = mc2 can be derived
directly, where the mass m is a constant, the same at all
speeds, all places, and all times. Many challenges in this
chapter test your ability to use these invariants.

40 Mad about Modern Physics


90. Spotlight Joseph Larmor in 1900,
stimulated directly by
Can a spot of light move faster than c, the speed of the Michelson-Morley
light? For example, if a lighthouse light beacon spins experiment, gave for
around at very high speed, will the spot of light seen far the first time the full
from the beacon cut across the sky with a speed greater Lorentz transformations
than 3 108 m/s? for coordinates and
time, as well as electric
and magnetic field com-
91. Quasar Velocity ponents, and showed
Quasars have been detected that have recessional veloc- that the Maxwell equa-
ities greater than the speed of light c based on the cos- tions remain exactly
invariant under these
mological relationship for the redshift z, namely, l + z =
transformations. It
exp[v/c]. That is, there are quasars with z > 3, for
has long appeared a
example. Also, to explain the present state of the uni-
historical anomaly that
verse, the inationary big bang model requires a faster- Larmors work, which
than-light expansion of space in the young universe. preceded Lorentzs by
Are these examples violations of the special theory of four years, is so little
relativity? known among physi-
cists. Earlier still, in
1897, Larmor had
92. Spaceship Approach discovered time dilation.
A spaceship is approaching Stephanie at the relativistic Woldemar Voigts paper,
speed of v/c = 0.98974. What does she see as the space- published in 1887, con-
ship nears and then passes? Hint: for simplicity, con- tains an early version of
sider a cube approaching in place of the spaceship. the Lorentz transforma-
tions that appear to be
almost the same except
93. Mass and Energy for a scale factor.
C. K ITTEL , L ARMOR AND
A symbol of the twentieth century is the famous Ein- THE P REHISTORY OF THE
stein relation between mass and energy. Here are four L ORENTZ T RANSFORMATIONS ,
possible equations: (1) E0 = mc2 (2) E = mc2 (3) E0 = A MERIC AN J OURNAL OF P HYSICS
(S EPTEMBER 1974)
m0c2 (4) E = m0c2. In the equations c is the velocity of
light, E is the total energy of a free body, E0 its rest
energy, m0 its rest mass, and m its mass.
Which of these equations expresses one of the main
consequences of the STR? Which equation was rst

Go Ask Alice 41
Albert Michelson was written by Einstein and was considered by him a con-
born in Strzelno, sequence of STR?
Poland, in 1852. The
town, about 150 miles
northwest of Warsaw, 94. Strain Gauge
was then under Pruss- A long rectangular bar of metal sits at rest in my refer-
ian rule. His parents
ence frame. The strain gauge attached to its middle
were Samuel Michelson
reads zero. Now I run in the direction parallel to the
and Rozalia Przylubska.
length of the bar at an enormous constant speed V at
Four years later the
family immigrated to nearly light speed. I measure the bar length to determine
California. Michelson, that Lorentz-Fitzgerald contraction has occurredthat
famous for the 1887 is, that the bar measures shorter than before. What
Michelson-Morley ether should the strain gauge show?
drift experiment, in
1907 became the first
naturalized American
95. Mass/Energy
citizen to win a Nobel Under certain conditions, mass can be converted into
Prize in physics. The energy la E0 = mc2. Under certain restricted condi-
Nobel committee tions, energy can materialize as mass. What is wrong in
awarded the prize for these statements?
an investigation to
determine whether
wavelengths of light 96. System of Particles
could provide a standard
A system of particles is composed of n freely moving
unit of length. The ether
drift experiment was not particles. Is the mass of this system equal to the sum of
even mentioned. the masses of the individual particles?
A DAPTED FROM D OROTHY
M ICHELSON L IVINGSTON ,
T HE M ASTER OF L IGHT : 97. Light Propagation
A B IOGRAPHY OF
A LBERT A. M ICHELSON Suppose Patricia is driving her car at nearly the speed
of light and turns on her headlights. For simplicity in
calculations, in the rest frame of an observer on the
Are not gross bodies ground the light takes one second to reach the stop sign
and light convertible 3 108 meters away. This ground observer then sees
into one another? the car reach the stop sign very soon after the initial
I SAAC N EWTON light reaches the stop sign.

42 Mad about Modern Physics


Patricia sees the light moving forward at 3 108 The notion of the
m/sec also, but she sees the stop sign approaching her dependence of mass on
at nearly light speed. Therefore she sees the arrival of velocity according to
the light ash at the stop sign and her arrival there in m/(1 v 2/c2)1/2 was
quick succession. introduced by Lorentz in
1899 and then devel-
Call the initial arrival of the light at the stop sign
oped by him and others
event A and the cars arrival event B. Will the elapsed
in the years preceding
time between events A and B be the same for the driver
Einsteins formulation
as for the observer on the ground? No, because the of special relativity in
ground observer sees both events occur at the same 1905.
location, at the stationary stop sign, so x = 0. As seen L EV B. O KUN , T HE
by Patricia, these two events occur at two different C ONCEPT OF M ASS , P HYSICS
locations separated by x 0. TODAY (J UNE 1989)

Who measures the longer time interval between


events A and B? Can you provide a conceptual argu-
ment for this nonintuitive result? If the speed of the car
is closer to the speed of light, how does the difference in Ill be so happy and
elapsed times measured by driver and ground observer proud when we are
change? together and can bring
our work on relative
motion to a successful
98. Sagnac Effect conclusion!
A LBERT E INSTEIN IN A LETTER
Suppose two identical clocks are in motion on Earths TO M ILEVA M ARIC , HIS FUTURE
Equator with constant speed v relative to Earth, one WIFE , M ARCH 27, 1901. J RGEN

moving east and one moving west around the Equator. RENN AND ROBERT SCHULMANN,
EDS ., A LBERT E INSTEIN , M ILEVA
Do they tick at the same rate? What do their elapsed M AR I C : T HE L OVE L ETTERS
times reveal when they meet again?

99. Light Flashes


A student riding in a
Suppose that a spaceship travels at constant velocity train looks up and sees
between two planets, A and B. The spaceship sends out Einstein sitting next to
a light ash in all directions every 10 minutes by its him. Excited, he asks,
own clock reading. Traveling toward B, its light ashes Excuse me, Professor.
are seen at 5-minute intervals on planet B. What is the Does Boston stop at
ash interval time as seen on planet A? One of these this train?

Go Ask Alice 43
In 1905 Einstein wrote
twenty-one reviews for
the Beibltter zu den
Annalen der Physik, a Sends flash
journal about journals. Sees flash every 10 min Sees flash
In addition to publica- every ? min every 5 min
tions written in German,
he also reviewed French A B
and Italian papers, being
familiar with both lan-
guages. Without his
work for the Beibltter,
he might have easily
missed the Festschrift
on the occasion of
Ludwig Boltzmanns possibilities is correct: 5-minute intervals; 10-minute
sixtieth birthday, which intervals; 15-minute intervals; 20-minute intervals.
included 117 contribu-
tions by prominent
authors and thus 100. Forces and Accelerations
offered an exceptionally
In Newtonian physics, an applied contact force acting
broad panorama of
on a rigid object will accelerate the object in the same
physics at the beginning
of the twentieth cen- direction as the applied force. Does this behavior hold
tury. Einstein discussed for applied contact forces in relativity physics (STR)?
three papers from this For example, if an applied contact force pushes on the
volume, and he probably same rigid object in the direction perpendicular to the
read the rest. direction of motion, will the resulting acceleration be in
A LBRECHT F LSING , A LBERT the direction of the applied contact force?
E INSTEIN : A B IOGRAPHY

101. Uniform Acceleration


Academic disputes are
vicious because so little Suppose an object starts at rest with respect to the lab
is at stake. frame and undergoes a uniform acceleration a as
A NONYMOUS measured by an observer on a spaceship moving at a
uniform velocity v with respect to the lab. In Newton-
ian mechanics, for speeds where v << c, the velocity
after t seconds in the moving frame has elapsed is

44 Mad about Modern Physics


V = a t as measured by the observer on the moving Henri Poincar, building
object. This velocity is V = v + a t after the elapsed time on Lorentzs work but
t in the lab frame, because in Newtonian physics the removing, at least for-
clocks in the different frames run at the same rates. mally, certain inconsis-
What is the velocity value in the lab frame when the tencies, arrived in 1905,
speed is allowed to become relativistic? Can the prod- and more fully in 1906,
at the expressions
uct at be greater than c in either reference frame?
E = m c2/(1 v 2/c2)1/2
and p = mv /(1 v 2/c2)1/2.
102. Long Space Journey Einstein obtained the
same relations, at the
Can a person go with a 1-g acceleration to a distant same time, on purely
location 7,000 light-years away and return without kinematic grounds. These
aging more than 40 years? That is, the bathroom scale are the well-tested and
in the spaceship must show a persons correct weight familiar expressions of
for the whole journey. Is this feat within the realm of today.
science or science ction? J. DAVID J ACKSON ,
T HE I MPACT OF S PECIAL
R ELATIVITY ON T HEORETICAL
P HYSICS , P HYSICS TODAY
103. Head to Toe (M AY 1987)

Can relativitic effects make your feet age more slowly


than your head?

Einstein simply
104. Neutrino Mass postulates what we
have deduced, with
Since their proposed existence in the 1930s, neutrinos
some difficulty and not
and antineutrinos of all three lepton families have been altogether satisfactorily,
thought to have zero mass and travel at light speed to from the fundamental
conserve energy and angular momentum in nuclear equations of the electro-
decays. In 1969 came the rst hints that at least one magnetic field.
type of neutrino can become another type of neutrino, H. A. L ORENTZ , 1906,
and a neutrino oscillation scheme was proposed. We Q UOTED IN A LBRECHT
F LSING , A LBERT E INSTEIN :
now know that muon neutrinos created in Earths A B IOGRAPHY
atmosphere can oscillate into electron neutrinos and
tau netrinos before reaching an underground detector.
Why cannot all three neutrino types still have zero
mass?

Go Ask Alice 45
Einsteins special rela- 105. Spaceship Collision
tivity paper (On the
Electrodynamics of Two spaceships, A and B, move toward one another on
Moving Bodies), pub- courses for a head-on collision. According to an
lished in 1905, observer at rest in an inertial reference frame, both
attracted very little have speed V along the x-axis. At the time of observa-
attention, perhaps tion, spaceship A is coincident with the observerthat
partly because it was is, has the same x value. Spaceship B is at a distance L
one of a number of con- away. One would like to know how much later the col-
tributions by many dif- lision occurs according to the observer and according
ferent authors in the to an observer aboard spaceship A.
general field of the
Let us propose a solution method. According to the
electrodynamics of
observer, the collision occurs when spaceship A or B
moving bodies. In the
travels L/2, half the distance between them, which
Annalen der Physik
alone there are eight requires the elapsed time T = L/2V. Put into a better
papers from 1902 up to format, three events occur:
1905 concerned with
this general problem. Event 1: X1 = 0 T1 = 0
Einstein himself always
Event 2: X2 = L T2 = 0
insisted on this aspect
of continuity, With
Event 3: X3 = L/2 T3 = L/2V
respect to the theory of
relativity it is not at all a
These same events can be specied in the inertial
question of a revolution-
(primed) frame of spaceship A as:
ary act, but of a natural
development of a line
Event 1: X1 = 0 T1 = 0
which can be pursued
through centuries. Event 2: X2 = ? T2 = ?
G ERALD H OLTON , T HEMATIC
O RIGINS OF S CIENTIFIC T HOUGHT :
K EPLER TO E INSTEIN Event 3: X3 = ? T3 = ?

106. Twin Paradox


On their twenty-first birthday, Peter leaves his twin
brother, Paul, behind on Earth and goes off in a straight
line for 7 years on his own wristwatch time (2.2 108
seconds) at 0.96 c with respect to an inertial reference

46 Mad about Modern Physics


frame at rest with respect to Earth, then reverses direc- From a historical
tion, and in another 7 years of his time returns at the perspective, Einsteins
same constant speed. Paul sees Peters wristwatch run- recognition of E = m c2
ning slower, so Peter ages
(1v
c
2/ 2) = 0.28 as much, or (where c is for celeri-
1.96 years for each direction. But Peter looks back to tas, from the Latin for
see Pauls clock running slower than his own wrist- swiftness) did not
quite come out of the
watch, so Paul should be aging slower by 0.28 as
blue. Already in 1881,
muchthat is, 1.96 years for each direction. On his
J. J. Thomson had cal-
return, Peter is surprised: I know that I aged 14 years,
culated that a charged
but Paul should have aged only 3.92 years. Why is Paul sphere behaves as if it
an old man with gray hair? had an additional
mass of amount 4/3c2
times the energy of its
Coulomb field. That set
off a quest for the
electromagnetic mass
of the electronan
effort to explain its
inertia purely in terms
of the field energy. In
1900, Poincar made
the simpler observation
that since the electro-
magnetic momentum of
radiation is 1/c2 times
the Poynting flux of
energy, radiation seems
to possess a mass
density 1/c2 times its
energy density.
WOLFGANG R INDLER ,
R ELATIVITY : S PECIAL , G ENERAL ,
AND C OSMOLOGIC AL

Go Ask Alice 47
6 Start
Me Up
E NGINEERING PHYSICS IS REALLY APPLIED PHYSICS,

but more general, with social, political, nancial, and


aesthetic issues to be considered that are often beyond the
immediate concerns of the applied scientist. We have
included the ability to understand the microscopic behavior
of atoms and their components in solid and liquids, a knowl-
edge that has begun to reap huge benets in improving the
materials and devices around us. In fact, we have entered the
era of ingenious devices and designer materials. A very small
sampling of the vast array of these advances is included in
the challenges and puzzles considered in this chapter.

49
The earliest objection on 107. Air-Driven Automobile Engine
record to Aristotles
theory of falling bodies, Can a normal four-cylinder gasoline engine actually
based on observing the operate on compressed air instead of gasoline as its
actual fall of two bodies, energy source?
is that of John Philo-
ponus (ca. 490570),
also known as John of 108. Coin Tosses
Alexandria, a Christian The behavior of many systems and materials can be bet-
philosopher, scientist, ter understood by considering the random walk of par-
and theologian. He
ticles in the system. To get some feeling for a random
writes, If you let fall
walk, consider the following exercise. Divide a group of
from the same height
two weights of which one people into two groups. Have each individual in one
is many times as heavy group toss a fair coin 256 times and write down in
as the other, you will sequence the outcome of each toss. Have each individual
see that the ratio of the in the other group write down what they would imagine
times required for the a typical sequence of 256 random tosses to be but not
motion does not depend actually do the tossing. Collect all the papers and mix
on the ratio of the them up thoroughly. Can you determine with reasonable
weights, but that the
accuracy which sets of data were obtained experimen-
difference in time is a
tally? How accurate should your selection be?
very small one. How-
ever, he was in no sense
a precursor of Stevin 109. More Coin Tosses
and Galileo, believing as
he did that in a vacuum Suppose we are really ambitious about tossing a fair
the speeds of falling coin. Indeed, suppose we toss a fair coin 1000 times,
bodies would indeed be and for each head we step one unit distance radially
in the ratio of their away from a lamppost, and for each tail we step back
weights, and ascribing radially the same unit distance. About how many times
the near equality of would you expect to be at the lamppost?
their speeds in air
entirely to the resist-
ance provided by the 110. Brownian Motor
medium. In his famous lectures, physicist Richard Feynman
J. L. R EDDING , A RISTOTLE S
T HEORY OF FALLING B ODIES ,
discussed the impossibility of violating the second law of
A MERIC AN J OURNAL OF P HYSICS thermodynamics by a ratchet mechanism. The simplest
(J UNE 1978) model for a ratchet is an overdamped Brownian particle
in an asymmetric but spatially periodic potential (with

50 Mad about Modern Physics


Movable wall Its important to learn
classical mechanics
before learning modern
physics so that you will
know how to wave your
hands correctly when
discussing things you
dont quite understand.
H OWARD G EORGI , U.S.
P HYSICIST

Asymmetric periodic potential


DISCOVERY OF
KINETIC ENERGY
asymmetry and period L). Due to the uctuating force Huygens (and, inde-
caused by the pushing molecules of the surrounding pendently, Christopher
uid or gas, the Brownian particle may overcome the Wrens) studies of rigid
potential barrier moving to the left or to the right. The colliding balls around
probabilities for both directions are equal, and on aver- 1655 led them to con-
age the particle does not move. Hence building a motor clude that there was
that turns thermal energy into mechanical work from a something special about
single heat bath is impossible. the product of mass and
But the ratchet can be turned into a so-called velocity-squared.
Brownian motor that seems to violate the second law Remarkably, adding
of thermodynamics. The idea is to turn the ratchet values of mv 2 for each
ball prior to a collision
potential on and off periodically. Under certain cir-
yielded a total that was
cumstances, this action may yield directed motion even
essentially the same
against an applied force f. Indeed, this device does
after the collision, even
work. (No pun intended!) though the velocities
Recall that a perpetuum mobile of the rst kind had changed.
violates the law of conservation of energy, while a per- E UGENE H ECHT, A N
petuum mobile of the second kind uses the free H ISTORICO -C RITICAL
ACCOUNT OF P OTENTIAL
energy around us in the form of heatthat is, random
E NERGY: I S PE REALLY REAL ?,
thermal motion of molecules and atomsto run an T HE P HYSICS T EACHER
engine without fuel. Why isnt a Brownian motor a (N OVEMBER 2003)

perpetuum mobile of the second kind?

Start Me Up 51
111. Magnetocaloric Engine

Magnet Ferrofluid
What is the origin of
the 260-day cycle in
the Mayan calendar?
According to Anthony Heat
Aveni, the 260-day Heat source sink
cycle has meaning only
in tropical latitudes,
being connected with A ferrofluid is a fluid containing small magnetic
the interval the noonday
particles that respond to an applied magnetic eld, so
sun spends north as
a ferrouid becomes magnetized in the presence of the
opposed o south of the
magnet. The diagram shows a closed tube loop con-
overhead position. These
intervals vary depending taining a ferrouid, a heat source, a strong magnet,
on the latitude, but in and a heat sink all working together to act as an engine
latitude 1412N , close to transporting the ferrouid around the closed loop. Its
the locations of the thermal efciency approaches the efciency of a Carnot
great Maya city of cycle, so demands for this device should increase.
Copan and the pre- Exactly how does this engine maintain the uid move-
Classic city of Izapa, ment around the loop? Can a solar heating system
the annual cycle divides operate in this way?
up neatly into 105-
and 260-day periods.
A DAPTED FROM A NTHONY 112. Magnetorheological Fluid
AVENI , E MPIRES OF T IME : C ALEN -
DARS , C LOCKS , AND C ULTURES In a beaker is 250 milliliters of corn oil to which has
been added about 0.5 kilogram of iron lings about 1
millimeter long. The mixture is stirred thoroughly and
a strong horseshoe magnet is brought up to straddle the
beaker. The iron lings align with the magnetic eld as
If you go against the expected to magnetize the uid mixture. What other
grain of the universe, physical property of the uid changes?
youre liable to get
splinters.
A NONYMOUS 113. Binary Fluids
The two possible phase diagrams show the miscible and
immiscible phases of a binary uid, a mixture of two

52 Mad about Modern Physics


A technician named
Miscible Miscible Richard Woodbridge III
T T coined the phrase
acoustic archaeology
in the August 1969
Immiscible issue of Proceedings
of the I.E.E.E. Wood-
bridge theorized that
Immiscible
there were many occa-
sions when sound might
0 50 100 0 50 100 innocently get scooped
out of the air and pre-
% Fluid B % Fluid B
served. For example,
when an ancient potter
kinds of uid, in a plot with axes of temperature versus typically held a flat
concentration. For example, coffee and cream are mis- stick against a rotating
cible at room temperature but oil and water are not. pot, he was accidentally
Consider the 50 percent mixture in each phase (and crudely) recording
diagram and start at a high temperature in the miscible into the clay the sounds
phase. The diagram to the left reveals that the binary around him. Woodbridge
fluids become immiscible upon being cooled, while wrote about experi-
the diagram to the right tells us that the uids become ments he performed
immiscible as the cooling proceeds but that even pulling basic noises off
further cooling brings back the miscible phase. Can a pot. Another experi-
ment involved setting up
both phase diagrams represent a real binary uid, or is
a canvas and then
one false?
talking while making
different brush strokes,
114. Baseball Bats hoping to record a
spoken word in an oil
Hitting a baseball well is not easy. Even professional
portrait. In this fashion,
baseball players have difficulty consistently making for instance, the word
solid contact with a pitched baseball. Once hit, the dis- blue was pulled off a
tance of ight of the ball is determined by its initial blue paint stroke.
velocitythat is, the initial speed and directionwhich J ACK H ITT, E AVESDROPPING
depends on how hard the ball has been hit by the bat. O N H ISTORY, N EW YORK T IMES
M AGAZINE (D ECEMBER 3, 2000)
All other factors being held constant, the initial velocity
can be said to depend on the speed of the bat just before
collision. A quicker swing would mean a faster bat

Start Me Up 53
Many experts believe speed during the collision to add distance to each hit
that Egyptian pyramids and also allow the batter more time to judge the pitch.
are aligned with true There have been proposals to put shallow, pea-sized
north because the depressionsdimplesin the surface of a baseball bat
more stationary stars to allow a greater swing speed. Another sports object,
near the North Celestial the golf ball, already is made with dimples on its sur-
Pole represented
face. How would these dimples affect the bats swing
permanency and
speed?
eternal life.

115. Old Glass


In old castles and houses in Europe can be found win-
dows with old glass in which many of the panes are
slightly thicker on the bottom than at the top. What are
some possible reasons for this result, and what is the
The adult brain as a
most likely reason?
whole consumes some
twenty-five watts of
power when in full 116. Ferromagnetism
action.
Why are so few substances ferromagnetic, yet practi-
cally all materials exhibit paramagnetic behavior?

117. Coupled Flywheels


Conservation of angular momentum does not always
help in understanding the behavior of rotating devices.
The more I have studied
The diagram shows two flywheels, 1 and 2, of
him, the more Newton
moments of inertia I1 and I2, mounted on parallel hor-
has receded from me.
R ICHARD W ESTFALL ,
izontal shafts along with pulleys of diameters D1 and
H ISTORIAN OF S CIENCE D2. The belt is slack at rst, and the two ywheels are

1 2

54 Mad about Modern Physics


running at angular velocities 10 and 20. Suddenly the MV 2 GAINS A
belt is tightened. One can write out the torque equa- FACTOR OF 1/2
tions and the angular momentum equation to get the It was not until 1807
relation I1 1 + I2 2 = k (N 1) I1 1. Here, k is a that Thomas Young, an
constant of integration and N = D2 / D1, the ratio of English physicist and
pulley diameters. When N = 1, angular momentum is physician, spoke of
conserved. If N 1 and 1 changes, the angular mv 2 for the first time
momentum is not conserved! Why not? as energy. Then in a
textbook published in
1829 Gustave Coriolis,
118. Superconductor Suspension a French physicist, was
the first to give the
A popular physics demonstration since the late 1980s
exact modern definition
involves floating a small piece of high-temperature
to kinetic energy and
superconductor, such as yttrium barium cuprate
work. He carried out a
(YBa3Cu3O7), over a strong permanent magnet. The calculation of the work
levitation is easy to see, and the suspended supercon- done in accelerating a
ductor rectangular solid spins rapidly about its long body and arrived at the
axis. The demonstration is done by rst cooling the change in the quantity
superconductor in liquid nitrogen and then using tongs 1/2mv 2. By the end of
to place the piece in the air above the permanent mag- the 19th century, most
net. The repulsive force between the magnet and the scientists were avoiding
superconductor is a demonstration of the Meissner Leibnizs old phrase
effect. Or is it? vis viva (living force),
and using instead
kinetic energy, a term
119. Nanophase Copper introduced in 1849 by
The hardness and strength of a metal are measured by Lord Kelvin to better
distinguish between
studying its deformation in response to an applied
force and energy.
force. A metal is deformed when its crystalline atomic
E UGENE H ECHT, A N
planes slide over each other. An analogy may be the H ISTORICO -C RITICAL
bump in a rug that can be pushed across the oor. In ACCOUNT OF P OTENTIAL
E NERGY: I S PE R EALLY R EAL ?,
other words, a dislocation in a plane of atoms is moved
T HE P HYSICS T EACHER
until a barrier is reached, such as a grain boundary, (N OVEMBER 2003)
where the micron-sized grains are differently oriented.
One interesting advance in metal technology is the
ability to assemble nanometer-size clusters of atoms in
grain sizes of less than 100 nanometers in diameter

Start Me Up 55
instead of having the micron-size grains found in a
typical metal. A graph of hardness versus grain size is
shown.

Jean Buridan (ca. Hardness (GigaPascals)


1295ca. 1358), rector 3
of the University of
Paris in 1327, in his
impetus theory intro- 2
duced the prescient
notion that the true
measure of the motion 1
of an object was not
speed alone, but the
product of speed and 0
quantity of matter
0 1 2 3 4 5
(quantitas materiae). In
an anticipation of New- Log of Grain Size (nm)
tons first law of motion,
he maintained that once With grain sizes averaging about 10 nanometers,
the initial impetus was this nanophase copper metal has a hardness more than
supplied, motion contin- three times the hardness of normal copper metal. Why?
ued indefinitely. The
spheres of heaven, for
instance, having been 120. Head of a Pin
put in motion by God,
What is the smallest amount of charge that can sit on
continued so and
required no constantly the head of a pin? Some people say that the smallest
working angels to keep nonvanishing amount of charge should be +e or e,
them moving. where e is the fundamental unit of electrical charge.
I SAAC A SIMOV, A SIMOV S What do you say?
B IOGRAPHIC AL E NCYCLOPEDIA
OF S CIENCE AND T ECHNOLOGY,
2 ND REV. ED. 121. Coulomb Blockade
The tunnel junction is a conductor-insulator-conductor
device. Suppose a very small tunnel junction is operated
at very low temperatures so that thermal uctuations
do not contribute to electron tunneling across the junc-
tion. Now connect the tunnel junction to a source of

56 Mad about Modern Physics


constant electrical charge. Will the flow of current
across the junction be steady?

Descartes regarded
122. Deterministic Competition the conservation of
Consider a simplied system, one that can be described momentum (quantity of
by Nt objects at time t. For example, one could consider motion) as divinely
the number of grasshoppers on the plains of Africa, or ordained. He wrote:
on some small plot of land. Let there be competition [God] set in motion in
between the growth processes and the decay processes many different ways
the parts of matter
so that the number of objects at time t + 1 is Nt + 1 = Nt
when He created them,
exp[r (1 Nt)], an exponential growth relationship.
and since He main-
This equation is deterministic, for Nt determines Nt + 1
tained them with the
unambiguously. One can think of r as a measure of the same behavior and with
ratio between growth and decay. Numerous mechani- the same laws as He
cal, hydrodynamic, chemical, and electrical systems can laid upon them in their
be approximately modeled by this relationship. creation. He conserves
How does the number of objects behave with continually in this mat-
elapsed time? If Nt = 1, then N remains 1 forever. In the ter an equal quantity of
general case, we can determine Nt as t to nd out motion.
whether N approaches the equilibrium value 1. For
instance, let r = 1 and begin with N0 = 0.5, and calcu-
late with a calculator or personal computer. Now try
different values for r. What behavior do you predict?
Earth is gradually slow-
123. Two Identical Chaotic ing down; the day is
Systems about 16 milliseconds
longer now than it was
A chaotic system exhibits a sensitivity to initial condi- 1,000 years ago. This
tions and will evolve rapidly and deterministically slowing is due largely to
toward different end states if begun in slightly different frictional tidal effects
states. Although the chaos is unpredictable, each possible of the Moon on Earths
outcome is deterministicthat is, an orderly behavior. oceans.
Consider two identical chaotic systems isolated
from each other. They will quickly fall out of step
because any slight difference between them would be
magnified. Assume that these systems have several

Start Me Up 57
In the fall of 1915 it was parts and that at least one of the parts is stablethat
widely expected that the is, subjected to a perturbation, the parts behavior
Nobel Prize in Physics changes a little but settles back to its normal operation.
was to be jointly shared Now drive both systems with the same chaotic signal
by Edison and Tesla. applied to the same stable part. Can the two systems be
Then a Reuters dispatch synchronized?
from Stockholm dropped
a bombshell. The Nobel
Committee announced
that the prize for 124. Tilleys Circuit
physics would in fact be
shared by William Henry This electrical circuit near the permanent magnet has
Bragg and his son W. L. two ideal switches and a galvanometer. When switch A
Bragg . . . . What had is closed and switch B, on the right, is opened, there is
happened? The Nobel a large change in the magnetic ux in the galvanometer
Prize Foundation circuit. What do you predict the galvanometer response
declined to clarify. One will be?
biographer reported
years later that the
Serbo-American had Magnet
declined the honor,
stating that as a discov- G
erer he could not share A B
the prize with a mere
inventor. Yet another
biographer advanced the
theory that it was Edison
who objected to sharing 125. Thermal Energy Flow
the prize. . . . The Nobel If two identical bodies at different temperatures are in
Foundation said simply, contact, thermal energy will always ow from one to
Any rumor that a person
the other in such a direction as to increase the total
has not been given a
entropy. In which direction will this flow be? That
Nobel Prize because he
has made known his
depends on two factors, the amount of energy and
intention to refuse the entropy the two bodies already contain. The second
award is ridiculous. law of thermodynamics implies that thermal energy
M ARGARET C HENEY, must ow toward the region of lower temperature
T ESLA : M AN O UT OF T IME that is, each unit of thermal energy acquires greater dis-
order as it moves into the cooler region. Why?

58 Mad about Modern Physics


126. Cadmium Selenide Is Galileo a beneficiary of
the Matthew effect? The
When atoms are arranged in nanometer-size clusters of latter is a term introduced in
diameters from less than 100 nanometers to as large as 1968 by Robert K. Merton
700 nanometers, interesting optical properties can be (19102003), a U.S. sociol-
demonstrated. For example, nanophase versions of ogist of science, that refers
pure cadmium selenide can be made almost any color to the disproportionately
in the spectrum simply by changing its cluster size. great credit given to eminent
Indeed, some types of lipstick are made in many differ- scientists for their contribu-
tions to science, while rela-
ent colors even though the predominant light-scattering
tively unknown ones tend to
molecule is the same in all color versions. What is the
get disproportionately little
physics here? for their occasionally com-
parable contributions. The
127. Optical Solitons term derives, of course,
from the Gospel according
A light pulse is a continuum of optical carriers of dif- to Matthew (13:12 and
ferent frequencies. Optical media are dispersive, so 25:29). In the New King
these carriers in the light pulse travel at different veloc- James Version the passage
ities, causing the energy to spread over time and dis- reads: For whoever has, to
tance. In addition, there is the optical Kerr effect, which him more will be given, and
instantaneously increases the refractive index of the he will have abundance; but
whoever does not have, even
medium by an amount proportional to the optical
what he has will be taken
power. Can one use these two effectsdispersion and
away from him. Recognition
the Kerr effectto ensure that a light pulse retains its tends to go to those who are
integrity while traveling thousands of kilometers already famous. In Galileos
through an optical ber? case, both Philoponus in
the sixth century and the
BelgianDutch scientist
128. Ceramic Light Response Simon Stevin in 1586 per-
Certain ceramic materials will change their shape upon formed the key experiment
exposure to light. What is the physics here? of dropping two different
weights simultaneously and
observed that they struck
129. Random Movements the ground at the same
timethe experiment that
Supposedly, research has revealed that random move-
today seems indissolubly, if
ments help explain how a tightrope walker stays aloft, incorrectly, wedded to the
for instance. If understood, robotics engineers could name of Galileo.

Start Me Up 59
In Galileos time a make their machines more stable by injecting a little
general impression noise into their systems. And persons having difculty
prevailed that a falling walking may be able to let some noisy vibrating shoe
body gained speed in soles help them walk condently again. What could be
proportion to the dis- the physics here?
tance through which it
fell, and Galileo himself
held this opinion for a
time. It appears also 130. Gravitational Twins
that he abandoned this Engineering physics involves the transport of people
idea, not because of and materials in space as well as practical applications
contrary results of here on Earth. So consider a pair of twins in free fall.
experiment, but
Imagine that one twin is in circular orbit around a star
because deductive rea-
and that her sister is shot out from this circular orbit
soning (not without a
location on a radial orbitthat is, the traveling twin
flaw) had led him to the
impossible conclusion will fall back to meet up with the stay-at-home sister in
that a body governed by circular orbit. For simplicity, let them meet after an
this law would fall integral number of revolutions around the circular
through a long distance orbit for the one left behind.
in the same time as Any clock system in a gravitational potential, such
through a short one. He as the clocks in the GPS system here on Earth, depends
therefore abandoned on two relativistic effects on the clock rate: (1) a clock
the space-acceleration ticks slower closer to a massive object than when far
idea and considered the away, and (2) the faster-moving clock ticks slower than
possibility that the gain the slower-moving clock.
in speed was propor- Initially, the clock rates of the twins are the same
tional to the time of fall.
because they start out in the same circular orbit at the
PAUL R. H EYL ,
TRANSCENDENTAL MECHANICS, same radial distance from the star. The traveling twin
A MERIC AN J OURNAL OF P HYSICS moves away from the star along the radial line, all the
(M AY 1941) while slowing down and eventually coming to a momen-
tary stop and returning with ever-increasing speed until
rejoining her sister in orbit. So on average, the traveling
twin experiences a smaller amount of gravitational time
dilation and a smaller amount of speed time dilation
than her stay-at-home sister. Therefore the traveling
twin returns home older than her sister, because her
clock ticked faster on average. What do you think?

60 Mad about Modern Physics


131. Photon Engine Nikola Tesla (18561943),
the Serbian-born inventor
The ideal Carnot heat engine converts heat to of the first practical
work without the engine itself being a source of any alternating-current
work. The reversible closed Carnot cycle consists of dynamo and power trans-
two isothermal (constant temperature) processes and mission system, was
two adiabatic (no external exchange of thermal known for unusual powers
energy) processes. No heat engine operating between of visualization. He was
two temperatures can be more efcient than a Carnot able to construct, modify,
cycle. and even operate his
imaginary devices, purely
by visualizing them. He
Hot atoms wrote in My Inventions
(Electrical Experimenter,
Mirrors
1919), It is absolutely
immaterial to me whether
T1 Piston T2 I run my turbine in
thought or test it in my
shop. There is no differ-
Photon beam ence whatever, the results
are the same. In this way I
am able to rapidly develop
and perfect a conception
without touching anything.
But Carnot could be wrong. The challenger is the When I have gone so far
new quantum Carnot engine, in which the radiation as to embody in the
pressure from photons drives a piston in an optical invention every possible
improvement I can think
cavity. The inward-facing surface of the piston is
of and see no fault any-
mirrored and the other cavity mirror is xed in place
where, I put into concrete
while exchanging thermal energy with a heat sink at
form this final product of
temperature T1. A second heat bath at a higher my brain. Invariably my
temperature, T2 , provides the source of thermal energy device works as I con-
for the photons. ceived that it should,
This source of thermal energy is a stream of hot and the experiment
atoms, which flows into the optical cavity and comes out exactly as I
exchanges thermal energy with the photons through planned it. In twenty
emission and absorption processes. These atoms exit years there has not been
the cavity at a cooler temperature and are reheated to a single exception.

Start Me Up 61
The Earth is a somewhat T2 in a second cavity, to be reinjected into the rst
irregular clock. Some cavity for the next cycle of the quantum Carnot engine.
years the length of the Therefore, the quantum and classical Carnot
day is found to vary by
engines operate in the same way as a closed cycle
as much as one part in
10 million, or three sec- of two isothermal and two adiabatic processes. How-
onds in a year of 31.5 ever, in its simplest form, when each bath atom is
million seconds. In addi- treated as a two-state system, the quantum Carnot
tion, there are also sea- engine cannot extract work from a single heat bath.
sonal fluctuations of a Why not? Will the engine work if each bath atom is a
few milliseconds per
year. In the winter the
three-state system?
Earth slows down, and in
the summer it speeds
up. Think of the Earth
as a spinning skater.
During the winter in the
northern hemisphere,
water evaporates from
the ocean and accumu-
lates as ice and snow on
the high mountains. This
movement of water
from the oceans to the
mountaintops is similar
to the skaters extend-
ing her arms. So the
Earth slows down in
winter; by the summer
the snow melts and runs
back to the seas, and
the Earth speeds up
again. This effect is not
compensated by the
opposite effect in the
southern hemisphere
because most of the
land mass is north of
the equator.
J AMES J ESPERSEN AND J ANE
F ITZ -R ANDOLPH , F ROM
S UNDIALS TO ATOMIC C LOCKS

62 Mad about Modern Physics


7 A Whole
New World
A TOMIC PHYSICS BEGAN IN THE 1840 S WITH

the identication of the emission lines of hydrogen


and of other atoms and ions in laboratory sources and in the
solar spectrum. In the early 1900s, the Bohr-Sommerfeld
model of the atom was the paradigm, but numerous prob-
lems with its predictions existed that were nally resolved
with the advent of quantum mechanics in 1925. The electron
in the atom occupies particular quantized energy states of
unequal energy spacing, and selection rules based on conser-
vation of energy and angular momentum dictate which
jumps between states to an available nal state are allowed.
In addition to a spontaneous electron jump to a lower energy
level with the emission of a photon, external photons with
the correct energy can stimulate the atomic absorption or
emission of photons. The eventual application of quantum
mechanics to the simple molecules proved very successful, if

63
not challenging, and today faster computers continue
to calculate the properties of atoms, inorganic and
organic molecules, and very large biomolecules such as
DNA and proteins. Enormous progress has also been
made in understanding the fundamental properties of
condensed matter of uids and solids such as crystals,
ionically doped materials, plastics, pseudocrystals, and
so on. Our lives are becoming more dependent on the
practical devices arising from this great endeavor called
molecular design. The challenges introduced in this
chapter are but a small sample of the wide range of
possible problems.

64 Mad about Modern Physics


132. Grain of Sand Both Niels Bohr and
his wife had a similar
If the atoms in a grain of sand were laid out side by side response to religion:
in a line, approximately how long would the line be? Margrethe has written
about Niels: There
was a period of about a
133. Forensics year . . . [he was] 14 or
Historically, paintings could be veried with reasonable 15 . . . where he took it
assurance of authorship by experts who knew the all very seriously; he got
brushstrokes and color and paint choices of the artist as taken by it. Then sud-
well as the overall style and character of the subjects. denly it was all over. It
However, in some cases, fraudulent artworks have been was nothing for him.
About her own feelings,
successfully passed as genuine. New techniques for
Margrethe reported:
assessing all types of artwork are always needed, and
You know it was often
the scientic community has been answering the call.
at that age . . . that
One scientic technique for checking the authenticity one got very religious
of old paintings uses laser lights. How might this feat and would listen to the
be accomplished? minister about confir-
mation. Then it all
dissolved. And for me
134. Doppler Elimination? it was exactly the
same; it disappeared
completely.
Intensity Doppler shifted
L ON ROSENFELD,
Natural width B OHR , N IELS H ENRIK DAVID,
D ICTIONARY OF S CIENTIFIC
B IOGRAPHY, VOL . 2

Doppler-broadened

Frequency

When an atom emits or absorbs a photon, there is


always a recoil of the atom and a Doppler shift in the
photon frequency. Is it possible to have recoilless
atomic emission or absorption?

A Whole New World 65


FIELD THEORY IN THE 135. Light Tweezer
EIGHTEENTH CENTURY?
In many ways this major Laser light
advance had its origin in
1758 with the publication Lens
of the immensely influen-
tial Theory of Natural
Philosophy by Roger 1 2
Joseph Boscovich
(17111787), a theory of F2
such importance that
f 1
nearly 150 years later
F1
Lord Kelvin could describe
himself as a true Boscov- 2
ichean. Boscovich was
born Rudjer Josip In science ction movies we often see light beams shot
Boskovic in the Republic out from handheld light guns supplying a tremendous
of Dubrovnik. Today he is
impulse to knock over an enemy storm trooper
claimed with equal vigor by
approaching along the direction of the beam. By New-
the Croats, the Serbs, and
the Dalmatians. As Mar-
tons third law, the light gun itself should have experi-
garet Wertheim writes in enced an equivalent recoil! We know that a ray of light
her Pythagoras Trousers, has energy and linear momentum, so its impingence on
His nationality is surely a any surface will produce a slight backward movement
significant part of the of that surface. However, we would like to know
reason that this visionary whether a light beam can be used to physically move a
physicist isnt more tiny object, such as a small one-celled animal, in a
famous todayit is diffi- direction perpendicular to the beam.
cult to imagine that any
Anglo-Saxon scientist of
such caliber would have 136. Fluorescent Lights
remained so outside the
spotlight. Trained as a The gas plasma inside a uorescent tube emits mostly
Jesuit priest, he became a ultraviolet radiation and very little visible radiation.
professor of mathematics Electrons are captured by the ions and jump down to
in Rome, and over the lower energies, emitting a characteristic UV photon for
course of his lifetime each uorescence jump. Why are uorescent tubes so
published over a hundred much more efficient in producing visible light than
books and papers, most of incandescent lamps?

66 Mad about Modern Physics


Why can the light from some uorescent lights be which still remain
dangerous to your health? Could it be that they emit untranslated from the
some UV? Are there types of uorescent tubes that are Latin. To get rid of the
better for human working environments? Are they bet- spooky action at a dis-
ter because they do not emit in the UV? tance, he introduced
the proposition that
atoms have no size;
137. Phase Conjugation Mirror they are geometrical
points of force that in
Can a light wave pass through a disturbing medium, be
turn create fields of
distorted, reect off a special mirror, and return to the
force, an idea later
source undisturbed? elaborated on by Fara-
day. Moreover, he sug-
138. Stationary States gested that all these
atomic forces along with
In the Bohr model of the hydrogen atom, the angular gravity, must be aspects
momentum for the orbital motion of the electron of of one all-encompass-
mass m at distance r is quantized in integral units of ing universal force, an
Plancks constant hthat is, assuming the proton posi- eighteenth century ver-
tion to be xed, mvr = nh/2, where n is an integer and sion of the theory of
v the electron velocity. Using mv = h/ , de Broglie was everything!
able to derive Bohrs quantization rule and n = 2r. If ADAPTED FROM LESLIE HOLLI -
DAY, E ARLY V IEWS ON F ORCES
f1 and f2 are the frequencies of the Bohr orbital motion BETWEEN ATOMS , S CIENTIFIC
of the electron in energy states E1 and E2, then if an A MERIC AN (M AY 1970)
electron jumps down from state 2 to state 1, why isnt
the energy of the emitted photon the difference energy
hf1 hf2?

139. Angular Momentum


In classical calculations, the quantity that often appears
in the result is the square of the angular momentum J 2.
One can often guess at the correct quantum mechanical
formula by replacing J 2 by j (j + 1) h2/4 2, where j is the
z-component of the angular momentum and h is
Plancks constant. Why is the square of the angular
momentum in quantum mechanics proportional to
j ( j + 1) instead of just j 2?

A Whole New World 67


Five Venus rounds 140. Kinetic Laser
(synodic periods) equal
eight solar years within A traditional laser involves the stimulated downward
about two days, and electron transition in an atom in a background sea of
equal also 99 lunar photons, with the emission of a characteristic photon
months with a discrep- matching in fequency and momentum the stimulating
ancy of less than four photons. This stimulated emission process was predicted
days. Specifically, by Einstein. In 1951, J. Weber at the University of Mary-
5 583.9 = 2919.5, land was the rst to calculate the operating principles of
8 365.25 = 2922.0 the ammonium maser and laser. However, as the story
99 29.53 = 2923.5. goes, upon asking for research monies to build the
For ancient astronomers
maser, a few hundred thousand dollars from the univer-
this was evidence of
sity, he lost out to the athletic departments request for
profound unity and even
money to build up the Maryland football program.
preestablished harmony
within the cosmos. The rst operating ammonium maser was subse-
quently built by C. Townes in 1954, and the rst oper-
ating device lasing in the optical part of the spectrum
was built in 1960 by T. H. Maiman. Laser action rst
in the microwave region is no coincidence, for sponta-
neous emission is proportional to the cube of the tran-
We must be clear that, sition frequency, and being extremely small in this part
when it comes to atoms, of the spectrum, can be neglected compared to stimu-
language can be used lated emission and absorption.
only as in poetry. Among the more exotic lasers is the kinetic laser,
N IELS B OHR
which is an exploding material that emits light and
X-rays. In its simplest form, the material would be a
foil of a single element such as copper that is exploded
by focusing powerful laser pulses on it. How does this
type of laser produce coherent laser light?
If all the empty space
were squeezed out of a 141. Noninversion Laser
person, the amount of
solid matter remaining For decades, lasers have been explained as the result of
would be no larger than an inverted population of states with stimulated emission
a speck of dust. of photons in a high-Q cavity. However, lasers can be
made without an inverted population. Can you explain
how this type of stimulated emission process works?

68 Mad about Modern Physics


142. X-ray Paradox
The index of refraction n gives the ratio c/v, the speed
of light in vacuum to the speed of the electromagnetic
wave in the material. Window glass, for example, can Why do all FM radio
have an index of about n = 1.5 for visible light, with a stations end in an odd
slight variation in the index with the color of the light. number? FM radio sta-
A paradox arises with X-rays because they have an tions all transmit in a
index of refraction value less than one in crystals! What band between 88 MHz
does this behavior mean? and 108 MHz. Inside
that band, each station
occupies a 200 kHz
143. Benzene Ring slice, and all of the
slices start on odd num-
The benzene molecule is a
ber frequencies. This is
ring of six carbon atoms,
completely arbitrary. In
each C atom having one H Europe, the FM stations
atom attached. There is a are spaced 100 kHz
mystery about the energy apart, and their fre-
contained in this molecule. quencies can end on
The benzene ring can be even or odd numbers.
broken up into pieces, and
chemists have measured
the energies associated
with the pieces and with When Einstein regis-
tered for the draft in
the single bonds and the double bonds by studying
Switzerland at the age
ethylene and so on. The expected total energy can be of 22, his height was
calculated from these data, but the actual total energy recorded as five feet
of the benzene ring is much lower, telling us that the seven and a half inches.
carbon atoms are much more tightly bound. Therefore, His contemporaries
the bond picture would make the benzene ring easily regarded him as tall. By
susceptible to chemical attack, yet the molecule is quite way of comparison,
Isaac Newton is thought
resilient to breaking up.
to have been about five
Using the Schrdinger equation by considering each feet five inches tall.
carbon atom on this ring as the potential home for a A DAPTED FROM B ARRY
single electron, one can calculate the possible energy PARKER , E INSTEIN : T HE
PASSIONS OF A S CIENTIST
levels for the benzene ring. Why does this method of
calculation work?

A Whole New World 69


144. Graphite
Atoms in a crystal make a regular array if there are no
dislocations. Most pure single-element crystals have a
cubic or a diamond crystal structure, with all orthogonal
Niels Bohr discovered directions showing the same structural spacing. Even for
his ideas in the act of a pure element substance, however, the spacing may be
enunciating them, shap- different in different directions. For example, take car-
ing thoughts as they bon atoms, which probably are components of more
came out of his mouth. than 75 percent of all known compounds. In diamond
Friends, colleagues, they have the same structure in all orthogonal direc-
graduate students, all
tions, but in graphite the third direction is definitely
had Bohr gently entice
quite different than the other two directions, which
them into long walks in
dene a plane of hexagonal carbon rings. How can this
the countryside around
Copenhagen, the heavy third direction be so different in an originally nonbiased
clouds scudding over- environment?
head as Bohr thrust his
hands into his overcoat
pockets and settled into
an endless, hesitant,
recondite, barely audi-
ble monologue. While he
spoke, he watched his
listeners reactions,
eager to establish a
bond in a shared effort
to articulate.
ROBERT P. C REASE AND
C HARLES C. M ANN , T HE
S ECOND C REATION : M AKERS OF
THE R EVOLUTION IN
20 TH -C ENTURY P HYSICS 145. Ozone Layer
Weve heard so much in the past few decades about the
ozone layer in the upper atmosphere and its possible
demise. Yet ozone is only a minor greenhouse gas, far
behind carbon dioxide, HOH vapor, and methane in
overall importance. So why is there all this fuss over the
ozone layer?

70 Mad about Modern Physics


146. Greenhouse Gases The special theory of
relativity predicts that,
Why are the greenhouse gases carbon dioxide, HOH for an observer moving
vapor, and methane important for human survival on at the speed of light,
Earth? If they are good for our existence, shouldnt distance traveled
having more carbon dioxide, etc., in the atmosphere be shrinks to zero while
encouraged? time slows to a stand-
still. Thus, as far as the
light itself is concerned,
147. LED vs. LCD it does not travel any
An LED is a semiconductor device that emits visible distance, and takes no
light when an electric current passes through it. time to do so. As Gilbert
Lewis showed back in
The light is not particularly bright and usually
1926 (Nature, vol. 117),
monochromatic, occurring at a single wavelength. The
from lights point of view
LED light output range is from infrared and red to
the Universe is so bent
blue-violet. The LCD is a type of display used in digital that there is no separa-
watches and many portable computers that utilizes two tion between the point of
sheets of polarizing material with a liquid crystal emission of light and its
solution between them. An electric current passed point of absorption. . .
through the liquid causes the crystals to align so that If light does not expe-
light cannot pass through, each crystal acting like a rience itself to have
shutter, either allowing light to pass through or block- traveled any distance, it
ing the light. does not need a vehicle
What is the difference in energy requirements in or mechanism by which
the operation of a light-emitting diode (LED) and a liq- to travel. . . . It is only
uid crystal display (LCD)? After all, they both require in our frame of refer-
encethe frame of
energy to operate. And how is a plasma display differ-
observers with mass who
ent from both of them in its energy requirements?
move at sub-light
speedsthat light
148. Sonoluminescence appears to travel
through space and time;
Sound energy is converted directly into light energy by and only in that frame
a phenomenon called sonoluminescence. Discovered in does the question of
the 1800s, the process lay dormant for more than 100 whether it is a wave, a
years, only to experience a revival in the 1990s. How particle or both arise.
does one convert a small amount of sound energy into P ETER RUSSELL , H ERE I S
a brief but brilliant ash of light? T HERE , L ETTERS , N EW
S CIENTIST
(N OVEMBER 23, 1991)

A Whole New World 71


Isidor Isaac Rabi, (Nobel 149. Siphoning Liquid Helium
Prize in Physics in 1944)
faced a crisis of faith
while only a child.
He decided to put the
tenets of Orthodox
Judaism to the test. Liquid helium
Jewish law prohibited
riding streetcars on the Glass dewar
Sabbath, so one Sabbath
he climbed onto a street-
car, just to see what would
happen. Emboldened by
an uneventful ride, he
conducted another test: Liquid helium can crawl up the wall of its container
I remember being in the without any additional help. How is this feat accom-
synagogue and the priests plished?
. . . would stand up and
with their hands out-
stretched, they would
bless the congregation.
150. Quantized Hall Effect
You were not supposed to The Hall effect was discovered by Edwin Hall in 1879.
look at their hands, you . . . A charged particle moving in a magnetic eld feels
might go blind if you did. a Lorentz force perpendicular to its direction of
Well I tried . . . with one motion and the magnetic eld. As a direct consequence
eye. His sight left unim- of this Lorentz force, charged particles will accumulate
paired, he dismissed to one side of a wire if you send current through it and
Jewish faith as supersti-
hold it still in a [perpendicular] magnetic eld. . . .
tion but, to please family
When the transverse voltage is measured at a xed cur-
and friends, consented to
rent, the Hall resistance is measured and increases lin-
a bar mitzvah, at which he
ended up lecturing them early with an applied magnetic eld.
in Yiddish on the workings The conduction electrons in a solid behave like a
of the electric light bulb. gas of electrons. So the discovery of the quantized Hall
PAUL H OFFMAN , T HE effect in 1980 by von Klitzing and his research group
H IGHER T RUTH OF P HYSICS , A when he was investigating the conductance properties
REVIEW OF R ABI : S CIENTIST AND
C ITIZEN BY J OHN S. R IGDEN ,
of two-dimensional electron gases at very low temper-
N EW YORK T IMES B OOK atures and high magnetic elds was a surprise. What is
R EVIEW (M AY 10, 1987) the physics behind this quantized Hall effect?

72 Mad about Modern Physics


151. Integrated Circuits Planetary atomic mod-
els were already popular
As integrated circuits (ICs) become crowded with a few years before
more semiconductor devices and internal connections, Rutherfords proposal.
one wonders how they will be connected to the The most elaborate
external world. We know also that cosmic rays and attempt was that of
other particle radiation from the environment already Hantaro Nagaoka,
disrupt some of the operations by random destruction, whose Saturnian
and these effects will become worse as the scale dimin- model was published in
ishes. However, neither connection to the external 1904. Nagaokas model
world via gold wires of any size nor the background was astronomically
inspired, in the sense
particle radiation is the major problem today. What is?
that it closely relied on
Maxwells 1856 analysis
152. Atomic Computers? of the stability of Sat-
urns rings. The Japan-
Atoms are busy collections of electrons and nuclear ese physicist assumed
particles that are ever changing their positions in a ran- that the electrons were
dom dance. In contrast, information storage requires placed uniformly on
stable states over reasonable time intervals. Can infor- rings moving around the
mation be stored on individual atoms in their restless attractive center of a
world? positive nucleus.
Nagaokas calculations
led to suggestive spec-
153. X-ray Laser? tral formulas and a
We know that there exist free electron X-ray lasers that qualitative explanation
have an electron moving past a rippled surface and of radioactivity. The
emitting X-rays, as well as X-ray laser sources based on model was, however,
severely criticized, and
disappeared from the
Cu-W scene only to reappear
in an entirely different
Cu(111) dressing with Ruther-
fords nuclear theory.
H ELGE K RAGH , QUANTUM
G ENERATIONS : A H ISTORY
OF P HYSICS IN THE
T WENTIETH C ENTURY
Cu X-rays
X-ray tube

A Whole New World 73


There was a time when plasmas such as the kinetic laser. However, an X-ray
physics and philosophy laser with a wavelength of about 1 or 0.1 nanometer
were allied disciplines. or less that can be operated on a tabletop would be
However, Niels Bohrs convenient and would be able to resolve details down
three long historic papers to nearly 1 wavelength. The uses in physics and medi-
on the structure of the cine are expected to be many.
hydrogen atom were
A very interesting tabletop device is the working
published in 1913 in
monochromatic X-ray source shown in the illustration
Philosophical Magazine.
that emits very intense, narrow beams at the Cu 1.54
The journal, first pub-
lished in 1798, was at characteristic emission line known as K1. There is a
that time accepting special bimetal X-ray tube source of Cu-W that emits
articles from most X-rays from both metals upon bombardment with
branches of science. high-energy electrons in the standard way. These
This alliance began to X-rays exit the tube and then Bragg scatter in an exter-
break up at the end of nal Cu crystal to produce a very narrow, intense beam
the nineteenth century. of Cu K1 X-rays. The rst surprise is the enormous
Today, even though the line intensity at a single wavelength, and the second
word philosophical surprise is that no Cu K2 X-rays appear in the output
persists in its title, it is from the external crystal. How does the external crys-
devoted primarily to tal affect the X-ray beam? Is this device an X-ray laser
condensed-matter
or a super-radiant X-ray source?
physics.

154. Bose-Einstein Condensate


A Bose-Einstein condensate is a new form of matter
made at the coldest temperatures in the universe. Essen-
Things that cannot go on tially the condensate is a collection of identical atoms
forever dont. behaving as one entity. How do the individual atoms
A NONYMOUS lose their self-identity?

155. Quantum Dots


Quantum dots are crystals containing only a few hun-
dred atoms and when illuminated with UV light, for
example, will uoresce at only one specic wavelength
of light. Why does the dot emit only one wavelength of
light when excited?

74 Mad about Modern Physics


8 Chances
Are
Q UANTUM MECHANICS ( QM ) ORIGINATED IN

1925 as a theory to understand the internal


behavior of the hydrogen atom. Since then, QM has
evolved to encompass the behavior of practically everything.
In its most rudimentary version, QM is based on three
fundamental rules. The main idea of QM is not quantized
energy and quantized angular momentum, for the classical
physics of strings, tubes, drumheads, and so on, involve
quantized states of energy and angular momentum.
The heart of QM is the coherent superposition of states,
as given in rule 2 below. From The Feynman Lectures on
Physics, the three fundamental rules of QM are:

1. The probability P of an event in an ideal experiment is


given by the square of the absolute value of a complex
number , which is called the probability amplitude (or
wave function):
P = || 2.

75
2. When an event can occur in several alternative
ways, the probability amplitude for the event is
the sum of the probability amplitudes 1, 2, 3
. . . , for each way considered separately; that is,
there is superposition and interference:
= 1 + 2 + 3 + . . .
P = | 1 + 2 + 3 + . . . | 2

3. If an experiment is performed (or could be done)


that can determine whether one or another alterna-
tive is actually taken, the probability of the event is
the (classical) sum of the probabilities for each
alternative; that is, the interference is lost:
P = P1 + P2 + P3 + . . .

We have no knowledge about a more basic mecha-


nism from which these rules can be deduced. Numer-
ous tests have veried their fundamental validity over
and over. You will need to apply them in the challenges.

76 Mad about Modern Physics


156. Schizophrenic Playing Card For centuries, Britain
and its colonies rang in
the New Year on March
25, Annunciation Day,
when according to the
biblical account the
angel Gabriel announced
to the Virgin Mary that
she would bear the child
of God. March 25 is
nine months before
Christmas.
D UNCAN S TEEL , M ARKING
An ideal playing card stands perfectly balanced on its T IME : T HE E PIC QUEST
TO I NVENT THE P ERFECT
edge. According to the rules of quantum mechanics,
C ALENDAR
this card will fall in both directions at once! That is, the
nal state of the card is the superposition of the two If you stood on the
alternative falling directions, with 1 for left and 2 for moons near side, you
right. The cards wave function changes smoothly and would see the Earth
continuously from the balanced state to the mysterious suspended against the
nal state = 1 + 2 with two alternatives that seem stars more or less in
to have the card in two places at once. Why havent we the same direction
seen this happen in the everyday world around us? with respect to your
horizonnever rising
or setting. But the Earth
157. Schrdingers Cat as seen from the moon
In one version of the famous Schrdinger cat gedanken would exhibit phases
over the course of a
experiment, a healthy cat is placed inside an ideal cat
month, just as the moon
playroom that is isolated from the rest of the world
does as seen from
whenever the door is closed. Inside is one deadly object
Earth.
left by mistake. The door is closed. After some time M ICHAEL Z EILIK AND J OHN
elapses, one wonders whether the cat is alive or dead, G AUSTAD, A STRONOMY :
the two classical possibilities. Rule 2 of QM tells us, T HE C OSMIC P ERSPECTIVE

however, that the state of the cat is = 1 + 2 , where


1 means alive and 2 means dead. So QM requires us
to consider the cat as being alive and dead simultane-
ously! However, you are curious. You push a button

Chances Are 77
The bourgeois ambiva- that opens the door just enough so you could look in to
lence of Werner Heisen- determine the status of the cat. You could peek in but
bergs childhood may you decide not to. Now what does QM predict for ?
have played a role in his
own adult ambivalence
toward the sweeping 158. Wave Functions
claims of every system
of thought and belief, Wave functions can be functions of many different
including science. At physical parameters of the system of interest. For
middle age and again example, one can dene a wave function in coordinate
near the end of his life,
space, in momentum space, in spin space, and so on as
Werner declared science
and religion to be com- long as the unit vectors of the space are orthogonal. For
plementary aspects of a single particle, the wave function (x 1,y1 ,z1) is the
reality, each with its own QM amplitude for finding the particle at the three-
language and symbolism dimensional conguration space point (x 1,y1 ,z1), which
and each with its own directly corresponds one-to-one to position space coor-
limited realm of validity.
dinates x1, y1, and z1 for this one-particle system. For
Different religiously or
intuitively apprehended the two-particle system, the wave function (x 1,y1 ,z1 ;
truths should be viewed x2 ,y2 ,z2) denes a six-dimensional conguration space.
as different sides of the Is there a direct correspondence to three-dimensional
same truth, while position space coordinates for this two-particle wave
rational sciencehis function as well? What about the multiparticle wave
own professionshould
function?
be viewed as just one
among a variety of ways
of perceiving reality.
Shortly before his
159. Wave Function Collapse?
death, Heisenberg Consider an electron in a box. Imagine partitioning the
remarked to a colleague, box into N identical cubes and assume that the
If someone were to say
amplitude for finding the electron in the box is
that I had not been a
Christian, he would be the superposition = 1 + 2 + 3 + . . . , that is, the
wrong. But if someone sum over all N imagined identical cubes in the box.
were to say that I had Now use a photon to observe where the electron is by
been a Christian, he recording the scattered photon and so on. Suppose
would be saying too your incident probe photon passes right through the
much.
box and does not interact with the electron, which you
DAVID C. C ASSIDY,
U NCERTAINTY : T HE L IFE determine because the photon took a straight-line path
AND S CIENCE OF W ERNER to your detector. What happens to the electron wave
H EISENBERG
function ?

78 Mad about Modern Physics


160. Quantum Computer QUESTION: WHAT IS IT?
Pascal did IT under
The new quantum computers rely on quantum coher- pressure.
ence. That is, the quantum computer system contains N Coulomb got all charged
identical quantum subsystemsfor example, atoms, or up about IT.
optical setups, or molecules, or resonant cavities. In Hertz did IT frequently.
general, each quantum subsystem can be in many pos- Boltzmann did IT in heat.
sible quantum states. Assume that the i for each quan- Ampere let IT flow.
tum subsystem has only two states, which we label 1 Heisenberg was never
and 0. If N = 3, then = 1 + 2 + 3 is the QM state sure whether he even
of the system. Therefore our quantum computer repre- did IT.
sents all eight states simultaneously: 000, 001, 010, Bohr did IT in an excited
011, 100, 101, 110, 111. state.
Pauli did IT but excluded
his friends.
Hubble did IT in the
dark.
Theorists do IT on paper.
That is, during calculations on all eight states Astrophysicists do IT
participate in each calculation! If the quantum com- with young starlets.
puter is actually a large molecule in a vacuum, then the ANSWER: IT = science, of
molecule must be kept away from the walls of the con- course!
tainer and away from other molecules. Why? C OPYRIGHT 2002 BY
J UPITER S CIENTIFIC

161. Cup of Java Quantum


Computer It has been proved that
One day while looking into her cup of java, Laura real- the 13th is more likely
ized that this slurry of caffeine molecules could be the to fall on Friday than on
worlds natural quantum computer. How could this any other day of the
inherent ability in coffee be possible? week. For a short proof
consult the reference
below.
162. Bragg Scattering of X-rays J OHN WAGNER AND
ROBERT M C G INTY,
Bragg scattering of X-rays of wavelength in an ideal S UPERSTITIOUS ?
crystal satises Braggs law: 2d sin = m , where d is M ATHEMATICS T EACHER 65
the spacing between adjacent scattering planes and is (1972): 503505

the angle measured from the surface of the crystal, not

Chances Are 79
Pauli once referred to the perpendicular. When this condition is met for vari-
quantum mechanics as ous integer values of m, constructive interference from
Knabenphysik boys the entire family of parallel planes occurs because the
physicsbecause so path differences are integral multiples of the X-ray
many of the main con- wavelength. One often reads that the Bragg scattering
tributors were still in of X-rays from an ideal crystal is a coherent scattering
their twenties. For
processthat is, all the Bragg-scattered X-rays arrive
example, in September
in phase at the detector. Why is it not so?
1925 Heisenberg was
23 years old, Pauli 25,
Jordan 22, and Dirac
had just turned 22. In
163. Beautiful Faces
1932 Friedrich von Why can we see a persons face in great detail in visible
Weizscker recalled, light? Hint: think about coherent scattering versus non-
The general attitude coherent scattering of the light.
was one of immense Why is the image of a persons face blurry in the
Hochmut, an immense infrared (IR) and in the ultraviolet (UV)? For simplicity
feeling of superiority, and idealization purposes, assume that we can see
as compared to old pro- equally well in the IR, visible, and UV so that our phys-
fessors of theoretical
iology is not the limiting factor.
physics, to every exper-
imental physicist, to
every philosopher, to 164. Gravitational Waves
politicians, and to what-
ever sorts of people you In addition to telescopes for photons in the -ray, X-ray,
might find in the world, UV, visible, IR, -wave, and radio parts of the electro-
because we had under- magnetic spectrum, new windows to the universe are
stood the thing and they opening up with neutrino and gravitational wave
didnt know what we observatories. Gravitational waves are expected to be
were speaking about. produced by a changing mass quadrupolefor exam-
H ELGE K RAGH , QUANTUM ple, two masses revolving about their common
G ENERATIONS : A H ISTORY
OF P HYSICS IN THE barycenter, such as the two stars in a binary star sys-
T WENTIETH C ENTURY tem. They would emit gravitational waves with wave-
lengths of many kilometers that interact with all
objectsthat is, they exhibit most wave phenomena
such as scattering, reection, and transmission through
objects in ways similar to other types of waves. The
classical scattering cross section of gravitational waves

80 Mad about Modern Physics


by a mass pair in a detector was worked out by physi- FROM AN INTERVIEW WITH
cist J. Weber about 50 years ago. THE AMERICAN PHYSICIST
For simplicity, assume that each pair of identical ISIDOR ISAAC RABI
atoms in a material is a mass pair quadrupole scatterer The nature of discover-
of gravitational waves. We would like to know whether ies is so remarkable, so
gravitational waves can scatter coherently in the detec- wonderfulif you want
torthat is, whether a gravitational wave can simulta- to think of the goal of
neously scatter from many mass pairs in the detector the human race, there it
is. To learn more about
(such as an aluminum bar) or whether a gravitational
the Universe and our-
wave must scatter from a single mass pair at a time.
selves. In physics, the
What is the physics here?
newest discoveries like
relativity and the uncer-
165. Coherent Neutrino tainty relation, uncover
new modes of thought.
Scattering They really open new
Another possible window or telescope for observing perspectives. A sudden
the universe is in the detection of neutrinos. The Super- sad look passed over his
Kamiokande neutrino facility in Japan and the Sud- face. And I thought
that, say, fifty years
bury Neutrino Observatory (SNO) in Canada house
ago, that this would hap-
two of the largest neutrino detectors, containing thou-
pen, that these revolu-
sands of tons of water. Already they have determined
tions and advances in
that the ux of solar neutrinos from the Sun agrees science would have an
with the standard solar model. In addition, research effect on mankindon
groups operating these neutrino detectors have veried morals, on sociology,
neutrino oscillations in matter, the conversion of one whatever. It hasnt hap-
type of neutrino to another. pened. Were still up to
The two neutrino detectors are enormous because the same things, or,
neutrinos are notorious for their extremely small prob- well, I think, regressed
ability to interact with matter. Billions of neutrinos pass in values.
through our bodies each second and do no harm! A sin- ROBERT P. C REASE AND
C HARLES C. M ANN , T HE
gle electron neutrino would pass through solid lead S ECOND C REATION : M AKERS OF
(Pb), lling space from Earth to Jupiter with only a THE R EVOLUTION IN

small chance of colliding with a Pb nucleus. However, 20 TH -C ENTURY P HYSICS

in 1984 physicist J. Weber proposed that neutrinos of


all energies could be coherently scattered by the nuclei
in large defect-free single crystals of silicon, ruby, or

Chances Are 81
diamond, thereby enhancing the neutrino scattering
probability by a factor of 1022. Therefore, in the ideal
case, practically all incident neutrinos would scatter at
least once from the carbon nuclei in a perfect diamond
crystal within the rst centimeter or less!
Normally, one might expect only neutrinos of wave-
Compared to the theory
lengths much greater than the spacings between the
of relativity, quantum
nuclei in the crystal to have any chance at coherent
mechanics developed
scattering, analogous to light scattering coherently from
rapidly, disseminated
very quickly, and met a surface of atoms spaced much less than the wave-
almost no resistance. length of the incident light. Otherwise, when the nuclei
Also contrary to relativ- are treated as scattering potentials, the phases con-
ity, quantum mechanics tributed by the scattering nuclei to the QM amplitude
attracted little public are random, and the scattering probability will be pro-
interest. Eddington was portional to N instead of N 2, like the result for X-rays
one of the few scien- discussed in a previous problem. What assumption have
tists who wrote about we made about the scatterers that Weber says leads to
the theory for a nonsci- an incorrect conceptual argument against coherent scat-
entific readership. tering for the shorter-wavelength neutrinos?
Although quantum
mechanics was no less
counterintuitive than
relativity, there was no 166. Magnetic Resonance
quantum counterpart to Imaging (MRI)
the antirelativistic liter-
Magnetic resonance imaging (MRI)
ature that flourished in Magnetic
the 1920s. is really the medical application Field
H ELGE K RAGH , QUANTUM of nuclear mag-
G ENERATIONS : A H ISTORY OF netic resonance,
P HYSICS IN THE T WENTIETH which physicists
C ENTURY
have been doing
since the 1940s.
A sample of liv-
ing tissue con-
tains numerous hydrogen atoms bound in molecules.
Each hydrogen nucleus has a spin with a magnetic
moment that can be aligned by an applied magnetic
eld. The sample is placed in a very strong uniform

82 Mad about Modern Physics


magnetic eld to align the spins of the hydogen nuclei. The number of photons
A pulsed electromagnetic eld is applied that would ip is in general not con-
just one hydrogen spin, for example. What alternative served in particle reac-
QM interpretation can one provide that treats the tions and decays. I . . .
nuclei as a collective whole? would like to note here
an ironical twist of his-
tory. The term photon
167. Heisenberg Uncertainty first appeared in the
title of a paper written
The Heisenberg uncertainty principle, also known as
in 1926. The title: The
the indeterminancy principle worldwide, states px x
conservation of pho-
h/4 , where x is the uncertainty in the x-position tons. The author: the
measurement, px is the uncertainty in the x-momen- distinguished physical
tum measurement, and h is Plancks constant. As some chemist Gilbert Newton
people say, the uncertainty principle places a limit on Lewis (18751946) from
the accuracy of knowing a particles position. What do Berkeley. The subject: a
you think? Some people claim also that the Heisenberg speculation that light
uncertainty principle is just an example of a more gen- consists of a new kind
eral uncertainty relationship for all waves, that the of atom . . . uncreatable
position can be determined only at the expense of our and indestructible [for
knowledge of its wavelength. Is this statement true? which] I . . . propose the
We also know that Niels Bohr, in his discussions name photon. This idea
was soon forgotten, but
with Albert Einstein over several decades on whether
the new name almost
quantum mechanics is a complete description of
immediately became
nature, would often invoke the uncertainty principle to
part of the language.
defend his point of view, known as the Copenhagen A BRAHAM PAIS , IN S OME
interpretation of QM. Bohr argued that if you pin S TRANGENESS IN THE
down the particles position more precisely for the P ROPORTION , EDITED BY
H ARRY WOOLF
famous double-slit experiment by observing with pho-
tons, their interaction with the particle disturbs its
momentum by giving it a random momentum kick.
That is, without looking, the particle exhibits an inter-
ference pattern on a distant screen behind the two slits.
However, if you look to see which way the particle
goes through the slits, the measurement disturbs the
system and theres no interference pattern on the dis-
tant screen. You just see a classical two-hump distribu-
tion. What do you think about Bohrs argument?

Chances Are 83
Quantum mechanics, in 168. Vacuum Energy?
matrix form, was born
when in July of 1925 Although the classical vacuum is a void, the quantum
Heisenberg, only vacuum is a virtual soup of particle-antiparticle pairs
twenty-three years of that interact with real atoms to produce the Lamb shift
age, had a creative (slight energy shift in atomic levels) and the Casimir
breakthrough on the effect (attraction of two plates in a vacuum). Does the
fog-shrouded island of quantum vacuum have energy content, or does the
Helgoland in the North energy in the soup average out to zero?
Sea. He took as his
guiding principle the
proposition that a the- 169. Casimir Effect
ory should not traffic
When two parallel uncharged metal sheets are placed in
with unverifiable
abstractions. He wanted a perfect vacuum, they attract each other with a tiny
to deal only with meas- force that is not gravitational. What is the source of
urable quantities. He this effect?
later told Einstein that
the idea of observable
quantities was actually 170. Squeezing Light
taken from his relativ- Laser light can be described in many ways. If one con-
ity, which had rejected siders just the amplitude and the phase of one ray in a
such concepts as laser beam, there will always be shot noisethat is,
absolute speed for the random variations caused by virtual particle interac-
same reason. In a letter
tions in the vacuum with the beam. Yet weve heard
to Pauli, dated 9 July
that there may be techniques to reduce the shot noise in
1925, Heiseberg wrote,
the amplitude, for example. Then what happens to the
My entire meager
efforts go toward killing shot noise in the phase?
off and suitably replac-
ing the concept of the
orbital paths that one
171. Electron Spin
cannot observe. Does the vacuum affect the spin of a particle such as an
H ANS C HRISTIAN VON electron?
B AEYER , TAMING THE ATOM : T HE
E MERGENCE OF THE V ISIBLE
M ICROWORLD
172. Superconductivity
One quantum mechanical effect that shows itself on the
macroscopic scale is superconductivity. Cooper-paired

84 Mad about Modern Physics


conduction electrons in superconductors have total Creative thinkers seem
spin zero, that is, their paired spins are opposite, even to possess the following
though their spatial separation can be enormouscen- characteristics in
timeters to meters, for examplebecause they have common:
opposite momenta. These pairs can act like bosons of an acutely sensitive
spin zero, which obey Bose-Einstein statistics. Any awareness of their
environment;
number of bosons can be in the same quantum state,
the ability to generate
that is, have the same four-momentum (e.g., dened by
a large number of
the energy and three-momentum) and spin. Therefore,
ideas in response to a
all bosons in the same collective superconducting state given problem;
have exactly the same energy. Yet this boson collective the ability to focus
state in a superconductor has a small energy width. their faculties in sus-
Any thoughts about the cause of this energy width? tained concentration;
in most cases the cre-
ative individuals work
173. Superfluidity place is likely to be a
He-4 below the lambda transition temperature 2.7 K cheerfully haphazard
can be analyzed as a two-uid liquid composed of He conglomeration of
atoms in the normal state and He atoms in the macro- complete disorder;
scopic superuid state. Superuidity is a property of the majority of truly
He-4 in the liquid state because He-4 atoms obey Bose- creative persons are
introverts;
Einstein statistics. Many He-4 atoms can be in the same
they tend to be much
macroscopic quantum statethat is, the same momen-
less concerned with
tum states for these atoms moving in the superuid. If
what others think of
so, then why can He-3 at low temperatures also them than most peo-
become a superuid? ple are; also, they are
often comparatively
indifferent to clothing
174. Gap Jumping and appearance. Cre-
Tiny detectors on a persons head have been used to ative people do not
sense tiny fluctuations in the brains magnetic field. seem to have a need
These SQUIDS, short for superconducting quantum to present themselves
interference devices, are the most sensitive of any kind in a favorable light to
and rely on the Josephson Effect, in which the Cooper others.
pairs of electrons in a superconductor can sometimes OTTO H. T HEIMER , A
G ENTLEMAN S G UIDE
jump a physical spatial gap in the material to another TO M ODERN P HYSICS
part of the superconductor. Manufactured SQUIDS

Chances Are 85
We all share a strange Oxide layer
mental time lag, a phe-
nomenon first brought
to light in the 1970s by
neurophysiologist Ben- Current in Current out
jamin Libet of the Uni-
versity of California at
San Francisco. In one
experiment, Libet docu-
mented a gap between
the time an individual
was conscious of the
Superconductor
decision to flex his fin-
ger (and recorded the
exact moment of that have a thin lm lling the gap. In fact, the direct cur-
consciousness) and the rent (DC) SQUID used in laboratories worldwide today
time his brain waves for sensing small magnetic elds is a superconducting
indicated that a flex was ring with two gaps! The best DC SQUIDS have an
imminent. The brain energy sensitivity capable of detecting a magnetic ux
activity occurred a third change corresponding to about 1034 joule in one sec-
of a second before the ond, about the mechanical energy required to raise an
person consciously electron 10 centimeters in one second. Why do the
decided to move his paired electrons jump the gap?
finger.
A NTONIO R. DAMASIO,
R EMEMBERING W HEN ,
S CIENTIFIC A MERIC AN
175. Nuclear Decay
(S EPTEMBER 2002) In the nucleus of an atom, neutrons and protons are
held by nuclear forces. Their total energy (ignoring the
mc2 contributions) is less than the barrier height poten-
tial energy. Yet some nuclear particles do escape. Any
thoughts about the reason for an escape?

176. Total Internal Reflection


In total internal reection of lightfor example, at a
glass-air interface or from a water-air surfaceif the
incident light is in the more dense medium, does the
light penetrate into the air beyond the interface?

86 Mad about Modern Physics


The first wrist watches
Light ray Air were decorative orna-
ments made for women,
and men consequently
shunned them as being
too effeminate. Again
war had its effectthe
first World War. A watch
that has to be fished
Glass prism out of a pocket in order
to be read is a lot less
convenient than one
177. Annihilation strapped to the wrist,
particularly when youre
We know that particles and their antiparticles annihi- trying to fire a machine
late each other. For example, the electron and the gun or charge over a hill
positron in positronium can annihilate into two pho- at a particular moment.
tons or three photons in the nal state, depending on Realizing this, govern-
the total angular momentum of the positronium. Why ments made wrist
would they do such a violent action? watches part of the
Hint: why does any event occur in nature? We standard equipment
know that the rate of any quantum mechanical event, issued to their soldiers.
by Fermis Golden Rule, is proportional to the proba- After the war, men sud-
denly felt that wrist
bility for the event times the density of nal states. Is
watches were accept-
this statement all we need to say?
able for civilian wear, a
trend which clearly has
178. A Bouncing Ball continued for both
sexes until the present
We see a kid bouncing a ball. According to quantum day.
mechanics, which applies to everything that happens, J O E LLEN B ARNETT, T IME S
why does the ball bounce? P ENDULUM : F ROM S UNDIALS TO
ATOMIC C LOCKS , THE F ASCINATING
H ISTORY OF T IMEKEEPING AND
H OW O UR D ISCOVERIES
179. The EPR Paradox C HANGED THE W ORLD
First of all, a short explanation. Although there are
other examples of the Einstein-Podolsky-Rosen (EPR)
paradox and violations of Bells inequalities, we
choose this version because we can provide you with

Chances Are 87
Einsteins unease with actual data to use in formulating your own solution to
quantum mechanics the paradox.
stemmed from a firm A source of two correlated identical particles of
belief in determinism. In opposite spins sits on the straight line between two
a 1931 essay The World identical particle detectors. Each detector can measure
as I See It, reprinted in the polarization state of the entering particle, and each
Ideas and Opinions,
detector has three polarization switch positions (1, 2,
he wrote, I do not at all
and 3) and two display lamps (green and red). Each
believe in human free-
time the experimenter pushes the button, the two cor-
dom in the philosophical
sense. Everybody acts related particles are shot out of the source in opposite
not only under external directions into the detectors. The data show two pat-
compulsion but also in terns: (1) For runs that have the same switch settings on
accordance with inner the two detectors, the same color lights ash on them.
necessity. Schopen- (2) For all runs, without regard for switch settings, the
hauers saying, A man pattern of ashing is completely random.
can do what he wants, This experiment gets to the heart of QM and the
but not want what he application of its three rules for events. We can use
wants, has been a very classical mechanics to explain the rst pattern: let the
real inspiration to me two particles carry the same instructions to be applied
since my youth; it has at the detectors. For example, this instruction set might
been a continual conso-
work: ash red at switch positions 1 and 3; ash green
lation in the face of
at switch position 2. But this classical scheme with pre-
lifes hardships, my own
determined instruction sets will not handle the second
and others, and an
unfailing well-spring of pattern. Why not? What is the surprising conclusion?
tolerance. Reproduced here is a small part of a data set for the
A DAPTED FROM experiment (from the Mermin reference in the answer).
A LBERT E INSTEIN , Each entry shows the switch settings and the colors the
I DEAS AND O PINIONS
lights ashed for each run. The switch settings are ran-
domly changed from run to run.

31GR 13RG 31RR 33GG

21RR 31RG 33GG 11GG

22RR 12RG 31RG 13RG

33GG 13GR 31RR 31RG

88 Mad about Modern Physics


11GG 22GG 33RR 23GR

23RR 12RG 32RG 31GR

32GR 12GR 31RG 23RG

12GR 22GG 11RR 22RR

12RG 23GR 23GR 12GR HEISENBERGS


UNCERTAINTY PRINCIPLE
11GG 33RR 12GG 32GR SIMPLIFIED
If you know where it is,
12GR 23GG 21GR 12GG
you dont know where
its going, or, If you
22RR 23GG 13GR 31GG
know where its going,
you dont know where it
12GG 33RR 33GG 32RG
is.
33RR 23GR 11GG 21GR

11RR 21GG 12RR 22GG

The ordinary adult never


180. Information and a Black Hole gives a thought to space-
time problems. . . . I, on
Classical information and quantum information are
the contrary, developed
not the same. Why? Because QM rule 2 tells us that in
so slowly that I did not
QM there can be a coherent superposition of quantum begin to wonder about
states. No such state exists in classical physics. So space and time until I
quantum information supersedes classical information. was an adult. I then
The classical and the quantum information content delved more deeply into
in a system, such as a chair, can be determined or esti- the problem than any
mated by standard techniques of classical and quantum other adult or child would
information theory. Suppose the chair is tossed into a have done.
black hole. The quantum information in the chair A LBERT E INSTEIN ( TO
N OBEL L AUREATE
seems to have gone with the chair into never-never J AMES F RANCK ) IN A LICE
land. Why should we worry about this information C ALAPRICE , T HE E XPANDED
loss? QUOTABLE E INSTEIN

Chances Are 89
9 Can This
Be Real?
A FTER QUANTUM MECHANICS EXPLAINED

the internal behavior of the atom in the 1920s


and the chemistry of atoms and molecules, physicists
turned toward understanding the atomic nucleus in the
1930s and 1940s. Rutherford in 1911 had determined that
practically all the atomic mass was in the nucleus, and of
course everyone knew that its positive protons balanced
the electron negative charges in the neutral atom. But what
held the nucleus of positive protons together? A nuclear
strong force was eventually identied in the 1970s as the
color interaction acting between quarks, and it is one of the
four known fundamental forces in nature. The second
nuclear force, the weak interaction, responsible for many
nuclear decays, was identied completely in the 1960s. By
the early 1980s three of the four fundamental interactions
had been unied into the Standard Model (SM) of Leptons

91
and Quarks. Only gravitation needs to be incorporated
into the unied model of nature. The selected chal-
lenges in this chapter range through the whole gamut
of nuclear and particle physics.

92 Mad about Modern Physics


181. Carbon-14 Dating Roughly once a second,
a subatomic particle
Carbon-14 is produced when cosmic rays collide with enters the earths
atoms in the atmosphere to create an energetic neutron atmosphere carrying as
that then collides with a nitrogen-14 atom (seven pro- much energy as a well-
tons, seven neutrons) to make a carbon-14 atom (six thrown rock. Somewhere
protons, eight neutrons) and a hydrogen atom (one in the universe, that
proton, zero neutrons). Carbon-14 is radioactive, with fact implies, there are
a half-life of 5,730 years. forces that can impart
These C-14 atoms combine with oxygen to form to a single proton 100
carbon dioxide, which plants absorb into plant cells million times the energy
achievable by the most
through photosynthesis. Animals and people eat the
powerful earthbound
plants and take in the C-14 as well as the normal non-
accelerators.
radioactive isotope C-12. The ratio of C-14 to C-12 in
J AMES W. C RONIN , T HOMAS
the air and in all living things at any given time is K. G AISSER , AND S IMON P.
assumed constant; about 1 in 10 trillion carbon atoms S WORDY, C OSMIC R AYS AT
THE E NERGY F RONTIER ,
are C-14. The C-14 atoms are always decaying, so after S CIENTIFIC A MERIC AN
an organism dies, no new carbon atoms are taken in (J ANUARY 1997)
and this ratio of C-14 to C-12 atoms decreases.
The carbon-14 radiocarbon dating of living and
once-living materials began with Willard Libby in the
1940s. Antiquities dated by C-14 agree with other date
records until they begin to disagree for dates more than
several thousand years ago. Why is there disagreement [Pierre Curie] was
impressed by Maries
in the dates between C-14 dating and the written
courage and her amaz-
records?
ing love of work and
fascinated by her
182. Nuclear Energy Levels lucidity, her challenging
questions, her reflective
In the 1930s and 1940s, physicists working on the answers.
energy states of the nucleus of an atom concentrated on J. A. DEL R EGATO,
various models, including a shell model using the R ADIOLOGIC AL P HYSICISTS
Schrdinger equation with an approximately constant
electrical potential inside the nucleus. Conceptually,
each nucleon is in a well-defined orbit within the
nucleus and moves in an averaged eld produced by all
the other nucleons. However, even though quantum

Can This Be Real? 93


On August 6, 1945, an
atom bomb dubbed
2p
Little Boy was dropped
from an American B-29 1f
bomber called the
Enola Gay on the city of
Hiroshima. It detonated
at 8:16 A.M. at a height
of 1,900 feet. Of 2s
Hiroshimas 330,000
1d
inhabitants, approxi-
mately 70,000 were Energy
killed instantly. By the
end of 1945, the death 1p
toll had risen to
140,000. Little Boy
used the gun assembly 1s
design and uranium-
235 as the fissionable
states such as n = 1, with l = 0, 1, 2, 3, etc., are possi-
material. Because the
gun design was an inef- ble in the shell model, the predicted energy levels did
ficient means of caus- not t the data. In fact, the actual energy levels were all
ing the chain reaction, scrambled compared to the shell-model theoretical
about 50 kilograms of predictions. Why?
89 percent U-235 and
14 kilograms of 50 per-
cent U-235 ended up 183. Nuclear Synthesis
being used. Of this it is
estimated that only The championship of nuclear binding energy is often
about 2 percent actually attributed to Fe-56, meaning that Fe-56 has the great-
fissioned. Three days est binding energy per nucleon and therefore is the
later another atom most stable nucleus. Most elements are synthesized in
bomb, dubbed Fat stars. Supposedly, elements higher on the periodic chart
Man, was dropped on than Fe cannot be synthesized in normal star burning
the city of Nagasaki. cycles. Why not? Actually, the sequence of nuclear syn-
Approximately 40,000 thesis does not stop at iron, because Ni also is synthe-
were killed instantly. sized. What happens to the Ni isotopes that are
synthesized?

94 Mad about Modern Physics


184. Heavy Element Synthesis By the time she enrolled
at the Sorbonne in 1891,
If we are truly the stuff of stars, then where do all the Maria Sklodowska was
heavier elements beyond iron come from if they are not twenty-four years old. In
made in normal star burning cycles? 1893 she passed the
license in physics, com-
ing first in her class. In
that year Maria
185. Neutron Decay
Sklodowska met Pierre
A free neutron will decay with a half-life of about 14.8 Curie. The meeting
minutes, but it is stable if combined into a nucleus. between the two was
Why would the neutron be stable in the nucleus? arranged for scientific
purposes, with little hint
of matchmaking. At the
time they met, both
186. Finely Tuned Carbon? Maria and Pierre consid-
Eventually a star exhausts its supply of hydrogen in its ered themselves des-
core, gravitational contraction occurs, the temperature tined for single lives.
reaches about 108 K, and helium burning can occur via After graduation Maria
the reaction 3He-4 C-12 + 2 photons. In fact, the intended to return to
Warsaw to look after her
nucleosynthesis of all the heavier elements essential for
aging father and teach
life relies on this reaction. However, the chance that
science. Pierre, mean-
three helium nuclei get together fast enough to form the while, at age thirty-four
carbon nucleus is negligible. So this critical reaction one of Frances leading
actually proceeds via an intermediate beryllium step young physicists, was
given by 2He-4 + (99 6) keV Be-8 followed by convinced that he would
never find a wife who
would tolerate his com-
7.70 MeV plete devotion to sci-
7.65 MeV ence. He was the first to
fall. Almost from the
beginning he realized
that in this severe Polish
7.40 MeV girl he had found the
woman of his dreams.
A DAPTED FROM M ARGARET
W ERTHEIM , P YTHAGORAS
T ROUSERS : G OD , P HYSICS , AND
THE G ENDER W ARS ; S USAN
0.00 MeV Q UINN , M ARIE C URIE : A L IFE

Can This Be Real? 95


An ingenious explana- Be-8 + He-4 C-12 + 2 photons. Since the Be-8
tion of the Michelson- lifetime of about 1017 second is much longer than the
Morley null result was He-4 + He-4 collision time in a star, the beryllium will
found by George F. be around long enough for the reaction to occur.
FitzGerald of Dublin in The total energy of the Be-8 nucleus and a He-4
1889. He suggested nucleus at rest is 7.4 MeV above the energy of the nor-
that the lengths of bod-
mal state of the C-12 nucleus. The radioactive state of
ies moving through the
the C-12 is 7.65 MeV above the normal state. If the
ether at velocity v con-
energy of the radioactive state were more than 7.7 MeV
tract in the direction of
their motion by a factor above the normal state, the formation of C-12 via Be-8
(1 v 2 / c2)1/2which plus He-4 would require the reactants to have at least
would just compensate 0.3 MeV of total kinetic energy, which is extremely
for the ether drift in the unlikely at the temperatures found in most stars.
Michelson-Morley The importance of this process is emphasized by
apparatus. A few years physicists who inject the Anthropic Principle, that cer-
later [Dutch physicist tain constants of nature have values that seem to have
Hendrik A. Lorentz] been mysteriously fine-tuned to just the values that
apparently independ- allow for the possibility of life. Recently, others have
entlymade the same introduced a further extension that claims that this car-
hypothesis and incorpo- bon nucleus coincidence can be explained only by the
rated it into his ever
intervention of a designer with some special concern
more comprehensive
for life. Both groups cite the closeness of the required
ether theory. This
energy to the actual limit, 7.7 MeV 7.65 MeV = 0.05
Lorentz-FitzGerald
contraction then MeV, a quantity less than 1% of 7.65 MeV, as their evi-
quickly diffused into the dence for the ne-tuning. Why is their reasoning sus-
literature. pect with regard to this carbon formation process?
WOLFGANG R INDLER ,
R ELATIVITY : S PECIAL , G ENERAL ,
AND C OSMOLOGIC AL
187. Proton-Proton Cycle
The thermonuclear reactions in the proton-proton
cycle inside the Sun convert four protons into an alpha
particle, two positrons, two electron neutrinos, and
two photons with the release of 26.7 MeV of energy.
First, two protons collide to form a deuteron H-2, then
this deuteron collides with a proton to form He-3, then
nally two He-3 nuclei must nd each other to collide

96 Mad about Modern Physics


and form an He-4. The overall representation of this Bertrand Russell would
proton-proton cycle is: sometimes liken the
scientific method to the
4H He-4 + 2e+ + 2 + 2.
following syllogism:
The six photons ultimately produced, including the Bread is made of
four 0.511 MeV photons from two positron-electron rock;
annihilations, take about a million years to reach the Rock tastes good;
Suns surface to be emitted eventually as visible pho- Therefore bread
tons, which then take about another eight minutes to tastes good.
reach Earth. The two neutrinos carry away about 3
In other words, you
percent of the energy to balance the energy conserva- can never be sure that
tion equation and to conserve lepton family number. correct conclusions
Presented as the primary source of our Suns energy, dont follow from incor-
this method of burning hydrogen is not the primary rect premises.
method for fusion energy in many stars. Why not?
What reaction sequence is the primary candidate? The illumination pro-
vided at eye level in
artificially lighted rooms
188. Oklo Nuclear Reactor is commonly from 50 to
In the 1970s, uranium samples from the Oklo uranium 100 footcandles, or less
mine in Gabon, Africa, were discovered to have abnor- than 10 percent of the
mally high concentrations of the isotope U-235, as high light normally available
as 3 percent, when only about 0.72 percent of the iso- outdoors in the shade of
a tree on a sunny day.
tope was expected in a natural source. Supposedly the
As a result, the total
high concentration of U-235 is explained by realizing
amount of light to which
that the uranium deposits at Oklo acted as a natural
a resident of Boston,
nuclear reactor. Could this natural reactor have been a say, is exposed in a
breeder reactor making its own Pu and U-235? conventionally lighted
indoor environment for
189. Human Radioactivity 16 hours a day is con-
siderably less than
Radiation doses are expressed in SI units as milliSievert would impinge on him if
(mSv) effective doses. This unit takes into account he spent a single hour
the type, the intensity and duration of radiation, the each day outdoors.
amount and type of body tissues irradiated, and the R ICHARD J. W URTMAN , T HE
E FFECTS OF L IGHT ON THE
different radiation sensitivity of the irradiated tissues.
H UMAN B ODY, S CIENTIFIC
The average natural background dose rate in many A MERIC AN (J ULY 1975)

Can This Be Real? 97


In early 1940, Paul countries is 15 mSv a year. On average, medical expo-
Harteck, a German physi- sures contribute about another 0.50.7 mSv a year. The
cal chemist, felt hed need current recommended limit for occupational exposure
up to 300 kilograms of in many countries is about 20mSv effective dose per
uranium to test his idea of year averaged over ve consecutive years.
using carbon dioxide as a
The typical human adult body has an inherent
moderator. He arranged to
internal radiation dose from its natural amounts of
get the frozen carbon
dioxide (dry ice) from I. G. radioactive elements, including its major contribution
Farben, and the necessary of about 40 milligrams of radioactive potassium as the
uranium from Heisenberg. isotope K-40, which has a half-life of about 1.3
But at the last moment, Gigayears. This isotope is not the result of articial
Farben declared they radioactivity but remains from the formation of potas-
could only supply the dry sium in the supernova that gave birth to our Solar Sys-
ice until early June; theyd tem about 5 billion years ago. There has not been
need it after that for enough time for all of the radioactive potassium to
keeping food fresh during decay, so that is why there is so much in our bodies.
the hot summer months.
Eileen wonders whether this inherent K-40 radioactive
Harteck scraped together
source is exposing our bodies to more than the recom-
about 200 kilos of ura-
nium, but with that low mended limit? Is the limit exceeded when several peo-
amount his results were ple gather together in a small circle?
inconclusive; Germany did
not go ahead with the 190. Nuclear Surprises?
easy, dry ice reactor that
would almost certainly Which of the following statements is true?
have given them plenty of 1. A typical coal burning power plant releases more
radioactive metal early on radioactive materials into the air than a typical
in the war. Thus was the nuclear reactor plant.
clear hot weather of that
summerso often cursed 2. Spreading all the nuclear waste equally around the
by the Allies for letting surface of the planet will hardly change the back-
Panzer armies advance ground radiation level at all.
into Francecentral to
forestalling this greater 191. Cold Fusion
evil.
M ARK WALKER , G ERMAN Is cold fusionthat is, the fusion of two deuterium
N ATIONAL S OCIALISM AND THE nuclei at about room temperaturea possibility, or
QUEST FOR N UCLEAR P OWER
can this process be eliminated by theoretical arguments
19391949
alone?

98 Mad about Modern Physics


192. Fission of U-235 Although we are quite
unaware of their pres-
ence, there are, on the
U-235
average, some 400
Fragments microwave photons in
any cubic centimeter in
Neutrons the universe left over
from the big bang.

In A.D. 499 the Indian


astronomer Aryabhata
presented a treatise on
mathematics and astron-
During World War II the Germans and the Allies were omy, the Aryabhatiya.
both working on projects related to nuclear weapons The Aryabhatiya is a
development. One can calculate the minimum mass of summary of Hindu math-
ematics up to his time,
U-235 required for a ssion weapon from present-day
including astronomy,
nuclear physics data sheets. That value is the amount
spherical trigonometry,
required if the neutrons produced by the fission of
arithmetic, algebra, and
U-235 encounter stationary target nuclei. The problem plane trigonometry. The
is much more difcult for two important reasons. Can Aryabhatiya presented a
you identify them? new treatment of the
position of the planets in
space. It proposed that
193. Minimal Nuclear Device the apparent rotation of
What is the minimum mass of pure U-235 or Pu-239 the heavens was due to
required in a device for a nuclear event? How would the axial rotation of the
you estimate this value? Earth. Moreover, Aryab-
hata conceptualized the
orbits of the planets as
194. Large Nuclei ellipses, a thousand
Small nuclei that become excited and deformed lose years before Kepler.
D ICK T ERESI , L OST
their energy by breaking up into smaller fragments. A D ISCOVERIES : T HE A NCIENT R OOTS
larger nucleus, with 150 or more nucleons, stores OF M ODERN S CIENCE FROM THE

most of its excitation energy as rotational energy. As B ABYLONIANS TO THE M AYA

they slow down and de-excite, these nuclei lose energy

Can This Be Real? 99


In the end wethat is, and return to their unexcited shape. What do these
Bohr, Pauli and Iknew nuclei emit, and how would you characterize the
that we could now be energy spectrum?
sure of our ground, and
Einstein understood
that the new interpreta- 195. Human Hearing
tion of quantum The human eardrum is sensitive to displacements of
mechanics cannot be
less than the diameter of an atomic nucleus. How have
refuted so simply. But
such minute displacements been measured via nuclear
he still stood by his
physics techniques?
watchword, which he
clothed in the words:
God does not play at 196. 1908 Siberia Meteorite
dice. To which Bohr
could only answer: But In an article by Andrew Chakin in Sky & Telescope in
still, it cannot be for us January 1984, pages 1824, the author states:
to tell God, how He is to A grande dame of scientic mysteriesthe Tun-
run the world.
guska eventturned 75 last summer, her charm
W ERNER H EISENBERG ,
E NCOUNTERS WITH E INSTEIN very much intact. She continues to seduce both
scientist and charlatan alike, both hoping to
explain what happened over a remote stretch of
Siberian taiga on June 30, 1908. All that can be
When Rutherford was said from direct eyewitnesses is that a fireball
offered a position at nearly as bright as the Sun streaked to Earth out of
Yale that required some a cloudless morning sky. The bolides plunge was
teaching, he turned it abruptly terminated by an explosion so great that
down, commenting, it registered on seismic stations across Eurasia.
They act as if the uni- The resulting shock wave circled the Earth twice.
versity was made for
students. The article relates that in 1908 in Siberia a huge
A DAPTED FROM E. S EGR , meteorite is supposed to have crashed in the forest,
F ROM X- RAYS TO QUARKS causing huge res and a crater many kilometers long,
but no rocky debris was ever found.
By radiocarbon dating tree rings from old trees that
have been living since 1908, Willard Libby and Edward
Teller in 1963 may have learned something very
important about the constitution of the meteorite.
What could the radiocarbon data have suggested?

100 Mad about Modern Physics


197. The Standard Model The Schrdinger equa-
tion, published in March
of 1926, was designed to
Electron neutrino Up quark explain almost all
Electron Down quark aspects of the behavior
of electrons in terms of
de Broglie waves, rather
Muon neutrino Charm quark than of matrices. Physi-
Muon cists now could visualize
Strange quark
the atom in terms of
continuous processes
the ripple and flow of
Tau neutrino Top quark
standing waves
Tau Bottom quark whereas with matrices
they had to deal with
Heisenbergs assertion
that the nature of the
The Standard Model (SM) of Leptons and Quarks is
microworld was discon-
the most successful physics model of all in terms of test-
tinuous and impossible
ing its concepts. The model has six leptons in pairs in to picture. Little wonder
three lepton families and six quarks in pairs in three that many physicists
quark families, with the quarks in three different col- threw away their matri-
ors. Aesthetically, the matching of three to three is ces and started working
pleasing. Mathematically, this matching of numbers of with Schrdingers
lepton and quark families cancels out innities in quan- methods. Even today,
tum eld theory calculations, such as the innities that most physicists would
would arise from the famous triangle anomaly. How- say that the Schrdinger
ever important this family matching may be, can you equation, being nonrela-
provide a fundamental physics argument for the spe- tivistic, has no right to
cific matching of the first lepton family to the first be this good.
A DAPTED FROM
quark family, of the second lepton family to the second ROBERT P. C REASE AND
quark family, and so on? C HARLES C. M ANN ,
T HE S ECOND C REATION :
M AKERS OF THE R EVOLUTION IN
198. Spontaneous Symmetry 20 TH -C ENTURY P HYSICS

Breaking
Spontaneous symmetry breaking is a concept rst intro-
duced by W. Heisenberg in describing ferromagnetic

Can This Be Real? 101


Maria Sklodowska was a materials. A ferromagnet has a perfect geometric sym-
daughter of a Polish metry until the Curie transition temperature is reached;
freethinker but reared then the material becomes magnetized and one particu-
by a Catholic mother. lar direction of magnetization is chosen. The theory is
She abandoned the symmetrical still, but the actual material is not. One can
church before she was summarize the process by stating that microscopic
twenty and her marriage
events can have macroscopic consequences. Near the
to Pierre Curie was a
critical point of a phase transition, small, random uc-
purely civil ceremony
tuations can grow to make their presence felt through-
because she says in her
memoir of him, Pierre out the material. A few aligned spins can propagate
belonged to no religion their inuence throughout the whole crystal, and the
and I did not practice symmetry is broken.
any. Other examples are the Schrdinger equation and
Maxwells equations. As successful in helping to
describe nature as they have been, these equations
have more symmetry than the underlying phenomena
they describe. Interest in their symmetry-breaking
applications has led to signicant new insights into new
connections between macroscopic and microscopic
phenomena.
In particle physics, the spontaneous symmetry
breaking is achieved by the Higgs mechanism. The
One must make of life a
Standard Model of Leptons and Quarks relies upon
dream, and of that
dream a reality. the Higgs particle to spontaneously break symmetry to
P IERRE C URIE provide three of the electroweak bosons with mass
while leaving the photon massless. Simultaneously, all
the leptons and quarks get their mass values. Moreover,
the effect of the Higgs eld is to provide a frame of ref-
erence in the vacuum for the isotopic spin directions
that distinguish the particles of each groupingfor
example, neutrons from protons.
Is spontaneous symmetry breaking by the Higgs
mechanism the only way to go? Are there other ways to
spontaneously break symmetry to achieve the Standard
Model of Leptons and Quarks?

102 Mad about Modern Physics


199. Proton Mass Contrary to the claim
found in some dictionar-
Kate sees that the chart of the fundamental leptons ies, the word algebra
and quarks shows that the up and down quark masses does not derive from an
are ~ 5 MeV/c2 each. Yet the proton, which is com- Arabic expression for
posed of two up quarks and one down quark as the bone setting but rather
combination uud, has an enormous mass of 938 it means compulsion,
MeV/c2. She asks why there is such a large mass differ- as in compelling the
ence between constituents and the nal product. unknown x to assume a
numerical value.

200. Right- and Left-Handed


Neutrinos?
The amount of ultra-
Neutrinos are lepton family partners to the electron, violet radiation that
muon, and tau particles of the Standard Model of Lep- penetrates the atmos-
tons and Quarks. Each neutrino is thought to be dis- phere varies markedly
tinct, the electron neutrino being different from the with the season: in the
muon neutrino, for example. We now know, however, northern third of the
that each lepton family neutrino type has a very small U.S. the total amount
mass and is actually a linear combination of three fun- of erythemal (skin-
damental neutrino states: 1, 2, and 3. inflaming) radiation that
reaches the ground in
For the weak interaction, there is the left-handed
December is only about
doublet state | L, eL> and the two right-handed singlet
a fifteenth of the
states | R > and | eR >, with the consequence that the
amount present in June.
right-handed states interact with the Z 0 boson but do R ICHARD J. W URTMAN , T HE
not participate in the weak interaction mediated by the E FFECTS OF L IGHT ON THE
W + and W bosons. The left-handed doublet interacts H UMAN B ODY, S CIENTIFIC
A MERIC AN (J ULY 1975)
with all three weak bosons. Must one resort solely to
the explanation that is how Nature behaves, or is
there another fundamental reason for left-handed dou-
blet and right-handed singlet states?

201. Physics without Equations


John von Neumann and Stanislaw Ulam in the
1940s were among the rst to consider attempting to

Can This Be Real? 103


In 1911 Marie
Sklodowska-Curie, by Cellular Automaton Rule 30 for 50 steps
then a double Nobel
Prize winner, was asked
to write a letter of rec-
ommendation for Albert
Einstein who was being
considered for a posi-
tion at the ETH, the
Zurich Polytechnic. She
wrote, In Brussels,
where I attended a sci- Time steps
entific conference in
which M. Einstein also understand natural phenomena via cellular automata
participated, I was able
and computers. Cellular automata (CA) involve adja-
to admire the clarity of
cent cells in a 1-D, 2-D, 3-D, and so on grid of cells (or
his intellect, the breadth
nodes) that take on new numerical values at each tick
of his information, and
the profundity of his of the clock according to given rules. The future state
knowledge. Considering of each cell is determined only by the present state of its
that M. Einstein is still local neighborhood. One can even remove the external
very young, one is justi- clock and still maintain a progression of states within
fied in placing great the CA grid to simulate the passage of time.
hopes in him and in Some people claim that all of nature will be simu-
regarding him as one of lated eventually on computers using cellular automata.
the leading theoreti- Certainly, uid ows and other large-scale systems in
cians of the future. nature can be simulated to a reasonable degree by CA.
A LBRECHT F LSING , A LBERT But concerning the motion of electrons and other fun-
E INSTEIN : A B IOGRAPHY
damental particles, which involves quantum mechanics
and the fundamental interactions, how will these parti-
cles show their behavior with this CA technique?

104 Mad about Modern Physics


10 Over My
Head
U NTIL THE 1920 S NO ONE WAS SURE THAT WE

were seeing stars outside our own Milky Way galaxy.


Then, after Edwin Hubble established in 1927 that extra-
galactic galaxies existed and had recession velocities pro-
portional to their distance from us, the cosmology game was
afoot. The rules of the game had been established already by
Einstein in 1916 with his general theory of relativity (GTR).
The verication of one of its major predictions by analyzing
the deection of starlight passing near the Sun during the
1919 total solar eclipse told everyone that solid theoretical
foundations were in place. But only in the 1990s did the vast
accumulation of data on distant objects, by orbiting satellites
such as the Hubble Space Telescope and the COBE micro-
wave detector, and by a new generation of ground-based
telescopes, transform a conjectural science into real testing
of models of the universe. We present a sample of challenges
from a vast range of possibilities.

105
The possibility that a 202. Olbers Paradox
massive object could bend
light rays was discussed While walking through the elds one night with his
by Newton as early as dog, Jan looked up to see a remarkably clear night sky.
1704, and later by Henry In an instant, a famous question ashed in his mind:
Cavendish. However, the Why is the sky dark at night? With his engineering
first actual calculation background, he determined that if the universe is uni-
of the deflection angle formly lled with stars, then their successive spherical
was published by a shells would contribute equal amounts, and the sky
Bavarian astronomer
should be ablaze with light from all directions. Yet the
named Johann Georg
night sky remains dark. What is the resolution of this
von Soldner in 1803.
Assuming that light was a paradox?
corpuscle undergoing the
same gravitational attrac- 203. Headlight Effect
tion as a material particle,
Soldner determined how We live in a universe in which very distant stars have
much bending would occur enormous cosmological redshifts of their light. This
for a path that skimmed fact is interpreted as a cosmological recession velocity
the surface of the Sun. at nearly the speed of light. Unusual relativistic effects
The deviation, while small, can be observed when looking at such fast-moving light
is calculable, and sources. We consider a more local version here.
Soldners value was
Suppose you are standing next to a straight test
0.875 seconds of arc. In
track that carries a vehicle with a light that shines in a
1911 Einstein, using the
principle of equivalence, cone with an apex angle 45 degrees about the forward
obtained the same result. direction. In the past, you have always seen the light as
Then in 1915, using the the vehicle approached. One day the vehicle for the rst
equations of the general time is able to reach its highest speed ever: v = 0.9999
theory of relativity, times the speed of light. But this time you do not see the
Einstein found that the light as it approaches. Why not? Do you see the vehicle
deflection had to be 1.75 as it passes and then recedes into the distance? Suppose
seconds of arc, twice the a distant star or galaxy is approaching you at this speed.
previous value. In recent What would you see? And if receding?
decades the result as
been confirmed to a
precision better than 0.1 204. Incommunicado?
per cent.
C LIFFORD M. W ILL ,
In later problems we will encounter the behavior of light
W AS E INSTEIN R IGHT ? near a black hole, particularly its inability to escape

106 Mad about Modern Physics


from a black hole. That is, if you are trapped inside a At age twelve Einstein
black hole and are still alive, you cannot communicate suddenly became com-
with your friends outside because nothing escapes. pletely irreligious. Ironi-
Meanwhile, consider a related problem in a normal cally, this conversion was
space environment. Suppose you and your friend are in the consequence of the
only religious custom his
separate rocketships that begin next to each other and
parents observed, namely
accelerate with respect to the stars in opposite direc-
to host a poor Jewish stu-
tions. You both maintain steady pulsed light communi- dent for a weekly meal.
cation with each other via intense, nondiverging laser The beneficiary was Max
beams. But your relative speed is increasing each sec- Talmud (later Talmey),
ond as the separation distance grows ever faster. Will a medical student from
there come a time when neither of you will receive the Poland, ten years older
others light beam? than Albert. Talmud
directed his attention to
popular science books as
205. Local Accelerations well as to various books in
mathematics. Einstein
summed up the results of
Test masses Talmeys influence:
Through the reading of
popular scientific books I
soon reached the convic-
Massive body tion that much in the sto-
Laboratory ries of the Bible could not
be true. The consequence
was a positively fanatic
Einstein formulated the general theory of relativity [orgy of] freethinking
(GTR) in 1915 based on his Equivalence Principle. In coupled with the impres-
prerelativistic terms, a uniform gravitational eld of sion that youth is inten-
strength g may be exactly simulated inside a rigid lab- tionally being deceived by
oratory in a completely gravity-free region of space by the state through lies.
Suspicion against every
accelerating this laboratory with a constant accelera-
kind of authority grew out
tion g m/s2 relative to an inertial frame. By releasing
of this experience.
two small test masses, their behavior reveals the physi- A DAPTED FROM M AX
cal environment. J AMMER , E INSTEIN AND
Suppose an unseen massive body is near the rigid R ELIGION P HYSICS AND T HEOLOGY

laboratory. What behaviors of the two small test masses


will reveal its presence?

Over My Head 107


Are the planets arranged 206. Twin Paradox
so that the gravitational
perturbations between The twins are ve years old when one of then is sent off
them are smaller than in a spaceship that travels nearly the speed of light and
would be expected from a the other remains on spaceship Earth. After 50 years
random configuration, thus Earth time the spaceship returns. The twins greet each
resulting in long-term sta- other and compare their experiences. We know that
bility of the system? Some the twin who experiences accelerations will age slower
argue that an example of and return to Earth much younger than 55 years old.
this favorable arrangement Precisely how does the general theory of relativity
is given by Neptune and explain the aging of the twin during accelerations?
Pluto, whose orbits appear
to cross if we neglect their
inclinations. Since their 207. Twin Watches
orbital periods are in the This problem came from Richard P. Feynman in the
ratio 3:2 Pluto never gets 1960s while one of us (F. P.), an undergraduate, was with
near to Neptune and actu- him in his car on the way to Malibu, California, where
ally approaches more he gave weekly physics lectures. The third person in the
closely to Uranus. It is
car, B. Winstein, then a graduate student in physics, con-
worth noting that the
tributed to a discussion that became quite involved!
orbital planes of the plan-
Charlotte holds two identical ideal watches at the
ets are not coincident with
each other or with the same height, one in each hand. She holds one steady in
equatorial plane of the sun, her left hand and tosses the other into the air straight
which is inclined at about up. At the instant the upward-moving watch is along-
7 to the ecliptic. side the other at the same height above the ground, she
sees that the two watches are synchronized, with the
exact same readout value. Later, on its downward free-
fall path, she reads the time on both watches when they
are again alongside each other and at the same height
above the ground. Assuming that the moving watch is
always in free fall, what would you predict for the two
watch readings?

208. Global Positioning Satellites


The global positioning system (GPS) is a modern marvel,
with a constellation of at least 24 satellites, each in a 12-
hour orbit at an altitude of about 20,200 kilometers,

108 Mad about Modern Physics


whizzing around Earth at enormous speeds with respect The Russian astrophysi-
to the ground beneath them. Each satellite knows its cist George Gamow
own position and sends out signals with this informa- decided early in life
tion. The GPS handheld receiver uses the signals from at that traditional religion
least four different satellites to calculate its own position could not be trusted.
to within a few meters or better when a local reference After watching Commu-
nion in the Russian
signal is present. Yet within minutes the accuracy would
Orthodox Church, he
reduce to many kilometers of error if one of Einsteins
decided to see for him-
discoveries were not an essential part of the calculations
self whether red wine
in the GPS system. What are we referring to? and bread could trans-
form into the blood and
209. Solar Redshift flesh of Jesus. He held
a bit of the blessed
The light emitted from the Sun shows a redshift of the
bread and wine in his
spectral lines even though our distance to the Sun is
mouth, ran home from
xed during the measurement process. Why so? church, and placed the
specimen under the
210. Orbiting Bodies lens of his new toy
microscope. It looked
When a body such as a planet orbits around a more
identical to an ordinary
massive body such as the Sun, the orbit does not close
bread crumb that he
on itself, as expected from Newtons universal law of
had prepared at home
gravitation and Keplers laws. The general theory of earlier for comparison.
relativity (GTR) calculates the correct value for this I think this was the
precession of the orbital ellipse, determining that its experiment which made
complicated equations reduce to an equation similar in me a scientist, he
form to that of the classical Kepler problem, with an recalled. In the 1940s,
additional quadratic term that causes the precession. Gamow and others pre-
Can you provide a conceptual argument in GTR for the dicted the existence of
precession of the ellipse? a cosmic background
radiation.
C OREY S. P OWELL , G OD IN
211. Gravitational Lensing THE E QUATION : H OW E INSTEIN
B EC AME THE P ROPHET OF THE
In examining the universe, astronomers utilize a tech- N EW R ELIGIOUS E RA
nique called gravitational lensing of the light from dis-
tance stars. Supposedly, space itself can act as a lens for
light rays. How can the emptiness of spacethe vac-
uum itselfaround stars and galaxies focus light?

Over My Head 109


In various writings, not
all published, Isaac New-
ton intimated that
comets were divine
agents destined to Earth Quasar
reconstitute the entire
solar system, to prepare
sites for new creations,
and to usher in the Galaxy lens
Millennium.
S ARA S CHECHNER G ENUTH ,
C OMETS , P OPULAR C ULTURE , 212. Cosmological Redshifts
AND THE B IRTH OF M ODERN
C OSMOLOGY The light from a distant galaxy can exhibit a signicant
cosmological redshift. If the cosmological redshift is
not a velocity redshift, what is its origin? Can the two
In a letter to E. Bsching,
the author of There Is effects be distinguished from each other by observing
No God, dated 25 the spectrum of the galaxy or other light source?
October 1929, Einstein
declared that a belief in 213. Tired-Light Hypothesis
a personal God seems
preferable to the lack of Since the 1920s there has been a popular hypothesis
any transcendental out- trying to explain the cosmological redshift as a so-
look of life. called tired-light effectthat is, the light loses energy as
M AX J AMMER , E INSTEIN its photons race through space, getting more tired with
AND R ELIGION P HYSICS
distance, like a long-distance runner completing a race.
AND T HEOLOGY
What two specic pieces of evidence rule out this expla-
nation for the cosmological redshift?
My comprehension of
God comes from the
deeply felt conviction of 214. Black Hole Entropy
a superior intelligence A black hole has an entropy proportional to its surface
that reveals itself in the area, so it must have a temperature above absolute
knowable world. In zero. What would be evidence for this temperature?
common terms, one
can describe it as
pantheistic (Spinoza). 215. Black Hole Collision
A LBERT E INSTEIN , 1923 IN Two black holes collide head-on. Will they coalesce
A LICE C ALAPRICE , T HE
E XPANDED QUOTABLE E INSTEIN into one black hole?

110 Mad about Modern Physics


216. Centrifugal Force Paradox RIEMANN ANTICIPATES
EINSTEIN
The general theory of relativity (GTR) predicts that in Ever since Newton, scien-
certain circumstances the centrifugal force may be tists had considered a
directed toward, not away from, the center of circular force to be an instanta-
motion. In fact, if an astronaut could steer a spacecraft neous interaction between
sufciently near to a black hole, the astronaut would two distant bodies. How-
feel a centrifugal force pushing inward, not outward, ever, over the centuries,
contrary to everyday experience! What is the concep- critics argued that this
tual explanation of the unusual result? action-at-a-distance was
unnatural, because it meant
that one body could change
217. Geodesics and Light Rays the direction of another
without even touching it.
In conventional geometry, the geodesic is the shortest
Georg Bernhard
curve between two points measured by counting how Riemann (18261866)
many rulers t along the curve. When learning relativ- developed a radically new
ity theory, one often reads statements that conict with physical picture, banishing
intuition, such as the following: In any space-time, the action-at-a-distance
with or without a gravitational field, light always principle. To Riemann,
moves along geodesics and traces out the geometry of force was a consequence
space-time. In a space warped by a gravitational of geometry. He con-
eld, the light rays are curved and in general do not cluded that electricity,
coincide with geodesics. Why are these phrases, taken magnetism, and gravity are
caused by the crumpling
from the general theory of relativity (GTR), not really
of our three-dimensional
in conict with each other?
universe in the unseen
fourth dimension. Thus a
218. Galaxy Rotation force has no independent
life of its own; it is only
the apparent effect
Velocity
caused by the distortion
of geometry.
M ICHIO K AKU , H YPERSPACE :
A S CIENTIFIC O DYSSEY THROUGH
PARALLEL U NIVERSES ,
T IME W ARPS , AND T HE T ENTH
Visible edge D IMENSION

Radius

Over My Head 111


Venus always presents One of the great surprises in astronomy is the rota-
the same face to Earth tional behavior of galaxiesthat is, all the stars in the
when the two planets galactic disk revolve at roughly the same tangential
are at their closest
speed! Two immediate conflicts with conventional
approach, suggesting
that its peculiar rotation physics arise: (1) If Newtonian gravitation and Keplers
may be due in part to laws apply, they would dictate a decrease in star veloc-
terrestrial action. A ity with increasing radius from the galactic nucleus,
simple calculation will like the planets of the Solar System do. (2) If the spiral
show, however, that arms of a spiral galaxy are to retain their integrity and
solar tidal action will persist for at least ten complete revolutions, as they
dominate that of Earth
have for the Milky Way, there must be something pre-
on Venus, and so it is
difficult to see how venting them from wrapping numerous times. By
such a situation has assuming that Newtons universal law of gravitation
evolved. applies to these galactic problems, what general type of
mass/energy distribution must be proposed to explain
God is what mind the rotational velocity curve? What further hypotheses
becomes when it has might you propose?
passed beyond the
scale of our comprehen-
sion. God may be either 219. Cosmic Background Radiation
a world-soul or a collec-
tion of world souls. So I Cosmic background radiation was rst detected in the
am thinking that atoms microwave region in the 1960s and exhibits a perfect
and humans and God blackbody spectrum equivalent to the radiation from a
may have minds that source at a temperature of 2.72 K. One would expect
differ in degree but not
in kind.
F REEMAN DYSON
Intensity (W/m3)

3
2.72 K
2

0.005 0.001 0.0015 0.002


Wavelength (m)

112 Mad about Modern Physics


lots of remnant starlight all over the universe in all NOT ALL STARS ARE
parts of the electromagnetic spectrum emitted for the ROUND!
past 10 billion years or so. Yet this microwave back- Many red giant stars,
ground radiation is not this light emitted from stars. including the bright star
How do we exclude this starlight? Betelgeuse and the well-
known variable star Mira,
exhibit peculiar egglike
220. Planetary Spacings shapes, presumably
For some people the orbital radii for the planets in the because of the huge
convection currents roil-
Solar System seem to follow a regular pattern. The pat-
ing their filmy outer lay-
tern was originally called the Titius-Bode law before
ers. Astronomers also
Pluto was discovered. According to this numerical
found huge cocoons of
scheme, the semimajor axis of a planets orbit a = 0.4 + hydrogen gas surround-
(0.3) 2n, where n is taken as negative innity for Mer- ing hot blue stars and
cury, zero for Venus, and has increasing integer value clouds of titanium oxide
by one unit for each successive planet. Neptune does billowing off red giants
not t in this scheme, and the scheme may not repre- surfaces.
sent an underlying physics. M OONBALL : A STRONOMERS
B EAT A PATH TO H IGH
R ESOLUTION , S CIENTIFIC
Jupiter A MERIC AN (J ULY 1993)

Actual
Many of the oldest
Bode stars and star clusters
in the galactic halo of
Nottale the Milky Way move in
retrograde orbitsthat
Planet distances is, they revolve around
the galactic center in a
In the 1990s, applying chaos theory to gravitation- direction opposite to
ally bound systems, L. Nottale found that statistical ts that of most other stars.
S IDNEY VAN DEN B ERGH
indicate that the planet orbital distances, including that
AND J AMES E. H ESSER , H OW
of Pluto, and the major satellites of the Jovian planets, THE M ILKY WAY F ORMED,
follow a numerical scheme with their orbital radii pro- S CIENTIFIC A MERIC AN
(J ANUARY 1993)
portional to the squares of integers n2 extremely well!
The planets were divided into two groups, the inner

Over My Head 113


Experience may suggest planets Mercury, Venus, Earth, and Mars being at
the appropriate mathe- n = 3, 4, 5, and 6, respectively, and the outer planets,
matical concepts, but starting with Jupiter at n = 1. The two sets can be
they most certainly can-
combined into one set with Mercury at n = 3, Jupiter at
not be deduced from it.
Experience remains, of n = 10, and so on. The lack of planets at some integers
course, the sole criterion can be attributed to the history of the Solar System and
of physical utility of a does not indicate a failure of the prediction.
mathematical construc- Other researchers claim that the Nottale sets of
tion. But the creative integers are not unique and that several alternative sets
principle resides in math- of integers exhibit excellent ts, raising the question of
ematics. In a certain
whether there is actually a unique pattern in the orbital
sense, therefore, I hold it
true that pure thought can spacings. In addition, there are known orbital reso-
grasp reality, as the nances for the satellites of the Jovian planets that cause
ancients dreamed. some of the apparent patterns in the satellite spacings.
A LBERT E INSTEIN , 1933, IN How would you determine whether any of the
G ERALD H OLTON , T HEMATIC O RI -
claimed patterns are physically signicant or simply
GINS OF S CIENTIFIC T HOUGHT
numerology?

Popular books claim that


Fred Hoyle coined the 221. Entropy in the Big Bang
term big bang to ridicule
The primordial reball was a thermal statea hot gas
the theory; but Hoyle dis-
puted that. The BBC was in expanding thermal equilibrium. But the term ther-
all radio in those days, mal equilibrium refers to a state of maximum entropy.
and on radio, you have no However, the second law demands that in its initial
visual aids, so its essen- state, the entropy of our universe was at some sort of
tial to arrest the attention minimum, not a maximum! How would you resolve
of the listener and to hold
this paradox raised by R. Penrose?
his comprehension by
choosing striking words.
There was no way in which 222. Gravitational Wave Detectors
I coined the phrase to be
derogatory; I coined it to Radio wave detectors are calibrated by sending out
be striking, so that people radio waves from a transmitter several wavelengths
would know the difference away and more. Why cant builders of gravitational
between the steady state
wave detectors do the same thing? After all, one could
model and the big bang
model. put two large masses at opposite sides of a rotating
K EN C ROSWELL , platform and spin them around to have a gravitational
T HE U NIVERSE AT M IDNIGH wave source for detector calibration.

114 Mad about Modern Physics


223. Space Curvature WHATS IN A NAME?
Planck happened upon
The general theory of relativity (GTR) has been Relativtheorie when
checked and veried at local distance scales. We know in 1906 he was groping
that the GTR may not explain the rotation curves of for a name to distin-
galaxies without the introduction of dark matter. We guish the Lorentz-
do not expect the GTR to work for extremely small dis- Einstein theory (of the
tances, for extremely short time intervals, or for cos- deformable electron)
mological distancesthat is, whether the GTR from Abrahams (theory
of the rigid, spherical
correctly explains the universe on a global scale. The
electron). It was in the
GTR, for example, allows for the overall curvature of discussion following
space but does not predict its global value. In better Plancks 1906 lecture
words, the GTR does not fully predict the geometry of on Kaufmanns experi-
space, neither determining the global shape nor the mental electron data
connectedness of space. and their theoretical
Suppose you were given the task of measuring the interpretation that
overall curvature of space. One way might be to count Plancks term
the number of stars at each radial distance, say, and plot Relativtheorie was
embellished by the
the number found versus distance. How does this
experimentalist
method determine the curvature of space? Does this A. H. Bucherer to
technique work for both continuous and discrete spaces? Relativittstheorie.
Although Einstein
had in his published
224. The Total Energy work from the very
The total energy in the observable universe can be first referred to the
shown to be zero by adding the total mass energy in Relativittsprinzip, he
matter and radiation to the total gravitational potential did not use the designa-
energy. That is: energy total = mass energy + gravita- tion Relativittstheorie
in print until 1907, as
tional energy. Does this result mean that the creation of
a variant of Plancks
matter out of nothing contradicts no physical conser- expression. Felix
vation law? Kleins suggestion of
Invariantentheorie did
not catch on.
225. Different Universes? E RIC S HELDON , R ELATIVITY
OR I NVARIANCE ?, A MERIC AN
The present limited understanding of our universe
J OURNAL OF P HYSICS
allows for much speculation with regard to whether (S EPTEMBER 1986)
we live in but one of many universes, possibly with

Over My Head 115


This world is indeed a connections among them. These wild conjectures are
living being endowed permitted simply because we do not know enough
with a soul and intelli- about the origins of the fundamental constants such as
gence . . . a single living Plancks constant, the gravitational constant, or the lep-
entity containing all ton and quark masses, for example. Indeed, the other
other living entities, possible universes could have different values for these
which by their nature
constants. Suppose that the lepton and quark mass
are related.
values are discovered to be determined by some funda-
P LATO
mental properties in mathematics such as the invariants
of elliptic functions. How might this discovery end
speculation about many universes?
Cosmic bombardment
of Mars by planetary
debris has sent hun-
dreds of tons of Martian
rocks falling to Earth.
Experts estimate that a
Mars rock lands as
often as every three
days, and that billions
have done so over
timeeven though only
13 of them have been
identified so far. The
rain of debris from
Mars was hardest,
experts agree, in the
Earths early days. And
the reverse trip was far
less likely because the
Sun pulls Earth debris
away from Mars and
toward itself.
W ILLIAM J. B ROAD, WANNA
S EE A R EAL L IVE M ARTIAN ? T RY
THE M IRROR , T HE N EW YORK
T IMES (M ARCH 14, 1999)

116 Mad about Modern Physics


11 Crystal Blue
Persuasion
S OME PROBLEMS THAT COULD HAVE BEEN PUT

in the previous chapters are presented in this chap-


ter. We have collected these special problems for this grand
nale. Some of the previous problems will provide signicant
clues toward answering challenges in this hodgepodge of a
collection, while many challenges here are new to most
readers. We hope that you enjoy them as much as we have.

117
HIMMELS THEORY OF 226. Iodine Prophylaxis
ACADEMIC TYPES
The greater the cer- Supposedly, in the event of a nuclear emergency, iodine
tainty of ones results, tablets offer protection from radioactive iodine. How
the less the concern can this preventive measure work? Isnt all the iodine,
with others opinions of
tablets or not, exposed to the ambient radiation?
oneself.
Thus at the end of
the spectrum occupied 227. Bicycle Tracks
by sociologists and pro-
fessors of literature, If you came upon this set of
where there is uncer- bicycle tracks meandering
tainty as to how to dis- around in the mud,
cover the facts, the could you determine
nature of the facts to
be discovered, and which way the bicycle
whether indeed there was going simply by
are any facts at all, all examining the tracks?
attention is focused on Remember that the
ones peers, whose front wheel and the rear wheel make separate tracks.
regard is the sole crite-
rion for professional
success. Great pains 228. Earth Warming
are taken in the devel-
opment of the impres- Over the past several decades there has been consider-
sive persona, with able concern over the possible slow rise of a few tenths
excessive attention of a degree Celsius in the average temperature of
given to distinguished Earths atmosphere and its surface. Most of the concern
appearance and fault-
less sentence structure.
seems to be associated with the greenhouse effect on
At the other end, the radiation from the Sun. However, assuming that the
where, as the mathe- average temperature of Earth can be dened unam-
maticians themselves biguously so that the small rise in average temperature
are fond of pointing out, is true, can there have been additional thermal energy
a proof is a proof, no
concern need be given coming from within Earth to cause this rise?
to making oneself
acceptable to others; 229. Frequency Jamming
and as a rule none
whatsoever is given. Suppose one desired a noisy emitter of electromagnetic
R EBECCA G OLDSTEIN , T HE waves at all frequencies simultaneously. Such a device
M IND -B ODY P ROBLEM : A N OVEL might be useful to jam undesired cell phone signals, for
example. How could you do this simply?

118 Mad about Modern Physics


230. Light Energy If the universe of sci-
ence is not evident to
We know that the speed of light is the same for all our ordinary senses but
observers in inertial frames. If so, are the momentum is elaborated from cer-
and energy of light as measured by all observers the tain key perceptions, it
same even when the light source is moving toward the is equally the case that
observer? these perceptions
require their appropriate
instruments: micro-
231. Acid Rain scopes, Palomar tele-
Chemists dene the quanity pH = -Log [H + ], where scopes, cloud chambers,
[H + ] is the aqueous concentration of H + ions. Pure and the like. Again, is
there any reason why the
rainwater is a neutral solution that has a pH of 7 when
same should not hold for
the droplets form. Will these droplets falling through
religion? A few words by
clean, unpolluted air have a pH of 7 by the time they
that late, shrewd lay the-
hit the ground? ologian, Aldous Huxley,
make the point well. It is
232. Electrical Current a fact, confirmed and
reconfirmed by two or
Upon turning on a lamp, Raymond wonders, About three thousand years of
how fast do the electrons in house wiring move as they religious history, he
provide electrical energy to the lamps and other elec- wrote, that Ultimate
tronic devices? Reality is not clearly and
immediately appre-
hended except by those
233. Earths Orbit who have made them-
Earths elliptical orbit is not xed in orientation with selves loving, pure in
respect to the stars. Why not? heart, and poor in spirit.
H USTON S MITH , B EYOND THE
P OST-M ODERN M IND

I swear to you that to


think too much is a
disease, a real actual
disease.
F EODOR D OSTOEVSKY, N OTES
FROM U NDERGROUND

Crystal Blue Persuasion 119


Even when all the possi- 234. Crystal Growth
ble scientific questions
have been answered, the Many children grow crystals in solution as a science
problems of life remain project. How does a crystal grow from a small seed
completely untouched. to its final size? That is, exactly how do the atoms
L UDWIG W ITTGENSTEIN know where to adhere to the growing structure without
fouling up the precise cubic crystal structure develop-
ment, for example? Do you see the dilemma? And the
The shooting stars are surprise?
meteors usually the size
of a grain of sand.
235. Ruby, Sapphire, and Emerald
How are ruby, sapphire, and emerald crystals related?
General relativity is How do they produce their colors?
actually less relativistic
than special relativity.
The complete feature- 236. Kordylewski Clouds
lessness of flat space-
By Keplers laws any object orbiting the Sun in an orbit
time, its homogeneity
and isotropy, are what smaller than Earths has a faster speed. So how can dust
ensure that positions particles placed in solar orbit along the Earth-Sun
and velocities are radial line but closer to the Sun have the same speed as
strictly relative. As soon Earth?
as spacetime acquires
bumps, or local regions
of curvature, it becomes
237. Twist
absolute because posi- Scooter
tion and velocity can be
There is a type of three-
specified with respect
wheeled scooter with a
to the bumps. Space-
handlebar extending ver- Feet go here
time, instead of being
merely a featureless tically upward from the
arena for physics, is apex of a V-shaped hori-
itself endowed with zontal metal frame that
physical properties. has three wheels. The
B RYCE S. D E W ITT, front wheel can rotate
Q UANTUM G RAVITY, about a vertical axis at
S CIENTIFIC A MERIC AN Flexible hinge
(D ECEMBER 1983) the foot of the vertical
handlebar at the apex

120 Mad about Modern Physics


intersection, and the two rear wheels are at the ends of A 1972 study of the polit-
the arms of the V. The two arms of the V are hinged at ical views of academic
their intersection at the front of the scooter so they can scientists shows that the
rotate about the vertical axis of the hingethat is, they high achievers in acade-
can form a wider or a narrower V angle within limits. mia, particularly the
physicists and mathe-
The rider places one foot on each arm of the V and
maticians, are signifi-
swivels his or her body from side to side to tilt the verti-
cantly more liberal than
cal handlebar from side to side to make the vehicle go the rank and file of the
forward or backward. Can you explain the physics of its academic scientists and
forward motion? engineers. The study
ranks the percentage of
238. Unruh Radiation scientists ranked liberal,
from the highest to the
Physicist J. Bekenstein determined that a particle accel- lowest, as follows: math-
erating in a vacuum experiences a blackbody radiation ematics, physics, biologi-
bath around itself at a temperature directly propor- cal sciences, chemistry,
tional to its acceleration. By the equivalence principle, geology, medicine, and
would a particle at rest in a gravitational field also engineering. The authors
experience this blackbody radiation bath? suggest that if the scien-
tists work is character-
ized by a high degree of
239. Star Diameters intellectuality, creativity,
One can determine the diameter of a distant star even and orientation toward
though the diameter cannot be measured by parallax. basic research, the sci-
The process uses the intensity interference, not the entist will very likely be
near the liberal end of
Star light the scale, and will tend to
Star light
be critical of existing
social institutions and
practices. Conversely,
if his work is character-
ized by practicality,
routine, and know-how,
the scientist will tend to
be conservative.
E. C. LADD J R. AND S. M.
Multiplier LIPSET, P OLITICS OF ACADEMIC
NATURAL SCIENTISTS AND ENGI -
Correlator NEERS , S CIENCE (J UNE 9, 1972)

Crystal Blue Persuasion 121


The surest sign that amplitude interference, between the light entering two
intelligent life exists identical photodetectors (telescopes) from the left side
elsewhere in the uni- of the star surface, and from the right side of the star
verse is that it has never surface. This process is valid even though the star is
tried to contact us. simply a point in the light-gathering optics of either
B ILL WATTERSON , C ALVIN
photodetector. Can you explain the physics?
AND H OBBES COMIC STRIP

240. Glauber Effect


Among those who Does a standard incandescent lightbulb emit single
become famous, one photons? Paired photons? Triplets?
specific category is
greatly overrepresented: 241. Bird Sounds
firstborn sons who lost
their fathers at a young Practically all birds and other animals emit sounds that
age and were emotion- have a fundamental frequency and several harmonics.
ally rejected by their Some birds, however, can emit just the fundamental
mothers. The foremost with no harmonics! Measurements inside the bird
example in physics: reveal that the original sound has harmonics. So how
Isaac Newton. does the bird eliminate them before they escape into the
great outdoors?

Q: Why do chemists call 242. Spouting Alligator


helium, curium, and Some alligators can submerge themselves slightly
barium the medical underwater and vibrate their bodies so that numerous
elements?
A: Because if you cant
helium or curium, you
barium! Water surface

Talented alligator

122 Mad about Modern Physics


individual water droplets are projected upward simulta- When Glenn T. Seaborg
neously a foot or more directly above the backs of their and his colleagues at the
heads. What is the physics here? Lawrence Berkeley
National Laboratory in
California were able to
243. Hair-Raiser Function make a new element in
1940 with 94 protons in
One of us (F. P.) rst heard about the hair-raiser func- its huge nucleus, they
tion (HRF) from physicist Richard Feynman. We intro- could not at first imag-
duce this function here as a curiosity to stimulate the ine that anything more
mind. And if one pulls a hair on top of the head massive would ever be
upright, this function is probably a good representation obtained, and so they
of its fast vertical increase with horizontal distance. called their new element
ultimium (later it would
be renamed plutonium).
1200 2 1024

A typical person age fif-


teen to forty-five grows
about an inch overnight.
0 0 When we lie down to go
0 1 2 3 to sleep at night, the
0 1 2 3 4
spaces in our spine
Hair-raiser function expand and we get taller.
In the morning when we
get up, gravity reasserts
Most mathematical functions are easy to define, its downward pull and
and so are their inverses. Physicists utilize a tremendous very quickly we go down
variety of mathematical functions, the two most com- a bit. Astronauts typi-
mon being the power of a quantity and the exponential cally stretch more than
of a quantity. Physicists also use mathematical opera- two inches in weightless-
tions that may not seem to qualify as a function, such ness, but as soon as
as the Dirac delta function (r r0). Powers are preva- they return to Earth
lent in fundamental laws dictated by geometrical sym- gravity, over a matter of
hours they shrink to
metries such as the universal law of gravitation and the
normal height.
Coulomb law, both having potentials proportional to
1/r and forces proportional to 1/r 2 for ideal point
sources. The exponential function increases more

Crystal Blue Persuasion 123


Cosmic radiation comes rapidly than practically any other known function and
from outer space. The is used whenever the change in a quantity is propor-
radiation dose from tional to the quantity itself, such as in growths and
cosmic radiation decays.
increases with altitude, The hair-raiser function HRF(x) can be dened by
roughly doubling every example on how it maps integers to integers. The
6,000 feet. Therefore,
HRF(1) is 1. The HRF(2) = 22. The HRF(3) = (33)3, and
a resident of Florida (at
so on. Notice the grouping with parentheses. One
sea level) on average
needs a calculator for most of the higher-integer HRFs.
receives about 26 mil-
lirem, one-half the dose Certainly the HRF is a one-to-one mapping.
from cosmic radiation How does one calculate the HRF of a non-integer?
as that received by a Of a complex number? How does one determine the
resident of Denver, inverse of the HRF? That is, given a number such as 42,
Colorado, and about how does one determine what number is mapped by
one-fifth of that by a the HRF into 42? And nally, of what potential use is
resident of Leadville, the hair-raiser function?
Colorado (about two
miles above sea level). A
passenger in a jetliner 244. Space Crawler
traveling at 37,000 feet In 1999, the U.S. Patent Ofce awarded a patent to a
would receive about 60 propulsion device that is a base frame with a sliding
times as much dose
carriage on the frame and two counterrotating masses
from cosmic radiation
that together couple and decouple to the base frame to
as would a person
move the carriage forward and backward in a compli-
standing at sea level for
the same length of time. cated motion. An onboard battery provides the energy
for all internal movement. When the rotating masses
are not coupled to the frame, and when placed on a
I dont think there is one nearly frictionless air table, the whole device simply
unique real universe. . . . oscillates forward and back repeatedly, as expected.
Even the laws of physics When the coupling is allowed to occur at specific
themselves may be phases in their rotational cycle, the whole device moves
somewhat observer- only forward in a continual sequence of spurts that are
dependent. longer with less air table friction! Will this device oper-
S TEPHEN H AWKING ate likewise in space?

124 Mad about Modern Physics


Answers
inside the bottle will begin to cool just
Chapter 1 as the burning nishes. The egg seals
The Heat Is On the opening, so the air pressure inside
decreases as the cooling progresses.
The total downward force will eventu-
1. Egg into a Bottle ally be greater than the total of the
Newtons second law explains the forces resisting the egg at the entrance,
result. When the egg is resting on the and the net force downward will accel-
bottle, the ambient air in the room erate the egg into the bottle. The
plus the gravitational force on the egg movement continues until the egg
by the Earth together exert a total drops to the bottom. Kerplop!
downward force on the egg that is
equal to the total upward force pro-
vided by the contact force of the bottle
2. Egg out of a Bottle
plus the force of the air inside. By The hard-boiled egg is in the bottle
Newtons second law, a downward and the goal is to remove this egg
acceleration begins when there is a net without damage. When the total out-
force downward. To get the egg to ward force acting on the egg exceeds
accelerate downward, you must the total opposing force resisting its
reduce the upward force of the air exit, then the egg will accelerate out of
inside the bottle. This action reduces the bottle. Assume that the bottle is
the total upward force, allowing a net held vertically upside down. Earths
force downward, with the resulting gravitational force acting on the egg
acceleration downward dictated by net that is, its weightand the force of the
force = mass times acceleration. air inside the bottle both contribute to
The correct timing requires one to the total downward force acting on
wait until the paper dropped inside the the egg. The upward force is the con-
bottle has stopped burning, then tact force of the bottle (which includes
immediately and carefully place the static friction) plus the force of the
egg on the opening. The warmed air ambient air in the room.

125
To remove the egg, you need to latticework with water molecules
create a pressure difference between loosely connected, so the holes in
the air inside and the air outside the this water lattice can accommodate a
bottle, with the greater pressure inside. large number of other molecules. The
Hold the bottle with its mouth near sugar molecules form temporary
your mouth and its bottom somewhat hydrogen bonds to the water mole-
higher so that the egg is positioned far cules, these bonds breaking and re-
forward, to the neck of the bottle, but forming continually. Essentially the
not completely blocking the neck. rule is like dissolves like. Of course,
Exhale a tremendous burst of breath the sugar molecules are quite large,
into the bottle to suddenly increase the and one cup of sugar molecules con-
inside air pressure. The Bernoulli tains only about one twenty-fth the
effect, the reduction in air pressure number of water molecules in a cup.
perpendicular to the air stream ow So there are many water molecules to
direction, caused by the rush of air every sugar molecule in the solution.
flowing around the egg plus the Wolke, R. L. What Einstein Told His Cook:
increased inside air pressure, will aid Kitchen Science Explained. New York:
W. W. Norton, 2002, pp. 2122.
in pushing the egg through the neck
and mouth of the bottle. When the net
force outward occurs, the egg acceler- 4. Kneading Bread
ates outward. Sometimes the egg just
pops out and must be caught in the Each successive kneading of the bread
mouth, and at other times the egg dough distributes the CO2 gas released
must be gently removed at the open- by the action of the yeast to make a
ing. The more vertical the bottle is, the finer texturethat is, smaller holes
more help one has from the gravita- more evenly distributed throughout
tional force of the Earth. the bread volume.
Initially, the concentration of yeast
is not uniform but has some non-
3. Sugar uniform volume distribution in the
One can usually dissolve about five bread dough. Where there is more
cups of sugar in one cup of water! yeast, there will be more CO2 gas pro-
Very simply, sugar molecules can duced by the yeast chemistry and usu-
squeeze into the empty spaces among ally bigger bubbles in the region. At
the water molecules, so they are not the molecular level, CO2 molecules
really occupying more space. The released by the yeast will diffuse into
water forms somewhat of an open the surrounding dough somewhat,

126 Answers
probably not moving very far in the If ice chunks were used instead of
available time. Some of the gas bub- butter, then the procedure is correct.
bles may even coalesce to form bigger As a check, when oating ice melts,
bubbles. Without further kneading, the water level does not change.
some places in the bread will possess Therefore, the recommended proce-
many bubbles or large bubbles, and dure is correct for measuring one-half
other places may have very tiny bub- cup of ice. But the density of butter is
bles or none. We have all seen bread not the same as that of the ice, and the
with a non-uniform distribution of density of melted butter is not the
bubbles, or even with one large bubble same as that of the water. Therefore,
somewhere. A more thorough knead- the immersed volumes of ice and but-
ing would eliminate these oddities, ter would be different.
unless they are planned. However, if the butter is held
under the water surface when water is
added, then the butter measurement is
5. Measuring Out correct when the water reaches the
one-cup level. One is simply measur-
Butter
ing the volume of the butter, and one
Butter oats on water because its den- does not use Archimedes principle for
sity is less than the density of water. this measurement.
The recommended procedure in many
cookbooks for measuring one-half cup
of butter is: put one-half cup of water
6. Milk and Cream
into the measuring cup and then add The milk is heavier, meaning more
pieces of butter until the water level is dense. Cream oats to the top in a milk-
pushed up to the one-cup level. Often cream mixture, so the cream is less
a reference to Archimedes principle is dense. Quite often people confuse mass
given. (According to Archimedes prin- density with liquid flow sluggishness.
ciple, a body wholly or partially The two properties are unrelated. Many
immersed in a uid will be buoyed up of these people will think that the cream
by a force equal to the weight of the is more dense. Anyone who has milked
uid that it displaces.) a cow or a goat has seen the cream at
However, this recommended meas- the top. A pint of light cream weighs
uring procedure does not use about 1.5 grams more than a pint of
Archimedes principle! And the meas- heavy cream on average. Why? Because
ured amount of butter is not exactly heavy cream has more fat per volume,
one-half cup! and fat is less dense than water.

Answers 127
Separation of materials by density on the top or the bottom of the potato.
has been an important method for mil- With a sudden thrust, drive the
lennia. Gold and platinum have been pinched straw into the potato held in
separated from other elements by the other hand. The straw goes right
dumping the ores into a hot bath of through. Why? The trapped air upon
lead. The specific density of lead is contact is compressed inside the straw
11.36, of gold 19.32, and of platinum and helps the straw remain rigid. The
21.45, so gold and platinum atoms paper or plastic is stretched taut and is
sink and practically all other elements more difficult to bend significantly.
and compounds oat. Of course, the Alternately, one could pinch the straw
workers must not breathe in the lead in a vertical position in a vise or a
atoms in the vapors. clamp and drop the potato onto the
straw (or a collection of straws).

7. Straw and Potato


Simply placing the straw in one hand
8. Blueberry Muffins
and trying to push the straw through The downward drift of the blueberries
the hard potato leads to a frustrating in the warm batter is caused by the
failure. The straw materialpaper or gravitational force of the Earth, so one
plasticcannot take much compres- must increase the friction between the
sion before bending sideways. So this batter and the surface of the berry to
sideways bending must be prohibited hinder this settling. One could make
if success is possible. We can use the batter thicker, but this solution
air pressure to help make the straw may not produce the desired muffin
more rigid. texture. Instead, before mixing the
Pinch the straw between your blueberries in the batter, dampen them
thumb and forefinger about two slightly and shake them in a bag of
inches from the end that is farthest our. The our attaches to the berry
from the potato and squeeze tightly. surface and increases the static friction
Hold the potato carefully but securely with the batter, keeping the berries
in the other hand placed in a horizon- uniformly distributed in the mufn.
tal plane with thumb on one side and The physics involves Newtons
ngers on the other side of the potato. second law, found in every high
Make sure that no part of the hand school textbook, that is, net force =
will be in the path of the strawthat mass acceleration. The downward
is, avoid having any part of your hand gravitational force on the blueberry

128 Answers
must be balanced by an upward force 9. Can of Soup
applied by the batter through friction
to produce zero net force in the verti- Turn the can of soup upside down and
cal direction and therefore zero accel- open the bottom. Then turn the can
eration downward from rest. over and watch the concentrate being
In this case, the maximum value of pushed out by the weight of the more-
the static frictional force (equal to the liquid stuff, assuming that the
coefcient of friction times the force of upward-acting static frictional force of
the batter perpendicular to the blue- the wall with the concentrate balances
berrys attempted movement) has not the weight of the concentrate. The
been exceeded. Without the our coat- weight of the liquid thus provides the
ing, the static friction coefcient is too nonzero net force downward to accel-
small. The maximum static frictional erate the liquid downward by New-
force upward provided by the batter tons second law.
alone is too small. The berry acceler- If there is delay in this evacuation
ates downward until reaching the crit- process, allowing some air into the liq-
ical velocity, for which the net force is uid region behind the solid concentrate
again zero, so the blueberry drifts might expedite the motion. Sometimes
downward with no acceleration. With the air seal at the wall is very good, so
the flour coating, the coefficient of that as the concentrate slips outward, a
static friction is large enough to not be signicant inward pressure difference
exceeded and the downward gravita- can build up to slow down the extrac-
tional force is always balanced by the tion process. In addition, the molecules
static frictional force upward. in the soup may interact more vigor-
At the atomic level, friction involves ously than expected via the electrical
electrical forces acting between atoms force between the concentrate and the
and between molecules. Still an active wall of the can (i.e., the viscous force,
research area, the inuence of quantum the surface tension, etc., may be large
mechanical effects is signicant. Even enough to make the extraction even
sound waves contribute good vibrations more challenging).
in this eld, called tribology!
Krim, J. Friction at the Atomic Scale. 10. Salt and Sugar
Scientific American 275, no. 4 (1996):
7480. Salt and sugar do their work on bacte-
ria by osmosis, dehydrating them so
Miller, J. S. The Kitchen Professor. Sydney:
Australian Broadcasting Commission, that they die or are deactivated. A
1972, pp. 2224. bacterium in very salty water has a

Answers 129
saltier environment outside its cell 12. Ice Cream Delight
membrane than inside. Water mole-
cules will move from its inside through Good ice cream contains abundant air
its water-permeable membrane to the bubbles to keep it light with very small
outside to balance the salt concentra- ice crystals so that the texture is
tions, a process called osmosis. The smooth. There are several good small
bacterium shrivels up and dies. Sugar appliances for making ice cream and
works by the same process to preserve sorbet. Some ice cream makers rely on
fruits and berries. In the markets today human musclepower to turn a large
one can buy cured hams and other spatula to control the ice cream tex-
pork products that use both salt and ture; others are electric. But with an
sugar to enhance the avor. abundant supply of liquid nitrogen at
Wolke, R. L. What Einstein Told His Cook:
196C and about an equal volume of
Kitchen Science Explained. New York: ice cream mixture, the freezing of the
W. W. Norton, 2002, pp. 137138. ice cream occurs so fast that only small
crystals have time to grow. As the
nitrogen furiously boils, plenty of small
11. Defrosting Tray
gas bubbles form in the mixture. All
The miracle defrosting tray is simply these effects make for a delicious treat.
aluminum metal, and one could use a Kurti, N., and H. This-Benckhard. Chem-
thick aluminum frying pan or other istry and Physics in the Kitchen. Scientic
piece of aluminum or copper metal American 270, no. 4 (1994): 6671.

tray to do the thawing as quickly, as Walker, J. The Physics of Grandmothers


Peerless Homemade Ice Cream. Scientic
long as there is no coating on the American 250, no. 4 (1984): 150153.
metal. Nonstick pans have coatings
that are poor heat conductors. Metals
are the best heat conductors because 13. Cooking a Roast
they have about 1023 conduction elec-
The bone-in roast cooks faster because
trons per cubic centimeter available to
the bone, even though porous, rapidly
transfer thermal energy from the hotter
conducts thermal energy to the inside
source to cooler regions. For thawing
faster than the meat itself does. So the
purposes, the metal tray will conduct
bone-in roast cooks from both direc-
thermal energy from the room air into
tionsoutside in and inside out. There
the frozen food very efciently.
will be some minor speedup effect
Wolke, R. L. What Einstein Told His Cook:
Kitchen Science Explained. New York: from the difference in specic heats of
W. W. Norton, 2002, pp. 202203. bone and meat, and there will be

130 Answers
slightly less meat if both roasts weigh chunks cook faster and therefore
the same, but in the simplied, ideal require less fuel. The faster cooking
case we ignore these differences. One occurs because (a) the inside of the
could use computer modeling with the small cube is closer to the heat source
appropriate physics equations to than for a thicker piece and (b) the meat
determine the temperature distribution is tumbled during stir-frying, exposing
in various parts of the meat and bone different small surfaces to the higher-
in the two cases, showing that some temperature direction. The temperature
parts of the meat are cooked more sensed by the meat tends to decrease
than others in both types of pieces. with distance from the thermal energy
Any general physics text discussing source, in this case the pan bottom.
thermal conductivity and specic heat The amount of cooking experi-
contains the pertinent information for enced by a small volume of interior
analyzing this problem, but the actual meat is proportional to the tempera-
temperature profile as a function of ture experienced and the duration of
the elapsed time is difficult without cooking at this temperature. Both of
some idealizations about the shape, the these quantities are changing during
uniformity, and other things. We have the cooking process. In addition, the
considered the idealized case above thermal conductivity and the thermal
that ignored the change in bone prop- heat capacity of the meat are changing
erties with temperature change, such because the meat material itself is
as the specific heat of the bone and changing. For example, if the outside
its thermal conductivity. Somehow becomes charred, its thermal conduc-
Nature has gured out all these things tivity is signicantly reduced, so that
without special computer modeling! the transmission of thermal energy
decreases compared to its prior rate.
Therefore hamburgers, which must be
14. Cooking Chinese cooked thoroughly inside to kill the
bacteria on the surfaces of the ground-
Style up meat, must never be charred on the
There are at least two good reasons for outside because the inside will tend to
cutting up meats into small volumes: (1) remain uncooked or partially cooked,
marinades and spices penetrate more creating a dangerous eating condition.
thoroughly into the meat in a shorter The physics here is found in any
time because the inner-volume elements high school physics text, but the
are closer to the surface; (2) smaller application to cooking was devised

Answers 131
millennia ago by ancient chefs who tension has exceeded its elastic limit.
desired a particular result with per- During the rupture process, bean skin
haps a minimum of fuel expenditure. molecules have been separated because
Certainly one can go to the other the electrical force holding one mole-
extreme by slow-roasting a whole car- cule to the next has been exceeded.
cass in a revolving spit or in a hot Miller, J. S. The Kitchen Professor. Sydney:
ember-lined pit in the ground. Australian Broadcasting Commission,
1972, pp. 8182.

15. Baked Beans


The hot beans have taken up water
16. Ice Water
and swelled so that the skins are under The ice at the top brings about faster
great tension. Blowing cool fast-mov- cooling of the water in the pitcher. As
ing air across the beans from pursed some ice melts, this cold water is more
lips (instead of an open mouth) lowers dense than the surrounding water and
their surface temperature and reduces sinks, cooling the water it passes
the ambient air pressure. The inside is through. The warmer, less dense water
still hot, so the larger pressure differ- at the bottom is buoyed upward into a
ence results in the hot, high-pressure cooler region. This mixing helps the
vapor under the skin pushing outward water cool faster than when the ice is
just a bit more. If the pressure gradient held at the bottom, because the cold
is great enough, the skin will rupture. water produced by the ice would
The slight cooling of the skin material remain at the bottom. Thermal con-
increases the skin tension to reduce the ductivity throughout the water would
time to rupture. eventually cool the water above, but
We can simplify the physics to the convection currents work faster.
applying Newtons second law perpen- Of course, vigorous stirring of the ice
dicular to the skin surface. Three water eliminates any need for the pre-
forces acting on the skin are impor- vious discussion!
tant: (1) the inward force of the ambi- One could say that the discussion
ent air pressure, (2) the inward force above is incomplete because in our ide-
of the tension in the skin, and (3) the alization we have ignored the ice ther-
outward force produced by the hot, mal interaction with the ambient air.
high-pressure gas within. Blowing the This interaction can be important,
air reduces the ambient air pressure especially on hotter days. The ice does
enough to create a net force outward, its job when thermally interacting with
and skin rupture occurs when the skin the water, not with the air! The ice held

132 Answers
in the water would be a more efcient swollen. Cold water on the outer sur-
direct interaction procedure. So when face of the hot beet causes the skin to
the ambient air temperature is great shrink, but the innards remain hot and
enough, they could be competitive. swollen. So the stretching skin bursts
By the way, this cooling process is in several places and becomes easier to
exactly the same as the sequence of remove with a paring knife without
events that occurs when a pond freezes being so messy.
over in winter. However, in that case, The procedure is exactly opposite
the pond water is prevented from cool- to placing an ice cube in hot water.
ing further and from freezing through Now the outside of the cube tries to
until all the water reaches 4C rst. expand, but the inside is still cold. One
This delay in freezing throughout can hear the thermal stresses crack the
saves the lives of pond organisms ice cube.
through the winter if spring comes
soon enough. Evaluated in a different
way, there would be no life surviving 18. Igniting a Sugar
the worst ice ages on Earth if water
Cube
did not reach its maximum density at
about 4C! Very small particles tend to ignite
more easily. The large surface-area-to-
volume ratio for a collection of small
17. Peeling Vegetables particles aids ignition, providing a
When a tomato is held carefully over a large combustion area for the chemical
ame and rotated, some of the thermal interaction of their surface molecules
energy gained by the tomato vaporizes with oxygen and also providing a
the water just under the skin to locally nearby heat source for sustenance.
rupture it. A paring knife can remove Therefore, rub the far corner of the
the skin easily after the tomato has sugar cube in some cigarette ash or
cooled. Often, just pulling on the rup- tiny ash particles from burned paper,
tured skin is enough. Very hot water then light the ashen cube with the
can be used instead of a ame, but the burning match. Ignition is now easy.
effects are not as dramatic, and the Oxygen molecules react with mole-
peeling is a bit more difcult. cules in the ash to produce thermal
The boiling raises the temperature energy and product molecules, includ-
of the beets to cook them and, simul- ing water.
taneously, a small amount of hot Historically, there have been many
water enters them, so they are slightly examples of the spontaneous ignition

Answers 133
of dust particles in the air, such as process is more complicated than the
explosions in granaries where grains simple, ideal version we have consid-
crops are stored and in mills that grind ered here, involving nucleate boiling,
grain into smaller particles. A small transition boiling, etc., in the real situ-
warm spot in the dusty air, perhaps ation. However, the complete analysis
produced by sunlight, by a match, or produces the same general argument.
by friction, can rapidly spread into a One should ask whether different
full-scale explosion. results would occur with sea salt, a
On a less violent scale, simply mixture of KCl and NaCl and dead
lighting a campfire outdoors begins organic matter in larger grain sizes
with kindling, very small sticks and than normal table salt. The slow rate
shavings of wood, which have a very of dissolving the larger sea salt grains
large surface-area-to-volume ratio. may delay the reboiling longer than
The slightly larger sticks can be added experienced for NaCl.
once the ame sustains itself. Finally, a The relevant physical data are in
whole faggot of sticks can be placed in most chemistry and physics hand-
the repit to generate a lasting re. books as well as in some textbooks.
The physics pertinent to the idealized
case discussed above is part of tradi-
19. Water Boiling
tional physics and chemistry courses,
The major thermal effect is the raising but the more detailed complete analy-
of the boiling point (i.e., boiling tem- sis can be found only in the technical
perature) by the added salt in solution literature.
from 100C (standard conditions at 1
atmosphere) to about 104C (if the
salt is pure NaCl), a signicant change 20. Put the Kettle On
that results in a small time lag before
boiling begins again if thermal energy Steam
input continues.
In contrast, the actual amount of
thermal energy needed to raise the Spout Gap
temperature of the sprinkled salt itself
is minuscule because the specic heat
of NaCl is much lower than for water,
and the amount of water by weight in
the pot is enormous compared to the
amount of salt. The actual boiling

134 Answers
No, you cannot see water vapor, that level. So at higher elevations potatoes
is, water molecules in their gaseous may not cook as quickly in the open
state. If you look closely at the orice pot, and the lukewarm water will not
of the spout, there is a clear region per- make good tea or instant coffee. In the
haps up to one inch long. Thats where mountains we would be wise to use a
the water vapor is before it condenses pot with a lid so that the total pressure
into the steam you can see. The tem- acting downward on the water surface
perature of the vapor in the clear can be highervapor pressure plus
region is still too high for droplets of atmosphereand so that the water
steam to formthat is, collisions of boils at a higher temperature than
water molecules are too violent to without the lid, hopefully almost at
allow them to bind together to form 100C.
droplets. Suppose the cooking at sea level is
In the clear region at the orice of done in a pot with a lid. Now the
the kettle, the water molecules are action of lifting the lid to watch the
moving so rapidly that when they do pot reduces the thermal energy in the
collide, the van der Waals attractive air above the liquid surface as some
forcean induced dipole-dipole elec- molecules escape. These escaping water
tromagnetic interactionat close molecules are among the most ener-
range cannot keep them together. As getic in the vapor, so they can carry
the water vapor cools farther away away much thermal energy. The pres-
from the end of the spout, these same sure above the liquid is now lower
collisions produce droplets that grow than before, the boiling occurs at a
in size. lower temperature, and the cooking
takes signicantly longer. One should
replace the lid and let the food cook
21. The Watched Pot
undisturbed! Hence the expression A
Put a pot of water on a ame atop a watched pot never boils. This state-
stove. The thermal energy from the ment actually refers to the extended
ame raises the water temperature. If cooking time and involves some good
the pot has no cover, soon the water physics.
vapor pressure above the water sur-
face equals the pressure of the ambient
atmosphere. The water is now boiling.
22. Ice in a Microwave
If we are high in the mountains, the Yes! The water molecules in the liquid
boiling has occurred at a lower tem- state rotate a bit in the microwaves
perature than when we are near sea and transfer energy to the surrounding

Answers 135
molecules to make them jiggle ran- given above. At the water molecules
domly. The water molecules in ice are resonant frequencies in the microwave
locked into crystals and are unable to region of the electromagnetic spec-
rotate. (Note: The actual details of trum, the applied field changes so
molecular bonding in the ice are more rapidly that very little energy is trans-
complicated and show that a minus- ferred to the nearby molecules.
cule amount of rotation is possible, Microwave ovens actually operate at a
but an insignicant amount to change frequency that is lower than the
the ice to water.) Using microwaves, frequency at which the absorption is
therefore, one can boil water inside an greatest. The food needs to be heated
ice block! throughout, and by lowering the
Boiling the water inside the ice applied frequency a bit, more micro-
block is an example of selective energy waves penetrate farther inside, past the
absorption. Numerous examples of outer layer.
selective absorption occur in the natu- Kurti N., and H. This-Benckhard. Chem-
ral world. For example, the green istry and Physics in the Kitchen. Scientic
American 270, no. 4 (1994): 6671,
leaves of plants have chlorophyll A 120123.
and B molecules that selectively Walker, J. The Secret of a Microwave
absorb bluish and greenish light for Ovens Rapid Cooking Is Disclosed.
photosynthesis. At an even smaller Scientific American 256, no. 2 (1987):
134138.
scale, nuclei are very selective in
absorbing gamma rays of specic ener-
gies. At the macroscale of meters, we
know that rooms can absorb and
23. The Glycemic Index
amplify sound energy at selected reso- The rate of conversion from one type
nance frequencies. Some materials are of molecule to another is a chemical
even useful for just the opposite process, with the ratio of surface area
behavior, such as window glass, which to volume for particles in the food
has no selective absorption in the visi- being converted being an important
ble part of the electromagnetic spec- factor. Smaller spherelike particles
trum. You can take your pick, but the have a higher ratio of surface area (SA)
game is played by the rules of nature. to volume than larger ones. Conse-
The selective absorption by water quently, since the reactions occur on
molecules (and some other molecules) the surface, material consisting of
in a microwave environment is a little smaller-diameter spheres convert to
different from the other examples sucrose faster than the same material

136 Answers
consisting of larger spheres. In fact, for the glycemic index for numerous foods
the limiting case of a sphere, the ratio and some of the recent results from
SA/Vol. = 3/R, where R is the radius of nutrition and diabetes research world-
the sphere. Physical and chemical wide. Excerpts from the book and
processes initiated by the environment many other resources on the glycemic
occur rst at the surface of the parti- index and its comparison to the insulin
cle. In addition, for biological systems, index can be found on the Internet.
larger particles must be reduced to
smaller ones before passing through
membranes. So a collection of small
24. Electric Pickle
particles equal in total mass to one Even though the electrical energy
large particle will be reduced to source is provided through an AC cur-
acceptable size faster than the large rent, the pickle glows predominantly
one because the same amount of at one end with a yellowish color that
chemical solution acts upon a much is determined by the pickling solution
larger total surface area. and the pickle type. Reliably predict-
The smaller the particles of ing which end will glow has not been
ingested food, the faster can be the achieved. There is no actual symmetry
digestion of the molecules in the intes- here in the shape or chemical compo-
tines because the surface-area-to- sition of the pickle, so alternate glow-
volume ratio is higher, and the sooner ing is less likely and never seen. The
is the uptake into the blood. The conjecture is that the pickle is now act-
higher temperature used for baking ing like an electrical diode, passing
the potato makes its particles smaller current in one direction only!
than for the same potato when boiled The authors listed below performed
at 100C, so its glycemic index is an experiment by taking a visible light
greater, reecting its faster uptake into spectrum of the glowing pickle, using a
the blood. spectrometer with a diode array detec-
Dates contain some maltose, a tor. A fiber-optic probe was used to
sugar that is even faster than glucose channel the yellow glow to the spectro-
in its basic conversion to sucrose in graph, and a calibration spectrum was
the blood, so their glycemic index is taken of a sodium chloride ame test.
above 100. The emission spectra of the two are
A popular book The New Glucose nearly identical
Revolution by J. Brand-Miller is avail- This pair of emission lines, at
able in many libraries and has tables of 589.00 nanometers (nm) and 589.59

Answers 137
25. Space-Age
Pickle
NaCl Cooking
Unlike electric cooktops, which gener-
ate thermal energy by the electrical
resistance of the burner coils, magnetic
induction cooktops generate thermal
energy by the magnetic resistance of
the metal cooking vessel itself. The 60
587.5 590.0 592.5
Hz AC current owing in the induc-
Wavelength (nm) tion coil beneath the ceramic surface
produces an alternating magnetic eld
nanometers, indicate a characteristic than interacts with the Fe atomsfor
of sodium emission, called the sodium example, in the iron frying panto
D line doublet. Josef Fraunhofer oscillate its magnetization 120 times
observed these lines in the emission per second. The magnetization direc-
spectrum of the Sun, in about 1817. tion changes have resistance, so much
We know now that these lines are due energy goes into thermal energy in the
specically to an electronic transition metal of the pan. Iron and stainless
of sodium atoms in the gas phase. steel pans will work, but aluminum,
The pickle conducts electricity due copper, glass, and ceramic pans and
to the vinegar (acetic acid) and sodium pots will not. The advantages are no
chloride salt used to make it. Sodium noise and no hot cooktop except
ions in the pickle liquid attach elec- where the pan has been in contact.
trons from the owing current. These Cooking with light is not done by
ions are neutralized electrically, form- lasers! Light is meant in the broader
ing excited sodium atoms in two dif- sense of the word, the infrared (IR)
ferent excited electronic states (hence through the visible into the ultraviolet
the emission doublet). Because of the (UV) part of the electromagnetic spec-
heat and sparks and general pandemo- trum. Banks of 1500-watt halogen
nium around the electrodes stuck in lamps in the oven walls put out about
the pickle, these sodium atoms are in 70 percent IR, 10 percent is visible
the gas phase. They emit yellow light light, and the remaining 20 percent is
as they relax to the ground state. simply heat. The IR is not thermal
Appling, J. R., F. J. Yonke, R. A. Edgington, energy, but when IR is absorbed by
and S. L. Jacobs. Sodium D Line Emission
from Pickles. The Journal of Chemical molecules, their random motions can
Education 70, no. 3 (1993): 250. be increased. Thermal energy (heat

138 Answers
in the vernacular) is the random
kinetic energy of molecules and atoms.
These frequencies penetrate meat only
now
about half an inch at most. Thermal future
conduction transfers some of this ther-
mal energy farther inside. Sun
However, these light ovens also Earth
have a microwave source to penetrate
with microwaves to cook the interior
of the meat. So while the outside is
being browned by the light, the inside North Pole will be alternating from
is cooking via microwaves. The overall the extreme of being pointed toward
benet is much faster cooking than is the Sun and to being pointed away
possible in conventional ovens. from the Sun in January. At present
Wolke, R. L. What Einstein Told His Cook: and for some years to come, the North
Kitchen Science Explained. New York:
Pole points away from the Sun when
W. W. Norton, 2002, pp. 303307.
Earth is at the perihelion position in its
orbit on about January 5 each year.
Gradually over the next 12,900 years
Chapter 2 the North Pole will precess around to
receive more and more radiant energy
Does Anybody in January.
Really Know What However, we need not wait nearly
so long because the ellipse of Earths
Time It Is? orbit is also precessing, so our summer
will coincide with perihelion in only
about 10,000 years!
26. January Summer
Yes, the Northern Hemisphere enjoys
summer in January quite often (in the 27. Proximity of Winter
cosmic scheme of things), repeating,
every 25,800 years, the period of
Solstice and Perihelion
Earths precession. Just like a top with The proximity of the two dates is an
its axis precessing, Earth experiences a artifact of the particular century we
precession of its axis with respect to live in. The date of perihelion does not
the stars with a 25,800-year period of remain xed, but slowly moves later
oscillation. So every 12,900 years the into the year at the rate of about one

Answers 139
full day every 58 years. It turns out 29. The Equinox
that the period from perihelion to
perihelion (the anomalistic year) is
Displaced
about 25 minutes longer than the year On the dates of the equinoxes, the day
dened from equinox to equinox (the is about seven minutes longer than the
mean tropical year). The date of night at latitudes up to about 25
perihelion thus moves completely degrees, increasing to 10 minutes or
through the tropical year in about more at latitude 50 degrees.
21,000 years. This slow change in the The moment of the equinox occurs
date of perihelion may have a long- when the geometric center of the Suns
term effect on Earths climate. At this disk crosses the celestial Equator. But
time the temperature extremes are the standard denition of sunrise is the
moderated somewhat in the Northern time when the Suns upper limb is just
Hemisphere, but that will change as breaking the horizon, and sunset when
the perihelion shifts in the direction the Suns upper limb is just disappear-
of summer. ing below the horizon. This adds one
Sun semidiameter (about 16 arc min.)
at both sunrise and sunset, extending
28. Earths Speed the duration of daylight by a little over
two minutes.
Since perihelion occurs in early Janu-
The other factor is atmospheric
ary, Keplers second law implies that
refraction, which causes the rays to
Earth is traveling faster during the
bend around the horizon. As a result,
winter months. The time for Earth to
we see the Sun about 34 arc minutes
travel from the autumnal to the vernal
higher at both sunrise and sunset,
equinox, taken as a fraction of the
adding roughly four minutes to the
year (T = 178.83/365.25), can be used
time that the Sun is above the horizon.
to nd an accurate value of the eccen-
In spring the days get longer as we
tricity of Earths orbit, = 0.5
approach March 20, and the date of
(0.5 T) = 0.01632, about 2 percent
equal day and night occurs several
away from the precise value of =
days before the March equinox, about
0.016713. A more accurate formula
March 17 at latitude 40 degrees.
based on T is found in the reference
Conversely, in the fall it takes
below.
several extra days for the time when
Snyder, R. Keplers Laws and Earths
Eccentricity. American Journal of Physics the Sun is seen above the horizon
57 (1989): 663664. to shrink to 12 hours. The date falls

140 Answers
on about September 26 at latitude north, the equation of time has the
40 degrees. dominant inuence over the changes
On their website, the U.S. Naval in sunrise and sunset times. Prior to
Observatory publishes excellent sun- December 8, however, the declination
rise and sunset tables for any location. effect is dominant, pulling the sunset
to its earliest time on December 8.
Then the equation of time takes over,
30. The Dark Days of and during the two weeks before win-
ter solstice all the shortening of the
December day comes from the later clock time of
There are two effects that, together, sunrise. After winter solstice the days
determine the local times of sunrise and lengthen, even as the sunrises continue
sunset. One is called the equation of to get later until January 5.
time; the other is the Suns declination. Steel, D. Marking Time: The Epic Quest to
Earths orbit around the Sun is Invent the Perfect Calendar. New York:
John Wiley & Sons, 2000.
slightly elliptical. As a result, the speed
of the Suns apparent motion across
the sky is a bit faster in winter than in 31. Days of the Year
summer. Clocks, however, run at a While the time interval to return to its
constant speed, so there is usually a same point in the orbit is 365.2422
discrepancyup to 16 minutes days, Earth executes 366.2422 rota-
between clock time and the solar time tions on its axis. One can demonstrate
shown by a sundial. We refer to this this result by taking two coins, holding
discrepancy as the equation of time. one in place on a table, and rolling the
The Suns declination, its angular second coin in contact with the xed
distance above or below the celestial coin without slipping. Therefore the
equator, determines the maximum number of solar days is 365.2422, but
height of the Sun in the sky on any the number of sidereal days (i.e., with
given day, thus causing our seasons. In respect to the stars) is one more for
late December, the daily rate of change one orbit of the Sun.
of the Suns declination is rather small.
It is, in fact, exactly zero at the Decem-
ber solstice (solstice means sun sta-
32. Leap Years
tionary). Hence in late December, or In years divisible by four, every four
more precisely from about December years is a leap year except years divisi-
8 to January 5 at latitude 40 degrees ble by 100. If the mean interval

Answers 141
between vernal equinoxes, called the time periods occurs because Earth is
tropical year, lasts 365.2422 days, moving with respect to the stars, so the
then in 100 years we should experi- Moon must travel slightly farther
ence 36,524.22 days. But there will be around its Earth orbit to reach its full
24 leap years in a century normally, so Moon position along the Sun-to-Earth
there will be 0.22 day left over. So radial line.
every 400 years is declared to be a leap U.S. Naval Observatory, Nautical Almanac
year with one extra day to approxi- Office. The Astronomical Almanac for
the Year 2000. Washington, D.C.: U.S.
mate the 0.88 day. The year 2000 was Government Printing Ofce, 2000.
the rst such leap year on a year divis-
ible by 100 since the modern calendar
began general use in the late 1600s. 34. Moon Time
By the time the British were ready The person is at the desk at 12:20 dur-
to go along with the rest of Europe in ing the noon hour, not at night. The
the 1700s, the old Julian calendar Sun must be at the upper left because
required a correction of eleven days! the Moon is illuminated from this
The Gregorian calendar was adopted direction. This daytime Moon is sel-
in Britain in 1752, with Wednesday, dom noticed because the sky is nor-
September 2, 1752, being followed mally bright, but the daytime Moon
immediately by Thursday, September is up as often and as long as the Moon
14, 1752. at night.
The famous physicist Isaac New- Perhaps the easiest way to appreci-
ton was born on Christmas Day, 1642, ate the appearances of the Moon at
on the Julian calendar but on January night and during daylight is to use a
4, 1643, on the Gregorian calendar in lamp for the Sun and two spheres, one
use today. Therefore, Newton was not representing Earth and the other the
born in the year of Galileos death, Moon. Fix the lamp position and the
1642! Earth position, but move the Moon
around Earth to observe its illumina-
tion phases. Stop the motion at several
33. Full Moons points in the orbit of the Moon to
observe its view from daytime loca-
No, the orbital period of the Moon is
tions on Earth. You also might recon-
27.554 sidereal days, and the average
struct the scene in the diagram.
interval between full moons was
Pryor, M. J. Phases, Models, and Car-
29.535 days for the twentieth century. toons. The Physics Teacher 3, no. 6
The difference between these two (1965): 264, 288.

142 Answers
35. Lunar Calendar the seed parts develop) and may vary
extremely from one variety to the next,
Modern farming methods tend to plant from days to months. Photoperiod sen-
crops at approximately the same time sitivity is a natural mechanism based
year after year, with minor adjustments on the plants ability to distinguish pre-
for quirks in the weather. Hence, a par- cise differences in the ratio of day
ticular crop is usually planted about length to night length. The biological
365 days after its planting the previous mechanism causing photoperiod sensi-
year, plus or minus about 10 days. One tivity is quite complex and involves sev-
example is spring wheat, usually eral genes. Essentially, some varieties
planted on about April 15 in the north- should be planted only during certain
ern Plains states of the United States. times of year to ensure that prevailing
Rice is a different kind of organism day length/night length conditions will
than wheat as far as its environmental trigger panicle initiation when desired.
needs. Rice planted at about the same Many varieties of rice are sensitive
date every year will sometimes permit to bright moonlight, which can inter-
two good rice crops per year, but in rupt their growth sequence. However,
most years the farmer will get only one newer varieties have been bred and
good rice crop. The cause is the some- others will be genetically modied to
times detrimental appearance at night decrease their light sensitivity during
of the full Moon, which can interfere critical photosensitivity times so that
with the growth cycles of the rice plant. the moonlight will have a minimal
By planting rice according to the effect.
same date on the lunar calendar The use of the Moon for timing of
instead of the solar calendar, farmers events is not restricted to rice farming
can often harvest two good rice crops and its related festivals worldwide. For
every year. The young rice shoots are Christians, Easter Sunday is the rst
very sensitive to the light intensity at Sunday after the rst full Moon after
night during their photoperiod-sensi- the vernal equinox!
tive stage, so the timing of the Moons University of the Philippines College of Agri-
brightness is essential for a good crop. culture in cooperation with the Interna-
Because the lunar calendar shifts with tional Rice Research Institute (IRRI),
comps. Rice Production Manual, rev. ed.
respect to the solar calendar dates each Manila: compilers, 1970.
year, the solar calendar provides bad Yano, M., et al. Hd1, A Major Photoperiod
timing for planting rice. Sensitivity Quantitative Trait Locus in Rice,
Is Closely Related to the Arabidopsis Flow-
The photoperiod-sensitive stage ering Time Gene Constans. Plant Cell 12
occurs before panicle initiation (where (2000): 24732484.

Answers 143
Jargocki, C. P. Science Braintwisters, Para-
36. The Sandglass doxes, and Fallacies. New York: Charles
The hourglass shape ensures that the Scribners Sons, 1976, pp. 6, 70.

time scale on the glass is uniform, with


equal distances between scale divisions
corresponding to equal time intervals.
37. Old Watch
If the sandglass didnt taper, the top of The old watch will run fast. The bal-
the sand column would descend at ance wheel is the basic component
increasing speed. We can mathemati- that oscillates exactly 300 times for
cally determine the proper shape. The each minute on the watch facethat
speed, V, at which the sand is escaping is, the wheel changes direction 10
from the opening is given approxi- times each second! The moment of
mately by Torricellis formula V ~ inertia of the balance wheel depends
(2gy),
where g is the local accelera- on how much ambient air is dragged
tion of gravity and y is the height of along during each oscillation. The
the sand column in the upper glass. Let source of energy is a wound spring
A = r 2 be the circular cross-sectional that is essentially unaffected by the air
area of the upper glass at the top of the because of its extremely small change
sand column and v the speed at which in conguration.
this top is falling, then Av = aV, where In the mountains, the viscosity and
a is the area of the opening, because density of the air decrease slightly,
the sand is approximately incompress- allowing the balance wheel to oscillate
ible. Substitution produces y ~ cr4, faster. Newtons second law applied to
with constant c = 2v 2/(2ga 2). A plot this rotational motion is required.
of y versus r will produce the familiar From its momentary stop to change
hourglass shape. direction, the balance wheel must
Sandglasses without time interval accelerate to its maximum angular
markings have been used since before velocity, then accelerate back to rest,
the fourteenth century to time speeches etc. The net torque equals the
at town meetings and other events. moment of inertia I times the angular
When the sand had run its course, the acceleration that is, = I. The
speakers time was done. They are still moment of inertia is determined by the
used today in some board games and mass distribution with respect to the
as kitchen timers. Sandglasses with rotation axis, and the air carried along
ruled markings havent been so popu- with the balance wheel motion adds to
lar, being replaced early on by mechan- the moment of inertia of the wheel
ical watches and clocks. alone. That is why the watch must be

144 Answers
recalibrated when the location of the as, say, 3.45 seconds 0.005 second,
owner is at a different elevation than which is an awkward notation,
the factory. because the display goes only to hun-
When the balance wheel drags dredths of a second, yet the uncer-
along less air mass at a higher eleva- tainty is smaller. Therefore, by
tion, the moment of inertia is less for agreement, the elapsed time is given as
the same net torque, so the angular 3.45 seconds 0.01 second so that the
acceleration is greater. Less time is number of decimal places is the same
needed to reach top angular speed, for the value and the uncertainty
and less time is needed to come to rest that is, there is an uncertainty of plus
again. or minus one digit in the smallest time
Jargocki, C. P. Science Braintwisters, Para- interval position of the display.
doxes, and Fallacies. New York: Charles
Scribners Sons, 1976, pp. 6, 71.
39. Eternal Clocks?
38. Reading a Digital The laser clocks and the atomic clocks
must maintain a vacuum within a rea-
Timer sonably small range of parameters to
For a digital timer that displays the function accurately. The temperature
elapsed time to one-hundredth of a and pressure must be maintained
second, the minimum uncertainty in within a certain tolerance because
the interval depends on the software temperature fluctuations or pressure
and/or hardware method used to dis- uctuations could bring about inaccu-
play the last digit. Suppose that the racies. Even the outgassing of atoms
hundredths digit fraction from 0.00 and molecules from the container
through 0.49 is displayed as zero hun- walls can create severe problems for
dredths and from 0.50 through 0.99 is some designs. Certainly, improvements
displayed as one hundredth. Likewise, will be made to ensure longer lifetimes
1.00 through 1.49 is displayed as one and more robust timepieces. But main-
hundredth, and 1.50 through 1.99 as taining vacuums, low temperatures,
two hundredths. Seeing a 1 in the hun- and so on, for decades and centuries
dredths place then corresponds to the will be constant problems in these
range from 0.50 through 1.49, so the sophisticated systems.
minimum uncertainty in the elapsed Whether a 10,000-year (or even a
time value is 0.50, or one-half of 1,000-year!) mechanical clock will
one-hundredth of a second. The ever exist and can stand the test of
reported elapsed time should be given time and environment is doubtful. A

Answers 145
group of engineers and futurists are and then a decrease to zero after the
presently developing such a clock con- light from the far-corner reections is
taining a stack of rotating metal rings received. The detailed intensity curve
connected to a torsion pendulum. Peri- could be simulated on a computer.
odic winding will be required, perhaps With the photodetector array, the
once a year or so. image shows the six nearest spotsthe
No special environment is needed, centers of each equidistant flat sur-
although the assumption seems to be facewhich grow bigger and then
that a standard atmosphere with lim- form rings of reflected light, then
ited pollutant content is a reasonable many arcs of light until eight corners
expectation. We do know from a vari- appear and disappear.
ety of scientic research projects that When the ash length is extended
the oxygen concentration in the to 1 microsecond, the light pulse is
atmosphere has been very nearly con- 300 meters long. There will be an ini-
stant at 21 percent for millions of tial detector response rise and the rings
years, so the rusting rate of the of light from the walls will be seen for
exposed metal can be predicted. But a very small fraction of the total imag-
we do not know what future chemistry ing, then ooded, then decreased.
will bring. Even a local environmental We are not accustomed to light
disaster such as excess acidity in the pulses lasting for milliseconds or less in
air from volcanic eruptions, a chemi- daily life. But even nanosecond pulses
cal explosion, or careless disposal are very long in some research elds.
could shorten the life span of the clock For example, femtosecond (1015 sec-
dramatically. ond) and shorter light pulses are used
Gibbs, W. W. Ultimate Clocks. Scientic in chemistry to watch molecular inter-
American 287, no. 3 (2002): 8693. actions in progress. The present record
for subdividing the second with a laser
40. Room Light strobe light is a few hundred attosec-
onds, an attosecond being a billionth
For a nanosecond ash, the light pulse of a billionth of a second!
length d = 30 cm is calculated with d =
ct, where c is the speed of light and t is
the time interval. When summing all 41. Right to Left Driving
the entering light, the photodetector
displays an initial rise from the light
Switch
scattering from nearby walls with Yes, as long as the accelerations and
increasing intensity until maximum, decelerations required are within the

146 Answers
normal driving ranges, there should be frames, in the laboratory frame, and in
no problems. One could check out the rest frame of the clock. But the tick
each roadway with physical trials, or rates will be different. In the frame
one could make an aerial video and moving with the clock, the light ashes
play back the video in the reverse follow the same path as before, reect-
direction, essentially reversing time. If ing perpendicularly to each mirror,
the car accelerations appear unusual keeping the same tick rate.
in the reverse sequence, there will be
driving problems. Mirror
Even nature at the most fundamen-
tal level is cognizant of left versus
right, a surprising discovery in the
1950s related to the weak interaction, Mirror
one of the four fundamental interac-
tions, the other three being the gravita-
In the laboratory frame, the light
tional, the electromagnetic, and color
continues to reflect off each mirror
(also called the strong interaction). For
repeatedly, but during transit from one
the latter three, the interaction
mirror to the other the path length is
strengths are the same for left-hand
longer, being the diagonal of a right
spinning particles and right-hand spin-
triangle. If the speed of light is the
ning particles. But for the weak inter-
same value in both reference frames,
action, the evidence shows that nature
then the time interval between reec-
actually excludes any weak interaction
tions (and clock ticks) will be longer in
behavior for a right-hand spinning par-
the lab frame now than for the clock
ticle! The origin of this behavior biased
at rest. Therefore, a moving clock ticks
toward left-hand spinning particles is
slower than an identical clock at rest.
described by the mathematics in the
And this phenomenon is true for all
standard model of leptons and quarks.
clocks, no matter how they are made.
Bartlett, A. A. A Simple Problem from the
Real World That Can Be Solved through A more complicated situation
Time Reversal. American Journal of occurs when the light clock is acceler-
Physics 42 (1974): 416417. ated parallel to the mirrors. We suggest
that you think about this case when
there is ample time in your schedule.
42. Light Clock Feynman, R. P., R. B. Leighton, and M.
Sands. The Feynman Lectures on Physics.
Yes and no. The clock will continue to Vol. 1. Reading, Mass.: Addison-Wesley,
keep accurate time in both reference 1963, pp. 15-515-7.

Answers 147
43. Time Reversal a critical essential protein coding does
not produce a viable organism even
Answer b: the acceleration is still though a change in the nonessential
downward. The reversed motion is DNA mightthat is, changes in the
upward, but the object is decreasing its DNA that do not affect the biochem-
speed because the acceleration is down- istry critically will be tolerated. There
ward. A good example of this behavior are vast regions of DNA where such
is the ight of a ball tossed upward. At ineffective changes can occur, but any
all moments the acceleration is down- change in the other regions pro-
ward, toward Earths center. Yet the grammed for the production of essen-
ball moves upward with decreasing tial biomolecules will be disastrous.
speed, turns around, and moves down- If we assume the ideal case, that in
ward. Even at the turnaround point its principle the changes would be equally
acceleration is downward. probable at any random location along
Quite often people become con- any DNA chain, and we assume that
fused between velocity and accelera- the organism will grow and reproduce
tion. They are two different vector the next generation, then we could have
quantities that should be separated a molecular clock. However, as we
conceptually, but they are mathemati- know, just as all genes are not equal in
cally related. Their directions can be value at any given time, all DNA
the same or opposite along the line of sequences are not equal in value. In par-
motion. Newtons second law relates ticular, some DNA sequences code for
forces and accelerations but says noth- proteins that control the expression of
ing about velocities, for example. And other DNA genes themselves, turning
we know that Aristotle was wrong them on and off at appropriate times in
when he proposed that a force was the development of cells in the organ-
required to keep an object moving. ism. The Hox protein in insects, for
The real world operates with just the example, will determine the structure of
opposite rule because no net force is several different body parts, and slight
required to keep an object moving in a changes in its amino acid sequence are
straight line at a constant speed! major contributors to insect evolution.
Therefore, both complementary DNA
strands at the critical location need not
44. Molecular Clock be affected for the appearance of obvi-
The changes in the DNA during the ous phenotype changes.
evolution of organisms do not occur at However, the whole DNA mecha-
a common rate because any change in nism and its subsequent biochemistry

148 Answers
in the cell are much more robust than problems in their circadian rhythms
originally realized. The fact that many because the bright artificial lighting
of the amino acids have several DNA means that people stay awake long
base code triplets of nucleic acids for past sunset, delaying and shifting the
their selection is a built-in resiliency maximum in certain biochemical
that can produce a viable organism cycles beyond their evolutionary time
even when the DNA has an error of of day. In particular, greenish light
this kind. In addition, if the erroneous from televisions and clock radios
amino acid substitutes at a location passes into the eyes, even through
that is not critical for the 3-D shape closed eyelids while asleep, to trigger
and the operation of the protein, once the pineal gland to initiate some of the
again there is a built-in resiliency. Lets biochemical circadian rhythm shifts.
face the fact that Nature is much more Wright, K. Times of Our Lives. Scientic
clever than we can ever hope to be! American 287, no. 3 (2002): 5966.
Gibbs, W. W. The Unseen Genome: Gems
among the Junk. Scientic American 289,
no. 5 (2003): 4653. 46. Two Metronomes
Ronshaugen, M., N. McGinnis, and W.
McGinnis. Hox Protein Mutation and For the case of the periodic perturba-
Macroevolution of the Insect Body Plan. tion of one metronome by the other,
Nature 415 (2002): 914917.
the mode-locking occurs when the per-
turbing frequency is sufciently close
to the unperturbed frequency of the
45. SAD metronome. When a metronome is
Yes, if they suffer from SAD. At rst placed on the skateboard, the move-
one might think that the variation in ment of the pendulum causes the
the length of day and night changes so skateboard itself to move slightly, usu-
little from January to June that no one ally in the opposite direction to the
living at the Equator would suffer from pendulum swing, since the metronome
SAD. This reasoning is true if everyone base is kept in place on the skateboard
went to bed at sunset and arose at sun- by static friction or could be bolted
rise. The increasing light at sunrise down. Some of the energy of the
would trigger the start of another cir- metronome base motion is transferred
cadian rhythm that brings about bio- to the skateboard, and this very small
chemical changes in our bodies. amount of energy is further trans-
But even people living near the ferred along the skateboard in several
Equator are tuned no longer into the directions, with some amount reaching
rise and setting of the Sun. There are the other identical metronome.

Answers 149
If at rst this energy arrives at some of motion for each oscillator is mathe-
random phase point in the oscillation matically equivalent to describing a lin-
of the second metronome, eventually ear spring in a viscous medium with a
its regular energy delivery becomes uctuating driving force.
more and more effective in synchron- According to the first reference
izing the pendulum oscillations. Of below, the mode-locking can occur
course, the second metronome is acting also for a wide range of aperiodically
on the rst metronome in the same way driven nonlinear oscillators in the
simultaneously. The synchronization is physical and biological sciences, from
normally in-phase, but antiphase syn- nonlinear electrical circuits to neural
chronization can occur in special con- systems. As in the periodically driven
ditions. (See the second reference below systems, the synchronization of ran-
for details.) domly driven nonlinear oscillators was
The behavior can be represented found to be structurally stable, which
by two equations for two harmoni- means that even in the presence of
cally driven oscillators with a signi- small amounts of noise an approxi-
cant amount of dampening. If the mate synchronization is achieved.
dampening were not signicant, then Jensen, R. V. Synchronization of Driven
we would see two coupled pendulums Nonlinear Oscillators. American Journal
of Physics 70 (2002): 607.
alternating their swing behavior out of
Pantaleone, J. Synchronization of Metro-
phase from maximum amplitude to nomes. American Journal of Physics 70
nearly zero amplitude. In the actual (2002): 9921000.
case, the pendulums simply synchro-
nize and keep nearly identical time.
For the case of one of the pendu-
47. Time Symmetry
lums being driven by a force random in No, nature does not need to obey time
time, their fluctuating behavior can symmetry at the most fundamental
converge to an identical response. Both level. Equations sometimes have more
pendulums would exhibit the same symmetry than the actual underlying
random fluctuations eventually. For physics behavior. For example, even
both periodic and aperiodic driving though the tensor equations of general
forces, asymptotic stability results for relativity are time-symmetric, they
linear oscillators properly damped. can be derived from a more funda-
That is, small changes in the parame- mental type of mathematical entity
ters of the linear oscillator or the driv- called a twistor. Twistor equations of
ing force result in only small changes in general relativity are not time-symmet-
the asymptotic behavior. The equation ric. In applications to a black hole, for

150 Answers
example, the tensor equations predict
time symmetry, but the twistor ones
do not. As a consequence, the forma-
tion of a black hole and the time-
reversed version cannot both represent
real physical behavior.
One might think that the quantum
theory described by the Schrdinger
equation is time-asymmetric, the equa-
tion being rst order in time. As Roger
Penrose points out in the reference
and draw the straight-line path. Of
below, quantum theory and its equa-
course, all parts of the path must be on
tions are indeed time-asymmetric. The
the faces, and the appropriate faces
wave function can be used to calculate
sharing a common edge must retain
the probability of a future state on the
their relative positions and orientations.
basis of a known past state, but not
Steinhaus, H. Mathematical Snapshots, 3rd
the other waythat is, one cannot cal- ed. New York: Oxford University Press,
culate the probability of a past state on 1983, pp. 173176.
the basis of a future state. You cannot
retrodict the past!
Hilgevoord, J. Time in Quantum Mechan-
49. Moon Distance
ics. American Journal of Physics 70 The laser light pulse traveling from the
(2002): 301306.
Earth to the Moon and back will
Penrose, R. The Emperors New Mind.
Oxford: Oxford University Press, 1989, encounter the Earths atmosphere
pp. 354359. twice. The pulse will have an initial
known rise time and decay time, but
these times will be extended by pas-
sage through the air medium. We
Chapter 3 assume ideal reection at the Moons
Crazy Circles corner reflectorthat is, no pulse
spreading in angle or in time.
First, the ideal case. We assume
48. Spider and Fly that the laser source and the reector
To nd the shortest path between any on the Moon are opposite each other
two points on a cube not on the same on the line connecting the centers of
face, one convenient method is to lay Earth and Moon and that Earths
out the six faces of the cube on a plane atmosphere does not affect the transit

Answers 151
time. The major source of uncertainty table in a gridwork. Imagine placing
will be the ability of the detection sys- the ball in appropriate adjacent tables
tem to locate the half-height point on at mirror image positions; then draw
the rise time of the outgoing pulse and the straight line to the pocket to nd
the same point on the incoming pulse. the collision points on the cushions.
If the system is good to about a
picosecond in detecting this point,
then a transit time of 2.56 seconds for
the 3.84 108 meter distance corre-
sponds to a timing uncertainty of bet-
ter than one part in 100 billion, with
an uncertainty in distance of less than Normal billiards and pool tables
4 millimeters. That is, with the pro- are marked around the perimeter to
posed laser system, one can measure accomplish precision bank shots. Using
the distance to the Moon to almost the these markers takes practice. These real
same distance uncertainty as one can tables with their markings are not very
measure the length of a table with a similar to the ideal table discussed
meterstick! above. Moreover, the rolling ball before
Of course, the atmosphere will the collision with the cushion may have
foul things up a bit. The index of additional spinEnglishabout an
refraction and the change of this index axis not parallel to the table and per-
with altitude will both alter the light pendicular to the travel direction. The
pulse speed and spread out the pulse professional player uses all these quan-
rise time and rise shape. Sophisticated tities in the particular shot, but we
signal processing techniques can elim- amateurs simply enjoy the results as we
inate most of these atmospheric practice more of the many possible
effects. So the nal uncertainty will be improvements to our games.
determined in the electronics creating Steinhaus, H. Mathematical Snapshots, 3rd
the laser pulse and detecting the ed. New York: Oxford University Press,
arrival of the pulses leading edge. 1983, pp. 6164.

50. Ideal Billiards Table 51. Wallpaper Geometry


On this ideal billiards table for which Standing inside the cube, to your right
the incident and reection angles at the you see your left side of your person in
cushion are equal, one simply consid- the cube to the right. To your front
ers adjacent mirror image copies of the you see your back. To the top you see

152 Answers
the soles of your shoes. You see a 3-D 52. Space-Filling
array of yourself from many different
views, at many different distances, at
Geometry
many apparent sizes, and at many dif- First consider a two-dimensional at
ferent image intensities. This view is space. A plane tesselation (or two-
not like being inside a cube with dimensional honeycomb) is an innite
reecting mirrors on all sides because set of polygons fitting together to
no image is reversed. cover the whole plane once, with every
Cosmologists are trying to deter- side of each polygon belonging to just
mine whether our 3-D space is mathe- one other polygon. A regular tessela-
matically and physically discretethat tion has regular polygons. There are
is, compartmentalized into large cubes three regular tesselations of the plane:
or regular dodecahedrons, each being equilateral triangles, squares, and reg-
perhaps as large as 10 billion light- ular hexagons. There are additional
years or bigger. If so, seeing a galaxy in plane tesselations with two or more
one direction could be complemented convex polygon shapes. One also can
by seeing the same galaxy in the oppo- cover the plane with Penrose tiles,
site direction. Of course, several prob- polygon pairs with at least one poly-
lems exist, such as the distance being gon not being convex.
greater in one direction than in the Now consider an additional spatial
other, with the consequence that the dimension. A three-dimensional honey-
galaxy is being seen not only from comb (or solid tesselation) is an innite
the other side but also at a different set of polyhedrons tting together to ll
time in its evolution of structure. all space once, so that every face of each
There may be multiple copies of the polyhedron belongs to one other poly-
galaxies to confound things. There hedron. If we require all the polyhe-
might even be multiple copies of each drons to be identical, then the only
of us! Any positive results will bring regular honeycomb is the one lled with
about a revolution in our thinking cubes, eight at each vertex. If we allow
about space and time in the universe. two different regular polyhedrons, one
Levin, J. How the Universe Got Its Spots: can fill the space with eight regular
Diary of a Finite Time in a Finite Space. tetrahedrons and six regular octahe-
Princeton, N.J.: Princeton University Press, drons surrounding each vertex. These
2003, pp. 132155.
space fillings and others determine
Thurston, W. P., and J. R. Weeks. The many of the natural crystal structures.
Mathematics of Three-Dimensional Mani-
folds. Scientific American 251, no. 1 From the apparent simplicity of a
(1984):108120. 3-D space filled with cubes, one may

Answers 153
think that this solid tesselation would headstone. Archimedes was killed in
be the most likely mathematically if real 212 B.C.E. during the capture of Syra-
space is discrete instead of continuous. cuse by the Romans in the Second
However, mathematicians can show Punic War after all his efforts to keep
that the most likely and interesting 3-D the Romans at bay with his machines
discrete space is the non-Euclidean tes- of war had failed. Plutarch recounts
selation by dodecahedrons, of which three versions of the story of his killing
there are two kinds, depending on the that had come down to him:
angle of twist in relating one dodecahe- 1. Archimedes was, as fate would
dron to the adjacent one. For further have it, intent upon working out
information see the Thurston and some problem by a diagram, and
Weeks reference below. having xed his mind alike and his
Coxeter, H. S. M. Regular Polytopes. New eyes upon the subject of his specula-
York: Dover, 1973, pp. 5874. tion, he never noticed the incursion
Levin, J. How the Universe Got Its Spots: of the Romans, nor that the city was
Diary of a Finite Time in a Finite Space.
Princeton, N.J.: Princeton University Press, taken. In this transport of study and
2003, pp. 132155. contemplation, a soldier, unexpect-
Thurston, W. P. and J. R. Weeks. The Math- edly coming up to him, commanded
ematics of Three-Dimensional Manifolds.
him to follow to Marcellus; which
Scientific American 251, no. 1 (1984):
108120. he declining to do before he had
worked out his problem to a demon-
stration, the soldier, enraged, drew
53. Archimedes his sword and ran him through.
2. A Roman soldier, running upon
Gravestone him with a drawn sword, offered
Archimedes (287?212 B.C.E.), perhaps to kill him; and that Archimedes,
the greatest mathematician of ancient looking back, earnestly besought
times, was the rst to calculate the vol- him to hold his hand a little while,
ume ratio of the sphere inside the cylin- that he might not leave what he was
der. With a sphere and a cone inside then at work upon inconclusive and
the cylinder touching top, bottom, and imperfect; but the soldier, nothing
sides, Archimedes determined that moved by his entreaty, instantly
their volumes are in the ratios 1:2:3! killed him.
The Roman general, Marcellus, 3. As Archimedes was carrying to
tells of how he searched for and found Marcellus mathematical instru-
Archimedes gravesite with this ments, dials, spheres, and angles, by

154 Answers
which the magnitude of the Sun Therefore, a model with 106 neurons
might be measured to the sight, will not work as a useful scale model
some soldiers seeing him, and think- of the real brain.
ing that he carried gold in a vessel, Of course, one could simply take a
slew him. small volume of the brain containing 1
Archimedes was buried in Syra- million neurons and ignore connec-
cuse, where he was born, where he tions to other parts. Or one could arti-
grew up, where he worked, and where cially modify the unusable computer
he died. On his grave there is an model above by ensuring a few con-
inscription of , his most famous dis- nections or more to each neuron.
covery. Also placed on his tombstone Whether the behavior that ensues is
is the figure of a sphere inscribed realistic must be determined. The more
inside a cylinder and the 2:3 ratio of practical approach is to model a small
the volumes between them, the solu- section of the brainperhaps tens of
tion to the problem he considered his thousands of neurons and all their
greatest achievement. interconnectionsin a focused study
His nicknames were, the Wise and simulation. A grid of computers,
One, the Master, and the Great each representing one small section,
Geometer. could then be used to simulate a larger
Plutarch. Lives of Noble Grecians and portion of the brain. Hopefully, when
Romans. Translated by A. H. Clough. New
quantum computers become a reality,
York: Random House, Modern Library,
1992, p. 517. they will be able to simulate the whole
brain. Whether the brain behaves
quantum mechanically and requires
54. Brain Connections quantum superposition for its opera-
A million neuron model of the brain is tions is presently unknown.
still quite a formidable programming There is the remarkable problem
task for a computer simulation, but of information storage in the brain
there would be no information trans- that is, where exactly is information
fers from neuron to neuron. Why not? stored? If each neuron stores only one
Because any neuron in this model of bit of information, then the human
the human brain would have on aver- brain is not large enough by many fac-
age nearly zero inputs. One calculates tors of ten! In 1989 Roger Penrose
as follows: if 1011 neurons have 1,000 suggested that each neuron must be
connections each, say, then the average capable of storing many bits of infor-
is 1 connection per 108 neurons. mation, in contrast to the prevailing

Answers 155
ideas. Subsequently, the numerous calculate the extent of the arm to
microtubules in each neuron were determine its end point distance, thus
found to participate in the information requiring three more numbers, the
storage game. There still remains the lengths of the three parts. The space of
question of what each stored bit of operation is nine-dimensional and is
information represents. called a 9-D configuration space to
Penrose, R. Shadows of the Mind. Oxford: distinguish between physical space and
Oxford University Press, 1994, pp. coordinate space. Of course, one could
358377.
have determined this result by realiz-
ing that each rod end point requires
55. Configuration three coordinate values to be specied.
The movement of the arm to touch
Space the point is the next challenge. If feed-
There are many ways to approach this back exists in the robot, such as visual
problem of describing the arm position feedback of the hand position and the
in physical 3-D space. We consider one desired point location, the movement
approach only. In all approaches, the algorithm can use a correction proce-
end of the rodlike hand must touch the dure that becomes ner and ner as the
specified point, so three numbers ngertip approaches the point, as we
dene the end point of the hand. humans tend to operate. If there is no
Lets start at the fixed shoulder continual feedback mechanism, then
position. Two numbers will describe the algorithm must move the arm to
the upper arm position, the angle in the point directly, somehow knowing
the vertical plane measured from a where the ngertip is at all times. A
xed vertical axis through the shoul- systematic error cannot correct itself if
der, and an angle about this vertical no feedback exists. Many robotic arms
axis. Two more numbers describe the operate in both modes, first without
forearm position, an angle in the verti- feedback for rapid deployment and
cal plane measured from a vertical axis then with feedback for ne adjustment.
through the end of the upper arm, and As humans, we learn to perform
an angle about this vertical axis. Like- many tasks and do many of them sev-
wise, two more angles are needed for eral times daily. As a result, we often
the hand. forget how we learned a particular
At least six numbers are necessary procedure and how much practice was
for the robot to locate the particular required. To relive that learning expe-
point in the room. The program will rience, try using the other hand to

156 Answers
punch in data in a calculator, or some 57. A Spooky
similar task. The learning curve is
sometimes very steep!
Refrigerator
Yes. Just as you could remove a dot
from a piece of paper with an eraser
brought in from the third spatial
56. Farmer Chasing a dimension, a 4-D being could enter the
Goose refrigerator without needing to open
the door and remove a piece of food.
If the farmer is restricted to chasing
That is, 3-D objects are open in the
the goose along the instantaneous
direction of the fourth dimension.
line of sight to the goose, the farmer
Conceptually, visualizing a 4-D
will never be able to catch the goose
object is difficult in our 3-D world.
unless there is a head-on encounter.
Some mathematicians suggest letting
The best strategy for the goose is to
the fourth coordinate direction be rep-
run in a straight line, for then the
resented by ashing color, such as the
farmers velocity is eventually in the
sequence of colors in the visible spec-
same direction as the gooses direction,
trum from red to indigo. Take any 3-D
and the relative distance remains
objecta sphere, for example. As the
constant. Note that even when the
sphere moves in the fourth coordinate
goose changes direction often, the
direction its flashing color changes
farmer cannot close the gap com-
from red to orange to yellow, etc. A
pletely because the closer they are, the
2-D sheet of paper moving in the
more toward being parallel are the
fourth dimension would be changing
velocities!
its flashing color also to indicate its
In the real open-field experience
fourth coordinate value. The inherent
without the restriction, one strategy
color of the object does not change, of
that might work if the goose is inexpe-
course.
rienced is for the farmer to anticipate
Descriptions of 4-D objects inter-
the position of the goose and to get
secting our 3-D world are quite fasci-
there at the same time the goose
nating. For example, a 4-D sphere
arrives. However, most geese read
intersecting our 3-D world would rst
the game plan and change course in
appear as a point, then an increasing
midight.
3-D sphere, then a decreasing 3-D
Behroozi, F., and R. Gagnon. The Goose
Chase. American Journal of Physics 47, sphere, then a point, then gone! The
no. 3 (1979): 237238. analog in fewer dimensions would be a

Answers 157
3-D sphere intersecting a 2-D sheet of eight cubes. In each case we obtain the
paper, being rst a dot, then a widen- number 2 raised to an integer power.
ing circle followed by a narrowing cir- We can make a table and generalize to d
cle, then a dot again, and then gone. arbitrary dimensions.
Although most people would
expect there to be more mathematical Figure Dimension No. of Copies
difficulty and complications in even Line segment 1 2 = 21
higher dimensions than four, this Square 2 4 = 22
expectation is false. The mathematics Cube 3 8 = 23
Doubling figure d n = 2d
actually simplies with ve dimensions
and more! Much geometry remains to
We can now determine the dimen-
be worked out in a 4-D space, whereas
sion of an interesting but strange geo-
the mathematics is better understood
metrical object, the Sierpinski triangle,
in the higher dimensions.
named after the Polish mathematician
Gardner, M. The Colossal Book of Mathe-
matics: Classic Puzzles, Paradoxes, and who originally thought it up in 1916,
Problems. New York: W. W. Norton, 2001, shown here with its holes being gray.
pp. 137149.
Double the length of the sides, and you
Peterson, I. The Mathematical Tourist. New
York: W. H. Freeman, 1988, pp. 82107.
get another Sierpinski triangle, similar
Pickover, C. A. Surng through Hyperspace:
to the rst. For example, if the rst
Understanding Higher Universes in Six Easy Sierpinski triangle has one-inch sides,
Lessons. Oxford: Oxford University Press, the doubled one has two-inch sides.
1999, pp. 4470.
How many copies of the original tri-
angle do you have? Remember that the
58. Fractional gray triangles are holes, so we cant
count them.
Dimensions?
Ignoring the hole in the center of
Yes. Noninteger dimensions are known the double-sized Sierpinski triangle,
as fractal dimensions. A pathway
toward understanding fractal dimen-
sions begins by considering duplications
of well-known objects. A line segment
can be duplicated to produce two line
segments. A square can be duplicated
in each direction to produce four
squares. A cube can be duplicated in
each of its three directions to produce

158 Answers
we learn that doubling the sides of the their applications to the familiar phys-
original gives us three copies, so 3 = ical world. One interesting question is
2d, where d = the dimension according whether two or more objects with the
to the scheme in the table. Using a same fractal dimension must be
calculator, one nds that its dimension related in some fundamental way,
d = 1.585 . . . , a noninteger! either mathematically or physically.
In general, the mathematical Peterson, I. The Mathematical Tourist. New
expression for the dimension of the York: W. H. Freeman, 1988, pp. 114142.
figure is given by the ratio of two
logarithms: 59. Platonic Solids
dimension = logarithm (number of
self-similar pieces)/logarithm
(magnication factor).

For simplicity:
1. A dimension between 0 and 1 is
supposed to correspond to the
capacity of a set of points to partly
ll a line without achieving it com- One must turn the second identical reg-
pletely, out of having the whole ular tetrahedron upside down and
value 1 that is needed. rotate by 30 degrees about the vertical
2. A dimension between 1 and 2 is axis before mathematically pushing
supposed to correspond to the them through each other to form a six-
capacity of a line to partly fill a vertex regular solid. Obviously these six
plane, without achieving it com- vertices correspond to the vertices of a
pletely, out of having the whole regular octahedron if the new object
value 2 that is needed. were placed inside one. However, the
3. A dimension between 2 and 3 is sides are not convex and at. Our bi-
supposed to correspond to the tetrahedron does have twofold symme-
capacity of a surface to partly ll a try axes even though the two tetra-
volume without achieving it com- hedrons are rotated with respect to each
pletely, out of having the whole other.
value 3 that is needed. We might have asked you to place
There is a whole world of mathe- the two regular tetrahedrons face-to-
matics to be learned with fractal face in congruence so that the com-
dimensions and fractal geometry and bined object has ve vertices dening

Answers 159
a triangular bipyramid. There are now that the Greeks were absolutely
three twofold rotational symmetry right. That is, what we will be saying
axes, each one being through an edge in a very profound way, the finite
of the joining faces. However, this groups of symmetries in 3-space
bipyramid is not a regular polyhedron. see the simple Lie groups (and
If we were to extend the discussion hence literally Lie theory) in all
to four spatial dimensions, there are dimensions.
six regular solid convex objects analo-
gous to the ve Platonic solids in three One of us (F. P.) has proposed that
dimensions. The number of regular the fundamental building blocks of
convex solids, called regular poly- matter, the leptons and quarks of the
topes, does not increase with more Standard Model of particle physics,
spatial dimensions. Instead, all dimen- are described mathematically by the
sions beyond four have only three reg- specific rotational symmetries of the
ular convex solids. 3-D Platonic solids for the leptons and
The Platonic solids are special in by their 4-D analogs for the quarks.
many ways, but perhaps their most The key arguments are that the
important mathematical property was mathematical symmetry groups for
pointed out by mathematician B. these regular solids are subgroups of
Kostant: the Standard Model symmetry group
and that the lepton and quark mass
The ancient Greeks, especially the ratios can be directly related to the
school of Plato, had great reverence ratios 1:108:1728 of the invariants of
for the regular polygons in the plane these subgroups. Whether the natural
and regular solids in 3-space. The world mimics this fundamental math-
latterthe tetrahedron, cube, octa- ematical behavior has yet to be deter-
hedron, dodecahedron, and the mined by experiments at particle
icosahedronare often referred to as colliders.
the Platonic solids. The Greeks Coxeter, H. S. M. Regular Polytopes. New
York: Dover, 1973, pp. 4157, 126144.
believed that these regular figures
Kostant, B. Asterisque. In Proceedings of
were fundamental in the structure of
the Conference Homage to Elie Cartan,
the universe. If symmetry or its Lyons, July 1984, p. 13.
mathematical companiongroup Potter, F. Geometrical Basis for the
theoryis fundamental in the struc- Standard Model. International Journal of
Theoretical Physics 33 (1994): 279305.
ture of the world, then one of the
points of our lecture is the statement

160 Answers
Potter, F. Geometrical Basis for the
60. Intersecting Standard Model. International Journal of
Spheres Theoretical Physics 33 (1994): 279305.

Two identical three-spheres can inter-


sect in a point, a circle, a sphere (two- 61. Arm Contortions
sphere), and a three-sphere. Now
bring in a third identical three-sphere
to intersect with the former two in
appropriate combinations of points,
circles, spheres, and a three-sphere,
the latter when all three are coinci-
dent. With three intersecting identical
three-spheres, a resulting single two-
sphere can be obtained only when the
three three-spheres form a symmetrical
conguration.
If the leptons and quarks of the
Standard Model of particle physics are
physical manifestations of the finite
rotational symmetries of the 3-D Pla-
tonic solids and their 4-D analogs as
proposed in a model by F. Potter (see
the reference below), then the intersec-
tions of three-spheres will become
important in fundamental physics. A
quark would be defined in a 4-D
space, and its mathematical behavior Yes, if one allows the arm to move over-
would depend on the properties of head. This second rotation untwists the
three-spheres. The proton, for exam- arm and brings the orientation of
ple, is a real particle composed of three the book back to the initial one again.
quarks in our 3-D worldthat is, One can say that the arm-object pair
three 4-D entities according to the pro- requires two 360-degree rotations to
posed model. So three three-spheres return to the initial orientation. Such an
(representing the quarks) must inter- entity is said to mathematically corre-
sect to form a two-sphere that lives spond to a spin 12 systemthat is,
in our 3-D space. related to the continuous symmetry

Answers 161
group SU(2). Any lepton or quark Coxeter, H. S. M. Regular Polytopes. New
York: Dover, 1973.
wave function, such as the electron
Rieflin, E. Some Mechanisms Related to
wave function, behaves in this way Dirac Strings. American Journal of Physics
with respect to rotations and angular 47 (1979): 379381.
momentum.
Spheres, cubes, and other objects
with spatial symmetry also can be clas- 62. The Rotating
sified as spin 12that is, their rota- Cup
tions are described by symmetry
Done properly, you would see the
groups that are subgroups of SU(2)
same sequence in both cases. The cup
and SU (2) = SU(2) Ci , where Ci is
appears to rotate with changing rota-
the two-element inversion group. For
tion rates as time passes. We have here
the Platonic solids, the rotational sym-
an example of Galilean relativity for
metry groups are discrete instead of
uniform motion. At these familiar
being continuous, and some of these
slow speeds compared to the speed of
symmetry groups are subgroups of
light, the behavior of the rotating cup
both SU(2) and SU (2) because among
produces no surprises. We can walk
all the elements of both can be found
past the cup, or the cup can move past
the elements of nite order for the dis-
our stationary location.
crete subgroups.
In a later chapter, where we intro-
As you know, our practical experi-
duce the special theory of relativity
ence is mostly with spin 1 entities
(STR), we examine an object such as a
that is, those needing a 360-degree
cube moving past a stationary
rotation to return to the initial orien-
observer at enormous speed, and we
tation. Mathematically, these spin 1
could consider the opposite case, of
properties can be constructed from the
the observer moving past the station-
spin 12 symmetry properties. Mathe-
ary object. Of course, one sees the
maticians know that even more funda-
same behavior in both cases, just like
mental are the reection groups from
the symmetry we observe in Galilean
which all spin 12 properties can be
relativity. However, in STR an effect,
derived as two reections in perpendi-
now called the Terrell effect, explains
cular planes. The two books listed
why a cup approaching and passing by
below discuss these hierarchical rela-
at near-light speeds appears addition-
tionships and many more.
ally rotated, the observer being able to
Altmann, S. L. Rotations, Quaternions, and
Double Groups. Oxford: Clarendon Press, see the back side of the cup as it
1986. approaches!

162 Answers
63. Space and Time describe the dynamics of motion in
three-space. Spinors are equivalent
Together mathematical entities used in quantum
mechanical wave functions to describe
Using three real spatial coordinates
the electron and other fermions in
and one imaginary time coordinate for
(3 + 1)-D space.
calculations works correctly when
Quaternions were first discov-
calculating the squares of space-time
ered by W. R. Hamilton in the 1800s,
coordinates and their sums and differ-
and the quaternion q has one real com-
ences. The important relationship is
ponent and three imaginary compo-
the space-time interval dened by 2
nents. Just as complex numbers are
= c2 t 2 x 2 y2 z 2, where the
formed from pairs of real numbers,
xs are the four distances. How-
quaternions are formed from pairs
ever, physics textbooks that use an
of complex numbers. Thus one should
interval dened by 2 = + x2 + y2 +
assign the time coordinate to the
z2 c2 t2 are making a mathemati-
real component and the three space
cal faux pas in choosing three real
coordinates to the three imaginary
space coordinates and one imaginary
components of a quaternion. Hence,
time coordinatethat is, the set (x, y,
mathematically, we live in a quaternion
z, ict) with i being the imaginary and c
world with an imaginary 3-D physical
being the speed of lightinstead of
space and a 1-D real-time clock!
vice versa. Fortunately, this fundamen-
Altmann, S. L. Rotations, Quaternions, and
tal error does not affect the calcula- Double Groups. Oxford: Clarendon Press,
tions of time intervals and spatial 1986.
separations because these calculations Pickover, C. A. Surng through Hyperspace:
involve the differences of squared Understanding Higher Universes in Six Easy
Lessons. Oxford: Oxford University Press,
quantities. To be mathematically cor- 1999, appendix D.
rect in the (3 + 1)-dimensional space-
time, one must use quaternions, which
are numbers in the form q = a + bi + cj 64. Space > 3-D?
+ dk, with i, j, and k being 1 and a, There are several arguments for why
b, c, and d being ordinary real num- space is not larger than three dimen-
bers, because they are the numbers in sions. Planetary orbits are not stable
four dimensions that properly handle when n > 3, except for a circular orbit
rotations, translations, and Lorentz for n = 4, because the attractive force
transformations. Today, quaternions and the centripetal force both do not
are used everywhere in science to have the correct dependence on radial

Answers 163
distance. In 1917 P. Ehrenfest showed The considerations could be
that one needs to consider the Poisson extended to a universe with more than
equation for arbitrary dimensions to one time dimension! However, this
determine orbit stability. When the matter and others we leave for future
n 4 circular orbit for a body around challenges.
a central mass becomes slightly per- Bchel, W. Why Is Space Three-Dimen-
turbed, one can show that the com- sional? Translated by I. M. Freeman from
Physikalische Bltter 19, no. 12 (1963):
parison of the central force to the 547549. American Journal of Physics 37
centripetal force for the orbit depends (1969): 12221224.
on the perihelion value r1 and the Ehrenfest, P. Annalen der Physik 61 (1920):
440.
aphelion value r2 according to [1/2
Pickover, C. A. Surng through Hyperspace:
(n 2)1]/r12 < [1/2 (n 2)1]/r22,
Understanding Higher Universes in Six Easy
which cannot be true for n = 4 and Lessons. Oxford: Oxford University Press,
larger. In a 4-D space, a satellite 1999, pp. 202205.
launched from Earth toward the Sun
would either y away to innity or spi-
ral into the Sun.
The hydrogen atom is not stable
Chapter 4
when n > 3 because there is no energy Fly Me To The Moon
minimum for n 5, which is shown
using the indeterminancy principle
that is, the Heisenberg uncertainty
65. Gunfight
principle. For the case n = 4, the rela- One could do good classical physics
tivistic energy equation must be exam- here, but the lmmakers have turned
ined to show that no energy minimum the scene into Hollywood exaggera-
is available and the atom is not stable. tion. The physics is determined by the
Several other physical phenomena conservation of linear momentum.
would be unusual for n 4 dimensions. Assume that the victim of the shooting
There is no satisfactory propagation of is initially at rest, so the total momen-
sound waves or electromagnetic waves tum initially is all in the bullet (or
free of distortion and reverberation in buckshot) of mass m and speed V
spaces other than n = 1 and n = 3. before hitting the victim, at mV. After
Also, axial vectors such as the mag- the collision, the nal total momentum
netic eld and the angular momentum is in the backward flying person
vectors do not exist in even-dimen- plus bullet. If the victim has mass M
sional spaces. and the combined victim-plus-bullet

164 Answers
object has speed v, the total final backward. Falling backward, perhaps,
momentum is (M + m)v. For all inter- but not ying!
actions, by the law of conservation of There is the story of a famous
linear momentum, the final momen- physicist back in the 1950s who loved
tum equals the initial momentum. to watch gunghts in Western movies.
In the simplest case (assuming no The bad guy always draws first, he
frictional drag at the feet and ignoring noticed, but the good guy wins the gun-
transfer of momentum away to the ght. How could this outcome happen?
earth, etc.), its application yields (M + His hypothesis was that psychology
m)v = mV. Solving for the velocity v of played an important role, slightly hin-
the victim afterward produces v = dering the man who had to make the
mV/(M + m). Substituting reasonable conscious decision to draw rst. The
values of M = 80 kg, V = 400 m/s, and second man simply had to react.
m = 0.03 kg yields a blow-back Even today, the psychology of
velocity maximum of v = 0.15 m/s. choosing a physical action is an impor-
Most people can walk about 2 m/s tant factor, particularly in sports.
(i.e., about 4 mph). So we can con- There are tennis coaches (and coaches
clude that any shooting victim in other sports) who preach the psy-
depicted as being blown backward by chology of playing tennis, saying that
the impact of the bullet (or shotgun when you think too much on the court
blast of pellets) is ridiculous and instead of simply reacting, as you learn
belongs in the fantasy world only! to do in practice, then you are in trou-
A physicist wouldnt actually need ble. You are letting self no. 1 (your
to calculate the velocity backward mind) control self no. 2 (your body),
using linear momentum conservation and your tennis game will suffer. We
explicitly. Simply watching the behav- wonder whether the concepts hold
ior and movement of the shooter hold- true for playing the physics game, too!
ing the gun before and after the shot
reveals approximately how much
momentum is available by using New-
66. Body Cushion
tons third law. If the shooter isnt We doubt whether landing on top of
blown backwards by the recoil force another body after such a long fall
of the shot, the victim wont be either. provides much cushion! The impor-
Of course, someone will suggest that tant parameter here is the extent of the
involuntary muscle contraction in the collision time tthat is, how long
stunned victim causes the flying the collision of the heros body with

Answers 165
the object actually lasts. The longer because the change in linear momen-
the t, the better. We also need to tum will be almost double, even
know the acceleration a versus time though the collision duration may be
profile. In better words, what is the increased slightly. Automobile colli-
maximum acceleration to be experi- sions provide plenty of evidence about
enced by the heros body? By deni- the damages done to internal organs
tion, the average a = v/t, where v by sudden collisions with very short
is the velocity change during the time collision times. We doubt whether our
interval t. Shorter ts make the hero will be able to walk away from
experience more painful. the body cushion. In fact, our hero
Stunt professionals are often seen will be lucky to survive!
leaping off buildings or falling through
windows in movies, but their colli-
sions are with huge air-lled balloons
67. Cartoon Free Fall
that effectively extend the total colli- When the cartoon character steps off
sion time to half a second or more. We the cliff, the fall should begin immedi-
do not see their collision with the bal- ately, of course. The natural path is
loon in the program because the edit- essentially a parabola, with approxi-
ing process substitutes the desired mately free-fall acceleration downward
body lying dead on the concrete. and a constant velocity horizontally.
Back to the hero landing on the Even a cartoon character must have
other body. The collision time here some mass; otherwise the character
will be less than one-tenth of a second, could not exert a force on anything,
producing dangerous accelerations. including the ground being walked on.
For example, if the body falls from the Unless the upward buoyant force of the
top of a two-story building, its speed air balances exactly the gravitational
will be approximately 11 m/sec just force downward, no cartoon character
before collision. The acceleration dur- stepping off the cliff would remain in
ing the collision will be greater than suspension at the height of the cliff.
110 m/s2, very dangerous. Even if Even if the buoyant force was suffi-
bones are not broken in bringing the cient, why would its upward push dis-
heros body to rest, the internal organs appear suddenly to allow the character
will continue to move until they suffer to free fall?
a collision inside the heros body. And We should see the character accel-
if the hero bounces back upward, the erating downward with ever-increasing
acceleration can be even worse, speed unless the terminal velocity is

166 Answers
reached or the buoyant force balances increases rapidly in time as the ball
the weight. The collision at the bottom and wall both bend a little during the
is also subject to analysis. To prevent collision. The initial kinetic energy of
this sudden collision, sometimes the ball just before the collision
another character is able to run down becomes distributed in the distortions
from the clifftop just in time to catch of the ball and the wall. The interac-
the falling character. And sometimes tions between the molecules of the
we even see another person falling with wall material change as some of the
a greater acceleration downward to available energy from the collision
arrive in time to catch the victim! If the spreads from the immediate impact
fall is nearly at the free-fall accelera- area. If the total collision time is
tion, the runner must be mighty swift! extremely short, the energy distribu-
There are measured examples of skiers tion will be quite limited in distance,
going down Mount Fuji with accelera- and much of the energy is available for
tions greater than the free-fall accelera- ripping. Otherwise, if the collision
tion, but no runner has achieved this time is much longer, a big portion of
feat yet. And yes, an anvil always has a the wall will respond by deforming
greater acceleration downward than just a little, and ripping may not occur.
any other object (sure!)! A bullet going through the paper
at a practice target makes a fairly clean
hole for two reasons: (1) the contact
68. Silhouette of time is extremely short; (2) the bullet
offers a nearly round prole, so there
Passage is symmetry about an axis perpendicu-
The condensed matter physicist knows lar to the hole. Even then, close exam-
a lot about the physical properties of ination of the bullet hole reveals an
liquids and solids, so he or she proba- irregular surface and additional tear-
bly would say, Wow! How was that ing beyond the actual round hole.
done? The only realistic possibility is You can check out the advantage of
that the wall cut should be quite messy a symmetrical object. Now, with
and the cartoonist cleaned the edges appropriate safety precautions, try to
for heightened dramatic effect! rapidly push any shaped prole other
We can estimate how difcult the than round through a sheet of target
cookie cutter hole would be to achieve paper. The lack of cylindrical symme-
by considering a ball thrown at try perpendicular to the surface usually
the wall. The impact surface area creates enormous problems for the

Answers 167
material because slightly more energy resulting centrifugal force is called a
goes into some directions. Moreover, pseudo-force because the actual force
tearing is required to occur at different is acting radially inward toward the
distances along a noncircular prole, axis of rotation to accelerate the object
so there may be points where the trans- from its inertial straight-line motion.
ferred energy density is significantly We must assume that the structural
higher or lower than the surrounding integrity of the space station remains
paper regions. All these factors and intactthat is, the station was
several more act against a very clean designed for the rotation and for the
cut through the material. The cut allowed distributions of mass on
through a thicker piece of paper or a board.
wall that has signicant depth would Using the relation for the angular
be even messier. velocity = v/r, we can express the cen-
Of course, the cartoon character trifugal force as Fc = mr2. The whole
has several other options for getting spaceship has the same angular velocity
through the wall when time permits: about the rotation axis, so an objects
(1) simply paint an exit onto the wall radial acceleration increases linearly
through which only he can pass, or (2) with distance r from the rotation axis.
the character can carry around a hole An astronaut at one end of the dumb-
to be afxed where needed! bell must climbthat is, walk up a lad-
der and then down a ladderfrom one
end to the other through the middle,
69. Artificial Gravity where the radial acceleration is zero.
Space stations and spaceships have The muscular effort required changes
been created by authors and screen- throughout the climb, so the sensations
writers in a vast array of shapes and must be wonderful!
sizes. A rotating dumbbell shape has
appeared in many space adventures,
the rotation providing a pseudo-
gravity force for Earth creatures. The
70. Small Wings
rotation about the center of mass per- The small wings on such alien beings
pendicular to the long axis provides a are probably much too small for a 20
pseudo-acceleration acting radially kg body. One could argue that the
outward called the centrifugal acceler- planets gravitational force at its sur-
ation ac = v2/r, where v is the tangential face is much less than the value here
velocity value and r is the radial dis- on Earth, so the alien beings weight is
tance from the axis of rotation. The much less also. That may be so, and

168 Answers
the proposition is not unreasonable. the weight F = g m of the alien being
However, we still require a sufcient with small wings, or 200 N if g =
air density for the wings to do their 10 m/s2.
work and a breathing atmosphere for We assume that a very strong 20
our Earthling on the foreign planets kg individual can stretch out his arms
surface. (After all, this example has the horizontally to the side and push
human standing there breathing with- upward against two supports with
out any special oxygen supply.) about 200 N downward force to lift
We need to determine the required the body. However, this same individ-
density of the atmosphere of this alien ual will not be able to use small wings
planet, assuming an adequate supply of the same length and perhaps just a
of oxygen molecules for breathing by little wider than the arms to beat
our visiting human. Earths atmos- against the air with equal effect. If you
phere at sea level has a total density of doubt this hypothesis, put some arm-
about 1.4 kg/m3, of which O2 com- length wings on a strong person and
prises about 20 percent by molecular observe how easily he or she can lift
composition. That is, 1 cubic meter of off and hover a few centimeters above
air weighs 1.4 kilograms. The remain- the ground!
der, of about 80 percent, is N2 , which If the alien being were hollow
has a molecular weight of 28, com- inside so that its total mass is signi-
pared to 32 for O2. For simplicity, we cantly less than expected for the body
assume they have the same molecular size, there may be no problem with
weight, so we require an alien atmos- hovering.
phere to have about 0.3 kg/m3 of oxy-
gen available for breathing.
The gravitational acceleration g at
its surface determines the air density at 71. Shrunken People
the surface for a given molecular com-
position and air temperature prole. We assume that the proposed shrinking
Most planets will have an acceleration to one-hundredth scale can be accom-
not much different from the value of plished. Unfortunately, your weight
9.8 m/s2 here on Earth, as one can would remain the same (unless you get
check out for the planets in the Solar rid of molecules somehow), and your
System, for example. So the wings density would increase a millionfold!
must be capable of exerting an upward The area of contact of your feet would
force at least as great as the downward be 10,000 times smaller, so the pres-
gravitational forcein our example, sure at your soles would be 10,000

Answers 169
times greater, rising to about 20,000 forces, but the gravitational force
psi. Every step would break the con- always acts toward the center of Earth,
crete, or you would sink into the sometimes being a help and sometimes
ground until the upward normal force being a hindrance. The major problem
could balance you. Among other is the enormous energy requirement in
changes, your metabolism must change getting from the surface of Earth to a
enormously, for your high ratio of sur- reasonable distance away. Once the
face area to volume will mean that the spaceship is more than a few Earth
rate of heat loss has increased at least diameters away, its nuclear engine
100 times. Of course, we choose to operation can be reasonably efcient
ignore any consequences inside the in accelerating the vehicle. However, to
body for simplicity. get off the surface requires a tremen-
Notice that if the opposite happens dous amount of energy, and its rocket
and you grow bigger and increase your engines must throw out a lot of
size by a factor of 100 in all directions, momentum in the exhaust gases at
without adding molecules, your den- high speeds to achieve escape veloc-
sity decreases a millionfold. You ity. Newtons third law dictates this
would be blown away by practically momentum requirement. The particles
any breeze! But your greater problem ejected backward act on the rocket in a
would be that your density is now force pair, the rocket pushing particles
much less than the density of air, so the backward while the particles are push-
buoyant force upward would be ing the rocket forward.
greater than your weight. You are now To reach outer space from Earth,
a giant balloon being pushed upward the vehicle must provide a large supply
toward the upper atmosphere! Also, of energy and be able to eject a large
your metabolism would change dra- amount of momentum, usually by
matically, but again, we ignore any having a large supply of mass to eject.
consequences inside your body. You The energy needs can be accommo-
could probably make the journey dated by a variety of engineering
around the world in 80 days without designs. However, the physics is quite
the hot air! demanding on the amount of mass
ejected per second. The fuel mass used
for this propulsion is not consumed
instantly, so this fuel mass adds to the
72. Spaceship Designs mass of the vehicle at launch time.
Landing a spaceship on Earth and then Consequently, even more propulsion
taking off for space involve the same fuel mass and energy are required for a

170 Answers
launch than simply accounting for the 73. Warp Speed
payload itself. The mass of the fuel
supply soon becomes many times Spaceships with a warp drive to accel-
larger than the actual payload erate beyond the local speed of light
launched into space. cannot be ruled out just yet! One
So when the spaceship leaves its example is the expansion of the uni-
Earth spaceport and doesnt eject a lot verse, which carries everything along,
of stuff backward out of its rocket and speeds can exceed the speed of
engines, the lm is expressing a mode light. Distant quasars with recession
of operation that is not achievable velocities greater than c are a reality.
with present technology. But perhaps The analog on Earth may be to have a
the propulsion will be different in the 100-meter race on a stretchable track
future, say the disbelievers. So lets that can change length during the race.
now go to the extreme propulsion However, if the spaceship has the
limit. The most efcient process would technology to be able to distort space-
be particle-antiparticle annihilation, time itself in its local vicinity, then
conversion of fuel and antifuel com- there is no need for enormous speeds.
pletely to energy in the form of high- Simply contract the space in the front
energy photons according to Einsteins to bring distant points closer, warping
famous E0 = mc2. If we ignore many space-time directly. That starbase that
problems such as a source for antipar- formerly was light-years ahead of you
ticles, radiation exposure, and so on, is now close to you, reachable by nor-
and also assume that all the photons mal propulsion in minutes!
Krauss, L. The Physics of Star Trek. New
are eventually directed rearward, each York: HarperPerennial, 1996, pp. 5658.
kilogram of fuel could provide 3
1016 joules of energy and 3 108 kg-
m/sec of linear momentum. To acceler-
ate upward at about 10 m/s2, a
74. North Pole Ice Melt
kilogram of this fuel can provide a mil- There would be no change in the sea
lion-kilogram spaceship with 30 sec- level if all the ice at the North Pole
onds of thrust. If one requires 3,000 melted. Why? Because this ice is oat-
seconds of thrust, simply use 100 kg of ing on water. Upon melting, the water
matter-antimatter fuel. We look for- molecules in the ice simply occupy the
ward to the future of space travel with space of the liquid displaced by the ice
antimatter engines, but for the present originally. Of course, we need to be
we can enjoy the entertainment pro- more careful on dening the extent of
vided by space travelers in lms. the North Pole. If we include ice on

Answers 171
some landmass, then this ice will add kilometers inland from the sea, and the
extra water molecules to the liquid arid climate made living off the land
seas and slightly raise the sea level. In very difcult. So they migrated along
contrast, most of the ice at the South the eastern coast of Africa all the way
Pole is several kilometers thick and is through the Middle East and India to
predominantly on the Antarctica land- Australia. The Aborigines in Australia
mass, so its melting could signicantly are descended directly from these peo-
raise the sea level. Some lms depict ples from Namibia and South Africa
oceans that have risen hundreds of and together with the Bushmen are the
meters from the worldwide ice melt. oldest civilizations on Earth.
Simple estimates easily reveal that this
conjectured amount of sea level
change is ridiculous. 75. Lightning and
Another concern might be the lin-
ear expansion of water when its tem-
Thunder
perature is above 4C of about 70 Lets consider identical explosions on
106/C. Even if the water temperature the battlefield at two different dis-
rose by 10C throughout the rst 10 tances from the observer (i.e., the cam-
kilometers of ocean depth, the era) but seen simultaneously. The
expected water level rise for a column sound intensity emanating from the
of water would be no more than 7 farther explosion should be less loud
meters if the surface area remained compared to the closer one, and the
constant. But the surface area will sound of the farther explosion should
expand, so the actual water level will be delayed more with respect to its
rise about 2 meters of so. For a 1C light ash than for the closer explo-
increase in temperature, the ocean sion. The extra distance affects both
level rise will be several centimeters. the sound intensity and its time delay.
Sea level changes played major Depending on the distance and the
roles in the migrations of our human temperature gradient in the air, there
ancestors. Some Bushmen left their could be additional effects, such as dif-
homeland and their cave dwellings ferent frequencies having slightly dif-
about 40,000 years ago, seeking better ferent speeds and/or paths en route.
climates and less arid lands. A mini ice For example, if the closer explo-
age had developed, so seawater sion is one-fth of a mile away and the
became locked into the ice at the poles. farther one is two-fifths of a mile
Their caves, which were originally near away, the differences in arrival times
the sea, were now several hundred should be clearly heard. The sound

172 Answers
from the closer one begins to be heard As a dramatic example, in 1987
one whole second after the light ash Supernova 1987A in the Large Magel-
and the weaker sound from the farther lanic Cloud, bound to our Milky Way
explosion should be heard one second galaxy, blew apart, dumping practi-
later, two seconds after the apparently cally all its energy into neutrinos, but
simultaneous light ashes. Count these there was still enough energy to pro-
time delays aloud and you will imme- duce a lot of photons for a bright ash
diately realize that most battlefield of visible light that was rst seen by
scenes do a false portrayal of the audio amateur astronomers in Japan. Today
timing. But who really cares? The dra- that light flash continues to expand
matic license enhances our viewing with decreasing intensity, and the gas
experience of the fantasy world of the cloud of particles also continues to
cinema! But experienced military peo- stream outward, impacting molecules
ple know the difference. and other stuff in various directions to
Many more audio violations can be help paint the region in beautiful
recalled from the movies. We simulta- colors. Data seem to indicate that the
neously hear and see airplane crashes in remains at the origin consist of two
the distance, cars falling and crashing small, massive objects orbiting a com-
into a chasm far below, jet airplanes mon barycenter.
passing overhead with no sound delay, The real problem with hearing an
etc., all for the benet of a moviegoing explosion in space is that there is no
public who should know the difference. medium to carry the sound waves, so
no sound from the original explosion
should be heard by the spaceship crew
76. Explosions in Outer or by the audience in the theater. The
light travels at nearly 3 108 m/s, and
Space the sound would transit at the snails
The colors of the explosion are proba- pace of 3 104 m/s or slower. The light
bly okay, and perhaps one can appre- ash comes before the sound for any
ciate the beautiful outward-going safe distance. Of course, any debris
streamers isotropically distributed. from the explosion hitting the space-
However, very seldom does a real craft would make an impact sound
explosion, even in a vacuum, distrib- carried by the walls of the ship and by
ute the stuff isotropically. One also the internal artificial atmosphere.
would expect bits and chunks of vastly Then there is the noise of the other
different sizes, with a large chunk or spaceship going past, but that is
two left over near the explosion origin. another story.

Answers 173
77. Space Wars scattering of the light to your eyes by
the molecules in the air. Spots where
Dont try to learn your physics from the beams strike walls, mirrors, or any
space battles. Practically everything is objects could be seen, but their
incorrect, except the ability to cause straight paths to the spots are invisi-
something to explode if your weapon ble. Therefore you have two ways to
can dump enough total energy into the make the laser beams visible so your
target in a short time! audience can see the sequence of
The laser beams, no matter how maneuvers required to succeed in the
powerful they are, would not be seen heist: (1) put something in the air itself
en route. Seeing them requires that a to scatter laser light such as chalk dust,
reasonable amount of light be scat- smoke, or a fog of liquid nitrogen
tered back to you along their paths. droplets or water mist, or (2) artifi-
But there is practically nothing in the cially put in the light beams during
vacuum of space to scatter off. A few editing, after the scene is done.
hydrogen atoms per cubic meter are We suppose that some real-world
all there is out there. locations may use an arrangement of
The explosion would not be heard crisscrossed laser beams for security
at all. Sound requires a medium for its aids, but we do not know of any. Cer-
transport, and the vacuum of space is tainly, infrared lasers are much
not a material medium able to conduct cheaper than visible lasers, and they
sound waves. You will feel something, would not be seen by injecting a fog
the impact of the bits of the exploding into the space. Nor would their spots
debris from the enemy battlecruiser on the wall be seen.
because they travel unimpeded from Some heist scenes show the substi-
the explosion volume to your space- tution of a mirror to fool the security
ship. Their speeds may be enormous, system while the protagonist steals
so they could do signicant damage if the item. Indeed, the mirror substitu-
your shields are not in place. tion may reect the beam in the cor-
rect direction, but the tenths of a
second disturbance in the original
78. Security Lasers beam would be very easy to detect by
As the director of the theft scene, you the security system electronics, which
would know that the laser light would can sense disturbance spikes quickly
not be visible in the air in a normal and sound an alert. Of course, the
room because there is not enough human security operator may choose

174 Answers
to ignore the alert, which so often hap- 80. Internet Gaming
pens in movies!
The Internet is usually not the culprit,
being extremely fast compared to the
79. Bullet Fireworks hand-eye coordination of the player.
Normal bullets are copper-clad lead The actual travel time between the
and do not spark upon impact with major switching stations along the
steel or any other surface. Just go to a Internet is extremely short because
grinder to check out the properties of the data packages travel at nearly the
copper versus other metals in the pro- speed of light in a vacuum. So a data
duction of visible sparks. The grinding package can go 20,000 kilometers in
of steel produces sparks everywhere, about 70 milliseconds with no delays.
seen even in sunlight. The many small, Delays at the switching stations along
hot particles of steel are actually burn- the Internet are typically in hundreds
ing. Now grind a piece of copper tub- of milliseconds, with your local Inter-
ing. No sparks. You might see an net service provider (ISP) contributing
occasional spark due to contamination most of the delay time, on the order of
on the grinding wheel or in the copper. 300 milliseconds or so.
The copper bits ground off do react In contrast, your local computer
with oxygen, but they do not get very and cable modem, for example, usu-
warm. Please do not grind lead because ally will be slower in responding to
toxic particles will be released into the your input and getting the data out to
air and, besides, lead is known not to your ISP and onto the Internet. The
produce sparks. So the conclusion is faster the equipment at your end, up to
that practically no bullets produce a a certain speediness, the quicker your
brilliant ash of light on impact. response will appear in the game. If
In support of the lm depictions is your equipment delay time becomes
the fact that the military does have less than the Internet delays, theres
machine gun bullets containing white nothing further to be improved by
phosphorus so that the point of impact spending more money on a faster com-
can be seen by the gunner. These bul- puter system.
lets are used also to ignite fuel tanks
and other possible containers of explo-
sives by producing sparks to ignite the 81. Cartoon Stretching
vapors. But phosphorous bullets are There are many ways to approach the
very rare outside the military. problem of determining the speed of

Answers 175
sound in the cartoon characters body. slow speed of sound indeed when com-
We consider one approach only. Start pared to most materials, which have a
with the application of an external speed of sound of about 300 meters per
force to the surface of the body at second. So we conclude that cartoon
some locationthe characters foot, characters are made of very unusual
say. We see the stretch region progress materials. Perhaps designer materials in
up the leg over a period of one to two the future will be able to mimic a car-
seconds. toon material. According to the worlds
Wait just one minute! you greatest detective, the game is afoot!
exclaim. How is the speed of sound in
the material related to its speed of
stretching in response to an applied
82. Infrared Images
force? The answer is that both The actual converted infrared image
processes require communication from would be a blurred black-and-white
one molecule to the next molecule out- image, not a sharp one. The physics
ward, from the applied force region to places a limit on the resolution of
the far reaches. Usually the much images we see in the infrared com-
smaller energy in the sound application pared to the visible. When we look
produces a very tiny stretch followed through binoculars or any lens system
by a relaxation and overshoot, then in the visible, these systems have a res-
another stretch, etc., repeatedly at olution limit that depends on the qual-
some frequency above 14 Hz or so. ity of the optic elements. No matter
The stretch produced by the tug of a how much the image is enlarged or
much larger applied force shows a enhanced by dithering, etc., the origi-
much larger displacement of the mole- nal resolution is not improved even
cules that may or may not relax when though the image looks cleaner. But
the applied force is lessened. The larger the physics is even more restrictive
displacement between molecules for when comparing an infrared image to
the stretch process may require slightly a visible image because the visible light
more time if the process cannot be has been coherently scattered from the
modeled by a collection of linear har- object, whereas the infrared light has
monic oscillators, but the speed of not, as explained below. Of course,
stretch will be very close to the value of one cannot exceed the Rayleigh crite-
the speed of sound for most materials. rion for resolution of approximately
The stretched cartoon characters one wavelength of the light, except by
body exhibits a communication speed using interference techniques, which
of about a meter per second, a very we do not consider here.

176 Answers
In the visible part of the electro- will be able to take an infrared image
magnetic spectrum, atoms absorb and and make a sharpened black-and-
emit the photons of light in a two-step white image true to the original object.
process, usually absorbing and emit- In the ultraviolet, the scattering
ting in about 1016 second. During this time is very short, but so is the wave-
time interval, the molecule holding length, so the extent of the coherent
that atom moves very little. Nearby scattering area also is very short; thus
atoms also scattering this impinging the image is blurred compared to the
beam of numerous photons tend to visible one. Nature has given us a
remain in place during their scatter- vision range in the visible that ensures
ings. In effect, during the scattering of the best resolution of detail.
each photon there always will be a
xed phase relationship among all the
atoms scattering light from the object
83. Light Sabers
to your light sensors. Hence, photons Yes, the light sabers would pass
scattering from different areas on the through one another as if nothing were
objects surface carry detailed phase there! The photons of the light are
information with fixed phases to bosons, which do not repel each other.
achieve nearly maximum resolution. If The clashing of the light sabers is pure
the phases actually varied randomly artifact and far beyond artistic license.
from one location to another, the visi- Its an outright lie to the public!
ble image would become blurred. There are two general categories of
In the infrared, the image is blurred particles in the universe: fermions and
because most of the infrared is bosons. Two identical fermions (e.g.,
absorbed and emitted by molecular think electrons, or any other funda-
vibrations and rotations that have ran- mental spin 1/2 particle) cannot exist
dom phases over the objects surface. in the same quantum state dened by
This scattering process takes much its 4-momentum and spin. The exis-
longerabout 1012 secondsufcient tence of all matter, including us
time for the molecule to move consid- humans, critically depends on the
erably during the scattering, so there inability of two identical fermions to
will not be a xed phase relationship get together in the same state, so mat-
among neighboring molecules on the ter occupies a volume. That is, objects
surface of the object. The same surface can be bigger than a point!
that appeared well resolved in the visi- In the case of bosons, one can not
ble will now be quite blurred in the only put as many identical bosons
infrared. No magical digital techniques (think photons or any integer spin

Answers 177
fundamental particles) into the same the battleeld scatters enough of the
quantum state, they also prefer to be light. But when the laser beam origi-
in one, with the probability to do so nates in space where there is no air (or
enhanced by the number N of bosons other scatterers in signicant density),
already in the state. There is no repul- one can still see the laser beams. Is
sion experienced. So two light sabers there no correct laser physics in these
intersecting at an angle would pass movies? Is there any correct physics
right through one another with no at all?
change in either. If the light beams
were powerful enough, though, they
could cut material objects made out of 85. Cold Silence of
fermionsthat is, the ordinary stuff
all around uswhenever this material
Space
stuff intercepts the beams because suf- Yes and no. The very good vacuum
ficient energy could be absorbed to between Earth and Venus, for exam-
change the physical state of the mate- ple, certainly does not conduct sound,
rial and result in vaporization. so the space is silent. But what is the
temperature of outer space? This ques-
tion is an improper question, for we
84. Force Fields must instead ask: What temperature
We dont know why we can see the would be recorded by a thermometer
good guys through the battleeld force placed in space between the Venus and
eld via visible light at the same time Earth orbital distances? Whatever
that the visible laser beams cannot get kind of thermometer we use, if the sen-
through! This result might be due to sor is not rotated, one side will be
some dramatic nonlinear natural effect exposed always to the direct rays from
not yet experienced in research labs, or the Sun and the other side will be in
the phenomenon is a pure artistic shadow. At thermal equilibrium, the
falsehood. We would bet on the latter. amounts of outgoing radiation energy
Obviously, the light transmission and incoming radiation energy in all
properties of a transparent material directions will balance.
can change dramatically on absorbing The equilibrium temperature T
energy, but the usual effect is hole- for an object at Earths average dis-
burning, not reection. tance from the Sun is about 280 K, or
Then there is the problem of why +7C, the actual value being lower
the laser beams can be seen along their because there will be some energy
paths, unless the dust in the air above reflected away and not absorbed by

178 Answers
the thermometer. One calculates T effective in limiting the initial spread
from this equation: flux absorbed = of the debris from the thermal explo-
ux emitted. If we assume for simplic- sion. No nuclear explosion would
ity that the sensor is a sphere of radius occur, or else everything around would
R, then the equation becomes S (1 A) be vaporized by the energy released, in
R2 = T4 (4R2), where S = 1.4 kW which case the depth in the water
s1 is the solar constant at Earth dis- would help very little in preventing the
tance and = 5.67 108 W m2 K4. spread of energy in many forms. The
The parameter A is the reflectivity, thermal explosion in a nuclear reactor
which we take as zero in the ideal case in the sub releases the fuel and
of a perfect absorber and radiator. coolant, so radioactive particles and
Real materials will have an A value debris will be sent out in all directions.
between 0 and 1. Some of this stuff would be slowed
If you are orbiting at the Earth dis- effectively by the water, and some
tance from the Sun, you may desire to probably would make the surface and
rotate your spacecraft slowly in space escape into the air.
so that all sides cook evenly! This pas- When the Chernobyl nuclear reac-
sive heating can be augmented by tor broke its containment vessel in the
active heating from within to maintain 1980s, its nuclear particles were
a cozy environment. detected around the world within
If you are orbiting closer to the hours to days. At the University of
Sun than Earth, the equilibrium tem- California at Irvine, the air filtering
perature will be higher, the solar ux system at the local nuclear reactor
increasing as the ratio of the squared recorded the radioactive cesium and
distances. Near the orbit of Mercury iodine particles in parts per billion
you may be too hot! If you are farther from the Chernobyl incident 10 days
away, the temperature decreases, so after the chemical explosion.
you may need articial heating. Some
rotation may produce a system that
requires less fuel for heating, but the 87. Plutonium vs.
details need to be worked out.
Uranium
A plutonium bomb would be much
86. Nuclear Submarine safer to handle. Weapons-grade pluto-
Submariners love to dive in their sub- nium (Pu-239) emits primarily alpha
marines. A dive to several hundred particles and low-energy gamma rays,
meters under the surface may be both being easy to shield. The trace

Answers 179
amounts of even-numbered Pu iso- 88. Nuclear
topes have spontaneous ssion reac-
tions that emit neutrons. However,
Detonation
neutron detectors would need to be We know of no nuclear explosive
within a few meters to detect these device that does not require at least
neutrons above background. So a lost very good spherical symmetry to be
plutonium bomb could be very dif- detonated. The simpler atomic devices
cult to nd. have either two hemispheres that must
The plutonium usually is coated be rapidly moved together into a
with beryllium or another appropriate sphere, or two spherical sections held
sealant because the exposed element apart until a slab of nuclear material is
will react chemically with the oxygen shot into the gap to start the ssion
in the air or in water and increase its reaction. The more complicated hydro-
temperature considerably. The person gen devices require a strong, spheri-
holding a plutonium bomb would cally symmetrical implosion from the
probably feel that the protective casing perimeter shell to initiate the fusion
is warm, because the alpha particles reaction.
deposit their kinetic energy in the cas- Dropping the weapon from any
ing material. height would damage the casing asym-
Of course, inhaled or ingested Pu is metrically. Even shooting pellets or
one of the worst carcinogens known. bullets, etc., through the casing into
Any explosion releasing Pu into the air the warhead would create an asym-
creates a hazard for all life that would metrical result but no explosion. Initi-
remain for a long time. ating the nuclear reaction and keeping
In contrast, weapons-grade U-235 the reaction going are not easy. The
releases gamma rays at several ener- process is certainly not worth worry-
gies, the most intense at 186 KeV. So ing about to the extent portrayed in
the detection of a device containing U- the movies. One should be more con-
235 is much easier than trying to nd cerned about whether the proper
plutonium. Some lms portray the dif- safety precautions are being practiced
ferences correctly, while other films against accidentally dropping the thing
dramatize any nuclear device and its on ones toes.
possible dangers with remarkably ado- By the way, the smallest practical
lescent scare techniques. nuclear weapon tips the scales at only
about 9 kilograms, about 20 pounds,
and is small enough to t into a bulky

180 Answers
attach case. For the physics details that is, time passes at its normal rate
and an estimate of the smallest device, for all particles and collections of par-
see the reference below. ticles. One possible explanation for all
Bernstein, J. Heisenberg and the Critical particles behaving the same with
Mass. American Journal of Physics 70 regard to the passage of time relies on
(2002): 911916.
the direction of time being built into
the quantum state denition of a fun-
89. Fabric of damental particle, with the opposite
direction of time for its antiparticle.
Space-time The phrase speed of gravity is
Space-time is not a piece of cloth, nor meaningless in the simplest interpreta-
does space-time behave like a real mate- tion, being an error in stating the
rial. The metaphor fabric of space- acceleration of gravity. Most films
time allows one to visually model confuse the concepts of speed and
space-time with its coordinates of space acceleration in the same manner that
and time in a way similar to the simpler most people do. Unfortunately for
two-dimensional construction of woven society, the scriptwriters did not learn
cloth. There is no way for space-time to their beginning physics, and we all
rip or tear, although mathematically continue to suffer from their intuitive
there can be singularities of various mistakes in describing the behavior of
dimensions and other mathematical nature. Or perhaps a more positive
properties that some people have view should be taken in that the enter-
stretched into real physical properties tainment value is improved by ignor-
in the name of dramatic license. ing the laws of physics!
As far as being able to go back in
time as a time traveler, the time dimen-
sion of space-time is not like the spatial
dimensions of space-time. Mathemati-
Chapter 5
cally, the passage of time operator in Go Ask Alice
quantum mechanics is antiunitary,
while the spatial displacement operator
is unitary. In addition, no one has
90. Spotlight
shown convincingly that a fundamen- Yes, a spot of light from the lighthouse
tal particle can progress either forward beacon when moving across your eld
or backward in time differently than of vision can move faster than c. But
we now experience and understand the actual light itself (i.e., the photons

Answers 181
in the beam) moves from the source 91. Quasar Velocity
to the reecting spot in the sky at c,
no faster. The special theory of relativity says
A good example from astrophysics that information cannot be transmit-
is the radio wave beam from the pulsar ted faster than light. The photons
in the Crab Nebula, which sweeps always travel at light speed in the local
across our Earth observatory thirty reference frame, but the space in which
times a second from a distance of a the photons travel may be expanding.
few thousand light-years. The electro- An analogy would be a 100-meter race
magnetic waves coming to us from the with the track lengthening during the
distant source travel at light speed, but race. The elapsed time to reach the
the sweep across our planet moves 100-meter tape depends on the model
faster than c. for the expansion rate for the runner
Occasionally one encounters other and for the photons from that distant
suggested examples of spots traveling quasar. Under these expansion condi-
faster than light speed, such as the tions, there can be recessional veloci-
intersection edge in a very long pair of ties greater than c!
Chown, M. All You Ever Wanted to Know
scissors progressing outward when about the Big Bang. New Scientist (April
closing. Unfortunately, this intersec- 17, 1993): 3233.
tion edges speed is limited by the Peebles, P. J. E., D. N. Schramm, E. L.
speed of sound in the metal of the scis- Turner, and R. G. Kron. The Evolution of
the Universe. Scientic American 271, no.
sors, which is quite slow compared to 4 (1994): 5257.
c. However, the spot of light on an . Out with the Bang. Scientific
oscilloscope trace can move across a American 272, no. 3 (1995): 10.
screen faster than c even though this Stuckey, W. M. Can Galaxies Exist within
spot is produced by the slower-moving Our Particle Horizon with Hubble Reces-
sional Velocities Greater than c? Ameri-
electrons striking a phosphor. can Journal of Physics 60 (1992): 142146.
Bergmann, P. G. Can a Spot of Light Move
Faster than c? The Physics Teacher 19
(1981): 127.
Rothman, M. A. Things That Go Faster 92. Spaceship
than Light. Scientic American 203, no. 1
(1960): 142152.
Approach
. Not So Fast. Scientic American The observer sees the highly relativistic
269, no. 6 (1993): 10. object approaching back side first!
Therefore, the spaceship seems to be
approaching tail rst! What is often
referred to as a contraction in the

182 Answers
direction of motion for a rela-
tivistically approaching object is
actually a rotation known as the
Terrell effect.
We need to discuss some aspects
of the Terrell effect to explain the
behavior of the approaching space-
ship. Consider a solid, opaque cube
approaching. At low speeds, the light
rays emitted off the back side of the actually a rotation for a real three-
approaching cube cannot pass through dimensional object. What we have
the box to reach the observer. At described above is a snapshot of the
higher speeds nearing the speed of spaceship (and cube)that is, what
light, however, enough of the box photons from different parts of the
moves out of the way for light emitted object would imprint on a camera
from part of the back side to reach the sensor simultaneously. E. Sheldon (see
observer. When this behavior happens, the reference below) discusses the
the observer will not see all of the stereoscopic appearance of a three-
front side because some of the light dimensional object that involves shear-
rays from the front are intercepted by ing and other distortions in addition to
the extremely fast moving box. The rotation, all these effects rst discussed
box appears rotated, with the away by J. Terrell.
side of the front hidden, and the near Sheldon, E. The Twists and Turns of the
Terrell Effect. American Journal of Physics
side of the back visible. The rotation 56 (1988): 199200.
angle increases with increasing speed, Terrell, J. Invisibility of the Lorentz Con-
nearing c and with proximity to the traction. Bulletin of the American Physical
trajectory. Additional complications Society 5 (1960): 272.
also occur, such as nonrigidity, which Weisskopf, V. F. The Visual Appearance of
Rapidly Moving Objects. Physics Today
we ignore in this simple explanation. 13 (1960): 2427.
So the spaceship approaching at
near light speed will appear rotated so
that the back end is almost totally
93. Mass and Energy
visible and the front end is almost The answer to both questions is equa-
totally hidden from view. J. Terrell in tion 1, although the majority of physi-
1959 was the first to recognize that cists seem to prefer equations 2 or 3!
what physicists had been calculating Their choices probably are caused by
as a Lorentz-Fitzgerald contraction is the confusing terminology widely used

Answers 183
in the physics literature that says that the measurement of the length compo-
a body at rest has a proper mass nent along my direction of motion of
or rest mass m0 , and a body in the metal bar that appears to be
motion has a relativistic mass m = rotated. The atoms do not move closer
m0 /
(1 v
2/c2) . to one another, so the strain gauge
There is only one mass in physics, remains at zero.
m, which does not depend on the ref- The apparent rotation is called the
erence frame. This mass m is the rela- Terrell effect: if a snapshot is taken of
tivistic invariant quantity in E2 p2c2 a moving object, the object does not
= m2c4, whereas the energy is different appear contracted, but rather rotated.
in different reference systems. There is A snapshot is understood to be a two-
no need to place the index 0 with the dimensional, nonstereoscopic photo-
mass. However, the total energy E graph. The stereoscopic appearance of
needs the 0 index if the particle has no a three-dimensional object is more
momentum in that reference frame complicated because shearing and
that is, E0 = mc2. other distortions can be present. In
For a complete and stimulating fact, there is no such thing as a rigid
discussion of these ideas and their object in relativity!
history see the L. V. Okun reference DeCampli, W. M. A Gedanken Experiment
below. to Demonstrate Lorentz Contraction. The
Physics Teacher 13 (1975): 420422.
Einstein, A. Zur Elektrodynamik bewegter
Krper. Annalen der Physik 17 (1905): 891. Terrell, J. Invisibility of a Lorentz Contrac-
tion. Bulletin of the American Physical
. Ist die Trgheit eines Krpers von
Society 4 (1959): 294.
seinem Energieinhalt abhngig? Annalen
der Physik 18 (1905): 639. Translated by . Invisibility of a Lorentz Contrac-
W. Perrett and G. B. Jeffery in The Principle tion. Physical Review 116 (1959):
of Relativity. New York: Dover, 1923, p. 10411045.
71. . The Terrell Effect. American
. Zur Theorie der Brownschen Journal of Physics 57 (1989): 910.
Bewegung. Annalen der Physik 19 (1906):
371.
Okun, L. V. The Concept of Mass. Physics
Today 42, no. 6 (1989): 3136.
95. Mass/Energy
Mass is energy. There is no distinction
to be made. There is no conversion! In
94. Strain Gauge 1905, Einstein states explicitly: The
The strain gauge continues to show a mass of a body is a measure of its
zero value. What I interpret as a length energy content . . . . What Einstein
contraction when I run past is really was stating is that mass and energy are

184 Answers
equivalent, that they are possibly two the amount equal to the total kinetic
different aspects of the same physical energy of all the particles as seen in the
quantity, only their units have been frame in which the total momentum is
chosen to be different. One does not zero. The exception yes occurs when
convert one to the other if they are all the particles move in the same direc-
equivalent. tion with the same speedthat is, have
We can imagine a conversation the same velocity.
with a student: Does a photon have The value of dening the mass in
mass? asks the student. Yes, because this relativistic fashion means that M
the photon has energy. The student determines the systems inertia, its
counters, But for a photon E = pc, so resistance to acceleration by a force
the relation E2 p2c2 = m2c4 becomes that acts on the system as a whole. A
E2 p2c2 = 0. Therefore m = 0 for box with a hot gas of particles has
the photon. Can you complete this more mass than the same box after the
dialogue? gas has cooled. Also, the box of hot
Einstein, A. Ist die Trgheit eines Krpers gas exerts a greater gravitational pull
von seinem Energieinhalt abhngig? on a test particle. In addition, a box of
Annalen der Physik 18 (1905): 639. Trans-
lated by W. Perrett and G. B. Jeffery, in The photons exerts a gravitational pull on
Principle of Relativity. New York: Dover, a test particle, and vice versa.
1923, p. 71.
Taylor, E. F., and J. A. Wheeler. Spacetime
. Relativity: The Special and General Physics: Introduction to Special Relativity.
Theory. New York: Crown, 1961, p. 47. San Francisco: W. H. Freeman, 1992, p.
135.

96. System of
Particles 97. Light Propagation
No and yes! Except in the special cir- According to the special theory of rel-
cumstance described below, the answer ativity (STR), (1) no object can move
is no. Energy and momentum are addi- at the speed of light, (2) the speed of
tive, but not mass. Mass is a measure light is the same for all observers, and
of the magnitude of the energy- (3) the space-time interval between
momentum 4-vector. From the total two events dened by 2 = c2 t 2 x2
energy E and the total momentum P y2 z2 is the same for all
can be determined the mass M of the observers, but the t and x may be
system: M2c4 = E2 P2c2. Therefore the different, for example.
mass M of the system is greater than For one-dimensional motion 2 =
the sum of the masses of its particles by c2 t2 x2. The driver has x 0, so

Answers 185
her t must be greater for the two One also could use a regular n-gon of
events than the elapsed time for the at mirrors to reect the light around
observer on the ground. Therefore the the Equator and then take the limit as
driver measures the longer time inter- n becomes infinite. The light leaves
val between events A and B. from point P on the Equator of the
Suppose the car makes a second rotating Earth and returns to point P
run at a great speed. The nearer to the in time T. The light going eastward has
speed of light the car goes, the smaller traveled the distance 2R + RT in the
is t for the ground observer, and inertial system, where is the angular
= c t is smaller in this case also. But frequency of rotation with respect to
again, as expected, the time interval is the inertial reference frame. The point
longer for the driver. The nearer to the P has traveled RT. The ratio of point
speed of light the car goes in the speed to light speed is R/c =
ground frame, the difference will be RT/(2R + RT), from which T =
the difference in arrival times as 2R/(c R). For the system at rest,
observed in the two frames. T = 2R/c. Hence, when 0, dene
Taylor, E. F. Light Propagation. The T = T 2R/c as the extra time
Physics Teacher 25 (1987): 252. required. Substitution for T gives T =
2R2/[c (c R)]. Upon returning to
point P on the Equator after one cir-
98. Sagnac Effect cuit, the clocks will differ by 2T for
No, they do not tick at the same rates. the measured elapsed times.
Their tick rates are different because Schlegel, R. Comments on the Hafele-
Earth is rotating with respect to an Keating Experiment. American Journal of
Physics 42 (1974): 183187.
inertial reference frame such as the dis-
tant stars. The clock moving eastward
has a higher velocity with respect to
the inertial frame than the clock mov-
99. Light Flashes
ing westward at all moments. Accord- The observer on planet A sees the
ing to the STR, the higher the velocity, flashes 20 minutes apart. From the
the slower the clock ticks. That is, a STR postulate, we know that no
clock ticks fastest when at rest in an observations of the light ashes only
STR inertial reference frame. can discern which inertial frame is at
The difference in the elapsed time rest. If the flashes sent out by the
for the two clocks can be calculated by spaceship at 10-minute intervals are
considering a light clock following a seen at planet B separated by 5 min-
circular light path around the Equator. utes, then ashes sent out from B at

186 Answers
10-minute intervals will be seen on the To understand why the object
spaceship at 5-minute intervals. slows in the x-direction when the
One also realizes that if there were a applied contact force is in the y-direc-
light ash every 5 minutes from planet tion, we begin with the space-time
A, the observer on planet B would see interval: (interval)2 = c2 t2 x2 y2
them at 5-minute intervals. What is the z2. For real objects traveling at
interval for these ashes from A as seen speeds less than c, the time term is
by the spaceship observer? The answer much larger than the spatial terms,
is every 10 minutes, by invoking the and the interval is called the proper
STR postulate above. So the spaceship time . The linear momentum px in the
sees planet As ashes to be spaced twice x-direction in Newtonian physics is
as much apart as the interval at the dened as px = m dx/dt (for an object
source on A; likewise, the 10-minute that is not changing its mass, i.e.,
ash intervals from the spaceship must excluding objects such as a leaking
be twice the interval at planet A, or 20 bucket of water). The correct STR
minutes apart. expression simply substitutes proper
Hewitt, P. G. Conceptual Physics, 6th ed. time for Newtonian time t so that px
Glenview, Ill.: Scott, Foresman, 1989, pp. = m dx/d. For a low-velocity object,
650656.
d ~ dt. But the actual relationship
between and t depends on the mag-
100. Forces and nitude of the objects total velocity, a
vector quantity, not just the speed
Accelerations component in the x-direction. There-
No. In the STR, all contact forces will fore, as the object speeds up in the y-
produce an acceleration in a direction direction, its speed in the x-direction
not parallel to the applied force! For must decrease to maintain a constant
example, a rigid sphere is moving total velocity magnitude, otherwise its
along the plus x-direction of an iner- x-component of linear momentum
tial reference frame. Now let an would change, forbidden by the law of
applied contact force act in the plus conservation of linear momentum.
y-direction to increase the speed of The pertinent relationship is dt/d =
the sphere in the y-direction. What 1/
(1 v
2/c2) . Remember, the mass is

happens to the speed in the x-direc- xed in value.


tion? The x-component of the speed One could write down the relativis-
decreasesthe object slows down in tic momentum px = mvx// (1 v
2/c2)

its original direction, corresponding to and argue that since m is constant, the
a negative acceleration! component of velocity in the original

Answers 187
direction must decrease to keep the lab frame, lets review the simpler prob-
momentum component constant. lem of how velocities are added in rela-
Ficken, G. W. Jr. A Relativity Paradox: The tivistic frames. If an object moves
Negative Acceleration Component. Amer- forward with the velocity V in the
ican Journal of Physics 44 (1976):
11361137. spaceship frame, then the objects veloc-
Gonzlez-Daz, P. F. Relativistic Negative ity V in the lab frame is determined by
Acceleration Components. American Jour- the law of addition of velocities V/c =
nal of Physics 46 (1978): 932934.
(V /c + Vs /c)/(1 + V Vs /c2), where Vs is
Tolman, R. C. Philosophical Magazine 22
the uniform velocity of the spaceship in
(1911): 458.
the lab frame. One can check the limit-
ing case for low velocities, when
101. Uniform V Vs /c2 is very small, to verify agree-
ment with Galilean relativitythat is,
Acceleration the two velocities simply add.
In the STR, the velocity in the lab To relate the acceleration of the
frame is no longer V = a t for a uni- object as seen by both observers, the
form acceleration a in the moving addition of velocities expression is
frame. However, in the moving frame differentiated with respect to the time
at each instant the expression V = a t in the lab reference frame to obtain
continues to be true. To convert from a = a /{(1 + VVs/c2) (1


Vs2/c
2))3 }, a

the moving frame to the lab frame, we messy expression. The velocity of the
must essentially convert the clock accelerating object in the lab frame is
readings and time interval using dt/d found by substituting V = a . There-
= 1/
(1 v
2/c2) . Here, is the proper fore a a and V < c.
timethat is, the clock reading on a An alternative mathematical tech-
wristwatch worn by an observer on nique using a velocity parameter
the spaceship, say, and d is the proper defined in terms of hyperbolic func-
time interval between two events at tions is given in the Taylor and
the same location. In the example, is Wheeler reference below.
the elapsed time on the wristwatch of Taylor, E. F., and J. A. Wheeler. Spacetime
the person on the moving frame. Physics: Introduction to Special Relativity.
San Francisco: W. H. Freeman, 1966, pp.
Hence, on the moving spaceship 4758.
frame, V = a. Tipler, P. A. Modern Physics. New York:
Before we determine the answer Worth, 1978, p. 27.
for the velocity of the object in the

188 Answers
102. Long Space time d. In the same astronaut time the
velocity parameter of the spaceship
Journey with respect to the lab frame changes
The 7,000 light-year journey with 40- from to + d. But d = a d/c. That
year aging is possible in STR physics is, each time interval d on the astro-
but not in Newtonian physics! nauts wristwatch is accompanied by
Dene V/c = tanh , where tanh is an additional increase d = a d/c in
the hyperbolic tangent. Substitute into the velocity parameter of the space-
the law of addition of velocities to ship. Since the spaceship starts from
obtain tanh = (tanh + tanh s)/(1 + rest, we get = a /c, telling us the
tanh tanh s). Some checking of the velocity parameter of the spaceship
mathematics of hyperbolic functions in the lab frame at any time in the
will reveal that the s are additive, just astronauts frame.
as velocities are additive in Newtonian Our solution is V = c tanh (a/c).
physics with Galilean relativity. That There is no limit to the product a,
is, = + s. Some people call the which can be much greater than c, but
velocity parameter. tanh 1, so the lab velocity V only
approaches c after a long wristwatch
elapsed time. The distance traveled in
the lab frame is d x = tanh (a/c) cdt. In
the lab frame, the astronauts wrist-
co-moving frame co-moving frame watch seems to be ticking slower than
the lab clock, so dt = cosh d, with
Astronaut time Astronaut time + d = a /c. Therefore dx = c sinh (a/c) d,
which can be integrated from zero
Back to the problem at hand: How astronaut time to the nal time T to
much velocity V in the lab frame does produce the distance traveled x = (cosh
the accelerating spaceship have after a (aT/c) 1) c2/a.
given time? We need three frames of The journey would be done by
reference: the lab frame, the spaceship accelerating to the halfway distance at
frame, and an instantaneously comov- 3,500 light-years, then decelerating to
ing inertial frame that for an instant the 7,000-light-year distance. Substi-
has the same velocity as the spaceship. tuting 3,500 light-years in units of
With respect to the instantaneously meters, g as 9.8 m s2, and the speed of
comoving frame, the velocity parame- light, one calculates a journey dura-
ter changes from 0 to d in wristwatch tion for the space travelers wristwatch

Answers 189
of T ~ 8.62 years. The round-trip and, to a rst approximation, the time
would require about 34.5 years. So interval between ticks differs by (r/r)
the space travelers would age less than GM/rc2 T, where r is the altitude dif-
40 years! ference, M is Earths mass, r is the
Are there any plans to make this radial distance from the center of
journey? Assuming human volunteers Earth, G is the gravitational constant, c
are available who want to achieve this is the speed of light, and T is the time
feat, other factors, such as a reliable interval between ticks on the reference
food supply, sufcient health care, and clock. Substituting r = 6.37 106 m
an energy source for the constant 1-g and dr ~ 1.5 m produces a value of 1.6
acceleration for 40 years would be dif- 1016 T, an incredibly small change
cult to provide with present technol- in rate. Over a lifetime of about 80
ogy. And, of course, more than 14,000 years, the head becomes about 0.4
years would have passed for civiliza- microsecond older than the toes.
tion here on Earth. Who or what To understand the effect of gravi-
would be here to greet them on their tation on the clock rate, we can utilize
return? the equivalence between an accelerat-
Taylor, E. F., and J. A. Wheeler. Spacetime ing rocket frame and being in a uni-
Physics: Introduction to Special Relativity. form gravitational eld. Consider two
San Francisco: W. H. Freeman, 1966, pp.
4758.
light ashes sent from the bottom of
the accelerating rocket to its top, as
shown in the animation diagram from
the view of our inertial reference frame
103. Head to Toe with respect to the stars. The two light
Yes, your feet and toes age slower than flashes are one second apart in our
your head. That is, whenever you are frame but arrive at the top of the
standing or sitting, a clock at the alti- rocket three seconds apart. Why?
tude of your head will tick faster than Because the top has moved away
an identical clock at the altitude of from the approaching light ash with
your toes. The ambient gravitational the appropriate acceleration value.
eld affects the tick rate of all clocks in Therefore the frequency of arrival is
the same way. A clock will tick fastest lower than the starting frequency. In a
at rest in an inertial reference frame. stroke of genius, Einstein realized that
The difference between clock rates in the only reason for different ash fre-
different gravitational environments is quencies would be if the clock at the
normally minuscule but measurable top ticked at a different rate than the

190 Answers
photon traveling at light speed, the
clock would not tick. As a photon
traverses the universe, no time elapses
in its reference system. The photon can
be absorbed by an atom and disap-
pear, but the photon cannot change
directly into another photon. Like-
wise, if all three neutrino types did not
have any mass, none could oscillate
into another neutrino type because
they do not experience the passage of
time. Therefore, for neutrino oscilla-
identical clock at the bottom. There-
tions to occur, at least two neutrino
fore, gravitation makes time run slow.
types must have mass. The data indi-
Is there a place where one can put
cate that the sum of the three neutrino
a clock so that the time interval
masses cannot exceed about 1 eV/c2,
between ticks becomes infinite? Yes,
very much smaller than the 0.511
near a black hole, at the event horizon.
MeV/c2 mass of an electron.
Hewitt, P. G. Conceptual Physics, 6th ed.
Glenview, Ill.: Scott, Foresman, 1989, pp.
671678.
105. Spaceship
Taylor, E. F., and J. A. Wheeler. Spacetime
Physics: Introduction to Special Relativity. Collision
San Francisco: W. H. Freeman, 1966, p.
154. The method of determining position
and clock reading for the three events
rst before answering the question is
104. Neutrino Mass a good one. However, the values
For a change in a system to occur inserted already are not all correct for
such as the change of a muon neutrino the observer. Simultaneous measure-
to an electron neutrino, for example ments at both the origin X1 = 0 and at
time must elapse. That is, the reference X2 = L cannot be made by the method
clock must tick in the rest frame of the assumed since they are not equidis-
muon neutrino. We know that the tant. Therefore, if the notation (X, T)
greater the velocity of a real clock in is correctly (0, 0) for event 1, then
our laboratory reference system, the event 2 is labeled by (L, L/c) because
slower is its ticking rate. In the speed the light from event 2 takes L/c sec-
limit of a massless particle such as a onds to travel the distance L to the

Answers 191
observer. Event 3 is not at position L/2 result in less aging than for his twin
between the two spaceships at T = 0 brother, who has remained at home on
because spaceship B has already trav- Earth. Even if the acceleration was
eled for L/c seconds. Therefore the simply an immediate turnaround at
distance between the two spaceships is the farthest distance, the spaceship
L VL/c. Thus T3 = L(1 V/c)/2V. We velocity vector reversed direction from
can summarize the events as: +V to V, a change of 2V, in a time
Event 1: X1 = 0 T1 = 0
interval T. Peter felt the acceleration.
Event 2: X2 = L T2 = L/c
Therefore, all observers will agree that
Event 3: X3 = L(1 V/c)/2 T3 = L(1 V/c)/2V
Peter was the traveler and that his
clocks ran slow, so he ages less than
These same events can be specied
his stay-at-home twin.
in the inertial frame (primed) of space-
Feynman, R. P., R. B. Leighton, and M.
ship A as: Sands. The Feynman Lectures on Physics.
Vol. I. Reading, Mass.: Addison-Wesley,
Event 1: X1 = 0 T1 = 0
1966, pp. 16-3 to 16-4.
Event 2: X2 = L(1 + V/c) T2 = L(1 + V/c)/c
Event 3: X3 = 0 T3 = 1 L(1 V/c)/2V

We have defined =
(1 V 2/c 2

and have used the normal Lorentz


Chapter 6
transformations x = (x Vt) and t = Start Me Up
(t Vx/c2) of the STR.
Now, nally, we can determine the
clock readingthat is, the elapsed 107. Air-Driven
timefor the observer who sees the Automobile Engine
collision a distance L(1 V/c)/2 away
Yes. Many companies worldwide have
as T = L(1 V/c)/2V + L(1 V/c)/2,
been operating compressed-air-driven
which reduces to T = L(1 V2/c2)/2V.
cars using a standard gasoline four-
The observer on spaceship A has an
cylinder engine but replacing the gaso-
elapsed time of 1 L(1 V/c)/2V.
line fuel input with compressed air
Chai, A.-T. Some Pitfalls in Special Relativ-
ity. American Journal of Physics 41
from a tank. Of course, there is no
(1973): 192195. combustion, so the electrical supply
for the spark plugs is not needed, nor
will there be any need to change the oil
106. Twin Paradox very often. The compressed-air tank is
Peter experiences actual accelerations stored in the trunk.
during his spaceship journey that will The piston upstroke compresses

192 Answers
and heats the atmospheric air in the the characteristics of randomness do
cylinder chamber until just about top not contain long runs, you should be
dead center, when cool compressed air able to distinguish them reliably.
is injected to drive the piston down The actual estimate of the number
and turn the crankshaft. The process of runs with 6 or more heads or tails is
repeats itself until the compressed air 4, meaning that you should be able to
is depleted. The exhaust is just cool air. nd about 4 of these long runs. For a
The horsepower rating is about 35 run of at least k heads in n tosses,
horsepower for some models, but the where k 1, the mean number of runs
value will increase to more practical is ~ n/2(k+1); thus 2 (256/27) = 4. The
values with further development. following table contains actual data
Using traditional electricity sources to for 256 coin tosses, with a 1 repre-
compress the air, there will be some senting heads. You can count the num-
carbon dioxide air pollution for the bers of the different run lengths.
overall process, but only about a fth
or less that of conventional autos. 10111110101101101011010010001100
Perhaps the best-known air-pow- 11011110100001001001110010100100
ered car is that designed by French 11001100111110001000001011111000
inventor and engineer Guy Ngre for 10110010001111100110111001110010
Motor Development International 11111000011011100000001011111000
(MDI) in France. The car has a maxi- 11110110110000001010000010111110
mum speed of about 110 kilometers 11111100111011001011100010111110
per hour and can travel about 300 01110110111100001111111000001100
kilometers at a cost of less than a cent
per kilometer. (Details can be found on Silverman, M. P. A Universe of Atoms, an
Atom in the Universe. New York: Springer-
the Internet.) Verlag, 2002, pp. 284291.

108. Coin Tosses


109. More Coin Tosses
You should be able to pick out the
experimentally obtained sequences Most people would expect to return to
with about 98 percent accuracy! In a the lamppost quite often20 or more
random sequence of 256 fair coin times during the 1,000 tosses. How-
tosses, you would expect to nd at least ever, returning more than 2 times is
1 run of 6 heads or 6 tails with a prob- unlikely! There will be a long drift
ability of 98.2 percent. If the sequences away from the lamppost for most of
imagined by students unfamiliar with the coin-tossing time.

Answers 193
40 if the switching time t is large enough
30 to assure that the particle can adjust in
20 the trapping minimum (adiabatic
10 adjustment time) and also is small
0
0 200 400 600 800 1000 1200
enough to fulfill the above require-
-10 ment for the variance. Roughly, one
-20 can say that a net flux to the left
-30 always occurs when thermal energy is
-40 signicantly smaller than the potential
Toss Number
maximum, the external force chosen is
One can do the actual coin tossing not too big, and the driving frequency
to experience the drift away from the matches the adiabatic adjustment time
origin for long time periods, or one needed for the particle to move into a
can run a computer simulation. The potential minimum.
expected distance after N tosses will Where does the energy come from
be N times the unit step distance, leading to a drift against the external
the random walk distance in one force? The energy does not come from
dimension. the heat bath but from the ratchet
potential when it is switched on. At
that moment the potential energy of
110. Brownian Motor the particle will suddenly be increased.
As long as the ratchet potential is off, In a simulation, this can be seen by a
there can be no net movement to the sudden increase of the energy. But
right or the left because the particles most of the energy pushed into the sys-
will move diffusively according to a tem will just be dissipated into the heat
(biased) random walk, leading to a bath due to the relaxation of the parti-
variance in position of x = (2D)
and cle into a potential minimum. Only a
a mean position of <x> = f /, where tiny portion will be used for doing
D = kT/ is the diffusion constant. work. Thus a Brownian motor does
When the ratchet potential is switched not violate any law of thermodynam-
on, one or more particles get trapped ics because it only turns one type of
in one of the potential minima. If L work into another one. Nevertheless,
x (1 )L for the variance holds, the uctuating force due to the heat
the particle on average gets trapped bath is essential for a Brownian motor.
into the minimum left of the starting For more details and possible
point. The maximum ux is obtained applications in biology and chemistry

194 Answers
read the following review articles. For which probably would be costly. The
a simulation, there are Java applets on alternative would be to have a small
the Internet. closed loop of ferrofluid in contact
Astumian, R. D. Thermodynamics and with a large loop of piping containing
Kinetics of a Brownian Motor. Science 276 the water to be heated in a heat
(1997): 917 922.
exchange device. The advantage over
. Making Molecules into Motors.
Scientific American 285, no. 1 (2001): typical systems would be no moving
5764. mechanical parts in the solar heating
system.
111. Magnetocaloric Rosensweig, R. E. Magnetic Fluids.
Scientific American 247, no. 4 (1982):
Engine 136145.

The ferrofluid is cycled around the


loop by the stationary permanent
magnet. A small volume of ferromag- 112. Magnetorheological
netic material has less energy where Fluid
the magnetic field density is greater,
The ow properties of the uid change
just like iron lings are pulled to the
so radically that the uid becomes gel-
poles of a magnet. So the ferrofluid
like and can be pushed to one side of
approaching the magnet becomes
the beaker where no relaxation may
magnetized and drawn into the loop
occur. The degree of solidification
volume between the magnetic poles.
depends on the inherent properties of
But the heat source nearby warms the
the uid and the strength of the mag-
ferrofluid to partially randomize the
netic eld. Of course, its solidication
magnetic dipoles in the ferrouid, so
may vary within the gel itself because
the energy of the system can be low-
the magnetic eld may vary with posi-
ered again by drawing in some more of
tion in the beaker. Practical applica-
the cooler magnetized ferrofluid,
tions of these materials with their
which pushes out the warmed fer-
unusual properties are being devised
rofluid. The heat put in by the heat
and tested. Perhaps automobile brak-
source is deposited at the heat sink,
ing systems may someday use these
and the cycle repeats.
types of uids to replace solid materi-
To use this engine for the solar
als that wear away.
heating of buildings, two avenues of
Klingenberg, D. J. Making Fluids into
operation are possible. One could Solids with Magnets. Scientic American
have all the piping contain ferrouid, 269, no. 4 (1993): 112113.

Answers 195
113. Binary Fluids low T, changing the entropy has a min-
imal effect because the product TS
Both phase diagrams can represent may be small. But at high T, the prod-
actual binary uids, although the dia- uct can be large. So systems at high T
gram to the right is quite rare. To tend to maximize their entropy, that
understand these phase diagrams, both is, their randomness or disorder.
energy and entropy must be consid- We now have a good argument for
ered. The energy part involves the van ruling out the diagram to the right,
der Waals interaction between adjacent with its reappearing miscible phase at
molecules, an induced dipole-dipole low temperatures. Not so! For some
electromagnetic interaction. In general, molecules, hydrogen bonding occurs
this attractive force between unlike with its very small angular spread,
molecules is much weaker than the locking two molecules together. This
attractive force between like molecules. hydrogen bonding occurs primarily
The stronger the force of attraction at lower temperatures because of
holding the molecules together, the the orientation dependence, with
lower the energy of the system. Hence, orientation entropy lost in forming
when most of the molecules neighbors the hydrogen bond being greater than
are of the same chemical species, the the compositional entropy gained.
system energy is lowest and immiscibil- Therefore both energy and entropy are
ity is favored. Even the increased ran- lowered, and the lowered energy of the
dom tumbling about of the molecules hydrogen bond has a large effect on
at higher temperatures doesnt disrupt the free energy. Water and butyl alco-
this clustering of like molecules. hol is one example of a binary liquid
However, energy considerations with the rare phase diagram.
alone do not explain the behavior of Walker, J. S., and C. A. Vause. Lattice
binary liquids. Why are they miscible Theory of Binary Fluid Mixtures: Phase
at all? The miscibility occurs at lower Diagrams with Upper and Lower Critical
Solution Points from a Renormalization-
temperatures because the system tends Group Calculation. Journal of Chemical
to minimize not its energy but rather Physics 15 (1983): 26602676.
its free energy, Efree = Esys TS. The . Reappearing Phases. Scientific
free energy is the energy of the system American 256, no. 5 (1987): 98106.
minus the product of the temperature
T and the systems entropy S. At a
given T, the free energy can be 114. Baseball Bats
decreased by decreasing the systems The main source of drag on the swing
energy or by increasing its entropy. At of a baseball bat is not air friction but

196 Answers
the retarding force produced by the 115. Old Glass
pressure difference across the bat from
front to back. As the bat carves its Many people have suggested that the
swath, the air in front gets separated glass experiences some ow downward
into two boundary layers that pass in response to the gravitational pull of
around the bat and recombine behind Earth. Contrary to popular conjecture,
the bat. In the wake of the bat, there is no evidence that any of this old
between the two separated boundary glass could flow enough during the
layers, the lack of air means a lower time interval of centuries to create the
pressure immediately in back of the difference from top to bottom.
bat, with a resulting backward force Another factor against the flow
due to the pressure difference. There- hypothesis is the actual prole, which
fore, some of the energy of the swing is essentially a linear relationship of
does work against this backward thickness to vertical distance. As a
force. simple model, assume that the proper-
ties of the glass are identical at each
vertical position along the pane. If a
Boundary layer Direction xed amount of glass material ows
of bat from position 10, say, the same
amount would replace this amount
from position 11, slightly higher up
Bat the glass. The major changes over a
long time interval would be a thick
Airflow
buildup at the very bottom and a
Low-pressure High-pressure
depletion at the very top, with practi-
region region
cally no thickness change between, in
contrast to the linear dependence of
A dimpled bat sends the boundary glass thickness to height.
layers tumbling in turbulent eddies In the old days, window glass pro-
into the space behind the bat, reducing duction made panes that varied
the pressure difference and cutting the slightly in thickness from one end to
drag. More swing energy is now avail- the other because the at support sur-
able to accelerate the bat and to trans- face had a slight tilt. The installers
fer to the ball, so the balls exit velocity simply put the thicker end on the bot-
will be increased. tom. Quality control must have been
Gibbs, W. W. To Fenway, with Love. marginal in some areas of the world,
Scientic American 271, no. 1 (1994): 98. because we have seen some large

Answers 197
differences in glass thickness between substances the dipoles are far enough
the two ends! apart to behave approximately inde-
Glass is normally elastic at temper- pendently, and when no applied eld is
atures below about 1000 K, and glass present, these dipoles have random
may break but never deform perma- orientations. Each dipole is affected by
nently because the solid is crystalline. the applied magnetic eld but not by
Delicate telescope and camera lenses its neighbors. The applied magnetic
would reveal such creep by changing eld competes with the random ther-
their optical characteristics in obvious mal motion to cause a net magnetiza-
ways. tion that increases nearly linearly with
Pasachoff, J. M. Comment on Magnetic the strength of the applied eld, the
Fluids. American Journal of Physics 66 ratio being known as the magnetic
(1998): 1021.
susceptibility.
Zanotto, E. D. Do Cathedral Glasses
Flow? American Journal of Physics 66 When the density of magnetic
(1998): 392395. dipoles becomes high enough for
neighbors to affect each other, only
neighbors in the head-to-tail congu-
116. Ferromagnetism ration will tend to align one another.
The side-by-side neighbors will be
oppositely aligned because all the
elds from its neighbors are opposite
at its location. Thus every other dipole
in each layer of the crystal will be
aligned and form one sublattice, like
the white squares on a checkerboard,
and the remaining dipoles (on the
black squares) will form a second sub-
lattice of dipoles pointing in the oppo-
Many atoms and molecules have an site direction. The two sublattices
inherent magnetic dipole moment. interact strongly or ferromagnetically,
When we assume that each dipole but they cancel each others magneti-
behaves independently of its neighbors zation. Therefore, when magnetic
except for its alignment, the magnetic dipole moments are crowded together,
field direction next to the dipole is they are more likely to disalign their
opposite to the direction in which the nearest neighbors than to align them.
dipole itself points. In paramagnetic So ferromagnetism is rare.

198 Answers
Then how can ferromagnetic sub- that accounts for the change in angular
stances exist at all? Via a cooperative momentum.
effect when the dipoles are very close If the pulleys are the same size, this
and no longer behave independently. additional torque does not exist unless
In these conditions a state of lower the belts are crossed.
energy can form if groups of dipoles
align each other into magnetic
domains that themselves point in ran-
118. Superconductor
dom directions. With an applied eld, Suspension
these domains will change their sizes The demonstrated superconductor
to find the lowest energy state. Of suspension does not illustrate the
course, domain formation cannot Meissner effect. Instead, this demon-
form above a certain temperature stration depends on the persistent
called the Curie temperature, because eddy current in the zero resistivity
the thermal agitation interferes with superconducting material induced by
the dipole interactions. Above the the magnet. The eddy current direc-
Curie temperature the substance tion is determined by Lenzs law to
becomes paramagnetic. produce a magnetic eld that ends up
Kolm, H. H. Why Are So Few Substances causing a repulsion between the super-
Ferromagnetic? The Physics Teacher 20
(1982): 183185. conductor and the permanent magnet.
To show the Meissner effect, the
sequence of events must be different.
117. Coupled Flywheels Place the superconductor on the mag-
net at room temperature rst, and then
The overall angular momentum of the cool the superconductor below its crit-
system must be conserved, so including ical temperature Tc. Then the magnetic
just the change in angular momenta of ux will be expelled by the Meiss-
the ywheels leads to an incomplete cal- ner effect and the superconductor will
culation. The tension is different in the become suspended above the magnet.
two sides of the belt, so the belt exerts a Wake, M. Floating Magnet Demonstration.
downward force on pulley 2 and an The Physics Teacher 28 (1990): 395397.
upward force on pulley 1. These forces
are counteracted by reactions at the
bearings, in addition to the reactions to
119. Nanophase Copper
the weight of the components. These With smaller grain sizes, one would
additional reactions produce a torque expect there to be many more grain

Answers 199
boundaries in the nanophase copper fraction of the charge of a single elec-
metal than in normal copper. The tron. This transferred charge is pro-
extra grain boundaries would stop or portional to the sum of the shifts of all
impede any moving dislocation, the electrons with respect to the lattice
thereby making the nanophase copper of nuclei. These electrons in the con-
much harder. However, the surprise ductor can be shifted as little or as
turned out to be that nanophase cop- much as desired, so the sum can
per is mostly dislocation-free! Lacking change continuously, and therefore so
large numbers of moving dislocations, can the transferred charge. The pin-
these nanophase metals are much head can have any charge value, not
stronger. just integer multiples of the fundamen-
Siegel, R. W. Creating Nanophase Materi- tal charge.
als. Scientic American 275, no. 6 (1996): Likharev, K. K., and T. Claeson. Single Elec-
7479. tronics. Scientific American 266, no. 6
(1992): 8085.

120. Head of a Pin


Experiments show surprising results.
121. Coulomb Blockade
Any fraction of the fundamental No, the current across the junction
charge, such as +0.5 e or 0.1 e, can will not be a steady current. There will
exist on the head of a pin! The con- be single electron tunneling (SET),
ceptual argument goes as follows. The with the voltage across the junction
metal pin is an electrical conductor. In changing periodically with a frequency
general, an electric current ows in the equal to the current divided by the
conductor because some free electrons fundamental unit of charge e.
move through the lattice of atomic The tunnel junction is a conductor-
nuclei. Any particular volume of the insulator-conductor device, so trans-
conductor has virtually no charge ferred charge flows through the
because the negative charges are bal- conductor to accumulate on the sur-
anced by the positive charges of the face of the electrode against the insu-
nuclei. lating layer of the junction. An
So the important physical quantity opposite surface charge of equal
is not the electric charge in any given amount accumulates on the other elec-
volume but instead how much charge trode across the junction. The actual
has been carried through the conduc- amount of surface charge has a con-
torthat is, the transferred charge, tinuous change in value as the charge
which can have any value, even a accumulates, including fractional

200 Answers
values such as +0.8642 e, because the electrode is changed by only half the
electrons near this surface can adjust charge of an electron!
their positions slightly. Likharev, K. K., and T. Claeson. Single Elec-
However, only discrete amounts of tronics. Scientific American 266, no. 6
(1992): 8085.
charge can tunnel through the insulat-
ing layerthat is, each electron tun-
neling through changes the surface 122. Deterministic
charge by +e or e, depending on the Competition
direction of tunneling. The tunneling
process is energy-dependent. If the The time evolution here depends on
charge at the junction is greater than the value of r. One nds that Nt = 1 is
+e/2, an electron can tunnel through to a stable equilibrium only when r lies
reduce the surface charge by e, thus between 0 and 2. If r = 2.3 with N0 =
reducing the electrostatic energy of the 0.5, then successive Nt will oscillate
system. And if the surface charge is between about 1.59 and about 0.40 as
less than e/2, an electron can tunnel a stable 2 cycle. For r > 3.102, no cycle
in the opposite direction to decrease is stable, all cycles are possible, etc.
the energy. But if the surface charge In the chaotic regimes, the equa-
value is greater than e/2 or less than tion results are deterministic, but the
+e/2, tunneling would not occur time evolution is indistinguishable
because the system energy would from that governed by probability
increase. This tunneling suppression is laws. One really needs to see the cal-
known as the Coulomb blockade, rst culations proceed to appreciate the
studied in the 1950s. amazing behavior of this simple-look-
The tunnel junction connected to a ing equation.
constant current source begins in the Gleick, J. Chaos: Making a New Science.
New York: Penguin, 1987, pp. 166186.
Coulomb blockade condition, then
reaches tunneling for the one-electron
condition, then back to the Coulomb 123. Two Identical
blockade, then one-electron tunneling,
etc. The analogue may be a dripping
Chaotic Systems
faucet. Yes, the two identical chaotic systems
Many electronic devices are being described can be synchronized.
made with SET operation. For exam- Chaotic systems are very useful for sev-
ple, an SET transistor can switch on or eral reasons: (1) Chaotic systems are a
off the ow of billions of electrons per collection of many regular, ordinary
second when the charge on the middle behaviors, none of which dominate.

Answers 201
(2) The proper perturbation can the rate of change of magnetic ux .
encourage the chaotic system to follow But work must be done for V to be
one of its many regular behaviors. (3) generated because the change in the
Chaotic systems are very flexible work dWork = V dt.
because they can rapidly switch among Nussbaum, A. Faradays Law Paradoxes.
different behaviors. (4) Chaotic sys- Physics Education 7 (May 1972): 231232.
tems are deterministic and, although
no one can say which output will 125. Thermal Energy
result, two identical chaotic systems of
the appropriate type will produce the
Flow
same output in response to the same The classical ow of thermal energy
signal input. toward the cooler region occurs
To synchronize two identical because the free energy of the com-
chaotic systems each with the stable bined system Efree = Esys TS becomes
subpart behavior, one can apply the less, where T is the temperature and S
appropriate pseudoperiodic signal is the entropy. If the free energy is the
(one type is called a Rssler signal) to same at two temperatures, one can see
coax them into step. For the reasons that for a given amount of system
listed above, the outputs will be the energy there is more disorder at the
same. The details can be learned in the lower temperature. Assuming that the
reference below, where the chaotic two-block system initially simply
attractor and the Poincar section are transfers thermal energy from the
discussed. Applications to secure com- warmer to the cooler block, with no
munications and to biological systems other energy transfers, then a cooler
are included also. system is preferred.
Ditto, W. L., and L. M. Pecora. Mastering Dyson, F. J. What Is Heat? Scientific
Chaos. Scientific American 269, no. 2 American 191, no. 3 (1954): 5864.
(1993): 7884.

126. Cadmium Selenide


124. Tilleys Circuit The wavelength of visible light is com-
The galvanometer does nothing! There parable to nanophase cluster sizes. For
is no induced potential because no example, greenish light has a wave-
work was done (assuming frictionless length of about 580 nanometers, ve
switches). This result appears to vio- to ten times the nanophase cluster
late Faradays law V = d /dt, where V sizes. Clusters behaving as particles
is the potential difference induced by ranging from about 1 nanometer to 50

202 Answers
nanometers in diameter are too small These pulses conserving their shape
to have any signicant scattering of vis- and integrity are exhibiting soliton
ible light, so these materials are effec- behavior. Optical solitons were first
tively transparent. Clusters of sizes observed in bers in 1980 and are now
comparable to particular wavelength fundamental components in optical
ranges of visible light are subject to transmission systems.
quantum connement restrictions. Desurvire, E. The Golden Age of Optical
Quantum mechanics predicts the Fiber Ampliers. Physics Today 47 (1994):
2027.
correct behavior at the small cluster
sizes. The smaller the nanophase clus-
ter size becomes, the greater are the 128. Ceramic Light
energy spacings for the electron states. Response
Which colors of light are absorbed and
emitted are determined by these energy Certain ceramic materials will change
spacings. If the energy spacings are too their shape upon exposure to light
great, the incoming light will not be because some molecules in the mate-
absorbed, and light of that wavelength rial have changed their shape upon
and longer will not be scattered. For absorption of particular frequencies of
example, a typical semiconductor is light. If the responses of many mole-
cadmium selenide. When the size of the cules are coordinated, the overall
cluster is 1.5 nanometers, the cadmium effect can be a macroscopic shape
selenide appears yellow, but when the change. Called the photostrictive
size is 4 nanometers, it will appear red. effect, research began in the 1990s,
And larger clusters appear black. and some practical devices are begin-
Therefore, the observed color of the ning to be developed, such as direct
clusters in the nanophase depends on conversion of light to mechanical dis-
their actual sizes. placement for speakers instead of con-
version to an electrical signal rst. A
telephone speaker could be one of the
127. Optical Solitons rst products.
Under the right conditions, the two These ceramics are examples of a
effectsdispersion and the Kerr effect new type of smart material. The
can be made to cancel exactly. The four most widely used classes of smart
nonlinearity of the Kerr effect can materials are piezoelectrics, electro-
delay the fast carriers relative to strictors, magnetostrictors, and shape-
the slow carriers, bringing them memory alloys. The resulting changes
together to counter the dispersion. in the shapes of these materials are

Answers 203
large enough to make them useful as more of them there are. But the typical
actuators. A sensor receives a stimulus human reaction time for such balanc-
and responds with a signal; an actua- ing acts is about 100 milliseconds, so
tor produces a useful motion or most of the wobbles are faster than
action. By denition, smart materials humans can react. Mathematical mod-
are both sensors and actuators, eling of human balancing acts match
because they perform both functions. the measured uctuations only when
Photostrictive materials such as the person or object is on the verge of
PLZTa combination of lead, lan- falling. Then the random uctuations
thanum, zirconium, and titanium cancel each other out and the object
someday may be used to control remains upright.
robots and machines. Engineers at Related research has found that
Pennsylvania State University, for elderly people and others with balance
example, are exploring applications problems showed signs of better bal-
for devices that move when light ance when they stood on a pair of bat-
shines on them and have created a tery-operated, randomly vibrating
two-legged stand that walks very insoles. The idea is that these vibra-
slowly when illuminated. tions amplify balance-related signals
Dogan, A., et al. Photostriction of SolGel from the feet to the brain and vice
Processed PLZT Ceramics. Journal of versa that may have become reduced
Electroceramics. 1, no. 1 (1997): 105111.
by age or illness. When people walk,
Newnham, R. E., and A. Amin. Smart Sys-
tems: Microphones, Fish Farming, and then turn or reach, they are most vul-
Beyond. ChemTech 29, no. 12 (1999): nerable to a fall. When a person leans
3846.
or sways to one side, the pressure on
the sole of that side increases, and the
nervous system senses the change in
129. Random pressure and sends a message to the
brain so that the posture can be
Movements
adjusted. In many people, those mes-
Wobbles in any system can be fol- sages can be altered by age, stroke, or
lowed with fast cameras. For most conditions such as diabetes. Further
human actions, from balancing a stick testing is under way to optimize these
vertically on a nger to balancing on a helpful insoles.
tightrope, wobbles occur that last
Cabrera, J. L., and J. G. Milton. On-Off
from seconds to tens of milliseconds. Intermittency in a Human Balancing Task.
Usually the shorter the uctuation, the Physical Review Letters 89 (2002): 158702.

204 Answers
Chow, C. C., and J. J. Collins. Pinned Then the Carnot engine efficiency
Polymer Model of Posture Control.
Physical Review E 52 (1995): 907912.
= (Qin Qout)/Qin.
Priplata, A. A., et al. Vibrating Insoles and
If the bath atoms are assumed to be
Balance Control in Elderly People. Lancet two-state systems that absorb and emit
362, no. 9390 (2003): 11231124. radiation at the same photon
frequency, then we need the thermo-
130. Gravitational Twins dynamic properties of a photon gas in
order to determine the theoretical ef-
The traveling twin actually returns ciency of this photon engine. Assuming
much younger than her stay-at-home thermal equilibrium for the photon
sister. The argument given was cor- gas, the average number of photons n2
rectly stated but incomplete. The local with energy coming in from the heat
gravitational tidal effects are not the bath at temperature T2 is given by n2 =
same for the twinsthat is, the rate of 1/(exp[/kT2] 1), while the average
change of gravitational potential expe- number of photons n1 leaving at tem-
rienced was different. These tidal perature T1 is n1 = 1/(exp[/kT1] 1).
effects contribute to the clock rates Since Qin n2 and Qout n1 , the
and, when included in the calcula- efficiency of the quantum Carnot
tions, contribute enough to change the engine is = 1 T1/T2, exactly the
result so that the stay-at-home twin same as for the classical Carnot engine.
ages faster and is older upon return of When there is only one heat bath, with
her sister. For a calculation, see the ref- T1 = T2, no work can be done.
erence below. A different quantum engine occurs
Bradley, M., and J. Higbie. Physics Teacher when the bath atoms have three states
22, no. 1 (1984) 3435.
instead of two, bringing in quantum
behavior called quantum coherence,
with a nonvanishing phase difference
131. Photon Engine between the two lowest atomic states
We can analyze the operation of the induced by a microwave eld. One can
quantum Carnot engine in the same eliminate the photon absorption
manner in which we would analyze a process (analogous to laser operation
classical Carnot engine. Let Qin be the without a population inversion). The
energy absorbed from the bath atoms temperature T2 becomes altered to a
during the isothermal expansion and different effective temperature, T. The
Qout be the energy given to the heat sink efciency = (T T1)/T1 can exceed
during the isothermal compression. the efciency of the classical Carnot

Answers 205
engine. This quantum engine can
Original Copy
extract work from a single heat bath,
even when T1 = T2! For the details of
the three-state quantum engines oper-
ation, see the reference below.
Scully, M. O., et al. Extracting Work from a
Single Heat Bath via Vanishing Quantum
Coherence. Science 299 (2003): 862864.

400 600
Wavelength (nm)
Chapter 7 Different atoms absorb and emit
A Whole New World their unique characteristic frequencies
of light in the visible and the ultravio-
let. The types of atoms present and the
132. Grain of Sand intensity of the characteristic spectrum
If one assumes that the grain of sand from each atom type will create a
has a diameter that is a reasonable spectral ngerprint for each artist.
fraction of 1 millimeter, then the line As you know, some artists simply laid
of atoms would be about 1010 meters out the design of the painting, for
long, about thirty times the distance to example, and lesser painters lled in
the Moon! the regions, with the master artist
completing the final touches. Even
these paintings have their own nger-
133. Forensics print of spectral colors.
Until the mass production of paints With a tunable laser capable of
became available in the late 1800s and scanning from the infrared frequencies
early 1900s, each paint used by an to the ultraviolet frequencies, the
artist is known to contain atoms in spectral ngerprint of any region of
particular characteristic amounts, the painting can be recorded and
depending on the source. Paints were compared to other paintings by the
originally made from natural materi- same artist or even other artists,
als, so when an artist mixed his or her including fraudulent painters. This
paints, there was usually a unique laser approach is normally combined
mixture of atoms and molecules for with other approaches to achieve the
each color and color combination. comprehensive evaluation.

206 Answers
The laser technique also permits possible energies for the two photons
the identication and removal of envi- emitted in this quadrupole emission
ronmental coatings on top of the paint process. A very small fraction of these
beneath, such as dust and grime, and two-photon emissions will spit out
ensures that no harm to the painting two photons of the same energy, go off
occurs. Famous paintings such as in opposite directions, and produce no
Rembrandts 1642 De Nachtwacht recoil of the atom. The two-photon
(The Nightwatch) in Amsterdams emission from hydrogen was the rst
Rijksmuseum have had the soot and atom to be measured and the rst to be
grime safely cleaned off to reveal a calculated by quantum electrodynamics
marvelously brighter background of (QED) in the 1940s. Two-photon
faces when compared to the somewhat emissions after laser excitations have
obscure dull background that had become commonplace for many uses in
existed for centuries. todays optics research.
Likewise, simultaneous two-photon
absorption is possible. A container of
134. Doppler
single atoms is placed between two
Elimination? counterpropagating laser sources,
Yes. First consider the emission shining two identical frequency laser
process. Normally, a typical electric beams on an atom so that energy and
dipole emission occurs with a single angular momentum will be conserved
photon exiting the atom as a result of and recoilless absorption can occur.
an allowed transition within the atom First achieved in the 1970s, the precise
that conserves energy and angular energy-spacing values within atoms
momentumthat is, the angular have been determined. Today, two-
momentum of the atom changes by 1 photon absorption with nonidentical
unit of Plancks constant h/2. The energies plays a critical role in the
probability for all other emission upconversion of laser light to higher
processes is lower by a factor of 1/137, frequencies to achieve coherent beams
or by a higher power of this factor. in the UV and for providing light
A two-photon electric quadrupole sources of precise frequencies.
emission process is possible between At the nuclear level, recoilless
two atomic states with angular gamma-ray emission and absorption
momentum quantum numbers differ- are possible if the whole crystal recoils
ing by zero or two units of h/2. There simultaneously with the photon emis-
is a broad continuous spectrum of sion or absorption. This Mssbauer

Answers 207
Effect transition, discovered in the greater than the wavelength of the inci-
1950s, relies on the inability in princi- dent light but smaller in size than the
ple of identifying the single nucleus diameter of the incident light beam. Let
involved and includes an exponential the light source be a parallel beam of
factor proportional to the negative light rays all of the same frequency,
ratio of the temperature of the crystal such as in a laser beam focused to the
to its Debye temperature. point f by a symmetrical lens. The
As an interesting historical note, object tends to focus the light rays
Albert Einstein in 1917 was among the somewhat, changing the direction of
rst to recognize that classical electro- the light rays. The sideways recoil of
magnetism cannot explain sponta- the object occurs to simply conserve the
neous emission of light from atoms. In linear momentum. If the light beam has
particular, he inferred that an atom an intensity gradient, brighter in the
must recoil upon spontaneous emis- center than near the edge, the object
sion, in conflict with the symmetric- will receive a net push back toward the
field distributions produced by optical axis in the center. There must
electromagnetic theory based on also be a recoil of the object in the
Maxwells equations. According to direction of the original light beam,
Einstein, . . . outgoing radiation in the which usually is taken up by the appa-
form of spherical waves does not exist ratus and Earth because the object is on
. . . for if an atom radiated a classical a horizontal platform. A one-celled
spherical wave it could not recoil. paramecium remains well trapped in a
Einstein, A. Zur Quantentheorie der microscope via this light tweezer tech-
Strahlung. Physika Zeitschrift 18 (1917): nique, begun at Bell Labs in the 1970s.
121128.
When the object is smaller than the
wavelength of the incident light, a
135. Light Tweezer more detailed analysis is required to
Yes. A focused laser beam can exert a understand the 3-D trapping and the
trapping force perpendicular to the quantum interference effects.
beam direction of 2 1012 Newtons Optical tweezers have been widely
or more to keep cells confined in a used for several decades in applications
microscope at the optical axis. The as diverse as experiments on molecular
intensity gradient across the light motors in biology and the movement of
beam is the source of the force. Bose-Einstein condensates in physics.
In the simplest geometry, consider a The capabilities of single optical tweez-
semitransparent object with a diameter ers have been greatly improved and

208 Answers
extended by the development of This powder absorbs the UV light and
tailored beams and by schemes for gen- fluoresces in the visible. Very little
erating large numbers of trapping sites heating of the uorescent lamp occurs,
and shapes simultaneously. so the efciency occurs before the pro-
Block, S. M. Making Light Work with Opti- duction of the visible light, with very
cal Tweezers. Nature 360 (1992): little electrical energy being converted
493495.
into thermal energy. The conversion
Chu, S. Light Trapping of Neutral Parti-
cles. Scientific American 266, no. 2 process in the powder makes the tube
(1992): 7076. useful for room lighting.
MacDonald, M. P., et al. Creation and So why is the incandescent lamp so
Manipulation of Three-Dimensional Opti-
inefficient, converting only about 4
cally Trapped Structures. Science 296
(2002): 11011103. percent to 12 percent of the electrical
Ulanowski, Z., and I. K. Ludlow. Compact energy to visible light? The incandes-
Optical Trapping Using a Diode Laser. cent lamp is simply a resistor whose
Measurement Science and Technology 11
(2000): 17781785.
lament temperature rises until it gets
rid of thermal energy at the same rate
that thermal energy is being generated
136. Fluorescent Lights in the filament. In a standard 100-
Today, articial illumination requires watt, 120-volt bulb, the lament tem-
more than 25 percent of the electricity perature is roughly 2550C, about
generated worldwide. There are two 4600F, so that the thermal radiation
trends in energy saving technolo- from the lament includes a signicant
gies. The rst trend is using improved amount of visible light.
lamps, such as fluorescent, mercury, The output is 17.5 lumens per
sodium, metal halide, and halogen watt, compared to a maximum of 240
lamps. The second trend is improving lumens per watt if all the energy could
the electronic circuit design for such be converted to visible light. The rea-
lamps. son for this poor efciency is the fact
Although fluorescent lights are that tungsten laments radiate mostly
four to six times more efcient than infrared radiation at any temperature
incandescent lamps, there now exist that they can withstand. An ideal ther-
many other types of light sources that mal radiator produces visible light
are even more efcient. For the uo- most efficiently at temperatures of
rescent lamp, its efcient production about 6300C (about 6600 K or
of the UV is extended into the visible 11,500F). Even at this high tempera-
by a powder coating inside the tube. ture, a lot of the radiation is either

Answers 209
infrared or ultraviolet, and the theo- vitamin D production in the skin can
retical luminous efficiency is 95 lead to rickets and other problems
lumens per watt. associated with the calcium and inor-
Most fluorescent lights predomi- ganic phosphate metabolism. Eskimos
nantly emit light in the visible part of and other indigenous peoples obtain
the spectrum and they do emit some plenty of vitamin D from the sh oils
UV light, but only in a narrow range in their diets.
of the UV spectrum. Unfortunately, Porter, J. P., ed. How Things Work in Your
their UV emission range does not over- Home (and What to Do When They Dont).
New York: Henry Holt, 1985, p. 158.
lap the two small ranges of UV light
needed by humans for the best func-
tioning of certain internal organs,
which receive some of the UV light 137. Phase Conjugation
that passes through the skin, as well as Mirror
vitamin D production from 7-dehy-
Yes, the light can return undisturbed if
drocholesterol in the skin.
the light wave retraces it original path
as its time-reversed twin and the
Intensity medium retains its previous integrity.
The phase conjugate of a wave pos-
sesses exactly the same spatial proper-
ties as the original wave, but it is said
to be reversed in time. This means that
a phase conjugate wave exactly
Sunlight retraces the path of the original beam.
Standard Fluorescence This method has the useful property
Special Fluorescence
that if a light beam propagates
400 Wavelength (nm) 700
through a distorting medium, then the
phase conjugate is produced and
Special fluorescent lights more exactly retraces the path through the
benevolent to human needs are avail- distorting medium, enabling the unfa-
able and mimic sunlight to produce a vorable effects of the distorting media
UV spectrum better matching the to be reduced or eliminated. Phase
needs of these internal organs. Indeed, conjugation is the general term for a
the lack of the required UV parts of process in which both the direction of
the ambient light spectrum can lead to propagation and the overall phase fac-
certain illnesses. Of course, the lack of tor of a wave function are reversed.

210 Answers
Phase conjugate mirror states that twice the kinetic energy
plus the potential energy add to
zero, so mv2 = ke2 r, from which the
electrons frequency of orbit is f =
n3h3/(42me4). The actual Bohr energy
Conventional mirror E = 22me4/(n2h2) is clearly a differ-
ent quantity, and for an electron jump
between two energy states, E2 E1
hf2 hf1.
Spielberg, N., and B. D. Anderson. Seven
Ideas That Shook the Universe, 2nd ed.
New York: John Wiley & Sons, 1995, p.
248.
Some laser sources come with opti-
cal phaser conjugators to remove dis-
tortion in the laser beam. Optical 139. Angular Momentum
phase conjugation occurs also when We take the space quantization of
there are four waves mixing with all angular momentum as given, so there
four waves of the same frequency. will be (2j + 1) components in the
Another useful application of a phase z-direction from j to j, decreasing by
conjugate mirror might be to put one an integer each step. Since there is no
in one reecting path of an interfer- preferred direction, J2 = Jx2 + Jy2 + Jz2,
ometer as a reference for detecting that is, J2 = 3 < Jz 2>avg , where avg rep-
changes in the other path. resents the average value given by [ j 2 +
Blaauboer, M., D. Lenstra, and A. Lodder. (j 1)2 + . . . + (j + 1)2 + (j)2] h2/(42
Giant Phase-Conjugate Reection with a
[2j + 1]). Using a math table or nding
Normal Mirror in Front of an Optical
Phase-Conjugator. Superlattices and the sum of the series of squared inte-
Microstructures 23 (1998): 937. gers directly, one can verify that J 2 =
Brignon, A., and J.-P. Huignard. Phase Con- j (j + 1) h2/4.
jugate Laser Optics. New York: John Wiley
& Sons, 2004, chap. 1. Feynman, R. P., R. B. Leighton, and M.
Sands. The Feynman Lectures on Physics.
Vol. II, Reading, Mass.: Addison-Wesley,
1965, p. 34-11.
138. Stationary States
In the Bohr model of the hydrogen
atom, one would calculate the fre-
140. Kinetic Laser
quency f = 2r/v of the electrons The explosion of the lasing material
orbital motion. The virial theorem creates many free electrons, some of

Answers 211
which have been blown out of low- and that could be used for applications
lying atomic states, creating the such as biological imaging, nonlinear
needed population inversion for possi- optics, holography, and so on, a pre-
ble lasing action. Practically any mate- pulse technique has been developed.
rial can be used. During an extremely This technique has been used success-
short time interval after the explo- fully to produce lasing in many lower-
sionon the order of nanoseconds Z neonlike ions such as titanium (Z =
stimulated emission may occur as 22), chromium (Z = 24), iron (Z = 26),
photons from the exploding material and nickel (Z = 28). The use of this
exit the expanding blast volume. prepulse technique has opened up a
These photons pass through regions of new class of neonlike X-ray lasers for
the expanding cloud of ionized debris investigation.
and can stimulate the emission of Chapline, G., and L. Wood. X-ray Lasers.
many more photons into the same Physics Today 6 (1975): 40.
Dunn, J., et al. Demonstration of X-ray
quantum state at the same wavelength.
Amplication in Transient Gain Nickel-like
The resulting coherent radiation at Palladium Scheme. Physical Review Let-
many frequencies, including the soft ters 80, no. 13 (1998): 28252828.
X-ray region, will show intensity Nilsen, J. Reminiscing about the Early Years
of the X-ray Laser. Quantum Electronics
spikes in particular directions. 33, no. 1 (2003):12.
Some of the first kinetic laser
explosions were done at Livermore
National Laboratory in the 1970s and 141. Noninversion Laser
1980s with exploding foils and the Yes, lasing without inversion (LWI)
Nova laser system. Since the first can occur whenever absorption can-
demonstration of soft X-ray lasing cellation is established. Light ampli-
emissions at about 10 nanometers or cation is then possible even when the
moreusing the collisional excitation upper-level population is less than the
mechanism in neonlike selenium, lower-level population. This cancella-
many other neonlike ions, ranging tion can be set up in a three-level sys-
from copper (Z = 29) to silver (Z = tem in an atom in which the two
47), have been made to lase. However, absorption transitions to the same
attempts to produce lower-Z neonlike nal state interfere and cancel, making
X-ray lasers have been unsuccessful. the absorption probability zero.
In the effort to develop a tabletop In the diagram, upper-level state
X-ray laser that would require smaller | a > is connected to lower levels | b >
high-energy laser drivers than Nova and | c >. Use incident photons of the

212 Answers
State a 142. X-ray Paradox
The index of refraction n for a mate-
E1 E2 State b rial is normally stated with regard to
the phase velocity, unless indicated
State c
otherwise. The phase velocity is vph =
3-state system c/n(k), where the index is a function of
the wave number k. If n(k)<1, then the
phase velocity is greater than the speed
appropriate energies E1 and E2, which of the light in the crystal. There is no
correspond to transitions | a > to | b > alarm that the energy is being trans-
and | a > to | c >, respectively. The ported faster than c, for the group
uncertainty in these atomic transitions velocity is still less than c.
leads to interference, since the transi- Essentially, travelling harmonic
tions end in the same nal state. There waves in all physical examples require
is no way to determine which absorp- wave packets or groups because of the
tion transition to the nal state actu- non-infinite extent of space and/or
ally occurred, so like the Young time. There are two velocities associ-
double-slit experiment, one must have ated with these wave packets or
the interference. There is no interfer- groups: the phase velocity and the
ence between the emission paths, since group velocity. Harmonic waves or
they have different final states. By harmonic components have a phase
arranging the phases of the two velocity vph = /k, where = 2f and f
incoming light rays properly, one can is the frequency. This phase velocity is
make the interference completely the velocity at which the wave fronts
destructive for absorption. Then stim- travel. A group of harmonic waves or
ulated emission is the only process left. wave packet has a group velocity vg =
For details of the probability calcula- d/dk, the velocity at which the packet
tions, see the references below. shape or envelope travelsthat is, the
Narducci, L. M., H. M. Doss, P. Ru, M. O. velocity at which information or
Scully, S. Y. Zhu, and C. Keitel. A Simple
Model of a Laser without Inversion. energy is transported.
Optics Communications 81 (1999): 379. On the atomic level, the slowing of
Scully, M. O., and M.S. Zubairy. Quantum light passing through a material may
Optics. Cambridge, Eng.: Cambridge be considered as a continuous process
University Press, 1997.
of absorption and emission of photons
as they interact with the atoms of the

Answers 213
material. One assumes that between
each atom, the photons travel at c, as
in a vacuum. As they impinge on the
atoms, they are absorbed and nearly
instantly re-emitted, creating a slight
delay at each atom, which (on a large
enough scale) seems to be an overall
reduction in the speed of the photons.
Both states should have the same
Quantum mechanically, the scattering
energy, and they do. Therefore we
is a two-step process of absorbing the
really have a two-state system, analo-
incident photon and emitting a new
gous to the hydrogen molecular ion or
photon.
the ammonia molecule, so the analysis
Experiments in other ranges of the
should be for a two-state system. There
electromagnetic spectrum, particularly
will be the possibility that congura-
in the visible, have shown that by stor-
tion A changes into conguration B. As
ing the phase information of the inci-
a result, quantum mechanics will
dent light beam in a gas vapor
reveal that two new stationary states
temporarily, one can even claim that
will occur, one state (the new ground
the light pulse can be brought to rest!
state) with energy below the ground
Addinall, E. The Refractive Index of X-
rays. Physics Education 6 (1971): 7778. (lowest) state determined before, and
one state with higher energy. The new
ground state will be neither of the two
143. Benzene Ring conguration states shown but will be
The benzene ring has six-fold rota- a linear combination of these two con-
tional symmetry about an axis perpen- figuration states. Only this state is
dicular to the plane of the ring. One involved in the chemistry of benzene at
simply requires a wave function solu- room temperatures.
tion of the Schrdinger wave equation Understanding benzene was one of
that has this six-fold symmetry, and the rst verications of the linear super-
such a solution is easy to find. One position of states that is at the heart of
would expect that knowing this solu- quantum mechanics and also indicated
tion would allow one to calculate the that quantum mechanics will be suc-
energy levels. cessful at larger scales than atomic.
However, we are not done! There Feynman, R. P., R. B. Leighton, and M.
Sands. The Feynman Lectures on Physics,
are two possible configuration base Vol III, Reading, Mass.: Addison-Wesley,
states, as shown in the diagram. 1965, pp. 10-10 to 10-12.

214 Answers
144. Graphite formation of intermediate structures,
and so on. Fullerenes tend to form by
Place identical atoms into a diamond rolling up a graphite sheet and
crystal structure. First, one would adding carbon pentagons to achieve
mathematically find a wave function curvature. If you just roll the sheet into
for the four bonding electrons using a cylinder and cap off the ends with
the Schrdinger equation, resulting in pentagon-curved hemispheres, you
what are called sp3 orbitals. Then one make a carbon nanotube. These nan-
would represent the periodic symmetry otubes are quite different from the tra-
in the crystal. Each carbon atom will ditional fullerene-type materials (i.e.,
make four orthogonal bonds with roundish cages), so they have quite dif-
tetrahedral symmetry if it can to its ferent properties.
nearest neighbors. This diamond struc- Collins, P. G., and P. Avouris. Nanotubes
ture is one way to do this bonding. for Electronics. Scientic American 283,
Another way to have four carbon no. 6 (2000): 6269.
bonds is for six carbon atoms to form Pauling, L. General Chemistry. New York:
Dover, 1988, pp. 168170, 207210.
a regular hexagonal ring with two
bonds in the ring for each carbon, and
the other two bonds extending per-
pendicular to the ring, one upward 145. Ozone Layer
and the other downward. Upon calcu- Ozone plays two important roles with
lating the energy states for the four regard to the energy balance for Earth.
carbon binding states, one learns that As a minor greenhouse gas in all parts
the two perpendicular binding states of the atmosphere, including near the
are held less securely than the ones in surface, ozone helps maintain Earths
the ring that form a plane. The struc- average temperature at about 13C
ture makes graphite, a layered crystal instead of a frigid 17C. The concen-
that slips easily between the planes. tration of ozone in the upper atmos-
Pencil writing surfaces have been phere, however, regulates the UV
made from graphite for several thou- intensity in sunlight reaching the sur-
sand years. face. All organisms need some UV
Carbon in the fullerene structure light to maintain a healthy existence,
is even more interesting. The structure but any reduction of ozone in the
of 60 carbon atoms that results upper atmosphere might allow dan-
depends on many factors, including gerously large amounts of UV to reach
the velocity distribution of the free the surface.
carbon atoms before collision, the The two polar regions are extremely

Answers 215
susceptible to ozone depletion, particu- equilibrium temperature of about
larly by chlorouorocarbon (CFC) mol- 13C. Without the greenhouse effect
ecules and others, because the ice in our atmosphere, Earths average
crystals in the air provide these uoro- surface temperature would be about
carbons with platforms for rapid ozone 256 K, or about 17C, much too cold
dissociation. Already, as a result of for many life forms. The greenhouse
ozone depletions in the upper atmos- effect involves the inux of sunlight,
phere above the polar regions, particu- its absorption by the atoms and mole-
larly above the South Polar region, cules of the stuff on Earth, and the
there has been an increase in eye prob- attempted emission of light and
lems in land animals such as sheep in infrared energy back into space.
the southern parts of South America Although carbon dioxide receives
and in Australia and New Zealand. the most attention in the press, HOH
Allgre, C. J., and S. H. Schneider. The Evo- vapor is the most important green-
lution of the Earth. Scientific American house gas because the HOH molecule
271, no. 4 (1994): 6675.
absorbs energy over practically the
Newchurch, M. J., et al. Evidence for Slow-
down in Stratospheric Ozone Loss: First whole range of visible and infrared fre-
Stage of Ozone Recovery. Journal of Geo- quencies, while carbon dioxide
physical Research 108, no. D16 (2003):
absorbs in a small region of the near
4507.
infrared only. Water vapor controls
about 60 percent of the greenhouse
146. Greenhouse Gases effect, carbon dioxide about 20 per-
cent, and the other trace gases in the
CO2
HOH atmosphere the remainder.
HOH Additional greenhouse gas concen-
O3 trations added to the atmosphere
would be expected to trap even more
280 K blackbody
Radiance infrared radiation and probably raise
the temperature further. However, a
Absorption convincing comprehensive model of
curve
this process has not been achieved.
There are many complications to any
400 1600 model of Earth, including the trans-
Wavelength (cm-1)
mission and reflection of light from
The greenhouse gases trap most of the clouds, the movements of the ocean
infrared, and this additional energy currents, the effects of human-made
helps heat Earth to its present average sources and sinks, the perturbations

216 Answers
by vegetation, land animals, and sea LCD is provided by the ambient light,
organisms such as plankton, the ther- whereas all the energy for the LED and
mal energy input from additional heat plasma displays must be provided by
sources such as mantle transport of the electronic power source itself, such
thermal energy from the interior of as a battery or the AC supply. In addi-
Earth, and the effects of the bombard- tion, considerable thermal energy can
ment by cosmic rays from the galaxy be produced in a plasma display, an
and beyond. energy requirement beyond simply
Many natural temperature records producing a picture on the screen. Of
have been mined in the past few course, there are LCD displays that
decades that provide the history of must provide their own ambient light
temperature changes, so uctuations in if they are to be used in a dark envi-
average temperature, a vaguely dened ronment, so these displays have addi-
concept, are not new. The claim seems tional energy requirements when
to be that the rate of increase of the operated in this manner.
average temperature is among the So LCDs consume much less
greatest ever experienced on Earth. power than LED and gas-display mod-
Whether this hypothesis is veried in els because LCDs work on the princi-
the near future will take better models, ple of blocking light rather than
meaning greater computing capability emitting it.
and more included physical and
chemical processes and/or a denitive,
unambiguous example.
148. Sonoluminescence
Gillett, N. P., F. W. Zwiers, A. J. Weaver, and The light produced by sonolumines-
P. A. Stott. Detection of Human Inuence cence must originate in atomic transi-
on Sea-Level Pressure. Nature 422 (2003):
292294. tions, electrons in excited states in
Herzog, H., B. Eliasson, and O. Kaarstad. atoms jumping down to lower energy
Capturing Greenhouse Gases. Scientic levels and emitting photons to con-
American 282, no. 2 (2000): 7279.
serve energy and angular momentum.
The apparatus consists of distilled
water with an admixture of a little
147. LED vs. LCD helium or other inert gas in a spherical
We assume that they all have the same flask surrounded by a piezoelectric
resolution, and we know that all three crystal or two to send in sound waves
types of displayLED, LCD, and at practically any frequency. The
plasmarequire energy to operate. details of the apparatus can be found
But the majority of the energy for the at many sites on the Internet.

Answers 217
The sound energy creates bubbles Baghdassarian, O., H.-C. Chu, B. Tabbert,
and G. A. Williams. Spectrum of Lumi-
in the water that rapidly collapse and nescence from Laser-Created Bubbles in
emit a ash of light from their central Water. Physical Review Letters 86 (2001):
region. Instead of sound waves, a 4934.
powerful laser pulse also can create
the bubbles for the pulse of light. The 149. Siphoning Liquid
spectrum of the emitted sonolumines-
cent light pulse is similar to a black- Helium
body spectrum of an object at about At temperatures near absolute zero,
8,000 K, hotter than the Suns surface normal liquid He I becomes superuid
temperature of about 6000 K! And the liquid He II by undergoing a second-
pulse of light lasts for picoseconds, order phase transition. Its He atoms
with such an intensity that it can be can move without viscosity in the
seen by the unaided human eye. superuid. Superuidity is a quantum
The reference below provides mechanical phenomenon, with a
experimental results that support the macroscopic volume (centimeter dimen-
popular theory that a plasma inside sions) of liquid acting like a single
the bubble causes sonoluminescence. macroscopic particle and described by a
The research team tted their pulses single-particle Schrdinger equation.
spectra to a blackbody radiation curve Immediately, superuid He II in an
and found the correspondence to open beaker will form a film that
plasma temperatures at about 8000 K. crawls up the walls, over the top, and
The gas in the bubble becomes a down the sides until the beaker is emp-
partially ionized plasma, and the radi- tied. Normal fluids also can be
ation is emitted by an energy cascade siphoned out of containers, but only if
from ions to electrons and nally to their motion is started externally! The
photons. solid surfaces in contact with He II are
More details will be understood covered with a lm 50 to 100 atoms
eventually as faster optical response thick along which frictionless ow of
systems become available to better fol- the liquid occurs. Supposedly, mass
low the time development of the light transport ow in the He II lm takes
emission process. In fact, how quickly place at a constant rate that depends
a state-of-the-art photodetector system only on temperature.
operates is measured against what ini- As the atoms of liquid He II move
tial parts of the sonoluminescent pulse up the wall, they gain potential energy.
of light can be discerned! What process provides the energy?

218 Answers
The answer lies in the ability of helium 150. Quantized Hall
atoms to wet any surfacethat is, nor-
mal liquid He I atoms cling to the
Effect
wall. The helium-helium force is the
3
weakest force in nature because the K RH (h/e2)
shell of electrons is complete and the
helium zero-point motion is signifi- 2
cant, so the helium-anything force is
stronger. Hence helium atoms would
1
rather be next to anything other than
another helium atom. So He atoms
quickly form a film when presented
with the wall of the container because 0 10 20 30
Magnetic Field (Tesla)
the heliumanything attraction lowers
the potential energy and so on, while In a two-dimensional metal or semi-
they gain gravitational potential conductor, the standard Hall effect is
energy. These He atoms clinging to the observed, but at low temperatures, a
wall are no longer in the superfluid series of steps appear in the Hall resist-
phase because their ow velocities are ance as a function of magnetic field
now lower than a critical velocity instead of showing the typical monoto-
value. nic increase. By conning the electron
The thickness of the lm is usually system in the third dimension to con-
limited to a few hundred atomic diam- fine the electron gas to two dimen-
eters because at some thickness the sions, only specific electron wave
advantage of being near to the wall is functions meet the boundary condi-
canceled by the increase in gravita- tions, so only certain quantized energy
tional potential energy. Then, while levels are available for the electrons.
the normal fluid is clamped to the These steps in the Hall resistance occur
wall, the superuid He II ows freely at incredibly precise values of resist-
as the He atoms on the wall act as a ance, which are the same no matter
siphon. what sample is investigatedthat is,
Goodstein, D. L. States of Matter. Engle- the resistance is quantized in units of
wood Cliffs, N.J.: Prentice-Hall, 1975,
p. 327. h/e2 divided by an integer. This amaz-
ing result is the quantized Hall effect.
Recall that electrons have a spin

Answers 219
1/2 and obey the Pauli exclusion prin- When the Fermi energy lies in a
ciple. As electrons are added to an gap between Landau levels, there are
energy band, they fill the available no available states to scatter into, so
energy band states, just as water lls a there is no scattering, and the electrical
bucket. The states with the lowest resistance falls to zero. The Hall resist-
energy are lled rst, followed by the ance for the Hall current cannot
next higher ones. At absolute zero change from the quantized value
temperature T = 0 K, the energy levels whenever the Fermi energy is in a gap
are all lled up to a maximum energy between Landau levels, so one meas-
called the Fermi level. At higher tem- ures a plateau. Only when the Fermi
peratures one nds that the transition energy is in the Landau level can the
region between completely lled states Hall voltage change and a nite resist-
and completely empty states is gradual ance value appear.
rather than abrupt and described by Kivelson, S., D.-H. Lee, and S.-C. Zhang.
the Fermi function, which has a value Electrons in Flatland. Scientic American
274, no. 3 (1996): 8691.
of 1 for energies that are more than a
few times kT below the Fermi energy,
equals 1/2 if the energy equals the
Fermi energy, and decreases exponen-
151. Integrated Circuits
tially for energies that are a few times Heat dissipation is the biggest prob-
kT larger than the Fermi energy. lem in ICs. Simple, old-fashioned ther-
Consider the ideal case of a xed mal energy limits the density of
Fermi energy and a changing applied electronic components. The ability to
magnetic eld. In the presence of the miniaturize continues to improve, but
magnetic eld, the density of electron unless thermal energy production per
energy states in 2-D is no longer con- volume is decreased or new geometri-
stant as a function of energy and cal paths for thermal energy transport
bunches into discrete energy levels, away from the sources are devised, the
called Landau levels, of finite width game is lost. At present, 3-D ICs offer
separated by the cyclotron energy, a temporary reprieve, but even they
with energy regions between the will nd their limit.
Landau levels where there are no Some short time scale solutions
allowed electron states. As the mag- may be possible. The best thermal con-
netic eld is swept to higher values, the ductor among crystalline materials is
Landau levels move relative to the diamond, so going to a diamond sub-
Fermi energy. strate may be a solution. However, the

220 Answers
technology of diamond is not yet com- 152. Atomic
petitive with silicon technology. Also,
components on these substrates that
Computers?
require significantly less energy to Yes. One can use electron spin direc-
function as a gate would delay the tions as binary holders, for example.
overwhelming impact of thermal prob- Even the nuclear spins can join in the
lems. Optical information transfer game. Quantum computers already
between components would eliminate use nuclear spins for storage. On a big-
electrical currents and their thermal ger scale, DNA molecules are being
effects, but silicon does not have the used for a DNA computer.
right optical properties; hence the Several difficulties in making
active research into doping silicon to atomic computers exist, but all of the
make the desirable optical properties. difculties can be overcome by clever
At the longer time scale of several techniques. Putting information in and
decades, perhaps the silicon and semi- reading information out of these
conductor technology will simply fade atomic systems have been done in the
away in favor of some other technol- laboratory already. Maintaining their
ogy on the time horizon that seems xed states is a different kind of prob-
unachievable today but that would lem that depends on which type of sys-
become viable then. Or the newer tem is being used. Nuclear spin systems
technology hasnt even been dreamed have been used quite successfully since
about yet! the 1940s with the development of
For any solid or liquid material, nuclear magnetic resonance (NMR).
quantum disturbances from cosmic Electron spin systems also are con-
rays may decide the ultimate limit in trolled quite nicely in labs. If isolation
electronic component density unless of the system is required, then vacuum
redundancy can solve this problem. chambers work well for long enough
For optical systems based on light time periods of particle isolation.
interference, and so on, who knows At the other extreme are proposed
what is possible? Whatever wins in the quantum computers utilizing the caf-
decades ahead will be numerous feine molecules in a cup of java. They
orders of magnitude smaller and are being bombarded constantly by the
faster, as well as more robust than other molecules in the liquid, so the
what we have today. liquid environment brings about a
rapid decoherence of the system. How-
ever, there are an awful lot of caffeine

Answers 221
molecules in the cup, at least 1020. The X-ray line has been used to analyze
quantum computer requires probably materials in minutes that formerly
only a million or so to retain their iso- required hours to days to accumulate
lation for the duration of the computa- enough data.
tion timemicroseconds, perhapsso Whether the population inversion
the numbers may win out. for the 2p1s transition in the external
Like the limits to integrated circuit Cu atoms actually occurs is unknown.
component density caused by thermal The emission X-ray line is uncharac-
effects and by cosmic ray bombard- teristically narrow and intense, and the
ment, atomic computers also may face absence of the other competing line
similar limits. The type of atomic com- indicates that whatever the selection
puter devised will determine how hos- process is doing must be very efcient.
tile the environment can be. Other element sources such as nickel,
based on the same mechanism, also
have been made.
153. X-ray Laser? DasGupta, K. CuKa1 X-ray Laser. Physics
The mechanism for the intense X-ray Letters A 189 (1994): 9193.
source appears to be the following
according to K. DasGupta, the origi- 154. Bose-Einstein
nator of this unique X-ray source. The
W X-rays from the Cu-W X-ray tube
Condensate
knock out K shell electrons and others A Bose-Einstein condensate is formed
in the Cu atoms in the external Cu at the coldest temperatures, which
crystal to produce a temporary (about means that the atoms have been
1015 second) population inversion, slowed in their motion to be almost
and the Cu X-rays coming simultane- stationary. By the de Broglie relation,
ously from the same tube then stimu- each atom of mass m has a de Broglie
late transitions in these Cu atoms to wavelength = h/p, where p is its
produce the Cu K 1 line at the Bragg momentum mv and h is Plancks con-
angle to the Cu(111) atomic planes. stant. As the velocity v is further
This mechanism is very selective, the reduced to cool the atoms, the de
line being so narrow and intense and Broglie wavelength increases accord-
the process being so efcient that one ingly. Eventually temperatures are
does not detect any of the competing reached for which the wavelengths of
Cu K 2 emission to the available 1s adjacent and nearby atoms begin to
state. The single-frequency intense overlap in space considerably. Further

222 Answers
cooling places all the atoms in intimate how many electrons end up in each
contact in one collective quantum quantum dot.
state. Individual atoms can no longer An electron in a 3-D box is con-
be discerned because they act like one strained to have a quantum mechanical
big atom. wave function that matches the bound-
The rst Bose-Einstein condensate ary conditions for the Schrdinger
was achieved in 1995, even though the wave equation, producing discrete
physics principles have been known energy levels that are inversely pro-
since Einstein and Bose proposed them portional to the square of the wave-
in the 1920s. About 2000 rubidium length. As the box is made smaller, the
atoms in the gas were cooled to 170 energy levels become farther apart. If
nanoK when they formed a Bose-Ein- the quantum dot diameterthat is,
stein condensate less than 100 micro- box diameteris made small enough
meters across. The condensate lasted in fabrication, only a few energy levels
for about 15 seconds and was cooled will exist inside for the electron. Hence
further, all the way down to 20 nanoK. one can make quantum dots small
Anderson, M. H., et al. Observations of enough to allow only one uorescence
Bose-Einstein Condensation in a Dilute transition possible in the visible part of
Atomic Vapor. Science 269 (1995): 198.
the spectrum.
Castin, Y., R. Dum, and A. Sinatra. Bose
Condensate Make Quantum Leaps and The data from the rst quantum dot
Bounds. Physics World (August 1999): 37. spectrum showed a rich harmonic series
Cornell, E. A., and C. E. Wieman. The of transitions between electron energy
Bose-Einstein Condensate. Scientific
levels. Subsequent tweaking of the elec-
American 278, no. 3 (1998): 4045.
trostatic potential was shown to reduce
Townsend, C., W. Ketterle, and S. Stringari.
Bose-Einstein Condensation. Physics the dot size and increase the energy
World (March 1997): 2934. spacings. Later researchers have been
able to magnetically link together quan-
tum dots with the hope of making
155. Quantum Dots arrays of them for quantum computing.
Quantum dots are crystals, essentially Flgge, S. Particle Enclosed in a Sphere. In
metal or semiconductor boxes, con- Practical Quantum Mechanics. Vol. I. New
York: Springer-Verlag, 1974, pp. 155159.
taining only a few hundred atoms
Reed, M. A. Quantum Dots. Scientific
and a well-defined number of elec- American 268, no. 1 (1993): 118123.
trons. The number of electrons can
Whitesides, G. M., and J. C. Love. The Art
be controlled by the electrostatic of Building Small. Scientific American
environment. The trick is to adjust 285, no. 3 (2001): 3847.

Answers 223
by rule 3. The cat is now either alive or
Chapter 8 dead, not both. The two QM alterna-
Chances Are tives reduce to just one possibility.
Note that this example with the cat
brings the connection between the non-
156. Schizophrenic intuitive behavior of Nature on the
Playing Card microscopic scale up to the macroscopic
scale of our everyday experiences.
According to the rules of QM, the nal
There has been an enormous amount of
state should be the superposition of
controversy over this example and its
the two alternative falling directions,
interpretation. Some of the issues are
with equal amplitudes 1 for left and
discussed in the references listed below.
2 for right. But we never see a card
Albert, D. Z. Bohms Alternative to Quan-
fall both ways simultaneously. Any air tum Mechanics. Scientic American 277,
molecule colliding with the card is no. 5 (1994): 5867.
equivalent to an observation, a meas- Loeser, J. G. Three Perspectives on
urement process, so QM rule 3 applies Schrdingers Cat. American Journal of
Physics 52 (1984): 10891093.
and the outcome reduces to the classi-
Wick, D. The Infamous Boundary: Seven
cal one, with equal probabilities P1 to Decades of Heresy in Quantum Physics.
fall to the left side and P2 to fall to the New York: Copericus Books, 1996, pp.
right side. 149152.

The term describing this reduction Yam, P. Bringing Schrdingers Cat to Life.
Scientific American 276, no. 6 (1997):
of the wave function to the classical 124129.
probabilities that have no QM inter-
ference is often called decoherence.
The Schrdinger equation, which is 158. Wave Functions
deterministic, controls the entire No. Beyond three dimensions there is
process. no direct one-to-one correspondence
Tegmark, M., and J. A. Wheeler. 100 Years
of Quantum Mysteries. Scientic Ameri-
between many-dimensional congura-
can 284, no. 2 (2001): 6875. tion space coordinates and the three-
dimensional coordinates of position
space.
157. Schrdingers Cat The misconception referred to here
In QM, it is irrelevant whether you shows up in discussing the wave func-
actually peek or not. If in principle you tion for two-particle systems, espe-
could have determined the status of the cially when the discussion refers to the
cat, QM reduces to the classical result two-particle wave function reducing to

224 Answers
the classical result. One often encoun- Dicke, R. H. Interaction-Free Quantum
Measurements: A Paradox? American
ters questions about how the wave Journal of Physics 49 (1981): 925930.
function can reduce instantaneously to Hilgevoord, J. Time in Quantum Mechan-
the result, as if there has been some ics. American Journal of Physics 70 (2002):
faster-than-light information transfer. 301306.
Fortunately, the two-particle wave
function reduces in configuration 160. Quantum
space, not in position space!
Computer
Hilgevoord, J. Time in Quantum Mechan-
ics. American Journal of Physics 70 (2002): A quantum computer relies on main-
301306.
taining its linear superposition of
Mermin, N. D. Is the Moon There When
Nobody Looks? Reality and the Quantum
quantum statesthat is, = 1 + 2 +
Theory. Physics Today 38, no. 4 (1985): 3, its coherence during the calcula-
3847. tionsso that all the states participate
Styer, D. F. Common Misconceptions in the calculation. Quantum decoher-
Regarding Quantum Mechanics. Ameri-
can Journal of Physics 64 (1996): 3134. ence is a bad thing for a quantum
Wick, D. The Infamous Boundary: Seven computer. A collision with the wall of
Decades of Heresy in Quantum Physics. the chamber or with another molecule
New York: Copericus Books, 1996, pp. will ruin the coherence because an
162166.
observation has been made. By QM
rule 3, we no longer sum over the
159. Wave Function amplitudes i . This decoherence then
ruins the quantum computation
Collapse? because only one state will be partici-
The original wave function = 1 + pating in the computations.
2 + 3 + . . . will change. The probe Maintaining coherence in a real
photon did not scatter off the electron physical system has been progressing
in particular imaginary boxes, so we slowly for the past decade, with coher-
know immediately that the wave func- ence times of tens of nanoseconds for
tion should not include their ampli- three identical subsystems working as
tudes. One could say that there has a quantum computer. No one knows
been a partial collapse of the wave what type of physical system will com-
function even though there has been pose the rst 18-subsystem quantum
no interaction! We believe that this computer in the future, but this com-
gedanken experiment was discussed puter probably will outdo all the other
rst by physicist Robert H. Dicke in classical computers combined in com-
the reference below. puting speed.

Answers 225
Awschalom, D. D., M. E. Flatt, and N. ip a down spin to an up spin for a
Samarth. Spintronics. Scientic American
286, no. 6 (2002): 6773.
stimulated absorption transition or
Lloyd, S. Quantum-Mechanical Comput-
can cause a stimulated emission of a
ers. Scientic American 273, no. 4 (1995): photon by a spin ip from up to down.
140145. Now for some coffee. The liquid
Nielsen, M. A. Rules for a Complex Quan- contains about 1020 caffeine molecules.
tum World. Scientic American 287, no. 5
(2002): 6775. Even if we assume that all of them par-
ticipate initially in bunches as coherent
states of many quantum computers in
161. Cup of Java the cup just before the calculation,
most bunches will experience colli-
Quantum Computer sions during the calculation time of a
Coffee contains caffeine molecules, nanosecond, say, and drop out from
which may be useful as quantum sub- the collection of coherent states of the
systems for a quantum computer system. However, a signicant number
because they contain two rings in a of bunches of coherent states may be
plane with many attached hydrogen participating still when the calculations
atoms. The nuclear spin states of the H are done, and these will provide a
atoms attached to the rings can be strong signal above the background
used for information storage la noise. At least thats the hope!
NMR. That is, a nuclear magnetic res- Gershenfeld, N., and I. Chuang. Bulk Spin
onance (NMR) system is a collection Resonance Quantum Computation. Sci-
ence 275 (January 17, 1997): 350.
of nuclear spin states in an external
magnetic eld that tend to align the
spins. In the simplest ideal case at tem- 162. Bragg Scattering
perature T, the external magnetic eld
B is uniform and there are two spin
of X-rays
states, up and down. Lets say that B Bragg scattering requires < d; there-
aligns most of the spins to the up state, fore there will not be any collective
with the ratio of down to up spins scattering from a group of scatterers at
being determined by the exponential different atoms within one wave-
factor Exp( B/kT), where is the length. The actual scatterers of the
nuclear magnetic moment and k is the X-rays are the electrons at each atom
Boltzmann constant. An external in these planes of the crystal. Coherent
radiofrequency pulse of the proper scattering requires fixed phase rela-
frequency v and energy hv = 2 B can tionships, but there is no xed phase

226 Answers
relationship between electrons at dif- total amount is proportional to N
ferent atoms nor between the electrons instead of N. Therefore = N 1, so
doing the scattering at any moment. P = N | 1| 2. The Bragg scattering of
Therefore, the X-rays scattered into X-rays is not a coherent scattering
the Bragg angle have a multitude of process.
random phases and not fixed-phase
relationships. The scattering probabil-
ity is proportional to N, the number of
163. Beautiful Faces
scatterers, and not N 2 , as it would be Coherent scattering of light by the
for coherent scattering. atoms in the skin is the reason for our
Here is the QM argument mathe- ability to see details of a face. The
matically. Let i represent the proba- ambient incident light is scattered by
bility amplitude to scatter an X-ray at the molecules of the skin. Two factors
the i th atom. We know from QM rule 2 are signicant for this two-step scat-
that = 1 + 2 + 3 + . . . , for alter- tering process: the time interval
native ways to go from the X-ray required and the number of coherent
source to the crystal to the X-ray detec- scatterers. In the visible region of the
tor. Each i represents one atom, and electromagnetic spectrum, this scatter-
we assume single scatterings on the ing process occurs in atoms in less
way to the detector for simplicity. Each than 108 second over an area of the
i = exp[i] i , which includes a phase skin involving about a million atoms
part exp[i] and the identical scattering within a circle with a radius of about
amplitude i at the identical atoms in one wavelength of the light. The wave-
the crystal. If the phase part at each length of greenish light is about 500
scattering atom is identical, then we nanometers.
would have = N 1 and the proba- Consider scattering one incident
bility P = N 2 | 1| 2, giving us coherent photon at a time. During the scattering
scattering proportional to N 2. time of a single photon by these one
However, there is no correlated million alternative paths there is
motion between electrons on different almost no movement of the scattering
atoms, so their phases are random. If atoms in the molecules, so alternative
the phase differences between scatter- paths have essentially xed phase rela-
ersthat is, the electrons on different tionships. By QM rule 2, = 1 + 2 +
atomsare not xed differences, then 3 + . . . , and = N 1 with probabil-
the sum is over random phases and, ity P = N 2 |1| 2, giving us coherent
like the random walk problem, the scattering proportional to N 2. With

Answers 227
incoherent scattering we would not see 164. Gravitational
much detail.
In the UV, both factors are smaller
Waves
than for light in the visible spectrum Yes, the coherent scattering of gravita-
the scattering occurs in less time, and tional waves is expected to occur, with
the area for each scattering is less and the scatterers being mass quadrupoles
involves fewer atoms because the that is, mass pairs in the antenna.
wavelength is much less. The face seen J. Weber, the same physicist who rst
in the UV would appear grainier with calculated the classical cross section for
less detail because the adjacent coher- gravitational wave scattering in 1959,
ent scattering areas are smaller and proposed in 1981 that the coherent
the shorter time interval means that scattering of gravitational waves would
they will have some effects of almost- enhance the scattering cross section for
random phases. certain detectors by a factor of 106 or
In the IR, most of the scattering more. The larger cross section might
involves molecular transitions, which explain the large responses of his two
are relatively slow processes, so the independent one-ton cylindrical alu-
scattering process involves a much minum bar gravitational wave detectors
longer time interval. But each mole- every time either end faced the nucleus
cule itself is completely involved in the of the Milky Way galaxy, approxi-
scattering. So even though the wave- mately twice per day. If his proposal for
length is large, involving many more a coherent scattering response is cor-
scattering centers, the molecular scat- rect, then solid bar antennas would be
terers move signicantly during the IR much more sensitive to gravitational
scattering process, producing random waves than large interferometers with
phases everywhere and a smearing of their small masses at the mirrors such as
the image. LIGO and VIRGO.
Organisms of many different types The QM calculation can be out-
see in the UV and/or in the IR to nd lined as follows. With wavelengths in
their nourishment, as well as in the vis- the kilometer range being much longer
ible. However, we humans evolved than the size of the Al bar antenna in
without being able to see either the UV the lab, all the mass pair quadrupoles
or the IR, our vision being conned to in the antenna are within this one
the visible part of the electromagnetic wavelength. Hence, their responses are
spectrum. Why our eye-brain system approximately in phase, and each mass
evolved in this way is not known. pair offers an equivalent alternative

228 Answers
scattering path. By QM rule 2, = 1 when the original classical response
+ 2 + 3 + . . . , and ~ N 1 with cross section is used. The rate of con-
probability P = N 2 | 1|2, giving us version of mass to energy at the galac-
coherent scattering proportional to tic nucleus should have devoured the
N 2, where N is the total number of whole galaxy by now! I suppose that
mass pairs in the bar, about 1024. we must wait for LIGO and VIRGO
However, the bar is actually composed to detect and calibrate gravitational
of many microcrystallites, so one really waves before we truly know whether
sums the QM amplitudes over the gravitational waves can scatter coher-
number of mass pairs within each ently in Weber bar antennas.
microcrystallite, then sums the proba- Gibbs, W. W. Ripples in Spacetime. Scien-
bilities over all the microcrystallites. tic American 286, no. 4 (2002): 6271.
The coherent scattering probability is Preparata, G. Superradiance Effect in a
Gravitational Antenna. Modern Physics
still more than 10 million times larger Letters 5 (1990): 15.
(after accounting for the crystalline Weber, J. Gravitons, Neutrinos, and Anti-
defects) than the classical non-coher- neutrinos. Foundations of Physics 14
(1984): 11851209.
ent scattering response that Weber rst
calculated in 1959.
Whether any bar antenna for grav- 165. Coherent Neutrino
itational waves behaves as a coherent
scatterer has not been unambiguously
Scattering
demonstrated. Instead of the classical In 1984, so the story goes, J. Weber
result with the bar oscillating at its res- proposed to build a detector for the
onant frequency and its harmonics coherent scattering of neutrinos in a
when hit by a pulse of gravitational proposal for research monies. The
waves, the coherent scattering bar proposal review committee challenged
would essentially have an almost equal him to write up the neutrino coherent
response to a wide range of frequen- scattering idea and publish the paper
cies. The actual experimental bar in a reputable physics journal. In
responses are complicated and require December 1984 he submitted the
elaborate methods to find gravita- paper, Method for Observation of
tional wave scattering signals buried Neutrinos and Antineutrinos, to
in background noise. Physical Review C, and the paper was
If the Weber bars were really accepted by a referee within eight days
detecting gravitational waves from the of the December 12 reception date!
galactic nucleus, there is an enigma The paper triggered an enormous

Answers 229
response in parts of the physics com- of the neutrino took place. Hence their
munity. Numerous rebuttals of his responses are in phase and offer equiv-
arguments appeared in the physics lit- alent alternative scattering paths. One
erature within months after this publi- must sum the amplitudes over all pos-
cation, but all of these rebuttals can be sible pathsthat is, all nucleito
refuted. Every paper erroneously obtain the total amplitude for the
assumes that the nuclear scatterers act neutrino scattering.
as potentials. Wrong! Weber shows in By QM rule 2, = 1 + 2 + 3 +
the rst section of the paper that such . . . , and = N 1 with probability
an assumption cannot lead to coherent P = N2 | 1| 2, giving us coherent scat-
scattering for neutrino wavelengths tering proportional to N 2 , where N is
less than the spacing between nuclei. the total number of nuclei in the bar,
However, everyone seems to ignore the about 1023. One gains the enormous
details presented by Weber, who cor- factor of 1023 for neutrino scattering
rectly explains why the nonrelativistic over the noncoherent cross section!
calculation does not predict coherent The only remaining contention is
neutrino scattering for neutrino wave- whether all the phase relationships are
lengths less than the atomic spacing. properly accounted for in this rela-
The QM argument is essentially tivistic calculation.
dependent on the fact that the scatter- Weber (now deceased) actually
ing phases among the nuclei will be conducted several experiments to
random, leading to a scattering proba- check his relativistic calculations for a
bility proportional to N instead of N 2. long defect-free single crystal detector.
In later parts of the paper Weber He claims to have veried the turning
does the relativistic QM scattering cal- on and the turning off of a nuclear
culations to show that coherent scat- reactor in blind tests, the leaking of tri-
tering for all energies occursthat is, tium from a highly radioactive tritium
neutrinos of all energies will suffer source, and the twice-daily passing of
coherent scattering. Included in these the Sun though the long axis of his
calculations are terms involving the crystal detector. In 1995 he deter-
stiffness of the defect-free crystal, and mined that the total measured solar
so on. The conceptual idea is that flux of neutrinosall three types,
when the crystal as a whole recoils, because the detector did not distin-
like a Mssbauer Effect scattering, guish among themwas equal to the
then one cannot determine (even in total neutrino flux expected by the
principle) where the nuclear scattering standard solar model. This predicted

230 Answers
result agrees with the 2002 results to allow different regions of the living
reported by the heavy water detector tissue to be seen separately. A com-
at the Sudbury Neutrino Observatory puter algorithm analyzes the data from
(SNO). numerous RF detectors surrounding
Ho, T. H. Comments on the Method for the body and constructs an artificial
Observation of Neutrinos and Antineutri- image on a display screen. A dynamic
nos. Physics Letters 168B (1986): 295.
MRI instrument has a fast response
Weber, J. Method for Observation of Neu-
trinos and Antineutrinos. Physical Review time to show changes occurring in
C 31 (1985): 14681475. the microscopic environment in sec-
onds or less, such as muscle action or
heart contractions.
166. Magnetic
Resonance Imaging 167. Heisenberg
(MRI) Uncertainty
Nuclear magnetic resonance experi- The uncertainty principle places no
ments began in the 1940s, and they limit to the accuracy of measuring the
continue to be very useful today. Their particles position. The uncertainty
alternative QM behavior is described principle pxx h/4 forbids the
as a collection of spins acting together. simultaneous measurement of both
Initially, the spin collection has a total position and momentum in the same
spin S in a collective quantum state direction to arbitrary accuracy, not an
= 1 + 2 + 3 + . . . and then the individual measurement. Of course,
pulsed magnetic eld rotates them all practical design limitations exist that
so slightly to S with respect to the probably limit the measurement, but
original directionthat is, they act conceptually there is no limit. The
collectively and coherently. No one same argument applies separately to
spin behavior is isolated from the oth- the momentum.
ers in the same microscopic atomic An application of the Heisenberg
environment. All hydrogen nuclei in uncertainty principle to the hydrogen
the same environment respond the atom is an insightful example. The
same, while those in a different envi- hydrogen atom is usually solved in
ronment respond slightly differently. spherical polar coordinates instead of
The MRI instrument for magnetic Cartesian coordinates. In spherical
resonance imaging uses the differences polar coordinates, the uncertainty
in the microscopic atomic environment relations are a bit more complicated

Answers 231
and the consequences can be some- together many single-frequency waves
what bizarre. For example, since the of different frequencies with properly
hydrogen wave function for the elec- chosen amplitudes and phrases, we
tron about the z-axisthat is, in the can build up a lump in a narrow
directionis known precisely for the region of space of approximate length
1s atomic state, and hence the angular x. The range of wavelengths
momentum has no uncertainty, the needed can be represented by the cor-
uncertainty in is maximum. There- responding range of wavenumbers k.
fore, in the direction, one nds an The approximate mathematical rela-
equal probability at all angles, produc- tionship x k ~ 1 can be established
ing the smeared-out probability distri- by considering several examples, as
bution in . seen in the Krane reference below.
Bohrs famous measurement dis-
Two waves added turbance argument is faulty. For half a
century physicists have regurgitated
this argument of how the uncertainty
principle acts to defend quantum the-
Twenty waves added
ory. In experiments that rst refuted
Bohrs argument, a beam of cold
rubidium atoms is split to travel along
two different paths; call them A and B.
The beams still overlap and combine
Many other uses for the uncer- at the end of their journeys to create
tainty relation exist because it lies at an interference pattern. Now the
the very heart of quantum mechanics. researchers looked to see which path
However, one can see that any descrip- the atoms followed by tweaking those
tion of a phenomenon using waves of on path B into a higher energy state by
any kind will require an uncertainty a pulse of microwaves. These atoms in
relation. Engineers are familiar with
the fact that about a one-MHz band-
width is required to reproduce a one- Collimated atoms Slit plate

microsecond pulse: ft ~ 1, for Atom wave


A
example. Suppose there is a single-fre-
quency wave dened by y = y1 sin k1x.
This wave extends from to + , B
and the question where is the wave Collimator
located? has no answer. By adding waves Screen

232 Answers
their internal states kept a record of Krane, K. Modern Physics, 2nd ed. New
York: John Wiley & Sons, 1995, pp.
which path they had taken. The 93106.
microwave pulse absorbed by an Styer, D. F. Common Misconceptions
atom is insignificant by a factor of Regarding Quantum Mechanics. Ameri-
about 10,000 and can cause little can Journal of Physics 64 (1996): 3134.
change to the atoms momentum, not Wick, D. The Infamous Boundary: Seven
Decades of Heresy in Quantum Physics.
enough to smear the interference pat- New York: Copericus Books, 1996, pp.
tern. Yes, QM worked still. With the 152156.
microwaves off, interference fringes
appear. Turn them on so you can tell
which path was taken, and the inter- 168. Vacuum Energy?
ference pattern vanishes. The uncer-
tainty principle is correct still, but the There is always the zero-point energy
argument that measurement disturbs in the vacuum. Whatever QM model
the system to explain the double slit for the vacuum is considered, all can
experiment is wrong. be reduced in a rst approximation to
So what may be the deeper mecha- a large number of harmonic oscilla-
nism at work in the double slit experi- tors, which have a zero-point energy
ment, for example? Perhaps quantum value that is non-zero. At present, QM
entanglement, in which every particle calculations of the energy density of
is linked to every other particle it has the vacuum seem to be too large by at
interacted with. Two-particle wave least 30 orders of magnitude! The vac-
functions are linked in a six-dimen- uum energy density should be about
sional configuration space with no 1011 J m3 if this vacuum energy is the
one-to-one correspondence to physical source of the accelerated expansion of
3-D space, so the entanglement of N the universe determined by the Type
particles will be described by a wave 1a Supernova measurements in 1998.
function in 3N-dimensional congura- One can do an energy estimate
tion space with no one-to-one corre- using the Heisenberg uncertainty prin-
spondence to 3-D physical space. And ciple. Or, if the vacuum has an effec-
now the mathematics becomes messier! tive potential for a scalar field, the
Drr, S., and G. Rempe. Can Wave-Particle
product of the visible matter density
Duality Be Based on the Uncertainty Rela- and the potential will give the energy
tion? American Journal of Physics 68 density for an assumed radius of the
(2000): 10211024.
universe. In either case, the assump-
Englert, B.-G., M. O. Scully, and H. Walther.
The Duality in Matter and Light. Scien- tions necessary to estimate this energy
tic American 271, no. 6 (1994): 8692. density would take us too far astray.

Answers 233
However, we can determine is a soup of virtual particle-antiparticle
whether an electrically neutral particle pairs that interact with the real atoms
of mass m popping into existence for in the metal plates, these pairs being
a time interval t can be detected by its created and annihilated in extremely
gravitational eld. We use the uncer- short time intervals in accordance with
tainty relation Et h/4 in the form the Heisenberg uncertainty principle.
c2 mt > h/4. Suppose we have the That is, the more the total energy E
most sensitive detector, a free particle in the pair, the less time duration t is
of mass M initially at a distance R its existence so that Et h/4. This
away from m; then in the Newtonian vacuum pair soup pushes inward at
approximation the detector will both plates when the plates are very
receive a pulse P = Ft. Substituting close to each other because certain
F = GMm/R2 into the uncertainty particle-antiparticle pairs are practi-
relation produces GMmt/R2 cally forbidden from momentarily
GMh/(4R2c2).The initial state of the appearing between them. Essentially,
detector also obeys the uncertainty if their deBroglie wavelength exceeds
relation PX h/4, so that m to the plate spacing, these pairs have a
be noticeable requires the impulse P to much lower probability to be between
be greater than about 2P, or X the plates. But these same pairs appear
4R (R/rg), where the Schwarzschild outside the plates and provide the
radius of the detector rg = GM/c2. For additional forces, whence the net
objects ranging in size from protons to inward force. Known as the Casimir
planets, rg lies within the object itself. effect, it was rst measured in 1958.
So the momentum transferred by the The Casimir force is too small to be
impulse will not be detected! observed for plates that are not within
Haroche, S., and J.-M. Raimond. Cavity microns of each other. Two mirrors
Quantum Electrodynamics. Scientific with an area of 1 cm2 separated by a
American 268, no. 4 (1993): 5462.
distance of about 1 m have an attrac-
Ostriker, J. P., and P. J. Steinhardt. The
Quintessential Universe. Scientic Ameri- tive Casimir force of about 107 N.
can 284, no. 1 (2001): 4653. Although this force seems very small, at
Stefanski, B. Jr., and D. Bedford. Vacuum distances of less than a micrometer the
Gravity. American Journal of Physics 62
(1994): 638639. Casimir force becomes the strongest
force between two neutral objects! At
separations of 10 nanometerroughly
169. Casimir Effect 100 times the size of an atomthe
Although the classical vacuum is a Casimir effect produces a force that is
void, the quantum mechanical vacuum the equivalent of 1 atmosphere of pres-

234 Answers
sure. The resurgence of interest in the that the photons are not spread out
Casimir force is because micromechan- evenly in time nor in spatial extent.
ical devices on the scale of tens of nano- Heisenbergs uncertainty relation dic-
meters must accommodate its effects! tates this behavior. The QM operators
Haroche, S., and J.-M. Raimond. Cavity of phase- and amplitude-quadrature
Quantum Electrodynamics. Scientific (i.e., for the perpendicular components
American 268, no. 4 (1993): 5462.
of the E eld) of the electromagnetic
Kleppner, D. With Apologies to Casimir.
Physics Today 43, no. 10 (1990): 911. eld do not commute, similar to posi-
tion and momentum of a particle. The
product of phase- and amplitude-
170. Squeezing Light uncertainty has a fixed lower limit.
Classically, a ray of light is an electro- The more precisely the phase of a light
magnetic wave having an amplitude wave is measured, the less determined
and a phase, both being expressed in becomes its amplitude and vice versa.
terms of the electric eld components States of the light with the smallest
Ex and Ey. Quantum mechanically, the possible amount of overall quantum
normal modes of the electromagnetic noise are minimum uncertainty states.
eld are quantized and treated as an The reduction in quantum noise in
ensemble of harmonic oscillators, one one observable of the light (e.g., the
harmonic oscillator per normal mode. phase) at the expense of enhancing it
The number of photons in each har- in the complementary observable (i.e.,
monic oscillator is the energy in the the amplitude) can be done by para-
corresponding oscillator. An harmonic metric amplication procedures. The
oscillator obeys the Heisenberg uncer- resulting states of the light are called
tainty principle, so one expects the elec- squeezed states, since the quantum
tromagnetic eld to behave likewise. noise got squeezed at a particular
As the electric eld in a light ray is phase angle. Their wave packets oscil-
reduced, even a ray from a laser late in time and get wider and nar-
source, the xed amount of intrinsic rowerthat is, they breathe.
quantum noise in the light intensity Alternately, the uncertainty in the
becomes more obvious. This quantum amplitude of a laser beam can be
noise in an electrical eld is ever pres- reduced to a level below that normally
ent. If you shine any light on a pho- allowed by the Heisenberg uncertainty
todectector such as a photodiode, principle, a level known as the zero-
there will be uctuations in the diode point quantum noise level. However,
current corresponding to the individ- this increased knowledge comes at the
ual photons being detected. One sees expense of greater uncertainty in the

Answers 235
frequency of the light. Essentially, one increases the magnetic moment of the
is using an uncertainty relation of the electron or muon to g = 2 (1 + a). The
form Ex Ey V, where V is a con- small correction of about 0.12 percent
stant. Reducing the uncertainty in Ex is called the anomalous moment but is
to gEx means that the uncertainty in Ey often referred to as g-2. Its meas-
becomes Ey /g to keep their product urement with gradually increasing
the same. accuracy presents spectacular agree-
Experiments with squeezed light ment with calculation to better than
promise to enhance our understandings 24 parts per billion.
of quantum mechanics at the individual The muon is 206 times heavier
atom and photon levels. Recently, a than the electron, so the muons mag-
new type of ultraprecise laser pointer netic moment is 206 times smaller, but
made by squeezing a beam in two the virtual particles in the quantum
directions was able to position the soup can be more massive. As a result,
beam with a precision of 1.6 , about the anomalous moment is 40,000
1.5 times better than the theoretical times more sensitive to undiscovered
limit for a conventional laser. particles and new physics at short dis-
Treps, N., et al. A Quantum Laser Pointer. tances. There is agreement to 4 parts
Science 301 (2003): 940943. per million that must be regarded as
the best test of the theory, but there is
also a small discrepancy that needs to
171. Electron Spin be explained, a difference in mean val-
Yes. Although the vacuum influence ues of the experiments and the theory
on the electron spin is extremely small, by 2.6 standard deviations.
the same effect of the vacuum on the The muon g-2 result cannot at
muons spin has been measured at present be explained by the established
Brookhaven National Laboratory. The SM. Recalculations of the predicted
interaction magnitude is predicted by theoretical value continue, and correc-
the Standard Model (SM) of Leptons tions have been done. Moreover, the
and Quarks and their interactions. All g-2 calculation involves three of
fundamental particle-antiparticle pairs the four fundamental interactions
momentarily appear in the vacuum weak, electromagnetic, and colorso
and disappear sporadically, so the there are many Feynman diagrams
electron (and muon) see them all, if that contribute.
only for a eeting moment. This vac- Perhaps this unresolved g-2 differ-
uum soup is slightly magnetic, so it ence is the harbinger of new physics

236 Answers
beyond the SM, such as new quarks, 2.7 millikelvins. The He-3 pairs form
or supersymmetric particles, or a sur- one momentum macrostate. Because
prise in the vacuum. the component He-3 atoms are not
Bennett, G. W., et. al. Measurement of the bosons, there should be some small
Positive Muon Anomalous Magnetic width to the macrostate momentum in
Moment to 0.7 ppm. Physical Review
Letters 89 (2002): 101804. addition to the small width because the
He atoms are composed of fermions.
The pairs of atoms are magnetic,
172. Superconductivity so the He-3 superuid is more com-
The paired electrons in superconduc- plex than its He-4 counterpart. In fact,
tors that are in the superconducting superuid He-3 exists in three differ-
state show Bose-Einstein condensation ent phases related to different mag-
to a single macrostate. There is some netic or temperature conditions. In the
small energy width to this macrostate A phase, for example, the superuid is
because the pairs are composed of spin highly anisotropicthat is, directional
1/2 particles, and they are showing like a liquid crystal.
remnant Fermi-Dirac behavior: no Scientic American. Special brieng on the
Nobel Prizes in Science, A New Super-
two identical fermions can ever be in fluid. Scientific American 276, no. 1
the same state as dened by their four- (1997): 1516.
momenta and spins no matter how
they behave collectively.
174. Gap Jumping
This Josephson effect is really quan-
173. Superfluidity
tum mechanical tunneling across the
The odd number of constituents in physical gap because the wave func-
He-3 (two protons, one neutron, and tion for the superconducting pair
two electrons) classies it as a fermion extends beyond the end of the material
that obeys Fermi-Dirac statistics. So no into the gap and to the other side. If
two He-3 atoms can share the same the superconducting material is actu-
quantum state defined by the four- ally in the form of a ring, then match-
momentum (energy and three-momen- ing of the wave function for the pair
tum) and spin. The surprise in the early around the ring must be made,
1970s was that He-3 can magnetically restricting their angular momentum
couple with another He-3 to form a quantization to multiples of h/2.
boson and become a superuid liquid Clarke, J. SQUIDS. Scientific American
at the extremely low temperature of 271, no. 2 (1994): 4653.

Answers 237
175. Nuclear Decay nuclear decay rate may be inuenced
by its environment, see the Peres refer-
The wave function extends through ence below.
the potential barrier to the outside Halliday, D., and R. Resnick. Fundamentals
world. Therefore the probability to be of Physics. New York: John Wiley & Sons,
outside the nucleus is not zero. So why 1988, pp. 10091010.
does the wave function itself extend Peres, A. Zeno Paradox in Quantum The-
ory. American Journal of Physics 48
into the barrier? All connement prob- (1980): 931932.
lems, classical and quantum, have
solutions with functions that extend
into the barrier, usually decreasing 176. Total Internal
exponentially to almost zero within a
few wavelengths. Atomic particles
Reflection
have relatively long wavelengths com- Yes, the light goes a little beyond the
pared to the barrier thickness. So why interface. One can treat this behavior
does the wave function itself not end either classically or quantum mechan-
in the barrier? Because the effective ically. In QM the wave function for
barrier height decreases with radial the photon extends beyond the glass-
distance. air interface into the air.
The probability to tunnel through You can see this behavior in the
the barrier is proportional to Exp following manner. Fill a drinking glass
[Ar
(U
(r)E)], where E is the energy partially full with water. Tilt the glass
of the incident particle, U(r) is the bar- and look down into it at the side wall
rier potential as a function of distance at such an angle that the light entering
r, and A is a constant that includes your eye has been totally internally
Plancks constant h. Some closely reected from the wall. The wall will
related problems to be treated as tun- look silvery when this condition holds.
neling through a barrier are: Then press your moistened thumb
1. Bare copper wire is cut and the two against the outside of the glass. You
ends are twisted together. In spite of will see the ridges of your ngerprint
the fact that the copper is coated because, at those points, you will have
with copper oxide, the twisted ends interfered with the total reflection
still conduct electricity readily. process. The valleys between the
ridges are still far enough away from
2. Tunnel diode operation.
the glass that the reflection here
3. Scanning tunneling microscope. remains total and you simply see a sil-
For a discussion about how a very whorl.

238 Answers
177. Annihilation action. The compression of the ball
(and the concrete being struck) sends
Fermis Golden Rule hints that we phonons (quantum sound waves) run-
should consider the phase space avail- ning around telling the material that
able for the final particles, and this compression is occurring and that the
phase space is related to the entropy of increased energy density in parts of the
the nal particles. If the entropy of the ball can be reduced by expanding back
nal state is greater than the entropy to its normal size. Of course, the
for the initial state, the process occurs. expansion overshoots and the ball
In the simpler case, when an electron rings as it leaves the concrete, each
at rest and its antiparticle, the positron extended state also increasing the
at rest, annihilate each other, two pho- energy density in parts of the ball. One
tons are produced to conserve quan- can model much of this behavior by
tum numbers as well as energy and assuming that the atoms and mole-
momentum. The entropy of the prod- cules are in a potential well somewhat
ucts is greater than the reactants. similar to the parabolic well of the
Why? Because there is much freedom harmonic oscillator. However, instead
in the direction of the photon polar- of a potential energy for an atom ver-
izations. The interacting particle and sus the atomic separation distance
antiparticle begin with their spins being proportional to r2 only, there
opposite but along a specic direction, must be additional terms proportional
thereby having a total spin of zero. In to r3, etc., where r is the distance from
the nal state with two identical pho- the equilibrium location.
tons emerging in opposite directions in Eventually the phonons help the
order to conserve energy and linear ball get back into its normal shape, but
momentum, the photon spins are the atoms and molecules never quite
oppositethat is, both spin +1 or both make their initial relative positions
spin 1 with respect to their momen- again, there being some residual dis-
tum directionsbut the polarization tortion. Even the concrete being struck
vectors can be in any direction in the by the ball never quite recovers. Wit-
plane perpendicular to the momentum ness the eventual wear of a concrete
directions. highway by cars and trucks compress-
ing the road, a more vigorous process
178. A Bouncing Ball but conceptually the same.

We present a simplied version of the


complexities of this bouncing ball

Answers 239
Silverman, M. P. A Universe of Atoms, an
179. The EPR Paradox Atom in the Universe. New York: Springer-
There seems to be no classical thinking Verlag, 2002, pp. 92102.

that would reproduce the data set. A von Baeyer, H. C. Taming the Atom: The
Emergence of the Visible Microworld.
predetermined instruction set would Mineola, N.Y.: Dover, 1992, pp. 210211.
be akin to an algorithm for generating
random numbersbut no such sets of
numbers are truly random. One must 180. Information and a
accept the conclusion that Nature is Black Hole
quantum mechanical and therefore
classical physics is only an approxima- For certain, one should worry about
tion. The rules of QM agree with the quantum information loss, especially
results, but the details are too compli- if quantum mechanics is to provide a
cated mathematically to present complete explanation for everything in
herein. The references provide the the world. Does the black hole infor-
extended discussion. mation increase with the inclusion of
Even more surprising is the sugges- the chair? Lets see. A black hole has
tion that locality is violated. That is, mass, spin, and possibly electric
information from the first detector charge, weak charge, or color charge.
passes to the second detector without Thats all! We cannot determine the
passing through imaginary spherical information content of the black hole
surfaces surrounding each, as if more from these quantities only. That is a
dimensions exist in our world! Some- problem. The most likely solution that
one, someday, will determine a funda- would prevent quantum information
mental reason for this behavior of loss is that the surrounding space
nature. just outside the event horizon of the
Eberly, J. H. Bell Inequalities and Quantum
black hole takes care of the informa-
Mechanics. American Journal of Physics tion equation to make everything
70 (2002): 276279. correct, emitting particles to compen-
Einstein, A., B. Podolsky, and N. Rosen. sate correctly.
Can Quantum-Mechanical Description of
Physical Reality Be Considered Complete? The actual physics calculation of
Physical Review 47 (1935): 777780. information change in the gravita-
Mermin, H. D. Is the Moon There When tional field of a black hole is much
Nobody Looks? Reality and the Quantum more complex and difficult. Among
Theory. Physics Today 38 (1985): 3847.
the necessary concerns is the fact that
Shimony, A. The Reality of the Quantum
World. Scientific American 258, no. 1 the black hole has performed a non-
(1988): 4653. unitary transformation on the state of

240 Answers
system when it devoured the chair. A also vary. As the variations in the
non-unitary evolution is excluded in a cosmic rays are determined by other
quantum theory because it fails to pre- independent methods, they can be
serve probabilitythat is, after a non- incorporated into the C-14 dating as
unitary evolution, the sum of the adjustments.
probabilities of all possible outcomes According to research literature,
of an experiment may be greater or tree ring counts indicate that C-14 dat-
less than 1. Quantum mechanics could ing has uctuations of the C-14 con-
not survive. Perhaps the QM of a centration in the atmosphere between
black hole will eventually be done and 1400 and 1700 B.C.E. Furthermore, a
quantum gravity will save us from this comparison of radiocarbon-deter-
catastrophe! mined ages with ages of archaeological
Bekenstein, J. D. Information in the Holo- materials accurately established by
graphic Universe. Scientic American 289, other methods reveals that for the
no. 2 (2003): 5865.
period from 100 B.C.E. to 1400, radio-
carbon dating gives values that are too
large, and that prior to 100 B.C.E. the
radiocarbon values are too small.
Chapter 9 At about 1600 B.C.E., the C-14
Can This Be Real? date values are about 175 years (5
percent) too small, increasing to about
300 years (6 percent) at 3000 B.C.E.
181. Carbon-14 Dating The discrepancy appears to be a result
The ratio of C-14 to C-12 in living of slight variations in Earths magnetic
organisms will depend on many fac- eld over the years, which would alter
tors, including the local climate and the cosmic ray intensities and hence
the amounts of C-14 in the atmos- C-14 production in the atmosphere.
phere, factors that can vary on time These corrections allow C-14 dates to
scales as short as tens of years. The be corrected, and even for 100,000
radiocarbon dating process assumes years ago the radiocarbon dates are
in its zeroeth order approximation good to within 5 percent.
no variation in these factors over Staff of McGraw-Hill, eds. McGraw-Hill
Encyclopedia of Science & Technology. Vol.
hundreds and thousands of years. But 15. New York: McGraw-Hill, 2002, pp.
the cosmic ray intensity reaching the 136144.
atmosphere may vary considerably,
so the amount of C-14 produced will

Answers 241
182. Nuclear Energy are higher in energy than the corre-
sponding neutron levels because there
Levels is the added Coulomb repulsion. Any
Even the shell model, often called the extra proton or neutron can be added,
independent particle model of the but this additional particle must
nucleus, fails to correctly predict many occupy a higher energy state, usually
of the energy level spacings unless the leading to an unstable nucleus.
spin-orbit LS interactions are included. Jolie, J. Uncovering Supersymmetry.
That is, the proton and neutron mag- Scientific American 287, no. 1 (2002):
7077.
netic moments interact with magnetic
Serway, R. A. Physics for Scientists &
fields produced by their orbital Engineers with Modern Physics, 3rd
motions. These LS interactions add ed. Philadelphia: Saunders, 1990, pp.
terms to the approximate constant 13521354.

potential of the shell model to domi- Tipler, P. A. Modern Physics. New York:
Worth, 1978, pp. 427432.
nate the quantum state sequence inside
the nucleus. As a result, many energy
levels change their relative positions
on the energy scale, with levels from 183. Nuclear Synthesis
different principle quantum numbers Look at the binding energy curve for
becoming interchanged! Once the LS the elements and you will see that at
interaction was properly accounted least one isotope of Ni is well bound.
for, all its predictions were shown to Unfortunately, this isotope has a rapid
agree with the empirical data. decay mode. In fact, all the Ni isotopes
This model of the nucleus also from Ni-49 to Ni-57 have half-lives of
explained why nuclei containing an only milliseconds to at most 10 days.
even number of protons and neutrons
are more stable than others. Like the
energy levels for the electrons in quan-
8.80
tum states outside the nucleus, the Fe-58
Fe-56 Ni-62
Fermi exclusion principle allows two
identical particles per quantum state
8.78 Cr-52
only. The nuclear quantum states for Ni-60
Cr-54
the protons are separate from the
nuclear quantum states for the neu-
trons, and any particular state is lled 8.76
when there are two identical particles 50 55 60 65
with opposite spins. The proton levels Mass number A

242 Answers
Although the championship of and thus they occur in the normal stel-
nuclear binding energy is often attrib- lar burning cycles. But since the iron
uted to Fe-56, this isotope actually groupthose elements with isotope
comes in third. The most tightly bound mass number of about A = 60is
of the nuclei is Ni-62. The binding at the peak of the binding energy
energies are 8.790 MeV/nucleon for Fe- curve, the fusion of elements above Fe
56 and 8.795 MeV/nucleon for Ni-62. requires energy, with the exception of
The binding-energy curve shows those the most tightly bound isotope
nuclides that are close to the peak. Ni-62, for example.
The most tightly bound nuclides The elements beyond Fe are
are all even-even nuclei. Fe-56 is about expected to be formed in the cata-
a factor of ten more abundant in stars clysmic explosions known as super-
than Ni-62. The Fewell reference novae in which a large flux of
below indicates that the reason lies energetic neutrons build up mass
with the greater photodisintegration approximately one unit at a time to
rate for Ni-62 in stellar interiors. Oth- produce the heavy nuclei. Following
ers have suggested that the very low neutron capture, some isotopes beta
rate of multistep production of Ni-62 decay to change a neutron into a pro-
from Co-59 is the culprit. ton plus an electron and an electron
Fewell, M. P. The Atomic Nuclide with the antineutrino, increasing the atomic
Highest Mean Binding Energy. American number by one unit. Some sample
Journal of Physics 63 (1995): 653658.
sequences are:
Shurtleff, R., and E. Derringh. The Most
Tightly Bound Nucleus. American Journal Fe-56 + n Fe-57 (stable)
of Physics 57 (1989): 552.
Fe-57 + n Fe-58 (stable)
Fe-58 + n Fe-59 Co-59 by
184. Heavy Element beta decay
Synthesis Co-59 + n Co-60 Ni-60 by
The synthesis of the heavier elements beta decay
beyond Fe is done during supernova In principle this process could con-
explosions, in a few days or less, and tinue indefinitely, but the elements
the atomic debris are spewed out into beyond uranium (Z = 92) are all
space to later collect into new stars radioactive.
and planets and be there for incorpo- The layers of the star containing
ration into life forms. the buildup of heavy elements may be
The fusion process for elements up blown off by the supernova explosion
to Fe in the periodic table yields energy, to provide the raw material of heavy

Answers 243
elements within expanding hydrogen contributing to the energy of the nal
clouds that much later can condense to state. Therefore, energy conservation
form new stars, planets, and the stuff dictates that the proton will be in a
of life. proton energy state that is lower in
Krane, K. S. Modern Physics, 2nd ed. New energy than the initial energy of the
York: John Wiley & Sons, 1995, pp. neutron. In many nuclei all available
290291.
proton statesthat is, those that are
not occupied by protonshave higher
185. Neutron Decay energies than the energy of the initial
neutron state, so the decay cannot
The failure of the neutron in a nucleus
occur.
to decay is a quantum mechanical
The equivalent energy levels of the
effect. According to quantum mechan-
protons in nuclei are higher than for
ics, the rate of decay is dictated by
the neutrons because their energies
Fermis Golden Rule, which states that
include the Coulomb repulsion
the rate is proportional to the proba-
between two protons and other prop-
bility of decay (i.e., the absolute value
erties of the nuclear force, especially
of the square of the matrix element
the spin dependence. Obviously the
connecting the initial and nal states)
stable nuclei will include those for
times the density of final states.
which neutron and proton decays do
Because the free neutron decays to a
not occur!
proton plus electron plus electron anti-
Asimov, I. Understanding Physics. New York:
neutrino, we know that the probabil- Hippocrene, 1988, p. 245.
ity for this beta decay process is not
zero and that there are available nal
states for the three product particles. 186. Finely Tuned
Energy conservation dictates that the
total nal state energy equals the total
Carbon?
initial energy of the free neutron. The crucial energy comparison to
Inside a nucleus, the decay of a make is not simply the radioactive
neutron is a transition from an initial state energy of 7.65 MeV to the prac-
energy state, the particular bound neu- tical limit value of 7.7 MeV, but one
tron state that the neutron occupies, to must include the comparison of the
a nal state consisting of a proton in radioactive state energy 7.65 MeV to
some nal proton energy state plus a the energy 7.4 MeV of the reactants at
free electron and a free electron anti- rest. This energy of 0.25 MeV misses
neutrino, the latter two particles being too high for the production of

244 Answers
carbon by the fractional amount of Called the carbon cycle, this sequence
0.05 MeV/0.25 MeV, or 20 percent, of reactions occurs much more rapidly
which is not so critical after all. than the proton-proton cycle sequence
Barrow, J. D., and F. J. Tipler. The Anthropic because the C-12 acts as a catalyst,
Cosmological Principle. Oxford: Oxford neither being produced nor consumed
University Press, 1986, pp. 252253.
by the totality of reactions. The net
Livio, M., D. Hollowell, A. Weiss, and J. W.
Truran. The Anthropic Signicance of the process is still the same: 4 protons
Existence of an Excited State of C-12. He-4, and the net energy produced is
Nature 340, no. 6231 (1989): 281284.
the same, but the rate of energy pro-
Weinberg, S. Facing Up: Science and Its
duction is much higher.
Cultural Adversaries. Cambridge, Mass.:
Harvard University Press, 2001, pp. The carbon cycle occurs at a higher
235237. temperature than the proton-proton
cycle because the C and H Coulomb
repulsion is greater than the H and H
repulsion, so the Sun, with its internal
187. Proton-Proton temperature of about 15 106 K, is
Cycle too cool to activate the carbon cycle,
The other stars are using the carbon which requires about 20 106 K.
cycle for their fusion energy. The com- Krane, K. S. Modern Physics, 2nd ed. New
York: John Wiley & Sons, 1995, pp.
mon proton-proton cycle reaction is 282285.
not the source of fusion energy in
many stars burning hydrogen because
the rst reaction in this sequence has 188. Oklo Nuclear
two protons combining to form a Reactor
deuteron H-2, a very unlikely event
The reaction sequence shows how to
that occurs slowly. A more likely
breed Pu from local U-238, which is
sequence of reactions involves having
the most abundant naturally occurring
C-12 be a catalyst:
uranium isotope. Initially, neutrons
C-12 + p N-13 + come from the ssion of U-235. How-
ever, the very high abundance of
N-13 C-13 + e+ +
U-238 means that this isotope will
C-13 + p N-14 + absorb some of the neutrons to
N-14 + p O-15 + become U-239, decay by beta decay to
neptunium 239, and then decay to Pu-
O-15 N-15 + e+ +
239. Some of the resulting Pu-239
N-15 + p C-12 + He-4 undergoes ssion.

Answers 245
U-238 + n U-239 Np-239 + proximity would have been emitting
e + anti- enough gamma radiation to exceed the
Np-239 Pu-239 + e + anti-
recommended limit.
We can estimate the exposure
However, because the natural reactors amount and compare its value to the
at Oklo probably operated for such a recommended limit today. There are
long time, the Pu-239 had time to approximately 105 decays of K-40 iso-
decay by alpha decay to U-235. Thus topes per second in your body, but the
the Oklo natural reactors were true decay chart tells us that only about 11
breeder reactors, fissioning more percent yield a gamma ray, producing
U-235 than originally existed in the about 1100 self-inflicted gamma rays
reactors. The evidence for the breeder per second, amounting to about 0.36
process remains in the reactor as more mSv per year, well below the recom-
of the ssion products than could pos- mended limit today. Even a group of 10
sibly be produced by the amount of U- people closely packed would not pro-
235 that has been lost from each of the vide a radiation exposure more than 3.6
reactor sites. mSv per year. So we are not radiation
A second piece of evidence for Pu dangers to ourselves nor to our friends!
ssion is the isotopic composition of Cohen, B. L. Catalogue of Risks Extended
the ssion products in the mass range and Updated. Health Physics 61 (1991):
100 to 110. To breed Pu and addi- 317335.

tional U-235, the reactors must have


operated for periods significantly
greater than the half-life of Pu 239, 190. Nuclear Surprises?
about 24,360 years. Both are true statements.
Cowan, G. A. A Natural Fission Reactor. 1. The only emissions from a
Scientic American, no. 7 (1976): 3647.
nuclear power plant are (a) water
vapor from its cooling towers, (b)
189. Human thermal energy in the external cooling
water, (c) any stray gamma rays not
Radioactivity shielded (unlikely to be above normal
At one time in the history of radiation background), (d) any radioactive iso-
safety, before extensive and long-term topes created in the external cooling
measurements, the recommended radi- water (unlikely to be above normal
ation limit was much less than the background), and (e) electrical energy.
limit today. During those times, in The emissions and safety proce-
the mid-1900s, two people in close dures at a coal-burning power plant

246 Answers
are not as strict and, because all coal Th-232, Ra-226, etc.) and cosmic
naturally contains radioactive material radiation (photons, muons, etc.) are
with many isotopes, some of these fairly constant over the world in the
radioactive isotopes escape into the air range of 815 rads per hour. Assum-
when the coal is piled in storage, when ing maximum damage to human tis-
the coal is burned, and so on. Measure- sue, this present background radiation
ments at coal-burning plants verify that level corresponds to about 1.8 mSv per
radioactive atoms and molecules are year.
released. If one spreads all the human-pro-
Scientific researchers in the duced artificial radioactive materials
McBride et al. reference below have equally around the surface of Earth,
concluded from measurements that the local increase in radioactivity is
Americans living near coal-red power expected to be minuscule compared to
plants are exposed to higher radiation this indigenous natural radioactive
doses than those living near nuclear background. Suppose we had a million
power plants that meet government metric tonnes of human-made radioac-
regulations. . . . The fact that coal-red tive material to be dispersed over
power plants throughout the world are Earth of approximately 5 1014 m2.
the major sources of radioactive mate- Each square meter would acquire an
rials released to the environment has additional 0.2 105 kg of radioactive
several implications. It suggests that material, compared to the natural
coal combustion is more hazardous to amount of radioactive material in the
health than nuclear power and that it top 10 centimeters of about 2 102
adds to the background radiation bur- kg, producing an insignicant amount
den even more than does nuclear of local radiation unless the half-lives
power. It also suggests that if radiation were short, on the order of minutes to
emissions from coal plants were regu- days. The additional amount adds
lated, their capital and operating costs only 1 part in 10,000 when dispersed
would increase, making coal-fired around the globe.
power less economically competitive. Aubrecht, G. J. Energy, 2nd ed. Upper Saddle
G. J. Aubrecht, in the reference River, N.J.: Prentice Hall, 1994.
below, states that the radioactivity Cohen, B. L. Catalogue of Risks Extended
danger from each coal-burning electri- and Updated. Health Physics 61 (1991):
317335.
cal plant is at least 100 times the dan-
ger from each nuclear plant. Eisenbud, M. Environmental Radioactivity:
From Natural, Industrial, and Military
2. Background radiation levels Sources, 4th ed. San Diego, Calif.: Acade-
combining terrestrial (from K-40, mic Press, 1987.

Answers 247
McBride, J. P., R. E. Moore, J. P. Wither- room temperature, H nuclei in neigh-
spoon, and R. E. Blanco. Radiological
Impact of Airborne Efuents of Coal and
boring lattice sites experience enor-
Nuclear Plants. Science 202 (1978): 1045. mous accelerations, as large as 1014
m/s2 in random directions. Sometimes
these accelerations are toward each
191. Cold Fusion other, so the two protons can approach
Cold fusion at room temperature is a very close and perhaps fuse into a He
real but unlikely possibility. The key nucleus. However, the actual calcula-
idea is the quantum mechanical over- tion reveals the rarity of this event.
lap of the wave functions of two Despite the extreme improbability
nearby H-2 nuclei, for example. Their of deuteron fusion at room tempera-
wave functions always overlap, no tures, so-called cold fusion, research
matter how far apart they are. How- groups worldwide continue its pursuit,
ever, the bigger the value of the wave as revealed in the references below.
function overlap, the more probable Iwamura, Y., T. Itoh, M. Sakano, and S.
will be the possibility of the fusion Sakai. Observations of Low-Energy
Nuclear Reactions Induced by D2Gas Per-
process to make a He-4 nucleus. meation through Pd Complexes. Innite
Of course, there is a Coulomb bar- Energy 47 (JanuaryFebruary 2003):
1418.
rier to be overcome. In the 1940s came
Mallove, E. F. The Triumph of Alchemy:
the proposal that muonic atomsa Professor John Bockris and the Transmuta-
proton with a muon replacing the elec- tion Crisis at Texas A&M. Innite Energy
tronmight allow fusion because the 32 (JulyAugust 2000): 924.
muonic atom ground state puts the Miles, M. H., B. F. Bush, and J. J. Lagowski.
Anomalous Effects Involving Excess
muon so close to the nucleus on aver- Power, Radiation, and Helium Production
age that the muonic atom appears neu- during D2O Electrolysis Using Palladium
tral to the approaching proton. Cathodes. Fusion Technology 25 (1994):
478486.
However, calculations have shown
that the produced He isotope decays
too quickly for this scheme to succeed
in fusion energy production. 192. Fission of U-235
In gaseous form at room tempera- There are two major problems to be
ture, two colliding H-2 nuclei do not overcome in designing a ssion device.
get close enough for a large wave func- The neutron distribution in a pure
tion overlap, being strongly repelled by U-235 solid would decrease as the
electrical forces acting between two inverse distance squared from each
positive nuclei. In a solid, however, at nuclear decay source, and the target

248 Answers
nuclei would be moving away during chain reaction, but pure Pu-239 can-
the expansion, so one has a diffusion not start itself because the loss rate of
problem complicated by moving tar- neutrons exceeds the production rate.
gets. The moving targets contribute at On the average, each U-235 fission
least two difficulties: the density of produces 2.5 neutrons for every inci-
targets is rapidly decreasing, and the dent neutron. At the critical mass of
neutron-capture cross section is a ssile material the chain reaction will
function of neutron kinetic energy as be sustained. For U-235 this critical
seen from the reference frame riding mass is about 7 kilograms for ideal
with each U-235 nucleus. Without the behavior, requiring a sphere about the
proper neutron capture rate by the diameter of a baseball of pure U-235.
receding U-235 nuclei, the chain reac- Surely this baseball would be too hot
tion zzles out. to handle!
Of course, the nuclear device can- Diffusion problems of an expand-
not be expected to be pure U-235 ing material would require a neutron-
because the isolation of enough quan- reflecting strong tamper material
tities of U-235 from U-238 is too dif- surrounding the U-235 sphere to delay
cult and too costly. Therefore, there is the expansion for a few microseconds
mostly U-238 in the expanding solid to achieve additional fissions before
with some U-235, so we have all the exploding. At 100 percent efficiency
previously listed problems to solve but the explosion would be equivalent to
also must account for the nuclear about 120 kilotonnes of TNT. How-
properties of the U-238 as well as the ever, no nuclear device is that efcient.
U-235. Declassified records indicate that
Apparently the Germans during about 60 kilograms of highly enriched
World War II did not solve these diffu- uranium was used in the nuclear device
sion problems satisfactorily. that was released over Hiroshima,
Bernstein, J. Heisenberg and the Critical Japan, in 1945. The explosive charge
Mass. American Journal of Physics 70, no. for the device detonated over Nagasaki
9 (2002): 887976.
three days later was provided by about
8 kilograms of plutonium-239 (>90
193. Minimal Nuclear percent Pu-239).
Bernstein, J. Heisenberg and the Critical
Device Mass. American Journal of Physics 70, no.
9 (2002): 887976.
Pure U-235 can be accumulated into a Pochin, E. Nuclear Radiation: Risks and
critical mass for a sustained nuclear Benets. Oxford: Clarendon Press, 1983.

Answers 249
194. Large Nuclei shapes. Upon excitation, the nucleus
rst deforms into a shape like a rugby
Small nuclei that become excited and football, with a length-to-height ratio
deformed prefer to lose their energy by of about two to one. Mg-24 appears to
breaking up into helium nuclei (alpha behave as if two C-12 nuclei are its
particles) or C-12 nuclei whenever major components and seems to behave
possible. In fact, researchers often talk as a superdeformed nucleus in this
about nuclear molecules composed rugby football shape. The next state
of these two entities. would have an elongated hyperde-
The larger nuclei, with more than formed shape as a result of perhaps six
150 nucleons, usually spin faster when alpha particles lined up along the long
energy is added, and the result of a axis. This nucleus is highly unstable,
higher angular momentum state is a and this nuclear sausage would produce
nucleus that is more deformed. As they an unmistakable debris pattern.
de-excite, up to about 40 gamma rays Recent detailed investigations of
are emitted by descending an excita- several Pb isotopes have yielded sur-
tion ladder, producing a characteris- prises. The angular distribution and
tic gamma ray emission spectrum. polarization of the gamma rays show
From this spectrum one can determine that they were not electric quadrupole
the nuclear angular momentum states (E2) transitions but magnetic dipole
and the nucleuss deformation shape. (M1). Classically, M1 radiation is pic-
Superdeformed nuclei were discovered tured as being emitted from a rotating
in this way. current loop, with the eld oscillating
Rotational motion of quantum at the same frequency as the frequency
objects such as atoms and molecules of rotation. Similar gamma-ray emis-
has a long and distinguished physics sion bands have recently been identi-
history. Quantized rotational motion ed in other nuclei in the mass region
of molecules was rst recognized from around 110, where the nuclei also are
the absorption spectra of infrared light nearly spherical. These spectra have a
in 1912. The occurrence of rotational pattern that is typical of transitions
motion of atomic nuclei rst became a between rotation states, which poses
topic of interest in the late 1930s in an an awkward problem: how can we
effort to explain observed nuclear explain these regular patterns of M1
excitation spectra by physicists gamma rays? Apparently there is
Edward Teller and John Wheeler in much more to understand.
about 1938. Clark, R. M., et al. Evidence for Magnetic
Quantum mechanics dictates the Rotation in Nuclei: Lifetimes of States in

250 Answers
the M1 Bands of Pb-198, Pb-199. Physical about 108 eV, so that the Fe-57 recoil
Review Letters 78 (1997): 1868.
energy of about 0.002 eV produces a
Macchiavelli, A. O., et al. Semiclassical
Description of the Shears Mechanism and Doppler shift so large that no absorp-
the Role of Effective Interactions. Physical tion in the cooled, thin lm normally
Review C. 57 (1998): R1073. occurs. One can cancel this Doppler
Nolan, P. J., and P. J. Twin. Superdeformed shift with a moving absorber or emit-
Shapes at High Angular Momentum.
Annual Review of Nuclear and Particle ter of only 0.0002 m s1. Therefore,
Science. 38 (1988): 533. when the eardrum moves forward
toward the stationary cooled thin lm,
there will be some resonance absorp-
195. Human Hearing tion of the gamma ray, so the detector
The Mssbauer Effect has been used count will decrease. When the
to determine these actual displace- eardrum moves opposite, there is no
ments of an eardrum. The Mssbauer absorption. Because the eardrum
Effect utilizes the recoil-less emission vibrates in a nonlinear fashion, the
of a 14.4 KeV gamma ray (photon) details are somewhat more compli-
from an Fe-57 nucleus, say, and this cated. From the geometries and the
gamma ray is normally absorbed by physical properties of the emission and
an Fe-57 nucleus in another object in the Mssbauer absorption, the dis-
its path. When the emitters (Fe-57 placement values of the eardrum can
atoms placed on the eardrum) are be calculated. The sensitivity of this
moving with the eardrum, the emitted technique allows eardrum displace-
gamma rays pass through the second ments that are only fractions of a
object, a cooled thin lm of Fe con- nuclear diameter to be detected.
taining some Fe-57 atoms, to be cap-
tured in a gamma-ray photon detector.
The important physical property 196. 1908 Siberia
here is that the natural linewidth of the Meteorite
emitted gamma ray is very narrow,
Willard Libby and Edward Teller
explored this event with a reasonable
hypothesis, since no rocky debris was
ever found, and the amount of damage
was enormous. If the meteorite were
made of antimatter, then the ensuing
matter-antimatter annihilation in the
atmosphere and during the ground

Answers 251
collision would create plenty of ener- the 1911 C-14 to C-12 ratio detected
getic photons at 0.511 MeV, 935 in the old oak tree from Wisconsin.
MeV, etc., due to electron-positron Later, a research group of C. Cowan,
annihilations, proton-antiproton anni- C. R. Atluri, and W. Libby (1965) did
hilations, etc. Many of these photons publish a similar result for the analysis
would interact with nitrogen N-14 in of C-14 content in a 300-year-old
the atmosphere directly to make an Douglas r from Arizona showing an
excited N nuclear state or indirectly increase in C-14 in 1911 with the same
via secondary neutron production in interpretation, supported by R. V.
the atmosphere. The excited state of Gentry (1966). However, C-14 meas-
N-14 decays to C-14, which increases urements of a tree by J. C. Lerman,
the atmospheric concentration of C-14 W. G. Mook, and J. C. Vogel (1967)
in the carbon dioxide that is taken in nearer the blast failed to show an
by plants immediately after the event. increase in the 1909 ratio.
An increase in the C-14 to C-12 Several other interpretations of the
ratio should appear in the radiocarbon 1908 meteorite event are possible.
dating of living organisms such as One of the main proposals is that an
plants, beginning in the year 1908 for ice-rock comet struck Earth, much like
local trees, and this increase in the comet Schumaker-Levy struck Jupiter
ratio should appear a few years later in 1994. Also, one cannot rule out the
for trees in North America, caused by possibility that a massive rocky mete-
atmospheric mixing of the C-14. One orite just burned up completelythat
of us (F. P.) was working for the sum- is, broke into small fragments that
mer in Willard Libbys laboratory and burned up before striking the ground.
was assigned to carefully separate the Chyba, C., P. Thomas, and K. Zahnle. The
tree rings from an old oak tree, putting 1908 Tunguska Explosion: Atmospheric
Disruption of a Stony Asteroid. Nature
pieces into vials, and then coding the 361 (1993): 4044.
vials so that only I knew which vials Cowan, C., C. R. Alturi, and W. F. Libby.
contained which tree rings. The sam- Possible Anti-matter Content of the Tun-
guska Meteor of 1908. Nature 206
ples were radiocarbon-dated, and then
(1965): 861865.
the results were plotted by the C-14 to
Gentry, R. V. Anti-matter Content of the
C-12 ratio versus the calendar year. Tunguska Meteor. Nature 211 (1966):
W. Libby, E. Teller, R. Berger, L. 10711072.
Wood, and F. Potter did not publish Lerman, J. C., W. G. Mook, and J. C. Vogel.
Effect of the Tunguska Meteor and
their radiocarbon-dating results, which Sunspots on Radiocarbon in Tree Rings.
demonstrated a signicant increase in Nature 216 (1967): 990991.

252 Answers
197. The Standard Up quark
Down quark
Model
Electron neutrino Charm quark
As far as we know, no such denitive Electron Strange quark
argument for matching specic fami-
lies exists in the Standard Model of
Muon neutrino Top quark
Leptons and Quarks and their interac- Muon Bottom quark
tions. As long as six leptons cancel out
the anomalies of the six quarks, for
Tau neutrino Top' quark
example, all is well! Indeed, one can Tau Bottom' quark
use the second family of quarks to can-
cel the anomaly contributions from
the first family of leptons, the third mathematics. Although the proposed
family of quarks to cancel the second scheme successfully predicted the mass
family of leptons, and the rst family of the top quark, this geometrical basis
of quarks to cancel the third family of for the Standard Model awaits conr-
leptons. In fact, any permutation of mation of other specic predictions for
the traditional lineup of cancellations collisions, which are under way at Fer-
would succeed. milab and soon to be done using the
This ambiguity in the cancellations Large Hadron Collider.
probably indicates that the Standard Glashow, S. L. Quarks with Color and
Model as understood is incomplete. Flavor. Scientific American 233, no. 4
(1975): 3850.
One would expect the traditional
Kane, G. The Dawn of Physics beyond the
scheme, but the conceptual under- Standard Model. Scientic American 288,
standing provided by the Standard no. 6 (2003): 6875.
Model does not dictate uniqueness. Liss, T. M., and P. L. Tipton. The Discovery
of the Top Quark. Scientific American
One of us (F. P.) has proposed an 277, no. 3 (1997): 5459.
interesting mathematical argument for Potter, F. Geometrical Basis for the Stan-
matching lepton families to quark dard Model. International Journal of
families based on correlations among Theoretical Physics 33 (1994): 279306.
nite rotational subgroups of the Stan-
dard Model gauge group for the lep- 198. Spontaneous
tons and quarks. In this scheme, each
lepton family and each quark family is Symmetry Breaking
in a unique subgroup, and the one-to- Yes. At least two other methods can
one correlations are dictated by the achieve the same symmetry-breaking

Answers 253
result without requiring the Higgs par- particle if this mechanism is truly the
ticle. The Standard Model is described source of symmetry breaking and the
by its continuous gauge group SU(3)C particle masses. If the Higgs particle
SU(2)W U(1)Y. The simplest way of does not show up, then spontaneous
all is to spontaneously break this con- symmetry breaking to a discrete group
tinuous group to a discrete symmetry remains an alternative possibility.
subgroup of the continuous group Coleman, S. Aspects of Symmetry. Cam-
SU(2). That is, the lepton and quark bridge, Eng.: Cambridge University Press,
1985, pp. 113130.
avor eigenstates would be associated
Icke, V. The Force of Symmetry. New York:
with finite rotational subgroups of Cambridge University Press, 1999, pp.
SU(2) instead of the continuous group. 232248.
An analogy from geometry would be Potter, F. Geometrical Basis for the Stan-
dard Model. International Journal of The-
to begin with a sphere and then sym-
oretical Physics 33 (1994): 279306.
metry-break to a regular tetrahedron,
t Hooft, G. Gauge Theories of the Forces
or a regular octahedron, or a regular between Elementary Particles. Scientific
icosahedron. Reconciling discreteness American 242, no. 6 (1980): 104140.
with the continuous symmetry group
U(1) of quantum electrodynamics may
be a problem, however, where phases
199. Proton Mass
are assumed to vary continuously. Quantum chromodynamics describes
Another symmetry-breaking approach the interactions of the quarks. The
is the quark condensate method, which up and down quark masses are listed
also does not require a Higgs particle. as ~ 5 MeV/c2 each. However, these
At present, no Higgs particle has current quarks are not what is
been detected at the accelerators, even meant by having them conned inside
though its mass is expected to be below a proton by the color elds. Instead
200 GeV/c2, within the energy range of one must use the effective massthe
the large accelerators. Of course, the constituent masswhich accounts
decay of such a Higgs particle is a a- for this confinement and which can
vor-changing neutral current weak be estimated from the Heisenberg
decay, which means that its decay rate uncertainty principle. Since xpx
is severely suppressed, so only a few h/4, and each quark is confined
Higgs decays would have been detected within the proton radius of about one
among the particle debris so far. When Fermi, we estimate px ~ 100 MeV.
the Large Hadron Collider comes In three dimensions, the total
online in 2005 or later there should dp ~ (px ) 2 + (py ) 2 + (pz ) 2 ~ 170
be copious production of the Higgs MeV/c2. So at least 510 MeV/c2 of the

254 Answers
proton mass is to be associated with Aniol, K. A., et al. Measurement of the Neu-
tral Weak Form Factors of the Proton.
the constituent mass of the three Physical Review Letters 82 (1999): 1096.
quarks within the proton. The remain-
der is the energy contributions of the
gluons holding the proton together. 200. Right- and Left-
Most of the properties of protons,
except the spin, seem to be determined
handed Neutrinos?
by these three valence quarks, much No. The weak interaction is associated
like the valence electrons determine with the SU(2)-weak part of the Stan-
the important chemical properties of dard Model gauge group that operates
atoms. However, when the protons in the unitary planea plane with two
innards are probed more energetically, complex axes. That is, particle funda-
more structure is found, up to four or mental lepton and quark states are
five more particles, called virtual dened in this unitary plane. All rota-
quarks. In addition, up to 30 gluons tions in the normal unitary plane
can be detected. The proton is reveal- involve only left-handed doublets and
ing its inner sanctum to investigators, right-handed singlets, dictated solely by
and the view is becoming quite inter- the mathematics of the geometrical
esting. Quarks, antiquarks, and gluons transformation. Mathematicians call
can be said to form a thick soup these transformations right and left
inside the proton, and theoretical and screw operations. So the physical prop-
experimental physicists are working erty of left-handed doublet states for
together to gure out the recipe. the weak interaction is dictated by the
Today we know that the three mathematical property of rotations in
valence quarks cannot alone account the unitary plane. Nature simply
for the protons spin. The whole sea knows the mathematics!
of quarks, antiquarks, and gluons each The antiparticle eigenstates are in
possess spin, so one must rst deter- the conjugate unitary plane, which is
mine the contribution made by each gauge-equivalent (not equivalent) to
individual member of this seething the normal unitary plane, so the
mass. The results so far suggest that the energy values of particles and antipar-
sea of quarks makes a minimal contri- ticles are the same, but all other prop-
bution to the overall spin of a nucleon! erties are opposites. In this conjugate
unitary plane the mathematics dictates
Abbott, D., et al. Measurement of Tensor
Polarization in Elastic Electron-Deuteron
right-handed doublets and left-handed
Scattering at Large Momentum Transfer. singlets. The existence of two gauge-
Physical Review Letters 84 (2000): 5053. equivalent but different 2-D complex

Answers 255
spaces conjugate to one another dic- by summing over all the phase
tates that the universe has both parti- information from its local environ-
cles and antiparticles. Why there exist ment. Of course, each particle also
so many more particles than antiparti- provides phase information to its envi-
cles in our present universe remains to ronment both near and far. The parti-
be resolved. cles new location is the region where
Altmann, S. L. Rotations, Quaternions, and the phases match best. The calculation
Double Groups. Oxford: Clarendon Press, game requires a dynamic limit to how
1986, pp. 121123.
many nearby cells are counted in order
to accumulate a good approximation
201. Physics without of the phase information and to main-
tain the local geometrical symmetry.
Equations A proof-of-concept calculation has
The best way to use cellular automata been done by one of us (F. P.) on a
(CA) on computers is to incorporate desktop computer using thousands of
the fundamental interactions of the nodes in a 3-D array, but a good cal-
Standard Model of Leptons and culation requires millions of cells or
Quarks plus the gravitational interac- the equivalent.
tion of the general theory of relativity, The marriage between physics and
or preferably its quantum gravita- mathematics has been a happy and
tional version when available. We fruitful one over many centuries.
know that all these fundamental inter- Mathematical equations, from simple
actions in nature correspond mathe- algebraic ones to the more challenging
matically to local phase changes, a differential equations, have allowed us
process that can be simulated with CA to summarize an enormous amount of
without using equations by using a physical phenomena into a simple for-
clever enactment of the path-integral mat. The underlying fundamental
approach to doing all of physics in real symmetries of nature have been the
time. Not yet fully achieved except by true source of many of these equa-
very crude approximation, the physics tions. However, formulating these
of many-particle interactions will be symmetries as the Schrdinger equa-
accomplished by large-scale grid com- tion and Maxwells equations, for
putation methods or perhaps by the example, and solving the equations are
equivalent on a quantum computer. human processes. We cannot expect
The fundamental idea is to deter- Nature to do the same when the sim-
mine the present behavior of a particle pler process of looking locally for

256 Answers
information is more direct. Therefore sources, was not needed even though
we think that understanding the uni- its argument would likewise produce a
verse by combining CA with path inte- dark night sky.
grals will be the physics of future The critical quantity is the ratio of
generations. the average lifetime tave of a star to the
Icke, V. The Force of Symmetry. New York: time T required for the universe to
Cambridge University Press, 1999, pp. reach thermodynamic equilibrium.
178206.
Starting with a uniform density of
stars, an observer can appreciate that
after a clock time t = tave there will be
Chapter 10 an expanding sphere of burned-out
Over My Head stars beyond which lies a shell of lumi-
nous stars. The radiation from this
shell has a maximum radiation density
202. Olbers Paradox equal to the surface radiation density
German astronomer Heinrich Olbers from the average star times the ratio
(17581840) was not the rst scientist tave/T as long as the clock time t << T.
to ask Why is the night sky dark? But tave is at most a little more than 10
but his name remains connected to this billion years, while T can be shown to
paradox. The night sky is dark because be tens of billions of years, so the night
the time required for the radiation eld sky remains dark. Harrison shows that
to reach thermodynamic equilibrium is this argument is true for all present
large compared to all other time scales models of the universe and does not
of interestthat is, the lifetime of stars require a cosmological redshift. He
is far too short for the sky to be as argues that Lord Kelvin (1901) was
bright as the paradox suggests. In addi- the first to give the correct answer,
tion, if all the matter in the universe which Edgar Allan Poe had antici-
were converted to radiation, the equi- pated in his qualitative cosmological
librium temperature of the universe speculations. For the detailed calcula-
would be about 20 K, illustrating that tions, see the references below.
there is insufcient energy to have a Harrison, E. R. Why the Sky Is Dark at
Night. Physics Today 27, no. 2 (1974):
bright sky. Edward R. Harrison in the
3036.
early 1970s determined this solution
. The Dark Night-Sky Riddle.
and also determined that the usual Science 226 (1984): 941945.
explanation, based on a cosmological Pesic, P. Brightness at Night. American
redshift of the light from distant Journal of Physics 66 (1998): 10131015.

Answers 257
203. Headlight Effect also are the different clock rates for
the two frames of reference, so the
In the special theory of relativity (STR) number of photons emitted per second
a result called the headlight effect on the star and received at Earth will
occurs. One considers the Lorentz- differ. Moreover, the spectrum of light
Fitzgerald contraction of distances in will be different as well.
the direction parallel to the constant Taylor, E. F., and J. A. Wheeler. Spacetime
velocity and no change in the perpen- Physics. San Francisco: W. H. Freeman,
dicular direction. If the primed frame 1966, p. 69.
is the vehicle frame, then the angle in
the two frames are related by cos =
(cos + v/c)/(1 + v/c cos ). Substi- 204. Incommunicado?
tuting the appropriate values tells us No and yes! No, in the normal sense
that cos ~ 1, or ~ 0! Therefore, all case because the relative velocity can
the light is in a very small solid angle never exceed the speed of light. The
in the forward direction, and only an successive pulses may arrive less and
observer directly along the line of less often, but you will never outrun
motion will see the light. You will not the light.
see the light from the relativistic vehi- And yes, you would lose commu-
cle passing nearby unless your eye is nication contact if we allow the space
within the very narrow light cone. itself to expand, analogous to the
In the rest frame of the source, the expansion of the universe in present
star emits light in all directions, yet the cosmological models. The addition of
calculation reveals that for an observer velocities is the old classical physics
of a very fast-approaching star, practi- one, not the relativistic one. The pho-
cally all its light will be shining along ton velocity is affected by the local
the direction of motion! A fast- environment; the local substratum
approaching star or galaxy will pos- (i.e., coordinate system) drags the
sess a very narrow bright headlight photon along. Imagine two local
beam that could miss Earth. Mean- regions in rapid recession from each
while, a fast-receding star or galaxy other. If the person in one region res
may not be seen at all because its light a photon toward the other, the sub-
is redshifted out of the visible range stratum of the rst region drags the
and practically all its light shines away photon along, slowing the photons
from us! progress toward its target. If the
So in observing stars, there is this expansion rate is high enough, the two
STR headlight effect to consider. There regions can be receding from each

258 Answers
other at light speed or greater, pre- four test particles inside their spaceship
venting communication between you and having the capability to measure
and your friend. their relative accelerations, they can
Higbie, J. Radial Photon Paths in a Cosmic succeed in determining all the compo-
Model: A Student Exercise. American nents of the Riemann tensor and
Journal of Physics 51 (1983): 11021107.
decide whether they are in the space-
time curvature of a rotating central
205. Local mass. Note that gyroscopes do not
Accelerations help here because one would need to
check their alignment with stars out-
The presence of the massive body can side, which is forbidden. The challenge
be determined by the trajectories of the here is to measure a new effect, called
two test masses upon their release. In intrinsic gravitomagnetism, introduced
the simple case in which the laboratory by the GTR, that the space-time geom-
is not moving with respect to the mas- etry and the corresponding curvature
sive body, when released equidistant invariants are affected and determined
from the object but separated from by both mass-energy and mass-energy
each other, the two test masses will currents relative to other massthat
move toward each other faster than is, by mass-energy currents that cannot
their mutual gravitational acceleration be eliminated by a Lorentz transforma-
as they fall toward the body. In addi- tion. See the Ciufolini and Wheeler ref-
tion, if they are separated vertically so erence below for the details.
that one test mass begins closer to the Ciufolini, I., and J. A. Wheeler, Gravitation
massive body than the other, their ver- and Inertia. Princeton, N.J.: Princeton Uni-
tical separation distance will change as versity Press, 1995, pp. 358360.
Kalotas, T. M., A. R. Lee., and R. B. Miller.
they fall. In a uniform gravitational
Einstein on Safari. The Physics Teacher
field their separation distance would 29 (1991): 122124.
remain xed in value in each test. Martin, J. L. General Relativity: A Guide to
One can extend this problem to Its Consequences for Gravity and Cosmol-
ogy. Chichester, Eng.: John Wiley & Sons,
consider a rotating massive body. Can Ellis Horwood, 1988, pp. 9394.
observers inside a spaceship determine
by local measurements onlythat
is, without looking outsideif they are
206. Twin Paradox
in the eld of a rotating central mass, Both special theory of relativity (STR)
or if they are just moving with velocity and general theory of relativity
V on a Schwarzschild background (GTR) explanations for the aging of
metric? Yes they can; by using at least the space-traveling twin should be

Answers 259
considered. If by the STR we consider spaceship can be approximated by an
inertial reference frames only and equivalent gravitational acceleration
ignore any accelerations experienced that is, using the Equivalence Princi-
by the space traveler, a symmetry plewe can expect the traveling clock
would exist between the two frames, to tick slower during the acceleration.
and the twins must both age at the And that behavior is why the traveling
same rate. Therefore, the accelerations twin ages less. As seen by a third
experienced by the space traveler observer at rest with respect to the
make the difference in the aging. stars and the stay-at-home twin, the
One can handle these accelerations clock on the spaceship is changing its
in the STR or in the GTR. Some peo- rate of ticking during the accelerations.
ple argue that this twin paradox prob-
lem requires only the STR because
there is no curved space-time in the
207. Twin Watches
problemthat is, both twins can be A watch ticks at its fastest rate when at
considered to be in a at space-time rest and when there is no gravitational
because no gravitational accelerations eld. So there are two effects to con-
near a mass are necessary. Then one sider: (1) from the special theory of
would handle the accelerations for the relativity (STR), the motion of the
space-traveler twin in terms of STR watch with respect to the laboratory
calculations, perhaps via the velocity frame affects the ticking rate; and (2)
parameter technique. A true GTR the change in gravitational potential
problem, by contrast, would require according to the general theory of rel-
the physics of the curved metrics near ativity (GTR) affects the clock ticking
a massive body. rate. For a watch in free fall, the two
The solution of the twin paradox effects are exactly opposite and cancel!
using the GTR relies on clocks ticking The two watches agree again when she
slower in a gravitational potential near takes the second reading.
a mass. The clock at a far distance Now for the details. First, are there
from the mass ticks at its fastest rate any symmetry considerations that
and, if brought closer to the mass, would simplify the calculation? Yes;
begins to tick slower and slower. the two parts of the journey for the
Therefore a person closer to the mas- moving watchthe upward and the
sive body, where the gravitational downward partsare time reections
acceleration is greater, ages slower. of each other, and these two parts
In cases where the acceleration of a require the same elapsed time in the

260 Answers
laboratory frame and in the moving- From the GTR, the time interval T
watch frame. between clock ticks at radial distance
Pick the laboratory frame of refer- R outside of a body of mass M is given
ence. As the watch goes upward in the by T = T (1 2GM / ( Rc 2 )). In the
lab frame at its maximum velocity ini- limit of very large R, the clock ticks
tially, the STR makes the watch tick at its fastest rate. By definition, g =
faster as the velocity decreases, and the GM/R2 at the surface of Earth. Substi-
GTR makes the watch tick faster as tute the above heights for the two dis-
greater height is achieved. On the tances from the massive body and take
downward journey, the watch ticks the difference. One calculates that T2 ~
slower and slower by both STR and T1 T g h/c2, a quantity propor-
GTR effects. So we need only calculate tional to the change in height and a
the changes in the tick rate when the quantity from the GTR that changes
watch has gone upward by a small as fast as the quantity from the STR.
amounth, say. So the total change in the tick rate
From the STR, the time interval T going upward is canceled by the total
between ticks at velocity v is given by T change in the tick rate coming down-
= T / (1 v 2 / c 2 ) , where T is the ward, to make no net change when
time interval between ticks of the they are once again at the same height.
watch in its own reference frame. At If this argument has any aws, do not
two different heightsh1 and h2 = h1 + blame either of my colleagues, Richard
hthe time intervals between ticks P. Feynman (deceased) or B. Winstein,
are T1 = T / (1 v12 / c 2 ) and T2 = for they know not what they had
T / (1 v 22 / c 2 ) ,respectively, because wrought!
the velocities will be different at the
two heights. Since v << c, and assum- 208. Global Positioning
ing a uniform acceleration approxima-
tion for free fall, by the third golden Satellites
rule of kinematics, v22 = v12 2 g h. The general theory of relativity (GTR)
Substitute the velocities squared into plays an important role! Corrections
the watchs time interval relations and must be made for clock rates in a grav-
expand the square roots in the denom- itational eld in addition to the special
inators by the Taylor series expansion theory of relativity (STR) corrections
1/ (1 ) ~ 1 + / 2 + . . . . One calcu- to the clocks for the movement of the
lates T2 ~ T1 T g h/c2, a quantity satellite. Both relativistic effects foul
proportional to the change in height. up what should have been a pretty

Answers 261
simple geometry calculation relating correction factor of about 4.8 1010
distance to time and velocity. The for this GTR effect, a little more than
clocks in the satellites tick at a slightly four times the STR effect, or about
faster rate than identical clocks on the 14.4 centimeters of error every second.
ground because they are in a slightly So in 10 minutes even this small effect
weaker gravitational eld, being far- produces an error of about 86 meters
ther from the center of Earth. They if not accounted for. Who would have
tick slower than the Earth-bound thought that both STR and GTR
clocks because they are moving faster effects are big enough to play impor-
with respect to the stars. tant roles in such a useful practical sys-
We can estimate the sizes of these tem as GPS!
effects. From the STR, the time inter-
val T between the ticks of a clock
moving at velocity v is given by T =
209. Solar Redshift
T / (1 v 2 / c 2 ) , where T is the time Even though there may be no relative
interval between ticks of the clock in radial motion between the Sun and the
its own reference frame. At slow observer on Earth, there is still a grav-
speeds v << c, one expands the square itational redshift dictated by the gen-
root by the Taylor series expansion eral theory of relativity (GTR). Recall
1/ (1 ) ~ 1 + / 2 + . . . to obtain T that the innitesimal distance ds in a
~ T (1+v2/(2c2)). For satellites orbiting at Euclidean space with coordinates
Earth in about 720 minutes, their (r, , ) is dened by ds2 = c2 dt2 dr2
speed makes the time correction factor r2 d2 r2 sin2 d2. In the gravita-
about 1.1 1010. Multiplied by the tional eld of mass M, this innitesi-
speed of light, this time factor corre- mal distance in the GTR becomes the
sponds to a distance error of about 3.3 Schwarzschild line element ds2 = (1
centimeters for each second. rg /r) c2 dt2 (1 rg /r)1 dr2 r2 d2 r2
From the GTR, the time interval T sin2 d2, where rg = 2GM/c2 and G is
between clock ticks at radial distance the gravitational constant.
R outside of a body of mass M is given We see that near a massive body
by T = T (1 2GM / ( Rc 2 )) . Con- such as the Sun, the time coordinate
sider two different radii: r1 = 6.37 includes a factor (1 rg / r ) , where r
106 m and r2 = 2.02 107 m. Substi- is the position of the light measured
tute the two radii for the two distances from the center of the Sun. By evaluat-
from the massive body and take the ing this factor at the Suns surface and
difference. One calculates a clock at Earths distance, one nds that the

262 Answers
clocks at the two distances are ticking precession of about 43 seconds of arc
at different rates, faster for bigger r. per Earth century. There are many
One approach is to assume that the other precessional effects acting on the
photon does not change its inherent orbit, including effects from all the
physical propertiesfor example, it other planets orbiting the Sun, all
maintains its characteristic frequency these perturbations amounting to a
established during the emission whopping 532 seconds of arc per cen-
process at the surface of the Sun. Then tury, all but the residual 43 seconds
the observer on Earth, who has the being explained with Newtonian
faster-ticking reference clock with mechanics.
respect to the stars, will measure a When the angular change around
lower photon frequency and see the the orbit is calculated with GTR in the
light as redshifted. -coordinate and then independently
A second approach determines in the r-coordinate, there is disagree-
that the change in r for the photon can ment, which is the conceptual source
be shown to correspond to a change in of the effect. Or one can assign an
gravitational potential. The photon additional equivalent mass distribu-
essentially begins in a gravitational tion for the energy in the gravitational
potential energy valley and climbs eld surrounding the Sun, creating a
upward to reach Earth. Its total energy metric that does not correspond to a
must remain constant, so the increase 1/r potential, perhaps 1/r2 or 1/r3, or
in gravitational potential energy is some other function of r instead of
matched by the decrease in photon the Newtonian inverse r potential. All
energythat is, a redshifted photon these functions of r will exhibit a pre-
because its energy E = h. cession of the orbit.
Krane, K. S. Modern Physics. New York: In addition, a body in orbit such
John Wiley & Sons, 1983, pp. 438442. as a planet orbiting the Sun actually
does not obey Keplers third law pre-
cisely. That is, even when we ignore
210. Orbiting Bodies the precession of the orbit by assuming
The general theory of relativity (GTR) its return to the same angle with
in the Schwarzschild metric approxi- respect to the stars, the period of orbit
mation for the space-time metric needs correction. This period of orbit
about the Sun predicts a precession of correction is a so-called fourth inde-
the planetary orbit. Mercury, for pendent general test of the GTR, in
example, accumulates a total GTR addition to the gravitational redshift,

Answers 263
the deection of starlight, and the pre- metric surrounding a star, for exam-
cession of an orbit. ple, the path of the light from a distant
The correction to the period of star is diverted toward the Sun from a
orbit, as calculated by Preston and straight-line path when passing near
Weber in the reference below, begins a massive body such as our Sun. Half
by putting the reference clock at the of the deflection angle is caused by
center of the orbit. The classical New- the Newtonian attraction of the Sun;
tonian period of orbit is given by the second half is caused by the
Keplers third law: T = 2 a3/2/
(GM) , geometrical modication, called cur-
where a is the semimajor axis of the vature, of space by the Sun. Gravita-
ellipse and M is the mass value of the tional lensing ideas have been around
central body. For the elliptical orbit of for about 200 years, but only in the
eccentricity , in the GTR one can cal- past decade or so has gravitational
culate the period of orbit in the radial lensing played an important part in
coordinate Tr = T (1/ + 3/2 rg/r) and astronomical measurements.
in the -coordinate T = T (1/ 3/2 If the intermediate massive body is
(rg /r) (2/)), where = (1 2)3/2. For a galaxy, then light from distance
orbiting bodies near potential black sources behind this galaxy will be
holes, this correction can get large as focused somewhat by the two effects,
the radial distance r approaches the
Schwarzschild radius rg = 2GM/c2.
Krane, K. S. Modern Physics. New York: Hubble space telescope
John Wiley & Sons, 1983, pp. 438442.
lensing image
Landau, L. D., and E. M. Lifschitz. The Clas-
sical Theory of Fields, 4th ed. Sydney: But-
terworth-Heinemann, 1987, pp. 328330.
Preston, H. G., and J. Weber, The Period of
Orbit as a Test of General Relativity.
Physics Essays 6 (1993): 465.

211. Gravitational
Lensing
The general theory of relativity (GTR)
tells us that all forms of energy are
affected by a gravitational field,
including the energy carried by pho-
tons of light. In the Schwarzschild

264 Answers
just like the light going through a glass Wambsganss, J. Gravitys Kaleidoscope.
Scientific American 285, no. 5 (2001):
lens is refracted to a focus. However, 6471.
the geometry of the light focusing is
much more complicated for gravita-
tional lensing than for a simple sym- 212. Cosmological
metrical convex lens, for several
Redshifts
reasons. The light may be focused
onto a line instead of a point, for There are three distinct causes for the
example, in an ideal case. Therefore, spectral shift of light emitted (or
astronomers can use galaxies as lenses absorbed) by a galaxy: the kinematical
to gather more light from far objects Doppler shift of the special theory of
better. The focus may be poor, but the relativity (STR), the gravitational red-
greater intensity allows many spectro- shift of the general theory of relativity
scopic techniques to work better. (GTR), and the cosmological redshift
Usually the image resolution of caused by the expansion of the uni-
the distant object is quite limited by verse. These three effects cannot be
inhomogeneities in the intermediate distinguished from one another by
galaxy. However, these properties of observing the spectrum of a single
the intermediate galaxy can be exam- galaxy or other single light source.
ined quite well! In fact, if the classical One can separate out the kinematical
application of the GTR to calculate the Doppler shift for a cluster of galaxies,
focusing effects in gravitational lensing however, via statistical methods.
is correct, then the total mass of the The standard explanation of the
intermediate galaxy and its mass distri- cosmological redshift says that the
bution can be determined. Galaxy coordinate system of the universe is
masses determined with gravitational expanding while the galaxies remain
lensing have disagreed with the very at their local coordinate values. One
successful proposal for a modified can use an expanding balloon to
Newtonian dynamics (MOND) within mimic this type of behavior. Inate
a galaxy region and have supported the a balloon enough to enable you to
dark matter models. However, the draw a coordinate system on its sur-
game is not over for MOND because face. Place some galaxies on the bal-
large-scale gravitational quantization, loon surface. Now inate the balloon
either from a version of M-theory and further. The galaxies are farther apart,
superstrings or from some other quan- but they maintain the same coordinate
tization scheme, may come to the res- values. Or one can use Earth. If Earth
cue eventually. began to expand, Philadelphia and Los

Answers 265
Angeles would move apart, yet each 213. Tired-Light
would retain its present longitude and
latitude. Distances are stretched, so
Hypothesis
wavelengths will become longer. The only two specific pieces of evi-
Perhaps another viewpoint will be dence are the time dilation arising
helpful. One does not need to expand from the expansion of the universe,
space in this view. According to S. and the spectral shape of the cosmic
Weinberg in the Chown reference microwave background. Astronomers
below, simply accept the fact that see that exploding stars in distant
every bit of the universe is rushing galaxies brighten and fade more
away from every other bitthat is, slowly than those nearby. If the star
the galaxies are exploding away from emits a light pulse on January 1 and a
each other, as any cloud of particles second pulse on February 1, these two
would do if they are set in motion pulses are separated by one light-
away from each other. The matter month. As they travel toward Earth,
inside the individual galaxies does not their separation distance increases,
take part in the general expansion perhaps doubling, so that they are
because the local gravity holds the received two months apart. The tired-
local matter together. The universes light hypothesis cannot explain this
expansion appears just beyond the extended time interval. In fact, distant
frontiers of the Local Group of galax- supernovas are observed to wax and
ies, about 4 million light-years from wane more slowly than nearby ones.
the Local Groups center of mass. The observed spectrum of the
M. L. Bedran in the reference listed microwave background radiation is a
below compares the Doppler redshift perfect blackbody shape, easily
to the cosmological redshift for a explained by the expansion of the uni-
galaxy with z = 1, where 1 + z = exp verse from a thermodynamic equilib-
[v/c], determining that a 2.4 percent rium condition. For the tired-light
difference between the two redshifts hypothesis, an initial blackbody spec-
exists for this galaxy with an STR trum does not remain a blackbody
value of v/c = 0.6. spectrum as the light becomes red-
Bedran, M. L. A Comparison between the shifted.
Doppler and Cosmological Redshifts. Croswell, K. The Universe at Midnight:
American Journal of Physics 70 (2002): Observations Illuminating the Cosmos.
406408. New York: Free Press, 2001, p. 76.
Chown, M. All You Ever Wanted to Know
about the Big Bang. New Scientist (April
17, 1993): 3233.

266 Answers
Hawking, S. W. A Brief History of Time:
214. Black Hole From the Big Bang to Black Holes. New
Entropy York: Bantam Books, 1988, pp. 104110.
Penrose, R., The Emperors New Mind.
There should be radiation from the Oxford: Oxford University Press, 1989,
black holethat is, from the sur- pp. 361363.
rounding space, not from inside the
black hole, because nothing can get 215. Black Hole
out. This Hawking radiation was rst
calculated in the 1970s and awaits
Collision
experimental verication. Yes. The two black holes should
By taking quantum mechanics into coalesce like two liquid drops. We
account, particles and antiparticles are need to ensure that the entropy after-
being created continually via virtual ward in the coalesced final state is
pair creation in the vacuum. When this greater than the entropy in the initial
process occurs near a black hole, one state. We do that by adding up the
particle of the pair may be eaten by entropy in the two states, separate
the black hole and the other may black holes versus one larger black
escape. In the thermal equilibrium hole with gravitational waves carrying
state, the amount of energy that the away some energy and entropy in the
black hole loses to Hawking radiation nal state.
is exactly balanced by the energy The black hole entropy is propor-
gained by swallowing other thermal tional to the event horizon area, which
particles that happen to be running grows as the mass to the fourth power.
around in the thermal bath in which Suppose we take two black holes, one
the black hole nds itself. of mass M1 and the other of mass M2.
The temperature of a nonrotating Their initial total entropy is propor-
black hole is given by T = hc3/(8kGM), tional to M 14 + M 24. If they merge and
where h is Plancks constant and k is their nal total mass is approximately
Boltzmanns constant. Note that this M1 + M2, then their nal total entropy
expression connects gravitation, ther- is proportional to (M1 + M2)4, which
modynamics, and quantum mechanics. you can verify is greater than the orig-
For black holes of a few solar masses, inal total entropy, so the reaction will
the temperature is only about 106 K! go. In cases where the nal entropy is
The smaller black holes with little mass only slightly greater than the initial
will be at a much higher temperature, entropy for the black hole parts only,
contrary to intuition. one may need to add in the entropy in

Answers 267
the gravitational waves to ensure a so that the axis of the tube and the
bigger inequality path of the ray coincide. Measure-
Thus a bigger mass means a bigger ments with a straight ruler verify that
surface area, which means a bigger the axis of the tube is circular, yet
entropy than if the two smaller black because of the bending of the light
holes remained apart. Simulations in rays, the tube is seen as absolutely
3-D of colliding black holes and their straight by an observer on the axis. A
emitted gravitational waves can be lamp held at the axis by a colleague
seen on the Internet. will appear dimmer to you as he or she
Bekenstein, J. D. Black-Hole Thermo- walks away along the axis, but the
dynamics. Physics Today 33, no. 1 (1980): lamp is never obscured, so you must
2431.
conclude that the tube is straight.
Therefore, along this circular path one
would expect no centrifugal force
216. Centrifugal Force
effects.
Paradox Instead of the tube being around
The conceptual resolution of this par- the circular light path, consider the
adox starts with the consideration of tube to be around a smaller circular
light paths near the black hole. The path centered on the black hole. With
general theory of relativity (GTR) pre- rulers one can again verify that the
dicts that there should be light paths tube curves to the left, with the black
around the black hole that are circular hole on the left as one walks forward.
at a radial distance of 1.5 times the The outward direction is to the right.
gravitational radius rg = 2GM/c2. Everyday experience tells us that the
Around one of these circular light centrifugal force pushes outward.
paths imagine a circular tube centered Again your colleague walks away with
exactly on the path of the circular ray the lamp held along the axis of the
tube. If somehow the light rays were
not bent by the gravitational eld of
Black hole Light ray the black hole, you would see the lamp
disappear behind the left side of the
tube, and you would conclude that
the tube bends to the left. If the path is
the one discussed above, the lamp is
always in view. But the tube is so close
to the black hole that the light rays
Tube bend even more than circular rays. So

268 Answers
you actually see the lamp disappear to In conventional geometry, the
the right. Therefore the outward direc- geodesic is the shortest curve between
tion is to the left, and you must predict two points measured by counting how
that the centrifugal force would push many rulers t along the curve. In a
to the left! at spacethat is, in a space free from
Several related paradoxes for paths gravitational eldsthe geodesic is a
near a massive body are discussed in straight line. In the GTR one can
the references, including the fact that dene the distance between two points
for a rocket to maintain a constant in space as half the time it takes for
speed the boosters would need to re light to travel from one point to the
perpendicularly to the path, but their other and back, multiplied by the
force is not dependent on how fast the speed of light. In at space, the two
rocket is moving. denitions agree.
Abramowicz, M. A. Black Holes and the In 4-D space-time, light always
Centrifugal Force Paradox. Scientific moves along geodesics and traces the
American 268, no. 3 (1993): 7481.
geometry of space-time. In a 3-D space
Abramowicz, M. A., and J. P. Lasota. On
Traveling Round without Feeling It and
warped by a gravitational eld, how-
Uncurving Curves. American Journal of ever, the light rays are curved and do
Physics 54 (1986): 936939. not coincide with geodesics in general,
so the geometry of space is not traced
by light rays.
217. Geodesics and Abramowicz, M. A. Black Holes and the
Centrifugal Force Paradox. Scientific
Light Rays American 268, no. 3 (1993):7481.
Abramowicz, M. A., and J. P. Lasota. On
The two statements are not in conict.
Traveling Round without Feeling It and
One must always distinguish geodesics Uncurving Curves. American Journal of
in four-dimensional space-time from Physics 54 (1986): 936939.
geodesics in three-dimensional space. Misner, C. W., K. S. Thorne, and J. A.
Wheeler. Gravitation. San Francisco: W. H.
Light rays always follow geodesics in
Freeman, 1973, pp. 3134.
4-D space-time, but these paths are
not necessarily geodesics in 3-D space.
An analogy is helpful. Each great circle
on a globe is a geodesic line on the
218. Galaxy Rotation
two-dimensional surface but, being a To retain a Newtonian gravitation
circle, the great circle is not a geodesic explanation for the rotation properties
line in the 3-D Euclidean space in of a galaxy, additional matter, called
which the globe sits. dark matter, in a halo surrounding

Answers 269
the galaxy has been proposed, its mass Briefly, the large-scale structure and
being about 10 times the mass of the behavior of the galaxy may result from
visible mass! Known particles such as the galaxy being in some quantization
electrons, protons, neutrinos and so state. In modeling this type of theory,
on cannot be its major constituents; all the disk stars would be in the same
otherwise the particles in the halo quantization state independent of posi-
would have been detected already. tion radially and, by the virial theorem,
Some exotic form of matter/energy is must have the same tangential velocity
required in this galactic halo. How- V = GM2/(nJ), where M is the amount
ever, any type or types of possible of visible mass, n is a small integer, and
dark matter in sufcient quantities J is the total angular momentum of this
has not yet been found. visible mass of the galaxy. Substituting
An interesting proposal that does reasonable values for our Galaxy (the
not require dark matter is called Milky Way), for example, one obtains
MOND, an acronym for modified a value near to the measured value of
Newtonian dynamics, which applies V = 220 km s1. This theory predicts
whenever the inward radial accelera- that the next quantization state would
tion in the galaxy is below the value a0 have exactly half the disk tangential
= 1.1 1010 m s2, an incredibly velocity and, indeed, in 2003 a mass
small amount by Earth standards but a current of stars circulating the Galaxy
value that occurs in most galaxies. just beyond the edge of the easily vis-
Essentially, MOND replaces the New- ible disk with a tangential velocity of
tonian acceleration gN by g =
(gN a0) . 110 km s1 was determined serendipi-
All galaxies examined so far seem to tously from data collected by the Sloan
obey the consequences of this ad hoc Digital Sky Survey (SDSS)! Whether
rule by using the galactic visible matter this proposed large-scale quantization
only, but its possible origin in terms of faithfully represents gravitational
fundamental physics principles is behavior in galaxies and in the uni-
being investigated still. The major verse remains to be fully examined.
problem for MOND has been its Cline, D. B. The Search for Dark Matter.
inability to accommodate the empiri- Scientific American 288, no. 3 (2003):
5059.
cal results on the focusing of distant
starlight by gravitational lensing. Milgrom, M. Does Dark Matter Really
Exist? Scientific American 287, no. 2
An even more exotic solution has (2002): 4252.
been proposed to explain galaxy rota- Preston, H. G., and F. Potter. Exploring
tion without requiring dark matter. Large-Scale Gravitational Quantization

270 Answers
without h-bar in Planetary Systems, Galax- high temperature now exhibits a low-
ies, and the Universe. E-print archive for
physics papers. http://lanl.arxiv.org/abs/
temperature blackbody spectrum
gr-qc/0303112 (2003). because the expansion of the universe
has stretched the wavelengths.
The stars we see are not in thermal
219. Cosmic equilibrium as a collective whole. One
cannot produce a perfect blackbody
Background Radiation spectrum at any temperature by sim-
Amazingly, cosmic background radia- ply using billions of stars that are not
tion (CBR) has a perfect blackbody in thermal equilibrium as a whole and
distribution! This CBR is uniform and adding up their radiation intensities in
isotropic in the universe to one part in the universe. One cannot obtain a
100,000 and amazingly at over large blackbody spectrum from many other
spatial regionsthat is, large solid hypotheses about the cosmological
angles. One suspects that these large redshift of light from distant objects,
spatial regions, even those in opposite such as the tired-light effect wherein
directions in the sky, have always the light loses energy during tranversal
been in communication with each of the universe.
other, to make them so uniform. Of One could, however, speculate that
course, smaller regions have their indi- the galaxies have never changed their
vidual characteristic galaxies, clusters average separations, that there has
of galaxies, and so on. been no coordinate expansion as
The most popular interpretation, stated in the standard inflationary
the standard inationary model of the model. The cosmological redshifts
universe, requires the universe to orig- would correspond to the redshift pro-
inate with the far regions much closer duced by an effective cosmological
to each other in thermal equilibrium gravitational potential wellfor
for the observed uniformity to develop, example, in which the source sits
then for a very fast ination to occur lower in the well than the observer,
that separated them out of communi- true for all sources and all observers in
cation reach. Now we see these galax- the universe. The galaxies would be
ies, once close together, in opposite closer together at all epochs, in com-
directions in the universe and large munication with each other, and in
regions with the same large-scale char- thermal equilibrium, so the measured
acteristics in all directions. Their origi- uniformity would be expectedthat
nal collective blackbody spectrum at a is, all directions should look the same.

Answers 271
One consequence would be that one proposed by Nottale, including many
could never see galaxies beyond about sets with larger integers.
12 billion light-years or so, the actual Extrasolar planetary systems with
distance value depending on the aver- three planets have been found, but
age matter/energy density of the their statistical ts allow several sets of
vacuum. The redshifts would be inter- integers also, so they are not yet the
preted as effective recession veloci- denitive test. Unfortunately, we must
ties that reach light speed at this far wait for a denite extrasolar system or
distance. a precise laboratory test to resolve
Hasinger, G., and R. Gilli. The Cosmic the issue of whether the patterns are
Reality Check. Scientific American 286, simply numerology or an exhibit of
no. 3 (2002): 6067.
part of a new fundamental gravita-
Peebles, P. J. E. Making Sense of Modern
Cosmology. Scientic American 284, no. 1 tional theory.
(2001): 5455. Lynch, P. On the Signicance of the Titius-
Bode Law for the Distribution of the
Planets. Monthly Notice of the Royal
220. Planetary Astronomical Society 341 (2003):
11741178.
Spacings Nottale, L. Scale-Relativity and Quantiza-
tion of Extra-Solar Planetary Systems.
The orbital radii of the planets only Astronomy & Astrophysics 315 (1996):
roughly follow the Titius-Bode law, so L9L12.
this specic pattern is probably bogus. Nottale, L., G. Schumacher, and J. Gay.
However, L. Nottale and his research Scale Relativity and Quantization of the
Solar System. Astronomy & Astrophysics
group have shown that the planets 322 (1997): 10181022.
obey a generalized Schrdinger-like
wave equation (with one unknown
parameter) that has solutions dictating 221. Entropy in the
a regular pattern for where orbiting Big Bang
bodies reach an equilibrium radius.
We quote from the reference listed.
The planets of the Solar System occupy
The standard answer attempting to
only these radial positions and leave
explain the paradox is:
some equilibrium radii unoccupied,
perhaps a consequence of their history True, the fireball was effectively
of formation. However, even though in thermal equilibrium at the
Nottales ts are extremely good, there beginning, but the universe at that
are several other sets of small integers time was very tiny. The fireball
that statistically t as well as the set represented the state of maximum

272 Answers
entropy that could be permitted for several kilometers or more. One could
a universe of that tiny size, but the place the rotating laboratory gravita-
entropy so permitted would have tional wave source several kilometers
been minute by comparison with away or more from the gravitational
that which is allowed for a universe wave detector, but the decrease in the
of the size that we find it to be radiation eld intensity with distance
today. As the universe expanded, squared combined with the low sensi-
the permitted maximum entropy tivity of the detectors make this
increased with the universes size, arrangement unlikely to work with
but the actual entropy in the uni- present detectors. Therefore, as far as
verse lagged well behind this per- we know, there has never been a true
mitted maximum. The second law test of the gravitational wave response
arises because the actual entropy is of any detector to gravitational radia-
always striving to catch up with tion using laboratory sources of grav-
this permitted maximum. ity waves.
There have been two fundamental
This answer cannot be correct if the
types of gravitational wave detectors:
universe will eventually suffer a big
the Weber bar antenna, named after
crunch, for then the argument would
pioneering physicist Joseph Weber,
apply again in the reverse direction!
who began this research field in the
We are at an impasse.
1950s with his one meter diameter
Penrose, R. The Emperors New Mind.
Oxford: Oxford University Press, 1989, suspended aluminum bar; and the
pp. 317330. interferometer type such as LIGO, rst
analyzed by the same Joseph Weber
222. Gravitational and his students. The classical calcula-
tion of the resonant response of the
Wave Detectors Weber bar reveals just how limited is
For all kinds of waves, for locations its sensitivity to gravitational waves
beyond several wavelengths, the solu- originating in our Solar System and
tions of the wave equation correspond Galaxy. However, if the Weber bar
to the radiation field transporting antenna actually behaves differently
energy and momentum from the than originally expected, as a collec-
source into the surrounding space. tive quantum oscillator responding
When considering possible sources of coherently, say, then it will respond
gravitational waves in the Galaxy and well to all frequencies of the incident
beyond, the wavelengths are typically gravitational waves. Hundreds to

Answers 273
thousands of vibrational modes could exceed the cubic curve for the three
be excited in a large range of frequen- types of spaces: positively curved, at,
cies with an increase in sensitivity of or negatively curved, respectively.
many powers of ten. On a small scale, when the total
At this time there has been no sub- number of sources is less than a few
stantiated detection of gravitational hundred, there can be a relatively large
waves by either type of detector. uncertainty in the general behavior of
Weber reported a twice-daily response the plotted curve. But as more and
of his two side-by-side almost-identi- more sources are counted at farther
cal bar antennas for orientations distances, the asymptotic behavior
pointing toward the center of the should become apparent. However,
Galaxy during a period of almost two adjustments must be made for the
decades, but no other researcher has nite velocity of light and for possible
verified this behavior with an inde- evolutionary changes in the sources.
pendent detector. So we must wait for Distant sources are sampled at an ear-
the first detection of gravitational lier time, possibly at a closer distance.
waves by LIGO or other detectors. If the universe is actually represen-
Unfortunately, interferometer types of tative of a discrete space, one can show
detectors such as LIGO and VIRGO that by counting many sources one can
cannot operate as a collective quan- determine the curvature in the limit as
tum oscillator. the number of sources becomes large.
Think of a lattice of points as one sim-
ple example, such as a regular lattice
223. Space Curvature of atoms in a solid. By counting the
The proposed method for determining nearest neighbors only, then the next
the curvature of space will work for nearest neighbors, and so on, one even-
both continuous and discrete spaces. tually approaches asymptotically to a
If we assume a uniform density of plotted line from which the curvature
stars, or galaxies if we choose to count can be determined.
galaxies, the number N of this particu- Of course, in a discrete space, one
lar kind of source within a sphere of must be careful not to count over and
radius R in a Euclidean space (zero over the images of the same source.
curvature space) is given by N = For example, imagine space divided
4R3/3, where is the uniform den- into identical cubes next to each other
sity. When N is plotted against the dis- and filling all space. Standing inside
tance, N will fall short, match, or one cube, we can look to our right to

274 Answers
see ourselves inside the rst cube on Wolfram, S. A New Kind of Science. Cham-
paign, Ill.: Wolfram Media, 2002, pp.
our right looking away to the next 433540.
cube, and so on. Each successive image
will be dimmer and will be earlier in
time because the light does not travel 224. The Total Energy
innitely fast. If our real space in the Yes, there can be the creation of mat-
universe is discrete, the cube size ter out of nothing with no violation of
would be enormous, certainly way conservation laws! First proposed in
beyond the size of our Local Group of 1958 by H. Margenau and later recal-
galaxies; otherwise we would have culated in more detail by N. Rosen
detected this discreteness already by and others in 1994, the gravitational
having seen multiple images of our energy cancels the mass energy in a
own Galaxy. closed, homogeneous universe.
If space in the universe is curved, The simplest general approach was
then cubes will not fill the space. done by Margenau. Consider a nite
Mathematicians point out that one of spherical universe of radius R filled
the dodecahedral spaces would be the with matter and radiation of equiva-
simplest space-lling for a negatively lent total mass M. The gravitational
curved discrete space, the most likely potential energy is the negative quan-
type of space curvature for the uni- tity kGM2/R, where G is the gravita-
verse. However, the curvature of the tional constant and k is a positive
universe is not known unambiguously numerical factor not greatly different
yet, although a at space with no cur- from 1. The total energy E in the uni-
vature will nicely t the present data in verse is then E = Mc2 kGM2/R. Using
the standard model of an inationary representative values such as R = 1.3
big bang universe. 1026 m and a mass density of 8 1027
Eckroth, C. A. Counting Distant Radio kg/m3, we estimate k ~ 2.4 when E =
Sources to Determine the Overall Curva- 0. Nathan Rosen and others showed
ture of Space. The Physics Teacher 30
(1992): 9293. that the gravitational energy cancels
Gruber, R. P., A. D. Gruber, R. Hamilton, out the mass energy without resorting
and S. M. Matthews. Space Curvature and to numerical estimates.
the Heavy Banana Paradox. The Physics Cooperstock, F. I., and M. Israelit. The
Teacher 29 (1991): 147149. Energy of the Universe. Foundations of
Levin, J. How the Universe Got Its Spots: Physics 25 (1995): 631635.
Diary of a Finite Time in a Finite Space. Jammer, M. Einstein and Religion: Physics
Princeton, N.J.: Princeton University Press, and Theology. Princeton, N.J.: Princeton
2003, pp. 132155. University Press, 1999, pp. 201203.

Answers 275
Margenau, H. Thomas and the Physics of will be overwhelmed by many other
1958: A Confrontation. Aquinas Lecture
23. Milwaukee: Marquette University Press,
particle decays into the same final
1958, p. 41. products. Perhaps when the Large
Rosen, N. The Energy of the Universe. Hadron Collider is turned on in a few
General Relativity and Gravitation 26 years, with its very high rate of pro-
(1994): 319321.
duction of quarks, the statistics will be
so much better that the b quark
should be easy to nd.
225. Different If the b quark is found, then we
Universes? expect that all other fundamental
physical constants should also be
If the lepton and quark masses are dic- derivable from mathematical invari-
tated by fundamental mathematical ants. If the proposed scheme is correct,
quantities, we are behooved to con- then our universe is the only universe
sider that all fundamental quantities in possible. Even exotic speculations such
nature have an origin in fundamental as time travel could be eliminated if
mathematics. There can be no alterna- the direction of time is one of the
tive universes, each supposedly having innate properties of the particle state
different fundamental constants, for denition. However, we must remem-
they must have the same mathematics ber always that Nature is more clever
dictating the same physical constants. than we can hope to be, so we must
In 1994 F. Potter, within the con- continue to test every reasonable pro-
nes of the Standard Model of Lep- posal for the truth.
tons and Quarks, related the lepton Linde, A. The Self-Reproducing Ination-
and quark mass ratios to a mathemat- ary Universe. Scientic American 279, no.
ical invariant called the elliptic modu- 11 (1994): 4855.
lar invariant J, which is invariant Potter, F. Geometrical Basis for the Stan-
dard Model. International Journal of The-
under all linear transformations. The oretical Physics 33 (1994): 279306.
critical prediction is a fourth quark Tegmark, M. Parallel Universes. Scientic
family with a b quark mass of about American 288, no. 5 (2003): 4051.
80 GeV/c2 and a t quark mass of
about 2,600 GeV/c2. Although
searches for a b quark have been
under way at the Fermilab collider for
several years, its existence cannot be
ruled out yet because the decay reac-
tions have very low probability and

276 Answers
Chapter 11
Crystal Blue
Persuasion

226. Iodine Prophylaxis


The iodine tablets, usually potassium information, but a little mathematics
iodine, work by topping up the thy- reveals the answer.
roid gland with stable iodine to reduce Note that the rear wheel of a bike
its accumulation of any radioactive always points to the place where the
iodine that may have been released front wheel touches the ground.
into the environment by a nuclear Therefore, the tangent to the rear
accident. Inhalation of the radioactive wheel track will always cross the front
iodine in the air is the major route of wheel track, while the front wheel
entry into the body and the thyroid. track does not exhibit this geometrical
For maximum benefit, the iodine property.
tablets should be taken before the Once we identify the rear track, we
radioactivity fallout reaches your area; can pick two random points on it and
otherwise the tablets themselves will extend the tangents to where they
have become radioactive. cross the front track in both direc-
tions, measure the segments, and
determine which direction yields seg-
227. Bicycle Tracks ments of the same length. Since a bicy-
In The Adventure of the Priory cle cannot change its length, we learn
School, Sherlock Holmes not only the direction of travel.
draws a map of the neighborhood of
the school but also examines several
sets of tyre tracks on the moor. He
228. Earth Warming
needed to determine the bicycles Yes, there can be uctuations in the
direction of travel solely from his thermal energy conducted and con-
inspection of the tracks. Holmes could vected to Earths surface from sources
tell from the depth of the wheel within. There are several types of
impression which track was made by thermal energy sources, including
the rear tyre. You dont have that radioactive nuclei emitting particles

Answers 277
that transfer their kinetic energy to wires in the process of making contact.
thermal energy, as well as friction Held near a radio, the small spark
between internal rock flows, which across the gap can be heard through
could create local hot spots and/or the radio, indicating that many fre-
temporary changes in the ow proper- quencies are being emitted. As an
ties of liquid rock or the thermal con- additional demonstration, one can
ductivity of the rocks. Therefore, small even move a radio near a small electric
changes in the rates of thermal energy motor of the kind that has brushes to
transport to Earths surface are possi- hear its rotation frequency because the
ble and most likely happen continu- brushes make and break contact each
ally. Are these internal sources the revolution.
culprits in the present slow average Of course, if one desires to have a
temperature increase? These fluctua- higher-current spark gap, a car battery
tions are expected to be too small, but or a transformer can be used with
no one knows for sure. proper safety precautions to provide a
healthy current that can be operated in
an intermittent mode or in a continu-
229. Frequency ous mode. Nearby radios, televisions,
Jamming and so on will be affected by this
larger-current spark gap device. Even
GPS transmissions between 1,000
MHz and 2,000 MHz may be affected,
so some care must be taken not to vio-
late federal transmission limitations.

A spark gap usually is a wonderful 230. Light Energy


noisy source of electromagnetic waves A light source emitting light of fre-
at all frequencies simultaneously. The quency f approaching an observer at
greater the current across the gap, the constant velocity v will appear blue-
more intense will be the total radia- shifted by an amount corresponding
tion emitted at each frequency. There to the relativistic Doppler effect for-
will be a distribution of intensity ver- mula f = f [(1 v 2 / c 2 )] / (1 v / c )
sus frequency that can be tuned a because the clock ticking rates will be
little by adjusting the gap spacing. different for the source reference frame
A simple spark gap would be a and the observer reference frame, and
small battery, such as a D cell, and two their separation distance is decreasing.

278 Answers
When v << c, we can expand the for- is dened to have a pH of less than
mula in a Taylor series to obtain f ~ f 5.0, a condition that tends to occur
(1 + v/c v2/2c2 + . . . ), so the leading more often in industrialized areas of
term in powers of v/c is positive, cor- the world than in remote regions.
responding to the blueshift. We Human activities can increase the
assume that an acceleration itself does amount of CO2 in the air, but so do
not produce an additional fundamen- many natural resources, such as volca-
tal frequency shift, although there will noes, lightning strikes, cows, bacteria,
be acceleration effects because the and res. When industrial and auto-
source is changing instantaneous mobile exhausts release sulfur com-
comoving inertial frames. pounds and nitrogen compounds,
For a photon, its energy is E = hf these molecules combine with oxygen
and its momentum is p = E/c, so both to form sulfuric acid and nitric acid,
energy and momentum are different in which can harm ecosystems, historic
different reference frames because the monuments and buildings, and the
observed frequencies are different. health of people around the world.
Notice that the recoil of the source on The reduction of sulfur and nitrogen
emission of light and of the observer compounds released into the air has
on detection are not accounted for in become a worldwide concern.
the discussion and that the energy and Trefil, J. The Nature of Science. Boston:
momentum input necessary to keep Houghton Mifin, 2003, pp. 67.
the relative velocity of the source and
observer fixed must be considered
232. Electrical Current
also. Of course, energy and momen-
tum conservation laws are obeyed in The electrons in house wiring move at
this example. a snails pace, with an average drift
velocity of about a millimeter per sec-
ond. These electrons, which are free to
231. Acid Rain move in the metal wires, are distrib-
Not so! The falling raindrops will not uted throughout, so when the switch is
remain neutral at pH 7. Pure rainwa- closed to make a complete circuit, they
ter falling through unpolluted air is an move en masse, sort of like water pass-
acid, with a pH of about 5.6, because ing through a continuous hose that
as the drops form and fall they dis- closes on itself. The electron velocity is
solve carbon dioxide in the air and limited because its negative electric
react to produce carbonic acid, charge interacts with the lattice of pos-
H2CO3. Ofcially, therefore, acid rain itive ions during the movement.

Answers 279
In addition to having a drift veloc- 234. Crystal Growth
ity, the electrons experience a random
sequence of pinball-like collisions to The speed and precision of the crystal
change their speeds and directions, growth depend on many factors,
essentially behaving as a free electron including the temperature, concentra-
gas. Consequently, the host metal tion, and purity of the solution.
gains some thermal energy, and its Assuming the ideal solution, each
temperature rises. If a lamp is the additional atom to be added from the
incandescent type, the tungsten alloy solution must rst nd a location on
filament will gain enough energy by the growing surface of the developing
this process to dramatically increase its crystal. But these atoms in solution are
temperature to a new equilibrium tem- randomly moving about, making ran-
perature of about 2000 K to glow in dom collisions with the crystal at ran-
the visible and the infrared. dom locations on the surface. How
can they build a perfect single crystal?
Their little secret is that some
233. Earths Orbit atoms that have been added at mar-
Although the general theory of relativ- ginal locations, say, can escape from
ity dictates a precession of the perihe- these surface locations to allow other
lion for all planets, including Earth, atoms from the solution to nd a bet-
this effect is very small compared to ter location nearby, better here
perturbations provided by gravitational meaning to be held electrostatically
inuences of all the planets. Apparently tighter to the crystal. But these better
Earths elliptical orbit will pass through locations do not occur in chronologi-
a repetitive cycling about every 93,000 cal order because they are determined
years from its present ellipse and orien- by the collective inuence of numerous
tation with respect to the stars, to a cir- atoms already in the crystal, and the
cle, then back to an ellipse with an best position one microsecond ago
orientation perpendicular to the pres- may not be the best position for an
ent orientation, to a circle again, back atom now. Therefore, the addition and
to an ellipse perpendicular again, etc., subtraction of atoms from the growing
until the present elliptical orientation is crystal surface proceeds almost by trial
recovered approximately. Of course, all and error! Consequently, one cannot
the planets are experiencing these write down an algorithm for placing
perturbation effects simultaneously, so atoms from the solution onto the
the detailed calculations become quite growing crystal.
interesting! When the crystal grows slowly,

280 Answers
there is plenty of time for the sampling Sapphire is aluminum oxide with
process to proceed to fruition, and the trace impurities of iron and titanium
crystals tend to form with fewer dislo- atoms, which are responsible for the
cations and inclusions. When the crys- deep blue color shades most people
tal grows rapidly, errors in the crystal associate with sapphire. Several other
structure become trapped, and these colors of corundum, such as yellow,
crystals tend to have many disloca- reddish-orange, and violet, also are
tions and inclusions. classified as sapphire. Synthetic sap-
phires have been produced commer-
cially since 1902 and are used for
235. Ruby, Sapphire, scratch-resistant watch crystals, opti-
cal scanners, and in applications where
and Emerald physical strength and transparency to
How are ruby, sapphire, and emerald ultraviolet irradiation are important.
crystals related? Ruby and sapphire Emeralds are quite different from
are color variations of the same min- rubies and sapphires in that emeralds
eral, corundum. Rubies contain a are the green form of beryl, colored by
small amount of chromium. Pure the presence of chromium or vana-
corundum is a colorless, trigonal crys- dium. The crystal structure of beryl
tal that occurs in a wide variety of col- emeralds is hexagonal (six-sided), with
ors due to infiltrations of other a hardness slightly higher than quartz
elements. All color variations of but considerably less than diamond.
corundum, with the exception of ruby, Emeralds are notorious for containing
are called sapphires. aws, and awless stones are rare and
Rubies are red variations of the greatly valued.
mineral corundum, a crystalline form The colors in these gemstones are
of aluminum oxide and one of the produced by characteristic downward
most durable minerals that exists. atomic transitions involving F-centers
Only diamonds are harder. Rubies (from the German word farbe, mean-
rich, red colors arise from the substi- ing color) at the chromium or other
tution of a small number of aluminum atoms. In a simple view of an F-center,
atoms by chromium atoms. When the ambient light excites one electron
exposed to high temperatures, rubies in the chromium atom, for example,
turn green, but they regain their origi- so the atom can be treated analogous
nal color after cooling. Some rubies to the hydrogen atom, with a large
phosphoresce with a vivid red glow average radius for the location of the
when illuminated by ultraviolet light. electron away from the chromium

Answers 281
nucleus. The electron transition back Applying Keplers third law for a
to a lower energy state causes the particle of mass between Earths mass
emission of a photon in the visible. m and the Suns mass M orbiting with
Nassau, K. The Causes of Color. Scientic Earths period T, one obtains after sev-
American 243, no. 4 (1980): 124154. eral steps GM/(rR)3 Gm/(R2(rR))
Perkowitz, S. True Colors: Why Things = GM/r3 where r is the EarthSun dis-
Look the Way They Do. The Sciences
(MayJune 1991): 2228. tance and R is the Earthparticle dis-
tance. The particle at L1 will be about
0.01 times the distance to the Sun. The
236. Kordylewski L3 point on the night side of Earth can
Clouds be calculated in the same way, replac-
ing rR with r+R. However, the other
Joseph L. Lagrange in the late 1700s
three points must be calculated with
calculated via Newtons laws that
the gravitational attractions of the
there are five special positions for
other planets included.
objects bound by any two-body
Similar calculations have been
system. Now called Lagrange points,
done for the ve Lagrange points for
positions L1, L2, and L3 are unstable,
the Earth-Moon system. Polish
while L4 and L5 are stable. Several
astronomer K. Kordylewski in 1961
spacecraft have been placed at or
reported the observation of dust clouds
near these Lagrange points, and
at the L5 point, but some observers
there have been proposals for
have not seen them. Particles here may
building space colonies at the L4 or L5
not remain long before being ejected,
positions.
according to calculations.
L4 Kordylewski, K. Photographische Unter-
suchungen des Librationspunktes Lim5
System Erde-Mond. Acta Astronomica 11
(1961): 165169.
ONeill, G. K. The Colonization of Space.
Physics Today 27, no. 9 (1974): 32.
L1 L2 L3

Moon 237. Twist Scooter


Earth
If the plane of the V arms of the twist
bike remained horizontal at all times,
there would be no forward motion
except by pushing with a foot on the
L5
ground. By tilting the vertical shaft of

282 Answers
the handlebar sideways about 10 phase with the photons from the right
degrees or so, the front of the scooter side. In other words, the photons are
is lowered a bit, and a slight unbal- not expected to be in phase. Their
anced outward push of the arms of the phase difference depends on three
V by the riders legs provides a for- parameters: their initial phase differ-
ward net force similar to the resulting ence, the distance to the star, and the
net force exercised by an ice skater. diameter of the star. By slowly chang-
The initial forward movement from ing the separation of the two photode-
rest may be difficult on an upslope tectors on the arms of an intensity
exceeding a particular angle deter- interferometer, one can sweep across a
mined by the size of the wheels and by range of phase differences to determine
the possible tip angle of the handlebar. the diameter of the source. One labo-
ratory analogy might be considering
how one would determine the spacing
238. Unruh Radiation between the slits of a two-slit interfer-
The Equivalence Principle tells us that ence experiment with a similar appara-
a particle accelerating in the vacuum is tus. Ultimately, the amplitudes and not
equivalent to a particle at rest in a really the intensities interfere. How-
uniform gravitational eld. If there is ever, the phase correlations depend on
radiation in one case, there must be the product of intensities, in contrast
radiation in the other equivalent case. to the two-slit interference example.
The former is called Bekenstein radia- The original experiment is known
tion, the latter Unruh radiation, as the Brown-Twiss experiment,
named after physicists who studied the named after the two researchers who
properties of the radiation theoreti- rst succeeded in using the technique
cally. No one has ever measured this to determine a star diameter back in
radiation because its intensity is many 1957. Interference associated with the
powers of 10 too faint to be detected. superposition of separate light intensi-
ties was viewed with considerable
skepticism. Apparently, as the story
239. Star Diameters goes, one of the original researchers
One can determine the interference was giving a physics talk at Caltech
diameter of a distant star even when soon after their rst measurements. In
optical parallax resolution of its diam- those days, several Nobel physicists
eter is impossible by utilizing quantum would sit in the front row along with
interference between the photons from Richard Feynman and other prominent
the left side of the star arriving out of physicists. About 10 minutes into the

Answers 283
talk, Feynman walked out, much to the stimulate the emission of a second
dismay of the speaker. About 40 min- photon from a nearby atom, and the
utes later, just near the end of the talk, two can stimulate the emission of a
Feynman walked back in and sat down third photon, and so on. In principle,
in his seat again. The speaker then the photons arriving at the receptor
asked why he walked out and then can be single, double, triple, and so on,
returned. Feynman responded that he the actual photon state depending on
had walked out because he did not how many stimulated photons were
believe that the physics was correct. He picked up before escaping the light
explained that he had gone back to his source. The receptor receives a differ-
office and worked out the problem, ent energy burst with each absorption.
only to discover that the physics had Since the probability for stimulated
been done correctly. He then returned emission into the same final state is
to acknowledge the cleverness of the proportional to the number of pho-
speaker and his colleague. Now the tons in that state already, these
speaker was in dismay again, amazed multiple photon processes occur quite
that someone could have worked out readily.
the many details in so short a time! Real light sources such as incan-
Brown, R. H., and Twiss, R. Q. A New descent bulbs have huge physical
Type of Interferometer for Use in Radioas- dimensions compared to the wave-
tronomy. Philosophical Magazine 45
(1954): 663. length of light. There will be numerous
Silverman, M. P. A Universe of Atoms: An ideal chaotic sources along the fila-
Atom in the Universe. New York: Springer- ment wire simultaneously and ran-
Verlag, 2002, pp. 102126.
domly emitting photons toward the
detector. These photons tend to arrive
in bunches, with the photons within
240. Glauber Effect any one bunch coming from several
Yes, a standard incandescent lightbulb places in the source. Very seldom does
does emit single photons, and some- one nd a steady stream of photons
times there are photon pairs, and with nearly equal time spacing arriv-
triplets, and so on. In the ideal chaotic ing from the lightbulb when one looks
photon sourcea hot, incandescent on the nanosecond time scale.
wire that has physical dimensions Loudon, R. The Quantum Theory of Light,
smaller than a wavelength of the 3rd ed. Oxford: Oxford University Press,
2000, chap. 6.
emitted light, for examplethe first
spontaneously emitted photon can

284 Answers
241. Bird Sounds 243. Hair-Raiser
Some birds can emit just a fundamen- Function
tal frequency with no harmonics. Just For the HRF of a non-integer, one
how the bird eliminates the harmonics needs to write down a few more exam-
originally generated within is being ples of the given integer description.
investigated. The present conjecture is Then take the logarithm of each exam-
that a cavity resonance amplies just ple to discover that they all can be
the fundamental before the sound is expressed as log N = nn1 log n. By tak-
emitted. If the fundamental frequency ing the exponential of both sides with
changes, then the cavity must change the proper grouping, the nal expres-
to accommodate the new fundamental sion becomes N = (n)^(nn1)that is, n
in live time. to the power (nn1). With HRF(x) =
(x)^(xx1), the HRF of non-integer val-
ues for x becomes an easy calculation
242. Spouting with the appropriate calculator, one
Alligator capable of many decimal places. What
is the limit as n approaches zero?
To eject water droplets upward, the
Complex numbers can be used, as well
alligator head vibrations must provide
as irrationals such as .
the initial energy to create nearly
A plot of the HRF using integers
standing waves in the shallow water
shows a remarkably steep rise for even
on the back of its head. As wave crests
small integers; hence its name! You
become larger, droplets of water break
might want to compare its rise to an
off and are projected high above the
exponential function. And if all you
surface. One can simulate this effect
desire is an approximate value for the
by sliding a styrofoam cup lled with
inverse or for the HRF of a non-integer,
water across a nished wooden sur-
the plot provides a visual image and a
face at about 10 centimeters per sec-
means to satisfy your curiosity.
ond. Water droplets will shoot upward
However, as far as we know, the
to about 20 centimeters.
inverse HRF is awkward, and no easy
Jargodzki, C., and F. Potter. Spouting Water
Droplets. In Mad about Physics: Brain- calculation algorithm is known. We
twisters, Paradoxes, and Curiosities. New dont even know whether the inverse
York: John Wiley & Sons, 2001, p. 39. can be expressed as the limit of a
series! One can determine the inverse
by successive approximation to any

Answers 285
number of decimal places with the required. However, during operation
appropriate calculator. the system does not require fuel or
Of what use is the hair-raiser func- other mass be expelled into the envi-
tion? The question reminds us of two ronment to move in space. The sys-
classic quotes from Michael Faraday tem is designed to operate in two
when he was attempting to explain a operational modes: in Mode I the
discovery to the visiting prime minis- system incrementally moves the pay-
ter. He was asked: But, after all, what load platform forward with each
use is it? To which Faraday replied, operational cycle. In this rst mode,
Why, sir, there is the probability that the velocity imparted to the payload
you will soon be able to tax it. And platform is not additive. In Mode II
when the prime minister asked of a the payload platform accelerates for-
new discovery, What good is it?, ward a discrete increment of velocity
Faraday replied, What good is a new- during each operational cycle. In this
born baby? second mode the increments of
velocity are additive.

There is no problem with energy


244. Space Crawler conservation because the onboard bat-
tery supplies the energy. The inventor
The U.S. Patent Ofce awarded patent
Virgil Laul claims that this propulsion
5966986 in 1999 to this propulsion
device when attached to spacecraft
device. We quote the patent abstract:
will be able to propel spacecraft out in
A propulsion system which is space. We leave this problem as a nal
designed to be used on a payload challenge. What is the physics here?
platform such as a spacecraft, satel- Are any conservation laws violated?
lite, aircraft, or an ocean vessel. To Will the device work in space as well
operate the system electrical power is as it does on the air table?

286 Answers
Index
absorption conservation of, 55 Rutherford, E., 73
atmospheric, 216 electron in atom, 67 axis of rotation, 24, 31
cancellation, 212 of ywheels, 54
academic quantization, 67 b quark, 276
disputes, 44 quantum, total, 67 Babylonians, 13, 20
types, 118 annihilation, 87, 97,171 ball, bouncing, 87
acceleration Anthropic Principle, 96 baseball bat, 53
angular, 144 antiparticles, 87, 255 BBs, 37
car driver, 146 Apollonius, 21 Be-8 synthesis, 95
collision time, 165 archaeology, 53 beans, parboiling, 6
free fall, 30 Archimedes beets, peeling, 7
local, 107 gravestone, 22 Bekenstein radiation, 283
movie of, 17 principle, 3 Bell inequalities, 87
pseudo-force, 168 Aristotle, 50 benzene
rocket ship, 107 arm contortions, 25 carbon atoms, 69
in sandglass, 144 asteroid, 36 energy levels, 69
skiers, record, 167 astronauts, 123 beta decay, 244, 245
time reversal of, 16 astronomers, 30 bicycle tracks, 118
twin paradox, 108 atmosphere big bang
uniform, 44 absorption by, 216 entropy, 114
acid rain, 119 carbon-14 in, 93 Hoyle, F., 114
action-at-a-distance cosmic rays, 124 inationary, 41, 112
Boscovich, R. J., 66 expansion of, 35 microwaves, 99
Riemann, G. B., 111 greenhouse, 70, 71 billiards table, 20
air ozone layer, 70 bird sounds, 122
boundary layer, 197 refraction by, 140, 152 birth dates, 15
can, air pressure, 7 UV, 103 black hole
density, 169 wings in, 31 collisions, 110
engine, 50 atom entropy, 110
pressure, 125, 126, 132 absorption by, 135 forces, radial, 111
soup can, 129 Bohr, N., 68 and information, 89
in straw, 128 Bose-Einstein, 74 time symmetry, 150
viscosity, 144 carbon, 69, 70 blackbody spectrum,
algebra, 103 clusters, 55, 59 cosmos, 112, 266, 271
alien beings, 31 emission by, 66, 67 plasma, 218
alligator, spouting, 122 quantum dots, 74 blueberry mufns, 4
alpha particle, 96 X-ray laser, 73, 222 body cushion, 30
amino acid triplet, 149 atomic model Bohr, N.
ammonium maser, 68 Bohr, N., 67 atom, 68
angular momentum Nagaoka, H., 73 completeness, 83

287
Bohr, N. (continued) carbon collision
double-slit, 232 cycle, in stars, 245 asteroid, 36
habits, 70 nuclear levels, 95 body cushion, 30
quantization, 67 synthesis in stars, 95 bullet, 30
religion, 65 carbon-14 dating, 93 molecules, 167
boiling point, 134 carbon dioxide spaceships, 46
boiling water air amount, 279 wall, in cartoons, 30
altitude effects, 5 in bread, 126 color
beets in, 7 greenhouse gas, 70, 71 cadmium, 59
in ice, 8 moderator, 98 F-centers, 281
salt added to, 7 plants, 93 nanophase, 59
watched pot, 8 in water, 9 ruby, 120
Boltzmann, L., 44 carbonic acid, 279 communication
book rotation, 25 Carnot cycle black hole, 106
Bose-Einstein ferrouid, 52 delays, 175
condensate, 74, 208 photon, 61 jamming, 118
Cooper pairs, 84 quantum, 61 spaceships, 43, 106
He-4, 85 cartoons, 30, 35 computer
statistics, 177 Casimir effect, 84 atomic, 73
bosons, 85, 177 cellular automata, 104 DNA, 221
Bragg scattering, 74, 79 centrifugal force, 111 java quantum, 221
brain CFC, 216 nuclear spins, 221
connections, 23 chaotic systems quantum, 79
magnetic eld, 85 competition, 57 concrete, 239
power needs, 54 hot wire, 284 conductivity
bread kneading, 3 electrical, 200, 220
identical, 57
Brownian motor, 50 thermal, 130, 131, 132
Chernobyl, 179
bubble collapse, 218 conics, 21
Chinese cooking, 6
bullet consciousness, 86
chlorophyll, 135
reworks, 34 convection, 132
circadian rhythm, 149
impact, 30 Cooper pairs, 84
classical mechanics, 51
Bushmen, 172 copper
clocks,
butter, measuring, 3 cladding, 175
atomic, 15 nanophase, 55
cadmium selenide, 59 eternal, 15 oxide tunneling, 238
caffeine, 7, 79 identical, 43 X-ray laser, 73
calendar light, 16 Coriolis, G., 55
Gregorian, 142 molecular, 17 cosmic rays, 93, 124, 221
Julian, 142 coal burning, 98 Coulomb
lunar, 14, 143 coffee, 7, 79 barrier, 244, 245, 248
Mayan, 52 coherent blockade, 56
rice planting, 14 light scattering, 227 Crab Nebula, 182
calories scattering, 80, 81, 176 creative thinkers, 85
from fat, 5 X-rays, 227 crystal structure
human needs, 4 coin tosses Bragg scattering, 79
campre, igniting, 134 random walk, 50 diamond, 70
can, pressure in, 7 randomness, 50 graphite, 70
car driver, 15, 42 cold fusion, 98 growth, 120

288 Index
lattice, 120, 248 fractional, 24 height, 69
quantum dots, 74 space, 21, 22, 24 letter to Mileva, 43
crystalline, 55, 198 space-time, 27 light bending, 106
cubes dimples in bat, 54 mass-energy, 41
array, 21 dipole-dipole, 135 paper in 1905, 46
space-lling, 22 dispersion, 59 photon emission, 68
symmetry, 24, 160 distance, 20 reality, 114
cup, rotating, 27 DNA relativity 1905, 27
cuprates, 55 clock, 17 space-time, 89
Curie, M., 93, 95, 102 computer, 221 electrical
Curie, P., 93, 95, 102 dodecahedron, 22, 24, 153 circuit, 58
Curie temperature, 102, 199 Doppler shift, 65 conductor, 200
cylinder, 22 double-slit, 83, 213, 232 current, house, 119
driving, switching, 15 diode, pickle, 137
dark matter, 269 electrical charge
dates, 9 Earth fractional, 200
day length atmosphere, 35 on pinhead, 56
Egyptians, 12 climate, 140 transferred, 200
hours in, 12 magnetic eld, 241 tunnel junction, 56
sidereal, 37 Moon, seen from, 77 electron
solar, 37 orbit change, 119 in a box, nding, 78
tidal effects, 57 rotations in year, 13 Cooper pairs, in, 84
winter, 61 speed in orbit, 12 density, metals, 130
de Broglie , 67, 222, 234 temperature, 178, 216 house current, 119
decay warming, 118 single, tunneling, 200
competitive, 57 ecliptic, 14 spin, vacuum effect, 84
neutron, 95 eddy Standard Model, 101
nuclear, 86 current, 199 elements
decoherence, 224 turbulent, 197 pun, 122
defrosting tray, 5 efciency synthesis of, 94, 95, 123
demonstrations, 34 light bulb, 209 ellipse, 21, 119
density, cream, 127 photon engine, 61 elliptic functions, 116, 276
Descartes, R., 57 thermal, 52, 61 emerald, 120
detector egg and bottle, 2 energy
calibration, 114 Egypt Earth warming, 118
gravitational, 80, 114 day length, 12 eld, charge, 47
neutrino, 81 pyramids, 54 kinetic, 51, 55
determinism, 88 Einstein, A. mass-energy, 40, 42
deterministic Beibltter, 44 rotational, 99
behavior, 57 Bible study, 107 in universe, 115
chaos, 57 completeness, 83 energy levels
detonation, 37 Curie letter, 104 benzene, 69
deuteron, 96 deductions by, 45 C-12 nucleus, 95
diamond, 70, 220 determinism, 88 hydrogen, 67
digital timer, 15 dice quote, 100 nuclear, 93
dimensions emission, 208 quantum dot, 223
dened, 159 EPR paradox, 87 engine
fractal, 159 God, 100, 110 air-driven, 50

Index 289
engine (continued) nuclear shell, 93 geodesics, 111
Brownian, 50 ssion glass
magnetocaloric, 52 Pu-239, 246 absorption in, 135
photon, 61 U-235, 99, 245 old window, 54
entropy U-238, 246 Glauber effect, 122
asymmetry in, 18 Fitzgerald, G. F., 96 gluons, 255
big bang, 114 uid glycemic index, 9
black hole, 110 binary, 52 gold density, 128
disorder, 58 ferrouid, 52 GPS signals, 108
nal state, 239 inmiscible, 52 grain
heat ow, 58 magnetorheological, 52 boundaries, 200
liquids, 196 uorescence sand, atoms in, 65
eponymy, 24 lights, 66 size, 55, 65
EPR paradox, 87 quantum dots, 74 graphite, 70
equations, time in, 18 ywheels, coupled, 54 grasshoppers, 57
equinoxes, 12, 14 FM radio frequency, 69 gravitational
eternal clock, 15 force bound system, 113
evaporation, 9 aperiodic, 150 clock rate, 60, 108, 260
explosions in space 33 applied, 44, 51, 55 detectors, 80, 114
exponential growth, 57 buoyant, 170 lensing, 109
centrifugal, 111 tidal effects, 205
faces, resolution of, 80 eld, in movies, 36 twins, 60
fame, 59 uctuating, 51 waves, 80
Faraday, M. geometry, 111 gravitomagnetism, 259
ux law, 202 torque, 144 gravity
quotes, 286 van der Waals, 135 articial, 31
farmer forensics, paint, 65 in galaxies, 112
goose chase, 23 4-D, 24, 27 geodesics, 111
rice, planting, 14 four-momentum, 40 geometry, 111
faster than c free energy, 196, 202 greenhouse effect
spotlight, 41 free fall gases, 70, 71, 216
quasar, 41 acceleration, 30 water vapor, 216
wave function, 225 body cushion, 30 Groundhog Day, 14
Fe-56 stability, 94 cartoons, in, 30 group velocity, 213
Fermi, E. Galileo, 59 growth, 57, 120
energy, 220 Philoponus, J., 50 gunght, 30
exclusion, 242 twin watches, 108
Golden Rule, 87 friction, static, 128 hair-raiser function, 123
Fermi-Dirac, statistics, Friday 13th, 79 Hall effect, quantized, 72
237 fullerene, 215 halogen lamp, 138
fermions, 177, 237 hamburgers, 131
ferromagnetism, 54, 102 g-2, muon, 236 hardness, metals, 55
Feynman, R. P. Galaxy center, 274 Hawking radiation, 267
Brown-Twiss, 283 galaxies, 110, 111, 258, 265 He-4
ratchet, 50 Galileo, G., 59, 142 burning in stars, 95
twin watches, 108 galvanometer, 58 superuid, 85
eld theory gamma rays, 250, 251 synthesis in stars, 96
Boscovich, J., 66 Gamow, G., 109 headlight effect, 106

290 Index
heat bath, 61 icosahedron, 22, 25, 160 leap years, 13
heat engine ignition LED, 71
air, 50 oxygen, 133 lens, 66
Brownian, 50 spontaneous, 133 Lenzs law, 199
photon, 61 inertia, 56 Lewis, G., 71, 83
Heisenberg, W. information Libby, W., 93, 100
matrix model, 84 black hole, and, 89 life, 120, 122, 133
on reality, 78 phase, 214, 256 light
uncertainty principle, 83 quantum, 89 ambient, 97
helium burning, 95 speed of, 182 approaching, 106
helium liquid, 72, 85 storage, 73 bending of, 106, 109
Hubble telescope, 264 infrared bulb, 122, 209
human images, 35, 80 and car driver, 42
aging, 45, 47, 60 resolution, 176 ceramic response, 59
brains, 23, 54, 85 insulation, fat, 10 clock, 16
collision damage, 166 integrated circuits, 73 cosmic background, 112
DNA, 17 interference efciency, sources, 209
energy needs, 4 double-slit, 83 faster than, 32, 41, 182
faces, 35, 80 intensity, 121 ash, 15, 33, 34, 71, 173
health in UV, 67 laser, 213 forensics, paint, 65
hearing, 100 quantum, 75 geodesics, 111
height, 69, 123 star light, 121 infrared, 80
memory, 156 Internet, game play, 34 moving source, 119
radioactivity, 97 invariants, 40, 41, 51 oven, 10
SAD, 17 iodine, 118, 179 sabers, 36
shrunken size, 32 iron lings, 52 scattering, 59, 80, 174, 177
space debris, 32 signals, 43
space journey, 45, 46 Josephson effect, 85 squeezing, 84
UV need, 210 therapy, 17
walking, 59 Keplers 3rd law, 264 tired-light, 110
Huxley, A., 119 Kerr effect, 59 tweezer, 66
Huygens, C., 51 kinetic energy UV, 80
hydrogen discovery, 51, 55 light tweezer, 66
atom, 67, 164 vis viva, 55 lightning, 33
bonds, 126, 196 Kordylewski cloud, 120
LIGO, 229
burning, star, 97
Lincoln, A., 32
MRI, 82 Lagrange points, 282
lipstick color, 59
nuclear device, 37 Lamb shift, 84
liquid helium siphon, 72
in space, 174 Landau levels, 220
locality violation, 240
stability of atom, 27 Larmor, J., 41
Lorentz, H.,
laser
contraction, 42
ice beams in space, 34
bond angles, 8 kinetic, 68 relativity, 41
oating, 6, 127 noninversion, 68
uorocarbons, 216 security system, 34 magnetic
in microwave, 8 strobe, 146 dipole, 198
polar melt, 33 Weber, J., 68 magnetic
in ponds, 133 X-ray, 73, 212 eld, 52, 72, 85
ice cream, 5 LCD, 71 induction stove, 10

Index 291
magnetic (continued) meteor, 120 MSG, 2
resonance, 82 meteorite in 1908, 100 muon, 101, 236, 248
Mars methane, 70, 71
debris on, 116 metronomes, two, 18 Nagaoka, H., 73
moons, 30 Michelson, A., 42 nanophase cluster
Martians, 33 microtubules, 156 cadmium, 59
maser, 68 microwaves copper, 55
mass quadrupole, 80 absorption, 135 energy levels, 203
mass-energy background, 99, 266 neurons, 23, 95
confusion, 42 heating water, 3 neutrino
creation of, 115 ice absorption, 8 detectors, 81, 231
denition, 40, 41 metals in, 6 handedness, 103
electron, 47 oven, 6, 10 mass, 45, 103
particles, system of, 42 milk and cream, 3 proton cycle, 97
Poincar, H., 45, 47 Milky Way, 112, 113, 274 scattering, coherent, 81
proton, 103 Miller, J. S., 2 solar, 81
material Minkowski, H., 27 supernova 1987A, 173
ceramic, 59 minute, origin, 13 Standard Model, 101
dispersive, 59 mirror types, 67, 211 neutron
empty space in, 68 miscible liquids, 52 capture, 246, 249
glass ow, 197 modular function J, 276 decay of, 95
hardness, 55 molecular Newton, I.
magnetic, 54 clock, 17 birth date of, 142
nanophase Cu, 55 displacement, 176, 177 comets, 110
smart, 203 scattering IR, 177 rstborns, 122
stretching of, 35 momentum, 57, 164, 170 height, 69
mathematicians, 118, 121 MOND, 265, 270 mass-energy, 42
Matthew effect, 59 Moon Principia, 23
Maxwells equations calendar, 14 quip about, 54
failure of, 208 in daylight, 142 Newtons Laws
invariance of, 41 distance, 20, 206 2nd law, 125, 128, 132
symmetry of, 102 Easter, 143 3rd law, 170
measurement full, 13 torques, 144
Bohr, 232 size of, 25 Ni isotopes, 94
Moon distance, 20 tidal effects, 57 night sky, 106
quantum, 78, 83 Mssbauer Effect nitrogen, liquid, 5
simultaneous, 191 eardrum, 251 NMR, 226
uncertainty principle, 83 gamma rays, 208 noise
meat Weber bar, 230 jamming with, 118
Chinese style, 6 movies robotic, helpful, 60
cooking roast, 5 battle sounds, 33 shot, electrical, 84
preserving, 5 body cushion, 30 north, nding, 18
Meissner effect, 55 gunght, 30 nuclear
memory, 17 sounds, 33 beta decay, 244, 245
men, 4, 10 spaceships, 32 binding energy, 242
metals warp speed, 32 decay, 86
density of, 128 western, 165 device, 36, 37, 94, 180
thermal conductivity, 130 MRI, 82 emergency, 118

292 Index
energy levels, 93, 95 random, 82, 227 positronium, 87
ssion, 99 space, 239 pot, watched, 8
gamma rays, 250 uncertainty, 235 potassium-40, 98
reactor, 97, 98, 179 velocity, 213 potato, 4, 9
spin ip. 226 philosophical, 74 Potter, F., 160, 253, 256, 276
spin-orbit, 242 phosphorous, 175 power plant, coal, 98
submarine, 36 photon precession, 109, 119, 139
synthesis, 94, 95 absorption, 65 preserving food, 5
waste, 98 bunching, 122 prism, 87
nucleus emission, 65, 67 probability
large, 99 engine, 61 amplitude, 75
models, history of, 73 scattering, 78, 177 quantum, 75, 76
neutron in, 95 UV, 66, 210 reaction rate, 87
rotation of, 250 photosensitivity, 143 procrastinating, 13
shell model, 93 photostrictive effect, 203 projectile impact, 30
stable, 94, 99 physicist, 18, 20, 121, 165 protein, 148
physics proton, 103, 161, 255
octahedron, 22, 24, 160 equationless, 103 psychology, 165
Oklo nuclear reactor, 97 mathematics, 114, 116 Pu-239, 99, 180
Olbers paradox, 106 pi, 22, 23, 26 pyramids, 54
optical pickle, electric, 9 Pythagoras, 20
cavity, 61 pinhead, charge on, 56 Pythagorean
color, lipstick, 59 Planck, M., 115 theorem, 24
solitons, 59 planet orbits triplets, 20
orbits, 27, 109, 119 Aryabhata, 99
oscillators, 150 extrasolar, 272 quadrupole
oscilloscope, 182 Kordylewski, 120 electric, 207
osmosis, 129 precession, 109 gravitational, 80
ozone layer, 70 spacings, 113 quantum
stability, 64, 108, 168 Carnot cycle, 61
paintings, 65 plasma coherence, 79, 205
paramagnetism, 54 bubble, 218 computer, 79
parboil, 6 display, 71 dots, uorescence of, 74
Pauli, W., 80 ions, 66 engine, 205
Pb bullets, 175 Plato, 24, 116 entanglement, 233
Penrose, R., 114, 151, 155 Platonic solids information, 89
Penrose tiles, 153 dened, 24 interference, 75, 232
perceptions, 119 importance of, 160 noise, 235
perihelion, 12 playing card, 77 state, collective, 231, 273
perpetuum mobile, 51, 124 plutonium, 36, 123 vacuum, 84
pH, 119 Poincar, H., 45, 47 quantum mechanics
phase polar molecules, 8 amplitude , 75
conjugate, 67 polytopes, 160 angular momentum, 67
diagram, 52 pond freezing, 133 bosons, 85, 177
xed, 177, 226 population bouncing ball, 87
information in, 256 growth, 57 decoherence, 224
local change in, 256 inversion, 68, 212 development of, 82
locking, 18 noninversion, 68 Einstein & Bohr, 100

Index 293
quantum mechanics (continued) rectangular array, 21 of galaxy, 111
EPR paradox, 87 redshift nuclear, 99
Fermi exclusion, 242 cosmological, 41, 110 space station, 31
fermions, 177, 237 Doppler, 65 ruby, 120
interpretation of, 83 solar, 109 Rutherford, E., 73, 100
matrix origin, 84 tired-light, 110
measurement, 83 reection, total internal, 86 Sagnac effect, 43
relativistic, 230 refractive index, 59, 152 salt
rules of, 75, 224 refrigerator, 10, 24 in water, 7
scattering, 80, 81, 214 relativity, general preservative, 5
stationary states, 67 centrifugal force, 111 sea salt, 134
superposition, 76 clock rate, 190, 191, 260 sand grain, 65
time asymmetry, 151 curvature, 115, 120 sandglass, 14
time operator, 181 equivalence, 107, 121, 190 satellite
tunneling, 85, 237, 238 geodesics, 111 debris, 32, 37
wave function, 75, 78 GPS accuracy, 109 GPS, 108
quark light bending, 109 scattering
b quark, 276 redshift, 109, 262 coherent, 80, 81, 176, 227
masses, 103, 116, 160 test masses, 107 constructive, 80
families, 101, 276 twin paradox, 46, 60, 108 neutrino, 81
proton mass, 103 twin watches, 108 photon, 78
Standard Model, 101 twistors, 150 Schrdinger equation
quasars, 41, 171 relativity, special benzene, 69
quaternions, 163 accelerations, 44 de Broglie waves, 101
charge, 47 nuclear, 93
Rabi, I. I., 72, 81 clock rate, 261, 262 symmetry lack, 102
radiation Doppler formula, 278 Schrdingers cat, 77
cosmic, 112, 124 Einstein, A., 41 Schwarzschild, K.
infrared, 8 energy dened, 40 metric, 262, 263
Unruh, 121 headlight effect, 106 radius, 234
radioactivity invariants, 40 science ction, 35
background, 98, 247 Larmor, J., 41 scientic method, 97
C-14 dating, 93, 100 Lewis, G., light, 71 scientist, 16, 23, 36, 59
K-40 in human, 98 light energy, 119 scissors closing, 182
safety, 36 mass-energy, 41, 42 scooter, twist, 120
Ramanujan, S., 23 Planck, M., 115 Seaborg, G. T., 123
randomness Sagnac effect, 186 SET diode, 200
Brownian motor, 50 Terrell effect, 162, 183 sexagesimals, 13
coin tosses, 50 twin paradox, 46, 60, 108 Sierpinski triangle, 158
crystal growth, 280 twin watches, 108 silicon, 81
molecular, 51, 136 rice, 9, 14, 143 sodium D lines, 138
phases, 227 Riemann, G. B., 111 solar heating, 195
stability, for, 59 robot, 23, 59 Solar System, 113
thermal, 51 Roman numeral V, 25 solitons, 59
in walk steps, 50 rotation solstices, 12, 13, 14
in water, 136 book, 26 sonoluminescence, 71
ratchet, 50 cup, 27 sound
recoilless absorption, 65 double, 25 archaeology, 53

294 Index
birdsong, 122 spider and y, 20 Cooper pairs, 84
existence of, 164 spin ip, 226 Meissner effect, 55
explosions, 33 spin one-half, 161, 177 quantum effects, 84
in friction, 129 spotlight, 41 SQUIDS, 85
into light ash, 71 square root, 27 suspension, 55
in space, 33, 36 SQUIDS, 85 superuidity, 72, 85
speed of, 35, 173, 182 St. Augustine, 11 supernova
in Weber bar, 229 stability, 27, 94, 108, 164 element synthesis, 243
soup, can of, 4 Standard Model intensity curve, 266
space b quark, 276 1987A, 173
conguration, 23, 78, 233 discrete subgroups, 254 Type 1a, 233
crawler, 124 families, 101, 276 superposition
curvature, 115 gauge group, 254 benzene, 214
dimensions of, 27 Higgs particle, 102, 254 computer states, 79
discrete, 115 mass ratios, 160 electron in box, 78
expansion, 41, 171, 258, weak interaction, 147 rule, 75
265 star two states, 77, 214
lling, 21, 22, 153, 275 C-12 synthesis, 95 super-radiance, 74
4-D, 24, 269 diameters, 121 Swift, J., on Mars, 30
imaginary, 27 element synthesis, 95 syllogism, 22, 97
spaceship Fe-56 in, 94 symmetry
accelerating, 44, 45 helium, 95 asymmetry, 50
black hole, 269 hydrogen, 97 breaking, 101
collisions, 46 lifetime, 257 discrete, 162, 254
communication from, 43 proton-proton cycle, 96 ferromagnet, 102
energy need, 170 shapes, 113 group theory, 160
in movies, 32, 34 stereoscopic view, 184 polyhedral, 160
nuclear, 32 Stevin, S., 59 potential, 50
propulsion, 171, 286 Stiglers law, 24 rotational, 24, 214
relativistic, 41, 43, 44 stimulated emission spherical, 180
temperature of, 179 laser, 68 SU(2), 162
warp speed, 32 noninversion, 68 tetrahedral, 160, 215
space-time strain gauge, 42 time, 18
Einstein, A., 89 straw in potato, 4
coordinates, 27 sugar tasting food, on, 2
curvature, 120 igniting, 7 tau, 101
distorting, 171 preserving with, 5 tea water, heating, 3
fabric, tear of, 37 sucrose in blood, 9 teakettle, water vapor, 8
interval, 40 in water, 3 Teller, E., 100, 250
imaginary space, 27 summer in January, 12 temperature
Minkowski, 27 sun boiling, 5
quaternions, 163 proton cycle, 96 Earth, 178, 216
spark gap, 278 redshift, 109 infrared, 8
spectral ngerprint, 206 sun rhyme, 34 nanoKelvin, 223
spheres sundial, 14 radiation, 8
intersecting, 25 sunlight spectrum, 210 Terrell effect, 183, 184
reactions on, 137 sunrise, sunset, 12, 13 Tesla, N., 58, 61
three-sphere, 25 superconductor tesselation, 153

Index 295
tetrahedron, 22, 24, 160 different ones, 115, 124 heating for tea, 8
theory and truth, 24 dimensions of, 153 helium in, 217
thermal energy expansion, 106, 110, 265 ice in, 6
burning in air, 133 inationary, 41, 271 salt in, 7
equilibrium, 178 potential well, 271 spouting, 122
ow direction, 58 saying about, 52 sugar in, 3
mantle, as source, 217 total energy in, 115 in universe, 3
random motion, 51 Unruh radiation, 121 water molecule, 8, 9
thermal stress, ice 133 water vapor
Thomson, J. J., 47 vacuum absorption, 216
thinking too much, 119 electron spin, 84 condensing, 135
thunder, 33 Casimir effect, 84 greenhouse gas, 70, 71
tidal effects, 57 Higgs particle, 102 teapot, 8
time Lamb shift, 84 wave function
coordinate, 27 muon g-2, 236 collapse, 78
direction, 276 quantum, energy in, 84 conguration space, 78
elapsed, 46, 108 particle soup, 84, 234 Cooper pair, 237
goes by, 12 Unruh radiation, 121 coordinate space, 78
gravity, 60 van der Waals force, 135 dened, 75
interval, 42, 43, 190 vapor pressure, 135 two-particle, 224
reversal, 16, 210 velocity weak interaction
St. Augustine, 11 limit c, 41 bosons, 103
symmetry, 18 parameter, 188, 189 doublet states, 255
travel, 34 redshift, 41 left-handedness, 147
twin watches, 108 relativistic, 41, 45 symmetry, 102
tomato, peeling, 7 Venus, orbit period, 68 Z boson, 103
tritium, 230 Verne, Jules Weber, J.
tunnel junction, 56 memorial, 35 bar detector, 81
tunneling Moon trip, 31 coherence, 81, 82
light, 86 weightlessness, 31 gravitation, 81
nuclear, 86 video games, 21, 35 laser and maser, 68
SQUIDS, 85 volume neutrinos, 81
twin paradox, 46, 60, 108 cooking meat, 6 weightlessness, 31
two-photon to surface area, 133, 135 Weinberg, S., 266
absorption, 207 Wells, H. G., 33, 37
H emission, 207 watch Wheeler, J. 250, 259
nding north, 18 wings, beating, 31
U-235, 36, 94, 97, 99, 180 free falling, 108 women, 4, 10
uncertainty mountains, rate in, 14 wrist watch, 87
in amplitude, 235 water
digital, 145 boiling, altitude, 5 X-ray
hydrogen, 231 boiling, pot, 8 Bragg scattering, 79
in phase, 235 carbon dioxide in, 9 laser, 73, 212
principle, 83, 233, 254 cloud weight, 33 refractive index, 69
relation, 232 cup of, spouting, 285
simplied, 89 density, max, 133 year length, 13, 15
universe evaporation, 9
curvature, 275 expansion, 172 zero-point energy, 235

296 Index

Vous aimerez peut-être aussi